Vous êtes sur la page 1sur 206

Remedial Law I| Page 1

Jurisdiction
Duero vs. Court of Appeals
G.R. No. 131282, January 4, 2002
Quisumbing, J.

Facts:

Gabriel Duero filed before the RTC a complaint for recovery of possession and ownership against private respondent, Bernardo
Eradel, and two others alleging that Eradel entered and occupied Duero’s land. As shown in the tax declaration, the land had an
assessed value of P5,240. Eradel was not able to file an Answer and he was declared in default. Duero presented his evidence ex parte
and a judgment was rendered ordering Eradel to vacate the land. Eradel filed a Motion for New Trial explaining that he turned over
the complaint and summons to Artemio Laurente in the honest belief that as landlord, the latter had a better right to the land and was
responsible to defend any adverse claim on it. Meanwhile, RED Conflict Case No. 1029, an administrative case between Duero and
applicant-contestants Romeo, Artemio and Jury Laurente, remained pending with the Office of the Regional Director of the
Department of Environment and Natural Resources in Davao City.

The RTC denied the Motion for New Trial. Eradel filed a Petition for Relief from Judgment which the RTC also denied. In his Motion
for Reconsideration, Eradel alleged that the RTC has no jurisdiction over the case, since the value of the land was only P5,240 and
therefore it was under the jurisdiction of the MTC. The RTC again denied the said motion. On appeal, the CA reversed the RTC
decreeing that Eradel was not estopped from assailing the jurisdiction of the RTC.

Issue:

Whether or not private respondent Eradel was estopped from assailing the jurisdiction of the RTC?

Held:

No. The fundamental rule is that, the lack of jurisdiction of the court over an action cannot be waived by the parties, or even cured by
their silence, acquiescence or even by their express consent. Further, a party may assail the jurisdiction of the court over the action at
any stage of the proceedings and even on appeal. Even if private respondent actively participated in the proceedings before said court,
the doctrine of estoppel cannot still be properly invoked against him because the question of lack of jurisdiction may be raised at
anytime and at any stage of the action. Precedents tell us that as a general rule, the jurisdiction of a court is not a question of
acquiescence as a matter of fact, but an issue of conferment as a matter of law. Also, neither waiver nor estoppel shall apply to confer
jurisdiction upon a court, barring highly meritorious and exceptional circumstances.

Here, petitioner, who claims ownership of a parcel of land, filed his complaint before a court without appropriate jurisdiction.
Defendant, a farmer whose tenancy status is still pending before the proper administrative agency concerned, could have moved for
dismissal of the case on jurisdictional grounds. But the farmer as defendant therein could not be expected to know the nuances of
jurisdiction and related issues. This farmer, who is now the private respondent, ought not to be penalized when he claims that he made
an honest mistake when he initially submitted his motions before the RTC, before he realized that the controversy was outside the
RTC’s cognizance but within the jurisdiction of the municipal trial court. To hold him in estoppel as the RTC did would amount to
foreclosing his avenue to obtain a proper resolution of his case.
Remedial Law I| Page 2

Donato vs. Court of Appeals


G.R. No. 129638, December 8, 2003
Austria-Martinez, J.

Facts:

Petitioner Antonio T. Donato filed a complaint for forcible entry and unlawful detainer against 43 named defendants. He alleged that
the defendants had oral contract of lease that expired at the end of each month but were impliedly renewed under the same terms by
mere acquiescence or tolerance. Sometime in 1992, they stopped paying rent and in 1994, Donato sent them a written demand to
vacate; and the non-compliance with said demand letter constrained him to file the ejectment case against them. Of the 43 named
defendants, only 20 filed a consolidated Answer. They contended that they cannot be evicted because the Urban Land Reform Law
guarantees security of tenure and priority right to purchase the subject property. The MeTC rendered two separate judgments: a
demand to vacate on the non-answering defendants and a judgment in favor of the 20 respondents sustaining their rights under the
Land Reform Law and thereby, dismissing petitioner’s complaint. Petitioner appealed to the RTC which affirmed the dismissal. The
CA also dismissed the petition for review on two grounds: a) the certification of non-forum shopping was signed by petitioner’s
counsel and not by petitioner himself and, (b) the only annex to the petition is a certified copy of the questioned decision but copies of
the pleadings and other material portions of the record as would support the allegations of the petition are not annexed. Petitioner’s
Motion for Reconsideration on the order of dismissal was likewise denied. Petitioner then filed a petition for review on certiorari
before the Supreme Court against the CA’s order of dismissal.

Issue:

Whether or not a petition for review (Rule 45) was the proper remedy in assailing the CA’s order of dismissal based on procedural
grounds?

Held:

No. [I]n order to determine whether the recourse of petitioners is proper or not, it is necessary to draw a line between an error of
judgment and an error of jurisdiction. An error of judgment is one which the court may commit in the exercise of its jurisdiction, and
which error is reviewable only by an appeal. On the other hand, an error of jurisdiction is one where the act complained of was issued
by the court, officer or a quasi-judicial body without or in excess of jurisdiction, or with grave abuse of discretion which is tantamount
to lack or in excess of jurisdiction. This error is correctible only by the extraordinary writ of certiorari.

Inasmuch as the present petition principally assails the dismissal of the petition on ground of procedural flaws involving the
jurisdiction of the court a quo to entertain the petition, it falls within the ambit of a special civil action for certiorari under Rule 65 of
the Rules of Court.
Remedial Law I| Page 3

Gonzaga vs. Court of Appeals


G.R. No. 144025, December 27, 2002.
Corona, J.

Facts:

Petitioner-spouses, Spouses Rene and Lerio Gonzaga, purchased a parcel of land from private respondent Lucky Homes, Inc. Said lot
was specifically denominated as Lot No. 19 and was mortgaged to the Social Security System (SSS) as security for their housing loan.
Spouses Gonzaga then started the construction of their house, not on Lot No. 19 but on Lot No. 18, as Lucky Homes mistakenly
identified Lot No. 18 as Lot No. 19. Upon realizing its error, Lucky Homes, through its general manager, informed Spouses Gonzaga
of such mistake but the latter offered to buy Lot No. 18 in order to widen their premises. Thus, Spouses Gonzaga continued with the
construction of their house. However, they defaulted in the payment of their housing loan from SSS. Consequently, Lot No. 19 was
foreclosed by SSS. After Lot No. 19 was foreclosed, Spouses Gonzaga offered to swap Lot Nos. 18 and 19 and demanded from Lucky
Homes that their contract of sale be reformed and another deed of sale be executed with respect to Lot No. 18, considering that their
house was built therein. However, Lucky Homes refused. This prompted Spouses Gonzaga to file an action for reformation of contract
and damages with the RTC. The RTC dismissed the complaint. A writ of execution was then issued by the RTC. Spouses Gonzaga
filed an urgent motion to recall writ of execution, alleging that the RTC had no jurisdiction to try the case as it was vested in the
Housing and Land Use Regulatory Board (HLURB) pursuant to PD 957 (The Subdivision and Condominium Buyers Protective
Decree). Conformably, Spouses Gonzaga filed a new complaint against Lucky Homes with the HLURB. Likewise, Spouses Gonzaga
filed before the CA a petition for annulment of judgment premised on the ground that the RTC had no jurisdiction to try and hear the
case. The CA denied the petition relying mainly on the jurisprudential doctrine of estoppel.

Issue:

Whether or not the petitioners (Spouses Gonzaga) were estopped in assailing the jurisdiction of the RTC?

Held:

Yes. While an order or decision rendered without jurisdiction is a total nullity and may be assailed at any stage, active participation in
the proceedings in the court which rendered the order or decision will bar such party from attacking its jurisdiction. As held in the
leading case of Tijam vs. Sibonghanoy:

“A party may be estopped or barred from raising a question in different ways and for different reasons. Thus we speak of estoppel in
pais, or estoppel by deed or by record, and of estoppel by laches.

“It has been held that a party cannot invoke the jurisdiction of a court to secure affirmative relief against his opponent and, after
obtaining or failing to obtain such relief, repudiate, or question that same jurisdiction x x x x [T]he question whether the court had
jurisdiction either of the subject matter of the action or of the parties was not important in such cases because the party is barred
from such conduct not because the judgment or order of the court is valid and conclusive as an adjudication, but for the reason that
such a practice can not be tolerated—obviously for reasons of public policy.”

Here, it was petitioners themselves who invoked the jurisdiction of the court a quo by instituting an action for reformation of contract
against private respondents. It appears that, in the proceedings before the trial court, petitioners vigorously asserted their cause from
start to finish. Not even once did petitioners ever raise the issue of the court’s jurisdiction during the entire proceedings which lasted
for two years. It was only after the trial court rendered its decision and issued a writ of execution against them in 1998 did petitioners
first raise the issue of jurisdiction—and it was only because said decision was unfavorable to them. Petitioners thus effectively waived
their right to question the court’s jurisdiction over the case they themselves filed.
Remedial Law I| Page 4

Escobal vs. Garchitorena


G.R. No. 124644, February 5, 2004.
Callejo, Sr.

Facts:

Arnel Escobal, petitioner, is a member of the Philippine National Police. When petitioner was conducting surveillance operations on
drug trafficking at the Sa Harong Cafè Bar and Restaurant, he somehow got involved in a shooting incident, resulting in the death of
one Rodney Rafael N. Nueca. An Information was then filed with the RTC charging petitioner and a certain Jun Bombita with murder.
When petitioner commenced the presentation of his evidence, he filed a Motion to Dismiss the case arguing that since he committed
the crime in the performance of his duties, the Sandiganbayan had exclusive jurisdiction over the case. The RTC denied the motion
but ordered the conduct of a preliminary hearing to determine whether or not the crime charged was committed by the petitioner in
relation to his office as a member of the PNP. Thereafter, the RTC issued an Order declaring that the petitioner committed the crime
charged while not in the performance of his official function. The trial court added that upon the enactment of R.A. No. 7975, the
issue had become moot and academic. The amendatory law transferred the jurisdiction over the offense charged from the
Sandiganbayan to the RTC since the petitioner did not have a salary grade of “27” as provided for in or by Section 4(a)(1), (3) thereof.
The RTC, however, reversed itself in the Motion for Reconsideration and ordered the public prosecutor to file a Re-Amended
Information and to allege that the offense charged was committed by the petitioner in the performance of his duties/functions or in
relation to his office; and, conformably to R.A. No. 7975, to thereafter transmit the same, as well as the complete records with the
stenographic notes, to the Sandiganbayan. The Presiding Justice of the Sandiganbayan ordered the return of the records to the RTC. It
reasoned that the RTC retained jurisdiction over the case since petitioner had a salary grade of “23.” Furthermore, the prosecution had
already rested its case and the petitioner had commenced presenting his evidence in the RTC; following the rule on continuity of
jurisdiction, the latter court should continue with the case and render judgment therein after trial. Petitioner then filed a petition for
certiorari assailing the Order of the Sandiganbayan.

Issue:

Whether or not the Sandiganbayan has jurisdiction over a criminal case of murder against a member of the PNP with salary grade of
23?

Held:

No. For the Sandiganbayan to have exclusive jurisdiction over crimes committed by public officers in relation to their office, it is
essential that the facts showing the intimate relation between the office of the offender and the discharge of official duties must be
alleged in the Information. It is not enough to merely allege in the Information that the crime charged was committed by the offender
in relation to his office because that would be a conclusion of law. The amended Information filed with the RTC against the petitioner
does not contain any allegation showing the intimate relation between his office and the discharge of his duties.

Moreover, even if the offender committed the crime charged in relation to his office but occupies a position corresponding to a salary
grade below “27,” the proper Regional Trial Court or Municipal Trial Court, as the case may be, shall have exclusive jurisdiction over
the case. In this case, the petitioner was a Police Senior Inspector, with salary grade “23.” He was charged with homicide punishable
by reclusion temporal. Hence, the RTC had exclusive jurisdiction over the crime charged.
Remedial Law I| Page 5

Agan, Jr. vs. Philippine International Air Terminals Co., Inc. (PIATCO)
G.R. No. 155001, January 21, 2004.
Puno, J.
Resolution

Facts:

Asia’s Emerging Dragon Corp. (AEDC) submitted an unsolicited proposal to the Philippine Government through the Department of
Transportation and Communication (DOTC) and Manila International Airport Authority (MIAA) for the construction and
development of the NAIA IPT III under a build-operate-and-transfer arrangement. The DOTC/MIAA invited the public for
submission of competitive and comparative proposals to the unsolicited proposal of AEDC. The Paircargo consortium then submitted
their competitive proposal to the Prequalification Bids and Awards Committee (PBAC). After finding that the Paircargo Consortium
submitted a bid superior to the unsolicited proposal of AEDC and after failure by AEDC to match the said bid, the DOTC issued the
notice of award for the NAIA IPT III project to the Paircargo Consortium, which later organized into herein respondent PIATCO. The
Government, through then DOTC Secretary Arturo T. Enrile, and PIATCO, through its President, Henry T. Go, signed the
“Concession Agreement for the Build- Operate-and-Transfer Arrangement of the Ninoy Aquino International Airport Passenger
Terminal III” (1997 Concession Agreement). This agreement was later on amended and supplemented. Thereafter, various petitions
were filed before the Supreme Court to annul the 1997 Concession Agreement, the ARCA and the Supplements and to prohibit the
public respondents DOTC and MIAA from implementing them. The SC granted the petitions and declared the 1997 Concession
Agreement, the ARCA and the Supplements null and void. Respondents now seek reversal of the said decision. One of their
contentions was that the principle of hierarchy of courts precludes the SC from taking primary jurisdiction over the case.

Issue:

Whether or not the Supreme Court can take primary jurisdiction over the case?

Held:

Yes. The rule on hierarchy of courts in cases falling within the concurrent jurisdiction of the trial courts and appellate courts generally
applies to cases involving warring factual allegations. For this reason, litigants are required to repair to the trial courts at the first
instance to determine the truth or falsity of these contending allegations on the basis of the evidence of the parties. Cases which
depend on disputed facts for decision cannot be brought immediately before appellate courts as they are not triers of facts.

It goes without saying that when cases brought before the appellate courts do not involve factual but legal questions, a strict
application of the rule of hierarchy of courts is not necessary. As the cases at bar merely concern the construction of the Constitution,
the interpretation of the BOT Law and its Implementing Rules and Regulations on undisputed contractual provisions and government
actions, and as the cases concern public interest, the Supreme Court resolved to take primary jurisdiction over them. This choice of
action follows the consistent stance of the Court to settle any controversy with a high public interest component in a single proceeding
and to leave no root or branch that could bear the seeds of future litigation. The suggested remand of the cases at bar to the trial court
will stray away from this policy.
Remedial Law I| Page 6

Liga ng mga Barangay National vs. Atienza, Jr.


G.R. No. 154599, January 21, 2004.
Davide, Jr., CJ.

Facts:

Petitioner Liga ng mga Barangay National (Liga) is the national organization of all the barangays in the Philippines which constitutes
the duly elected presidents of highly-urbanized cities, provincial chapters, the metropolitan Manila Chapter, and metropolitan political
subdivision chapters. Section 493 of the Local Government Code empowers the Liga to adopt its Constitution and By-Laws to govern
its internal organization. In implementing its Election Code, the Liga set October 21, 2000 for the synchronized elections for highly
urbanized city chapters, such as the Liga Chapter of Manila, together with independent component city, provincial, and metropolitan
chapters. Meanwhile, the respondent City Council of Manila enacted Ordinance No. 8039, Series of 2002, providing, among other
things, for the election of representatives of the District Chapters in the City Chapter of Manila and setting the elections for both
chapters thirty days after the barangay elections. Thereafter, the Liga filed a petition for certiorari before the Supreme Court assailing
the validity of the said ordinance as it contradicts the Liga Election Code. On the other hand, the respondents seek the dismissal of the
petition on the following grounds: (1) the petitioner availed the wrong remedy as certiorari is available only to one aggrieved by the
decision of a tribunal, officer, or board exercising judicial or quasi-judicial functions; since the City Council and the City Mayor
exercise legislative function and executive functions, respectively, certiorari cannot be availed of; (2) the SC cannot take cognizance
of the case based on the doctrine of hierarchy of courts.

Issues:

(1) Whether or not the petitioner availed of the wrong remedy (Rule 65)?
(2) Whether or not the Supreme Court can take cognizance of the case?

Held:

(1) Yes. The enactment by the City Council of Manila of the assailed ordinance and the issuance by respondent Mayor of the
questioned executive order were done in the exercise of legislative and executive functions, respectively, and not of judicial or quasi-
judicial functions. On this score alone, certiorari will not lie. Second, although the instant petition is styled as a petition for certiorari,
in essence, it seeks the declaration by this Court of the unconstitutionality or illegality of the questioned ordinance and executive
order. It, thus, partakes of the nature of a petition for declaratory relief over which this Court has only appellate, not original,
jurisdiction. As such, this petition must necessary fail, as this Court does not have original jurisdiction over a petition for declaratory
relief even if only questions of law are involved.

(2) No. The Supreme Court’s original jurisdiction to issue a writ of certiorari (as well as of prohibition, mandamus, quo warranto,
habeas corpus and injunction) is not exclusive, but is concurrent with the Regional Trial Courts and the Court of Appeals in certain
cases. This concurrence of jurisdiction is not, however, to be taken as according to parties seeking any of the writs an absolute,
unrestrained freedom of choice of the court to which application therefore will be directed. There is after all a hierarchy of courts. That
hierarchy is determinative of the venue of appeals, and also serves as a general determinant of the appropriate forum for petitions for
the extraordinary writs. A becoming regard of that judicial hierarchy most certainly indicates that petitions for the issuance of
extraordinary writs against first level (“inferior”) courts should be filed with the Regional Trial Court, and those against the latter, with
the Court of Appeals. A direct invocation of the Supreme Court’s original jurisdiction to issue these writs should be allowed only
when there are special and important reasons therefor, clearly and specifically set out in the petition. This is [an] established policy. It
is a policy necessary to prevent inordinate demands upon the Court’s time and attention which are better devoted to those matters
within its exclusive jurisdiction, and to prevent further overcrowding of the Court’s docket.
Remedial Law I| Page 7

Manila Bankers Life Insurance Corporation vs. Ng Kok Wei


G.R. No. 139791, December 12, 2003
Sandoval-Gutierrez, J.

Facts:

Eddy Ng Kok Wei, respondent, sent a Letter of Intent addressed to Manila Bankers Life Insurance Corporation, petitioner, to purchase
a condominium unit at Valle Verde Terraces. When respondent was able to pay 90% of the purchase price, petitioner’s President
executed a Contract to Sell in favor of the respondent. The contract expressly states that the subject condominium unit “shall
substantially be completed and delivered” to the respondent “within fifteen (15) months” from February 8, 1989 or on May 8, 1990,
and that “(S)hould there be no substantial completion and fail(ure) to deliver the unit on the date specified, a penalty of 1% of the total
amount paid (by respondent) shall be charged against (petitioner)”. In October 1990, the subject condominium unit was still inlivable.
Exasperated, respondent demanded from petitioner payment for damages he sustained. As the demand went unheeded, respondent
filed a complaint for specific performance and damages before the RTC. During the pendency of the case, respondent finally accepted
the condominium unit and therefore his cause of action has been limited to his claim for damages. The RTC ruled in favor of the
respondent which the CA affirmed. In a petition for review before the SC, petitioner contends that the trial court has no jurisdiction
over the instant case.

Issue:

Whether or not petitioner Manila Bankers is estopped in assailing the jurisdiction of the RTC?

Held:

Yes. Based on Presidential Decree No. 1344, as amended, it is the HLURB which has jurisdiction over complaints for specific
performance with damages by a lot or condominium unit buyer against the owner or developer. While it may be true that the trial court
is without jurisdiction over the case, petitioner’s active participation in the proceedings estopped it from assailing such lack of it. The
Court held that it is an undesirable practice of a party participating in the proceedings and submitting its case for decision and then
accepting the judgment, only if favorable, and attacking it for lack of jurisdiction, when adverse. Here, petitioner failed to raise the
question of jurisdiction before the trial court and the Appellate Court. In effect, petitioner confirmed and ratified the trial court’s
jurisdiction over this case. Certainly, it is now in estoppel and can no longer question the trial court’s jurisdiction.
Remedial Law I| Page 8

Office of the Court Administrator vs. Sardido


A.M. No. MTJ-01-1370, April 25, 2003 (Formerly A.M. No. 00-11-238-MTC)
Carpio, J.

Facts:

Judge Braulio Hurtado, an RTC Judge, was charged with the crime of Falsification by Private Individual and Use of Falsified
Document which case was assigned to Judge Agustin Sardido, the presiding judge of the Municipal Trial Court of Koronadal, South
Cotabato. The crime arose from a deed of absolute sale which was notarized by Judge Hurtado at the time when he was still the Clerk
of Court of RTC-Koronadal and ex-officio notary public. Judge Hurtado filed a motion praying that the criminal complaint against
him be forwarded to the Supreme Court. Judge Hurtado claimed that Circular No. 3-89 requires “all cases involving justices and
judges of the lower courts, whether or not such complaints deal with acts apparently unrelated to the discharge of their official
functions, such as acts of immorality, estafa, crimes against persons and property, etc.” to be forwarded to the Supreme Court. Judge
Sardido then excluded the name of Judge Hurtado from the Information and the case against him was forwarded to the Court
Administrator for appropriate action. The Court Administrator returned the case to MTC Koronadal and opined that Circular No. 3-89
refers only to administrative complaints filed with the IBP against justices and judges of lower courts. The Circular does not apply to
criminal cases filed before trial courts against such justices and judges.

Issue:

Whether or not the trial court has jurisdiction over a criminal case filed against a judge?

Held:

Yes. Circular No. 3-89 does not refer to criminal cases against erring justices of appellate courts or judges of lower courts. Trial courts
retain jurisdiction over the criminal aspect of offenses committed by justices of appellate courts and judges of lower courts. This is
clear from the Circular directing the IBP, and not the trial courts, to refer all administrative cases filed against justices of appellate
courts and judges of lower courts to the Supreme Court. The case filed against Judge Hurtado is not an administrative case filed with
the IBP. It is a criminal case filed with the trial court under its jurisdiction as prescribed by law.
Remedial Law I| Page 9

Katon vs. Palanca, Jr.


G.R. No. 151149, September 7, 2004
Panganiban, J.

Facts:

In 1963, George Katon filed a request with the District Office of the Bureau of Forestry in Puerto Princesa, Palawan, for the re-
classification of a piece of real property known as Sombrero Island, located in Tagpait, Aborlan, Palawan. A survey was conducted
and in 1965, the subject land was certified and released as an agricultural land. Records show that respondents (Fresnillo, Gapilango
and Palanca) filed and were issued homestead patents for various hectares within the Sombrero Island. Petitioner now assails the
validity of the homestead patents and asked for the reconveyance of the lands to him before the RTC. Respondents filed a Motion to
Dismiss on the ground of the alleged defiance by petitioner of the trial court’s Order to amend his Complaint so he could thus effect a
substitution by the legal heirs of the deceased, Respondent Gapilango. The Motion to Dismiss was granted by the RTC in its Order
dated July 29, 1999. Petitioner then filed a Petition for Certiorari before the CA. The CA, instead of ruling only on the alleged grave
abuse of discretion, ruled on the merits. On the MR, the CA acknowledged that it had erred when it ruled on the merits of the case. But
still dismissed the case pursuant to its “residual prerogative” under Section 1 of Rule 9 of the Rules of Court. It ruled that petitioner’s
action was brought 24 years after the issuance of Palanca’s homestead patent. Under the Public Land Act, such action should have
been taken within ten years from the issuance of the homestead certificate of title. Second, it appears from the submission of petitioner
himself that Respondents Fresnillo and Palanca had been occupying six hectares of the island since 1965, or 33 years before he took
legal steps to assert his right to the property. His action was filed beyond the 30-year prescriptive period under Articles 1141 and 1137
of the Civil Code. Petitioner contends that the CA erroneously invoked its “residual prerogatives” under Section 1 of Rule 9 of the
Rules of Court when it motu proprio dismissed the Petition for lack of jurisdiction and prescription. According to him, residual
prerogative refers to the power that the trial court, in the exercise of its original jurisdiction, may still validly exercise even after
perfection of an appeal. It follows that such powers are not possessed by an appellate court.

Issue:

Whether or not the Court of Appeals properly exercised its power of residual prerogative?

Held:

Yes. Petitioner has confused what the CA adverted to as its “residual prerogatives” under Section 1 of Rule 9 of the Rules of Court
with the “residual jurisdiction” of trial courts over cases appealed to the CA.

Under Section 1 of Rule 9 of the Rules of Court, defenses and objections not pleaded either in a motion to dismiss or in the answer are
deemed waived, except when (1) lack of jurisdiction over the subject matter, (2) litis pendentia, (3) res judicata and (4) prescription
are evident from the pleadings or the evidence on record. In the four excepted instances, the court shall motu proprio dismiss the claim
or action.

On the other hand, “residual jurisdiction” is embodied in Section 9 of Rule 41 of the Rules of Court. The “residual jurisdiction” of trial
courts is available at a stage in which the court is normally deemed to have lost jurisdiction over the case or the subject matter
involved in the appeal. This stage is reached upon the perfection of the appeals by the parties or upon the approval of the records on
appeal, but prior to the transmittal of the original records or the records on appeal. In either instance, the trial court still retains its so-
called residual jurisdiction to issue protective orders, approve compromises, permit appeals of indigent litigants, order execution
pending appeal, and allow the withdrawal of the appeal.

In this case, petitioner’s complaint did not sufficiently make a case for declaration of nullity of the free patent and certificate of title
or, alternatively, for reconveyance. Also, the action should be dismissed for being time-barred.
Remedial Law I| Page 10

Figueroa vs. People


G.R. No. 147406, July 14, 2008

Facts:

Petitioner was charged with an information for reckless imprudence resulting in homicide before the RTC of Bulacan. Petitioner was
convicted by the RTC. On appeal, petitioner questioned for the first time the trial court’s jurisdiction. The Court of Appeals, however,
found petitioner estopped from questioning the jurisdiction of the RTC as he had actively participated in the trial. The CA also
affirmed his conviction.

Issue:

Whether or not the petitioner is already estopped in assailing the jurisdiction of the RTC?

Held:

No. As the imposable penalty for the crime charged was less than 6 years, jurisdiction to hear and try the same is conferred on the
Municipal Trial Courts (MTCs). Clearly, therefore, the RTC of Bulacan does not have jurisdiction over Criminal Case No. 2235-M-
94.

The general rule should be, as it has always been, that the issue of jurisdiction may be raised at any stage of the proceedings, even on
appeal, and is not lost by waiver or by estoppel. Estoppel by laches, to bar a litigant from asserting the court’s absence or lack of
jurisdiction, only supervenes in exceptional cases similar to the factual milieu of Tijam v. Sibonghanoy. Indeed, the fact that a person
attempts to invoke unauthorized jurisdiction of a court does not estop him from thereafter challenging its jurisdiction over the subject
matter, since such jurisdiction must arise by law and not by mere consent of the parties. This is especially true where the person
seeking to invoke unauthorized jurisdiction of the court does not thereby secure any advantage or the adverse party does not suffer any
harm.

Here, the petitioner is in no way estopped by laches in assailing the jurisdiction of the RTC, considering that he raised the lack thereof
in his appeal before the appellate court. At that time, no considerable period had yet elapsed for laches to attach. True, delay alone,
though unreasonable, will not sustain the defense of “estoppel by laches” unless it further appears that the party, knowing his rights,
has not sought to enforce them until the condition of the party pleading laches has in good faith become so changed that he cannot be
restored to his former state, if the rights be then enforced, due to loss of evidence, change of title, intervention of equities, and other
causes.

Estoppel, being in the nature of a forfeiture, is not favored by law. It is to be applied rarely—only from necessity, and only in
extraordinary circumstances. The doctrine must be applied with great care and the equity must be strong in its favor.
Remedial Law I| Page 11

Serana vs. Sandiganbayan


G.R. No. 162059. January 22, 2008.

Facts:

Petitioner was appointed Student Regent of University of the Philippines and discussed with then President Estrada the proposed
renovation of Vinzon’s Hall. She and her siblings registered with the SEC the Office of the Student Regent Foundation, Inc. (OSRFI).
The OSRFI received from President Estrada 15 million pesos as financial assistance for the proposed renovation which failed to
materialize.

The successor of the petitioner filed a complaint for Malversation of Public Funds with the Office of the Ombudsman. The
Ombudsman found probable cause and charged the petitioner with Estafa. Petitioner filed a motion to dismiss on the ground that the
Sandiganbayan has no jurisdiction over her person as she was not a public officer.

The Sandiganbayan denied her motion. Hence, this petition.

Issues:

(1) Whether or not the Sandiganbayan has jurisdiction over the subject matter of the offense charged?

(2) Whether or not Serana (a student regent) may be considered as a public officer?

Held:

Yes. Petitioner avers that estafa is not one of the crimes enumerated under Sec. 4(1) of PD 1606, as amended. Petitioner is mistaken.
Sec. 4 must be read in consonance with the other paragraphs of Sec. 4. It must be isolated with the succeeding paragraphs.
Sec. 4(b) of PD 1606, as amended states -
b. Other offenses or felonies whether simple or
complexed with other crimes committed by the public officials and employees mentioned in subsection a of this section in relation to
their office;

Evidently, the Sandiganbayan has jurisdiction over other felonies committed by public officials in relation to their office.

(2) Yes.

A public office is the right, authority, and duty created and conferred by law, by which for a given period, either fixed by law or
enduring at the pleasure of the creating power, an individual is invested with some portion of the sovereign functions of the
government, to be exercised by him for the benefit of the public. The individual so invested is a public officer.

Sec. 4(A)(1)(g) explicitly vests the Sandiganbayan with jurisdiction over Presidents, directors or trustees, or managers of government
owned or controlled corporations, state universities or educational institutions or foundations. Petitioner falls under this category. The
Board of Regents performs functions similar to those of a board of trustees of a non-stock corporation. By express mandate of the law,
petitioner is, indeed, a public officer as contemplated in PD No. 1606, as amended.

Moreover, compensation is not an essential element of public office.

Petitioner likewise aver that the offense was done in her private capacity. This should also likewise be answered in the negative. As
long as the public office is facilitated to commit the crime, the Sandiganbayan has jurisdiction over the offense.
Remedial Law I| Page 12

Pat-og, Sr. vs. Civil Service Commission


G.R. No. 198755, June 5, 2013

Facts:

Complainant Bang-on was second year high school student who was punched by the petitioner on his stomach for not falling in line
properly.

Complainant filed an administrative case to the CSC-CAR. CSC-CAR found the petitioner guilty of simple misconduct. Pat-Og then
filed an appeal before the CSC which affirmed the finding of CSC-CAR and ordered the dismissal of Pat-Og. Pat-Og then filed an MR
questioning for the first time the jurisdiction of CSC over the case. He contended that administrative charges against a public school
teacher should have been initially heard by a committee to be constituted pursuant to the Magna Carta for Public School Teachers.

The CSC dismissed the MR. It ruled that Pat-og was estopped from challenging its jurisdiction considering that he actively
participated in the administrative proceedings against him, raising the issue of jurisdiction only after his appeal was dismissed by the
CSC. On appeal, the appellate court affirmed the decision of the CSC.

Issues:

(1) Whether or not the CSC has jurisdiction over an administrative case involving a public school teacher?
(2) Whether or not petitioner is already estopped in questioning the jurisdiction of CSC?

Held:

(1) Yes. In the case of Puse vs. Santos-Puse, it was held that the CSC, Department of Education and the Board of Professional
Teachers-Professional Regulatory Commission (PRC) have concurrent jurisdiction over administrative cases against public school
teachers.

Under Article IX-B of the 1987 Constitution, the CSC is the body charged with the establishment and administration of a career civil
service which embraces all branches and agencies of the government.

Executive Order (E.O.) No. 292 (the Administrative Code of 1987) and Presidential Decree (P.D.) No. 807 (the Civil Service Decree
of the Philippines) expressly provide that the CSC has the power to hear and decide administrative disciplinary cases instituted with it
or brought to it on appeal. Thus, the CSC, as the central personnel agency of the government, has the inherent power to supervise and
discipline all members of the civil service, including public school teachers.

Indeed, under Section 9 of R.A. No. 4670, the jurisdiction over administrative cases of public school teachers is lodged with the
investigating committee constituted therein.14 Also, under Section 23 of R.A. No. 7836 (the Philippine Teachers Professionalization
Act of 1994), the Board of Professional Teachers is given the power, after due notice and hearing, to suspend or revoke the certificate
of registration of a professional teacher for causes enumerated therein.

Concurrent jurisdiction is that which is possessed over the same parties or subject matter at the same time by two or more
separate tribunals. When the law bestows upon a government body the jurisdiction to hear and decide cases involving specific
matters, it is to be presumed that such jurisdiction is exclusive unless it be proved that another body is likewise vested with the
same jurisdiction, in which case, both bodies have concurrent jurisdiction over the matter.

Where concurrent jurisdiction exists in several tribunals, the body that first takes cognizance of the complaint shall exercise
jurisdiction to the exclusion of the others. In this case, it was CSC which first acquired jurisdiction over the case because the
complaint was filed before it. Thus, it had the authority to proceed and decide the case to the exclusion of the DepEd and the Board of
Professional Teachers.

In CSC v. Alfonso, it was held that special laws, such as R.A. No. 4670, do not divest the CSC of its inherent power to supervise and
discipline all members of the civil service, including public school teachers. Pat-og, as a public school teacher, is first and foremost, a
civil servant accountable to the people and answerable to the CSC for complaints lodged against him as a public servant. To hold that
R.A. No. 4670 divests the CSC of its power to discipline public school teachers would negate the very purpose for which the CSC was
established and would impliedly amend the Constitution itself.

To further drive home the point, it was ruled in CSC v. Macud that R.A. No. 4670, in imposing a separate set of procedural
requirements in connection with administrative proceedings against public school teachers, should be construed to refer only to the
Remedial Law I| Page 13

specific procedure to be followed in administrative investigations conducted by the DepEd. By no means, then, did R.A. No. 4670
confer an exclusive disciplinary authority over public school teachers on the DepEd.

(2) Yes. At any rate, granting that the CSC was without jurisdiction, the petitioner is indeed estopped from raising the issue. Although
the rule states that a jurisdictional question may be raised at any time, such rule admits of the exception where, as in this case, estoppel
has supervened. Here, instead of opposing the CSC’s exercise of jurisdiction, the petitioner invoked the same by actively participating
in the proceedings before the CSC-CAR and by even filing his appeal before the CSC itself; only raising the issue of jurisdiction later
in his motion for reconsideration after the CSC denied his appeal. This Court has time and again frowned upon the undesirable
practice of a party submitting his case for decision and then accepting the judgment only if favorable, but attacking it for lack of
jurisdiction when adverse.
Remedial Law I| Page 14

Boston Equity Resources, Inc. vs. Court of Appeals


G.R. No. 173946. June 19, 2013.

Facts:

In 1997, Boston Equity Resources, Inc. filed a complaint for sum of money against the spouses Manuel and Lolita Toledo. In March
1998, Lolita Toleda filed an Answer which she amended in May 1998 alleging that her husband was already dead. Petitioner then filed
a Motion for Substitution. In 2004, after the presentation of evidence of the petitioner, respondent filed a Motion to Dismiss (instead
of a demurrer); one of the grounds alleged being invoked was that the trial court never acquired jurisdiction over the person of
Manuel. The RTC denied the MTD on the ground that “defendants’ attack on the jurisdiction is now barred by estoppel by laches”
since respondent failed to raise the issue despite several chances to do so. The respondent filed a petition for certiorari before the CA.
The CA granted the petition because according to it, jurisdiction may be raised at any stage of the proceeding, even for the first time
on appeal. By timely raising the issue on jurisdiction in her motion to dismiss, respondent is not estopped from raising the question on
jurisdiction.

Issue:

Whether or not jurisdictional estoppel applies to jurisdiction over the person of the defendant?

Held:

No. The aspect of jurisdiction which may be barred from being assailed as a result of estoppel by laches is jurisdiction over the subject
matter. Here, what respondent was questioning in her motion to dismiss before the trial court was that court’s jurisdiction over the
person of defendant Manuel. Thus, the principle of estoppel by laches finds no application in this case. Instead, the principles relating
to jurisdiction over the person of the parties are pertinent herein.

Since the defense of lack of jurisdiction over the person of a party to a case is not one of those defenses which are not deemed waived
under Section 1 of Rule 9, such defense must be invoked when an answer or a motion to dismiss is filed in order to prevent a waiver of
the defense. If the objection is not raised either in a motion to dismiss or in the answer, the objection to the jurisdiction over the person
of the plaintiff or the defendant is deemed waived by virtue of the first sentence of the above-quoted Section 1 of Rule 9 of the Rules
of Court.
Remedial Law I| Page 15

People of the Philippines vs. Henry Go.


G.R. No. 168539. March 25, 2014

Facts:

Respondent was charged with violation of RA 3019 for conspiring with then DOTC Sec. Arturo Enrile, now deceased, in relation to
contracts entered by them which is grossly disadvantageous to the government.

Respondent filed a motion to quash on the ground of lack of jurisdiction as he is only a private individual and not a public officer.
Independently of the deceased Secretary Enrile, the public officer with whom he was alleged to have conspired, respondent, who is
not a public officer nor was capacitated by any official authority as a government agent, may not be prosecuted for violation of
Section 3(g) of R.A. 3019.

Issue:

Whether or not respondent, a private person, may be indicted for conspiracy in violating Section 3(g) of R.A. 3019 even if the public
officer, with whom he was alleged to have conspired, has died prior to the filing of the Information?

Held:

Yes. It is true that by reason of Secretary Enrile's death, there is no longer any public officer with whom respondent can be charged for
violation of R.A. 3019. It does not mean, however, that the allegation of conspiracy between them can no longer be proved or that
their alleged conspiracy is already expunged. The only thing extinguished by the death of Secretary Enrile is his criminal liability. His
death did not extinguish the crime nor did it remove the basis of the charge of conspiracy between him and private respondent. Stated
differently, the death of Secretary Enrile does not mean that there was no public officer who allegedly violated Section 3 (g) of R.A.
3019. In fact, the Office of the Deputy Ombudsman for Luzon found probable cause to indict Secretary Enrile for infringement of
Sections 3 (e) and (g) of R.A. 3019. Were it not for his death, he should have been charged.

The requirement before a private person may be indicted for violation of Section 3(g) of R.A. 3019, among others, is that such private
person must be alleged to have acted in conspiracy with a public officer. The law, however, does not require that such person must, in
all instances, be indicted together with the public officer. If circumstances exist where the public officer may no longer be charged in
court, as in the present case where the public officer has already died, the private person may be indicted alone.
Remedial Law I| Page 16

City of Manila vs. Judge Cuerdo


G.R. No. 175723 February 4, 2014

Facts:

Respondent City of Manila assessed taxes SM and the other respondents. Because the payment of taxes was a precondition for the
issuance of their business permits, they were constrained to pay the said taxes under protest. Respondents then filed a Complaint
before the RTC of Pasay City for the Refund or Recovery of Illegally and/or Erroneously-Collected Local Business Tax, Prohibition
with Prayer to Issue TRO and Writ of Preliminary Injunction. The RTC granted private respondents' application for a writ of
preliminary injunction. Petitioner filed an MR but was denied. Thus, the petitioner filed a special civil action for certiorari with the CA
assailing the RTC’s order granting the writ of preliminary injunction. The CA denied the petition on the ground that it has no
jurisdiction over the petition as this is lodged with the CTA.

Issue:

Whether or not the CA has jurisdiction over a special civil action for certiorari which assails the interlocutory order issued by the RTC
in a local tax case?

Held:

No.

Section 5 (1), Article VIII of the 1987 Constitution grants power to the Supreme Court, in the exercise of its original jurisdiction, to
issue writs of certiorari, prohibition and mandamus. With respect to the Court of Appeals, Section 9 (1) of Batas Pambansa Blg. 129
(BP 129) gives the appellate court, also in the exercise of its original jurisdiction, the power to issue, among others, a writ of
certiorari,whether or not in aid of its appellate jurisdiction. As to Regional Trial Courts, the power to issue a writ of certiorari, in the
exercise of their original jurisdiction, is provided under Section 21 of BP 129.

The foregoing notwithstanding, while there is no express grant of such power, with respect to the CTA, Section 1, Article VIII of the
1987 Constitution provides, nonetheless, that judicial power shall be vested in one Supreme Court and in such lower courts as may be
established by law and that judicial power includes the duty of the courts of justice to settle actual controversies involving rights
which are legally demandable and enforceable, and to determine whether or not there has been a grave abuse of discretion amounting
to lack or excess of jurisdiction on the part of any branch or instrumentality of the Government.

On the strength of the above constitutional provisions, it can be fairly interpreted that the power of the CTA includes that of
determining whether or not there has been grave abuse of discretion amounting to lack or excess of jurisdiction on the part of the RTC
in issuing an interlocutory order in cases falling within the exclusive appellate jurisdiction of the tax court. It, thus, follows that the
CTA, by constitutional mandate, is vested with jurisdiction to issue writs of certiorari in these cases.

Indeed, in order for any appellate court to effectively exercise its appellate jurisdiction, it must have the authority to issue, among
others, a writ of certiorari. In transferring exclusive jurisdiction over appealed tax cases to the CTA, it can reasonably be assumed that
the law intended to transfer also such power as is deemed necessary, if not indispensable, in aid of such appellate jurisdiction. There is
no perceivable reason why the transfer should only be considered as partial, not total.

If this Court were to sustain petitioners' contention that jurisdiction over their certiorari petition lies with the CA, this Court would be
confirming the exercise by two judicial bodies, the CA and the CTA, of jurisdiction over basically the same subject matter –
precisely the split-jurisdiction situation which is anathema to the orderly administration of justice. The Court cannot accept that
such was the legislative motive, especially considering that the law expressly confers on the CTA, the tribunal with the specialized
competence over tax and tariff matters, the role of judicial review over local tax cases without mention of any other court that may
exercise such power. Thus, the Court agrees with the ruling of the CA that since appellate jurisdiction over private respondents'
complaint for tax refund is vested in the CTA, it follows that a petition for certiorari seeking nullification of an interlocutory order
issued in the said case should, likewise, be filed with the same court. To rule otherwise would lead to an absurd situation where one
court decides an appeal in the main case while another court rules on an incident in the very same case.

Stated differently, it would be somewhat incongruent with the pronounced judicial abhorrence to split jurisdiction to conclude
that the intention of the law is to divide the authority over a local tax case filed with the RTC by giving to the CA or this Court
jurisdiction to issue a writ of certiorari against interlocutory orders of the RTC but giving to the CTA the jurisdiction over the
appeal from the decision of the trial court in the same case. It is more in consonance with logic and legal soundness to conclude
that the grant of appellate jurisdiction to the CTA over tax cases filed in and decided by the RTC carries with it the power to issue a
Remedial Law I| Page 17

writ of certiorari when necessary in aid of such appellate jurisdiction. The supervisory power or jurisdiction of the CTA to issue a writ
of certiorari in aid of its appellate jurisdiction should co-exist with, and be a complement to, its appellate jurisdiction to review, by
appeal, the final orders and decisions of the RTC, in order to have complete supervision over the acts of the latter.
Remedial Law I| Page 18

St. Mary Crusade Foundation vs. Riel


G.R. No. 176508, January 12, 2015

Facts:

The petitioner applied for the judicial reconstitution of Original Certificate of Title for the issuance of a new OCT alleging that his
copy had been burnt and lost in the fire that gutted the Quezon City Register of Deeds in the late 80s. Initially, the judge gave due
course to the petition but upon the opposition of the LRA and UP, the judge dismissed the complaint. Upon the denial of its MR, the
petitioner directly went to the Supreme Court and filed a petition for certiorari and mandamus contending that the respondent judge
acted with grave abuse of discretion in dismissing its complaint. In their comments, UP and OSG contend that petitioner failed to
observe the doctrine of hierarchy of courts, despite the Court of Appeals (CA) having concurrent jurisdiction with the Court over
special civil actions under Rule 65.

Issue:

Whether or not petitioner violated the doctrine of hierarchy of courts by directly filing a petition for certiorari before the SC?

Held:

Yes. The filing of the instant special civil action directly in this Court is in disregard of the doctrine of hierarchy of courts. Although
the Court has concurrent jurisdiction with the Court of Appeals in issuing the writ of certiorari, direct resort is allowed only when
there are special, extra-ordinary or compelling reasons that justify the same. The Court enforces the observance of the hierarchy of
courts in order to free itself from unnecessary, frivolous and impertinent cases and thus afford time for it to deal with the more
fundamental and more essential tasks that the Constitution has assigned to it. There being no special, important or compelling reason,
the petitioner thereby violated the observance of the hierarchy of courts, warranting the dismissal of the petition for certiorari.
Remedial Law I| Page 19

Duncano vs. Sandiganbayan


G.R. No. 191894, July 15, 2015.

Facts:

Danico Duncano, petitioner, is at the time material to the case, the Regional Director of the Bureau of Internal Revenue (BIR) with
Salary Grade 26. The Office of the Special Prosecutor (Office of the Ombudsman) filed a criminal case against him before the
Sandiganbayan for violation of Section 8, in relation to Section 11 of R.A. No. 6713. He filed a Motion to Dismiss contending that the
Sandiganbayan has no jurisdiction to try and hear the case because he is an official of the executive branch occupying the position of a
Regional Director but with a compensation that is classified as below Salary Grade 27. The Sandiganbayan denied the Motion to
Dismiss ruling that the position of Regional Director is one of those exceptions where the Sandiganbayan has jurisdiction even if such
position is not Salary Grade 27. It was opined that Section 4(A)(1) of R.A No. 8249 unequivocally provides that respondent court has
jurisdiction over officials of the executive branch of the government occupying the position of regional director and higher, otherwise
classified as Salary Grade 27 and higher, of R.A. No. 6758, including those officials who are expressly enumerated in subparagraphs
(a) to (g).

Issue:

Whether or not the case against petitioner who is a Regional Director with SG 26 falls within the jurisdiction of the Sandiganbayan?

Held:

No.

Those that fall within the original jurisdiction of the Sandiganbayan are: (1) officials of the executive branch with Salary Grade 27 or
higher, and (2) officials specifically enumerated in Section 4(A)(1)(a) to (g), regardless of their salary grades.31 While the first part of
Section 4(A) covers only officials of the executive branch with Salary Grade 27 and higher, its second part specifically includes other
executive officials whose positions may not be of Salary Grade 27 and higher but who are by express provision of law placed under
the jurisdiction of the Sandiganbayan.

Petitioner is not an executive official with Salary Grade 27 or higher. Neither does he hold any position particularly enumerated in
Section 4(A)(1)(a) to (g).

In the same way, a certification issued by the OIC-Assistant Chief, Personnel Division of the BIR shows that, although petitioner is a
Regional Director of the BIR, his position is classified as Director II with Salary Grade 26.
Remedial Law I| Page 20

CE Casecnan Water and Energy Co. Inc. vs. Province of Nueva Ecija
G.R. No. 196278, June 17, 2015.

Facts:

In 1995, petitioner and the National Irrigation Administration (NIA) entered into a build-operate-transfer (BOT) contract relative to
the construction and development of the Casecnan Multi-Purpose Irrigation and Power Project in Pantabangan, Nueva Ecija and
Alfonso Castaneda, Nueva Vizcaya. The Casecnan Project is a combined irrigation and hydroelectric power generation facility using
the Pantabangan Dam in Nueva Ecija. In 2005, petitioner received from the Office of the Provincial Assessor a Notice of Assessment
of Real Property which indicates that for the years 2002 to 2005, its RPT due was P248,676,349.60. Petitioner assailed the assessment
with the Nueva Ecija Local Board of Assessment Appeals (Nueva Ecija LBAA) which dismissed it on January 26, 2006. Undeterred,
petitioner filed a Notice of Appeal with the Nueva Ecija Central Board of Assessment Appeals (Nueva Ecija CBAA). During the
pendency thereof, respondents collected from petitioner the RPT due under the said assessment as well as those pertaining to the years
2006 up to the second quarter of 2008, totalling P363,703,606.88. Petitioner paid the assessed RPT under protest; it also initiated
proceedings questioning the validity of the collection with respect to the years 2006 up to the second quarter of 2008. Petitioner
received another notice that it has RPT in arrears. Again, petitioner questioned this assessment through an appeal before the NE
LBAA. While the same was pending, petitioner received a letter from the respondent demanding payment. Hence, petitioner filed with
the RTC for injunction and damages with application for temporary restraining order (TRO) and preliminary injunction praying to
restrain the collection of the 2008 RPT Reassessment. RTC denied petitioner’s application for a 72-hour TRO. Subsequently, the RTC
granted the 20-day TRO. But later on, it denied petitioner’s application for a writ of preliminary injunction. Petitioner then filed a
Petition for Certiorari before the CA. The CA dismissed the petition observing that the Petition for Certiorari before it was actually an
offshoot of the 2008 RPT Reassessment. And since in resolving the issue of whether the RTC committed grave abuse of discretion in
denying petitioner’s application for a writ of preliminary injunction, the issue of the validity of the assessment and the collection of the
RPT against petitioner must also be resolved, thus jurisdiction over the case lies within the Court of Tax Appeals (CTA). Petitioner
maintains that its petition relates to an ordinary civil action for injunction and not to a local tax case.

Issue:

Whether or not the CTA has jurisdiction over an order of the RTC denying an application for writ of injunction (which seeks to enjoin
the assessment and collection of RPT)?

Held:

Yes.

Jurisdiction over the subject matter is required for a court to act on any controversy. It is conferred by law and not by the consent or
waiver upon a court. As such, if a court lacks jurisdiction over an action, it cannot decide the case on the merits and must dismiss it.

With respect to the CTA, its jurisdiction was expanded and its rank elevated to that of a collegiate court with special jurisdiction by
virtue of Republic Act No. 9282. This expanded jurisdiction of the CTA includes its exclusive appellate jurisdiction to review by
appeal the decisions, orders or resolutions of the RTC in local tax cases originally decided or resolved by the RTC in the exercise of
its original or appellate jurisdiction. In the recent case of City of Manila v. Grecia-Cuerdo, the Court ruled that the CTA likewise has
the jurisdiction to issue writs of certiorari or to determine whether there has been grave abuse of discretion amounting to lack or
excess of jurisdiction on the part of the RTC in issuing an interlocutory order in cases falling within the CTA’s exclusive appellate
jurisdiction.

In praying to restrain the collection of RPT, petitioner also implicitly questions the propriety of the assessment of such RPT. This is
because in ruling as to whether to restrain the collection, the RTC must first necessarily rule on the propriety of the assessment. In
other words, in filing an action for injunction to restrain collection, petitioner was in effect also challenging the validity of the RPT
assessment. As aptly discussed by the CA:

The original action filed with the RTC is one for Injunction, with an application for Temporary Restraining Order and a Writ of
Preliminary Injunction to enjoin the province of Nueva Ecija from further collecting the alleged real property tax liability assessed
against it. Simply because the action is an application for injunctive relief does not necessarily mean that it may no longer be
considered as a local tax case. The subject matter and the issues, not the name or designation of the remedy, should control. While an
ancillary action for injunction may not be a main case, the court [still has] to determine, even in a preliminary matter, the applicable
tax laws, rules and jurisprudence.
Remedial Law I| Page 21

Ferrer vs. Bautista


G.R. No. 210551. June 30, 2015

Facts:

The Quezon City Council enacted Ordinance No. SP-2095, S-2011 or the Socialized Housing Tax of Quezon City, which impose one-
half percent (0.5%) tax on the assessed value of land in excess of One Hundred Thousand Pesos (Php100,000.00). They also enacted
Ordinance No. SP-2235, S-2013 which impose garbage fees on residential properties in Quezon City. The petitioner then filed a
Petition for Certiorari under Rule 65 before the Supreme Court seeking to declare unconstitutional and illegal Ordinance Nos. SP-
2095, S-2011 and SP-2235, S-2013 on the Socialized Housing Tax and Garbage Fee, respectively, which are being imposed by the
respondents.

Issue:

Whether or not the Supreme Court can take cognizance of the case?

Held:

Yes. Petitioner has adduced special and important reasons as to why direct recourse to the SC should be allowed. Aside from
presenting a novel question of law, this case calls for immediate resolution since the challenged ordinances adversely affect the
property interests of all paying constituents of Quezon City. As well, this petition serves as a test case for the guidance of other local
government units (LGUs). Indeed, the petition at bar is of transcendental importance warranting a relaxation of the doctrine of
hierarchy of courts.

Citing Senator Jaworski v. Phil. Amusement & Gaming Corp., the Court said:

Granting arguendo that the present action cannot be properly treated as a petition for prohibition, the transcendental importance of the
issues involved in this case warrants that we set aside the technical defects and take primary jurisdiction over the petition at bar. x x x
This is in accordance with the well-entrenched principle that rules of procedure are not inflexible tools designed to hinder or delay, but
to facilitate and promote the administration of justice. Their strict and rigid application, which would result in technicalities that tend
to frustrate, rather than promote substantial justice, must always be eschewed.

Note: A Petition for Certiorari is not the proper remedy because the respondents are not exercising quasi-judicial functions. And it
actually partakes of a declaratory relief over which the Supreme Court has only appellate, not original, jurisdiction. Despite these, a
petition for declaratory relief may be treated as one for prohibition or mandamus, over which the SC exercises original jurisdiction, in
cases with far-reaching implications or one which raises transcendental issues or questions that need to be resolved for the public
good. The judicial policy is that this Court will entertain direct resort to it when the redress sought cannot be obtained in the proper
courts or when exceptional and compelling circumstances warrant availment of a remedy within and calling for the exercise of Our
primary jurisdiction.
Remedial Law I| Page 22

Lomondot vs. Balindong


G.R. No. 192463. July 13, 2015.

Facts:

Petitioners Omaira and Saripa Lomondot filed with the Shari’a District Court (SDC), Marawi City, a complaint for recovery of
possession and damages with prayer for mandatory injunction and temporary restraining order against respondents. They alleged that
they are the owners by succession of the subject parcel of land. The SDC ruled in favor of the petitioners and ordered the respondents
to vacate. The SDC decision became final and executory. Petitioner then filed a motion for the issuance of a writ of execution. The
SDC granted the motion but it subsequently issued an order that the resolution for the motion for the issuance of a writ of demolition
should be held in abeyance. Later on, it denied the motion for demolition stating that the respondents were claiming that they no
longer encroach the plaintiff’s land. Upon the denial of their MR, petitioners filed a petition for certiorari with CA-Cagayan de Oro
City. The CA dismissed the petition on the ground of lack of jurisdiction. It opined that it cannot take cognizance of the instant case
because it emanates from the Shari’a Courts, which is not among those courts, bodies or tribunals enumerated under Chapter 1,
Section 9 of BP 129, as amended, over which it can exercise appellate jurisdiction.

Issue:

Whether or not the Court of Appeals has jurisdiction over decisions of SDC?

Held:

Yes. The Shari’a Appellate Court has yet to be organized with the appointment of a Presiding Justice and two Associate Justices. Until
such time that the Shari’a Appellate Court shall have been organized, however, appeals or petitions from final orders or decisions of
the SDC filed with the CA shall be referred to a Special Division to be organized in any of the CA stations preferably composed of
Muslim CA Justices (Tomawis v. Balindong).
Remedial Law I| Page 23

Municipality of Tangkal vs. Balindong


G.R. No. 193340, January 11, 2017

Facts:

The private respondents, heirs of the late Macalabo Alompo, filed a Complaint with the Shari'a District Court of Marawi City (Shari'a
District Court) against the petitioner, Municipality of Tangkal, for recovery of possession and ownership of a parcel of land. They
alleged that Macalabo was the owner of the land, and that in 1962, he entered into an agreement with the Municipality of Tangkal
allowing the latter to "borrow" the land to pave the way for the construction of the municipal hall and a health center building. The
agreement allegedly imposed a condition upon the Municipality of Tangkal to pay the value of the land within 35 years, or until 1997;
otherwise, ownership of the land would revert to Macalabo. Private respondents claimed that the Municipality of Tangkal neither paid
the value of the land within the agreed period nor returned the land to its owner. Thus, they prayed that the land be returned to them as
successors-in-interest of Macalabo. The Municipality of Tangkal filed a Motion to Dismiss on the ground of improper venue and lack
of jurisdiction. It argued that since it has no religious affiliation and represents no cultural or ethnic tribe, it cannot be considered as a
Muslim under the Code of Muslim Personal Laws. Moreover, since the complaint for recovery of land is a real action, it should have
been filed in the appropriate Regional Trial Court of Lanao del Norte. The Shari'a District Court denied the Municipality of Tangkal's
motion to dismiss. It held that since the mayor of Tangkal, Abdulazis A.M. Batingolo, is a Muslim, the case "is an action involving
Muslims, hence, the court has original jurisdiction concurrently with that of regular/civil courts." The Municipality of Tangkal
elevated the case to the SC via a petition for certiorari, prohibition and mandamus.

Issue:

Whether or not the Shari’a District Court has jurisdiction?

Held:

No. The matters over which Shari'a district courts have Jurisdiction are enumerated in the Code of Muslim Personal Laws, specifically
in Article 143. Consistent with the purpose of the law to provide for an effective administration and enforcement of Muslim personal
laws among Muslims, it has a catchall provision granting Shari'a district courts original jurisdiction over personal and real actions
except those for forcible entry and unlawful detainer. The Shari'a district courts' jurisdiction over these matters is concurrent with
regular civil courts, i.e., municipal trial courts and regional trial courts. There is, however, a limit to the general jurisdiction of Shari'a
district courts over matters ordinarily cognizable by regular courts: such jurisdiction may only be invoked if both parties are Muslims.
If one party is not a Muslim, the action must be filed before the regular courts.

In determining whether the Shari'a District Court has jurisdiction over the case, the threshold question is whether both parties are
Muslims. It is clear from the title and the averments in the complaint that Mayor Batingolo was impleaded only in a representative
capacity, as chief executive of the local government of Tangkal. When an action is defended by a representative, that representative is
not-and neither does he become-a real party in interest. The person represented is deemed the real party in interest; the representative
remains to be a third party to the action. That Mayor Batingolo is a Muslim is therefore irrelevant for purposes of complying with the
jurisdictional requirement under Article 143(2)(b) that both parties be Muslims. To satisfy the requirement, it is the real party
defendant, the Municipality of Tangkal, who must be a Muslim. Such a proposition, however, is a legal impossibility. A juridical
person exercises no religion. As a government instrumentality, the Municipality of Tangkal can only act for secular purposes and in
ways that have primarily secular effects35-consistent with the non-establishment clause. Hence, even if it is assumed that juridical
persons are capable of practicing religion, the Municipality of Tangkal is constitutionally proscribed from adopting, much less
exercising, any religion, including Islam.
Remedial Law I| Page 24

Presidential Commission on Good Government (PCGG) vs. Dumayas


G.R. No. 209447. August 11, 2015.

Facts:

The Sandiganbayan rendered partial summary judgments in Civil Case No. 0033 revolved around the provisional takeover by the
PCGG of COCOFED, Cocomark, and Coconut Investment Company and their assets and the sequestration of shares of stock in UCPB
CIIF corporations (CIIF oil mills and the 14 CIIF holding companies), or CIIF companies, so-called for having been either organized,
acquired and/or funded as UCPB subsidiaries with the use of the CIIF levy. SB’s partial judgments were affirmed with modification
by the Supreme Court. In 2012, a petition for declaratory relief was filed by respondent UCPB in the RTC of Makati (Civil Case No.
12-1251) against the six CIIF oil mills and 14 holding companies (CIIF companies), PCGG and other corporations “similarly
situated.” A similar petition was also filed by respondent United Coconut Planters Life Assurance Corporation (COCOLIFE) against
the same defendants (Civil Case No. 12- 1252). UCPB alleged that the capital or equity used in establishing the CIIF companies was
not exclusively sourced from the coconut levy funds. Like UCPB, COCOLIFE asserted that the CIIF OMG and 14 CIIF holding
companies are not wholly owned by the Government. PCGG filed a Motion to Dismiss in both cases on the ground of lack of
jurisdiction over the subject matter of the case. Both MTDs were denied.

Issue:

Whether or not the RTC has jurisdiction over suits involving the sequestered coco levy assets and coco levy funds?

Held:

No. Under Section 4(C) of P.D. No. 1606, as amended by R.A. No. 7975 and R.A. No. 8249, the jurisdiction of the Sandiganbayan
included suits for recovery of ill-gotten wealth and related cases.

Under Section 2 of the President’s Executive Order No. 14 issued on May 7, 1986, all cases of the Commission regarding “the Funds,
Moneys, Assets, and Properties Illegally Acquired or Misappropriated by Former President Ferdinand Marcos, Mrs. Imelda
Romualdez Marcos, their Close Relatives, Subordinates, Business Associates, Dummies, Agents, or Nominees” whether civil or
criminal, are lodged within the “exclusive and original jurisdiction of the Sandiganbayan” and all incidents arising from, incidental to,
or related to, such cases necessarily fall likewise under the Sandiganbayan’s exclusive and original jurisdiction, subject to review on
certiorari exclusively by the Supreme Court.

Respondents’ petitions for declaratory relief filed in the RTC asserted their claim of ownership over the sequestered CIIF companies
and indirectly the CIIF SMC Block of Shares. Undeniably, these are related to the ill-gotten wealth cases (Civil Case Nos. 0033-A and
0033-F) involving the issue of ownership of the aforesaid sequestered companies and shares of stock, which have been tried and
decided by the Sandiganbayan, and the decision had been appealed to and finally disposed of by this Court in G.R. Nos. 177857-5831
(COCOFED and Lobregat, et al.’s ownership claim over the CIIF companies and CIIF SMC Block of Shares) and G.R. No. 18070532
(Eduardo M. Cojuangco, Jr.’s claim over UCPB shares under an Agreement with PCA).
Remedial Law I| Page 25

Regulus Development, Inc. vs. Dela Cruz


G.R. No. 198172. January 25, 2016.

Facts:

Dela Cruz leased two units of petitioner’s apartment. The contract of lease provides for a period of one-month subject to automatic
renewals. Petitioner sent respondent a letter to terminate the lease of the two subject units. Due to the respondent’s refusal to vacate
the units, the petitioner filed a complaint for ejectment before the Metropolitan Trial Court (MTC) of Pasay City. The MTC resolved
the case in the petitioner’s favor and ordered the respondent to vacate the premises, and pay the rentals due. The respondent appealed
to the Regional Trial Court (RTC). Pending appeal, the respondent consigned the monthly rentals to the RTC due to the petitioner’s
refusal to receive the rentals. The RTC affirmed the decision of the MTC. The CA reversed the two lower courts and dismissed the
ejectment case. This decision became final and executory. Petitioner then filed a motion praying for the withdrawal of the rentals
consigned with the RTC. The RTC granted the motion. The CA also upheld the RTC’s order and held that they were issued pursuant
to its equity jurisdiction. The CA ruling became final and executory as the SC denied respondent’s petition for insufficiency in form.
The petitioner then returned to the RTC for the issuance of the lease payments deposited by respondent. However, the withdrawn
deposits were insufficient to cover the rentals due from 2001 to 2004. Hence, the petitioner filed a manifestation and motion praying
that the RTC to levy upon the respondent’s property to satisfy the judgment credit. The RTC granted the motion. Respondent went to
the CA. The CA initially denied the petition on MR, the CA reversed and set aside the orders of the RTC directing the levy of the
respondent’s property. The CA held that while the approval of the petitioner’s motion to withdraw the consigned rentals and the
posted supersedeas bond was within the RTC’s jurisdiction, the RTC had no jurisdiction to levy on the respondent’s real property. The
CA explained that the approval of the levy on the respondent’s real property could not be considered as a case pending appeal,
because the decision of the MTC had already become final and executory. As such, the matter of execution of the judgment lies with
the MTC where the complaint for ejectment was originally filed and presented.

Issue:

Whether or not the RTC had jurisdiction to levy on the respondent’s real property?

Held:

Yes. The appellate jurisdiction of courts is conferred by law. The appellate court acquires jurisdiction over the subject matter and
parties when an appeal is perfected. On the other hand, equity jurisdiction aims to provide complete justice in cases where a court
of law is unable to adapt its judgments to the special circumstances of a case because of a resulting legal inflexibility when the
law is applied to a given situation. The purpose of the exercise of equity jurisdiction, among others, is to prevent unjust
enrichment and to ensure restitution.

The RTC orders which allowed the withdrawal of the deposited funds for the use and occupation of the subject units were issued
pursuant to the RTC’s equity jurisdiction. The RTC’s equity jurisdiction is separate and distinct from its appellate jurisdiction on the
ejectment case. The RTC could not have issued its orders in the exercise of its appellate jurisdiction since there was nothing more to
execute on the dismissed ejectment case. As the RTC orders explained, the dismissal of the ejectment case effectively and completely
blotted out and cancelled the complaint. Hence, the RTC orders were clearly issued in the exercise of the RTC’s equity jurisdiction,
not on the basis of its appellate jurisdiction.

The levy of the respondent’s property was made pursuant to the RTC orders issued in the exercise of its equity jurisdiction,
independent of the ejectment case originally filed with the MTC.

An examination of the RTC order dated June 30, 2008, directing the levy of the respondent’s real property shows that it was based on
the RTC order dated July 25, 2003. The levy of the respondent’s property was issued to satisfy the amounts due under the lease
contracts, and not as a result of the decision in the ejectment case. The CA erred when it concluded that the RTC exercised its
appellate jurisdiction in the ejectment case when it directed the levy of the respondent’s property. Execution shall be applied for in the
court of origin, in accordance with Section 1,45 Rule 39 of the Rules of Court. The court of origin with respect to the assailed RTC
orders is the court which issued these orders. The RTC is the court with jurisdiction to order the execution of the issued RTC orders.
Remedial Law I| Page 26

Rule 1: Commencement of Actions

Alday vs. FGU Insurance Corporation


G.R. No. 138822, January 23, 2001

Facts:

FGU Insurance Insurance Corp. filed a complaint with the RTC of Makati against Evangeline Alday alleging that the latter owed it
owed it P114,650.76, representing unliquidated cash advances, unremitted costs of premiums and other charges incurred by Alday in
the course of her work as an insurance agent for FGU. Alday filed her Answer and by way of Counterclaim asserted her right for the
payment of P104,893.45, representing direct commissions, profit commissions and contingent bonuses earned and for accumulated
premium reserves amounting to P500,000.00. In addition, Alday prayed for attorney’s fees, litigation expenses, moral damages and
exemplary damages for the allegedly unfounded action filed by FGU. FGU then filed a Motion to Dismiss Alday’s counterclaim
contending that the trial court never acquired jurisdiction over the same because of the non-payment of docket fees. In response, Alday
asked the trial court to declare her counterclaim as exempt from payment of docket fees since it is compulsory and that FGU be
declared in default for having failed to answer such counterclaim. RTC granted the Motion to Dismiss because it found Alday’s
counterclaim to be merely permissive in nature and held that her failure to pay docket fees prevented the court from acquiring
jurisdiction over the same. The Court of Appeals affirmed the RTC’s decision.

Issue:

Whether or not Alday’s counterclaims are merely permissive and accordingly, the trial court did not acquire jurisdiction over the same
for non-payment of docket fees?

Held:

Alday’s counterclaim for commissions, bonuses, and accumulated premium reserves is merely permissive. The evidence
required to prove her claims differs from that needed to establish FGU’s demands for the recovery of cash accountabilities from
Alday, such as cash advances and costs of premiums. The recovery of FGU’s claims is not contingent or dependent upon establishing
Alday’s counterclaim, such that conducting separate trials will not result in the substantial duplication of the time and effort of the
court and the parties. One would search the records in vain for a logical connection between the parties’ claims. This conclusion is
further reinforced by Aldays’s own admissions since she declared in her answer that respondent’s cause of action, unlike her own, was
not based upon a Special Agent’s Contract.

However, Alday’s claims for damages, allegedly suffered as a result of the filing of its complaint are compulsory.

There is no need for petitioner to pay docket fees for her respondent of its complaint, are compulsory. On the other hand, in
order for the trial court to acquire jurisdiction over her permissive counterclaim, Alday is bound to pay the prescribed docket
fees.
In this case, instead of dismissing the counterclaim, trial court should have given Alday a reasonable time, but in no case beyond the
applicable prescriptive or reglementary period, to pay the filing fees for her permissive counterclaim.

Discussion:

A compulsory counterclaim is one which, being cognizable by the regular courts of justice, arises out of or is connected with the
transaction or occurrence constituting the subject matter of the opposing party’s claim and does not require for its adjudication the
presence of third parties of whom the court cannot acquire jurisdiction.

The criteria or tests that may be used in determining whether a counterclaim is compulsory or permissive, summarized as follows:
1. Are the issues of fact and law raised by the claim and counterclaim largely the same?
2. Would res judicata bar a subsequent suit on defendant’s claim absent the compulsory counterclaim rule?
3. Will substantially the same evidence support or refute plaintiffs claim as well as defendant’s counterclaim?
4. Is there any logical relation between the claim and the counterclaim?

Another test is the “compelling test of compulsoriness” which requires “a logical relationship between the claim and counterclaim,
that is, where conducting separate trials of the respective claims of the parties would entail a substantial duplication of effort and time
by the parties and the court.”

Rule on the payment of filing fees:


Remedial Law I| Page 27

1. It is not simply the filing of the complaint or appropriate initiatory pleading, but the payment of the prescribed docket fee,
that vests a trial court with jurisdiction over the subject-matter or nature of the action. Where the filing of the initiatory
pleading is not accompanied by payment of the docket fee, the court may allow payment of the fee within a reasonable time
but in no case beyond the applicable prescriptive or reglementary period.

2. The same rule applies to permissive counterclaims, third- party claims and similar pleadings, which shall not be considered
filed until and unless the filing fee prescribed therefor is paid. The court may allow payment of said fee within a reasonable
time but also in no case beyond its applicable prescriptive or reglementary period.

3. Where the trial court acquires jurisdiction over a claim by the filing of the appropriate pleading and payment of the
prescribed filing fee but, subsequently, the judgment awards a claim not specified in the pleading, or if specified the same has
been left for determination by the court, the additional filing fee therefor shall constitute a lien on the judgment. It shall be the
responsibility of the Clerk of Court or his duly authorized deputy to enforce said lien and assess and collect the additional fee.
Remedial Law I| Page 28

Korea Technologies Co., Ltd. vs. Lerma


G.R. No. 143581. January 7, 2008

NOTE: The doctrine in this case is no longer controlling.

Facts:

Korea Technologies Co., Ltd. (KOGIES) and Pacific General Steel Manufacturing Corp. (PGSMC) executed a Contract in the
Philippines, whereby KOGIES would set up an LPG Cylinder Manufacturing Plant in Carmona, Cavite. Subsequently, the obligations
of the parties regarding installment of the plant were complied with. However, after the installation of the plant, the initial operation
could not be conducted as PGSMC encountered financial difficulties affecting the supply of materials, thus forcing the parties to agree
that KOGIES would be deemed to have completely complied with the terms and conditions of their contract. For the remaining
balance for the installation and initial operation of the plant, PGSMC issued two postdated checks. When KOGIES deposited the
checks, these were dishonored. Thus, KOGIES sent a demand letter to PGSMC. Later on, PGSMC informed KOGIES that it would
cancel their contract on the ground that KOGIES had altered the quantity and lowered the quality of the machineries and equipment it
delivered to PGSMC, among, others. PGSMC filed before the OCP an Affidavit-Complaint for Estafa against the President of
KOGIES. KOGIES sent a letter to PGSMC that it could not unilaterally rescind the contract and that their dispute should be settled by
arbirtration as agreed upon in their contract. Later, KOGIES instituted an Application for Arbitration before the Korean Commercial
Arbitration Board (KCAB) in Seoul, Korea pursuant to their contract. KOGIES also filed a Complaint for Specific Performance,
against PGSMC before the Regional Trial Court (RTC). PGSMC filed its Answer with Compulsory Counterclaim asserting that it had
the full right to dismantle and transfer the machineries and equipment because it had paid for them in full as stipulated in the contract;
that KOGIES was not entitled to the PhP 9,000,000 covered by the checks for failing to completely install and make the plant
operational; and that KOGIES was liable for damages amounting to PhP 4,500,000 for altering the quantity and lowering the quality
of the machineries and equipment.

The RTC ruled in favor of PGSMC. The Court of Appeals affirmed the trial court and held that the counterclaim of PGSMC is
compulsory in nature which did not require the payment of docket fees.

Issue:

Whether or not the counterclaim of PGSMC is compulsory and is therefore exempt from the payment of the docket fees?

Held:

No. Effective August 16, 2004 under Sec. 7, Rule 141, as amended by A.M. No. 04-2-04-SC, docket fees are now required to be paid
in compulsory counterclaim or cross-claims.
Remedial Law I| Page 29

Mercado vs. Court of Appeals


G.R. No. 169576, October 17, 2008

Facts:

Mercado had been distributing San Miguel Corporation’s (SMCs) beer products in Quiapo, Manila since 1967. SMC extended to him
a P7.5 million credit line allowing him to withdraw goods on credit. To secure his purchases, Mercado assigned three China Banking
Corporation (CBC) certificates of deposit to SMC and executed a continuing hold-out agreement. He also submitted three surety
bonds from Eastern Assurance and Surety Corporation (EASCO). Later, SMC notified CBC that Mercado failed to pay for the items
he withdrew on credit. Consequently, citing the continuing hold-out agreement, it asked CBC to release the proceeds of the assigned
certificates of deposit. CBC approved SMBs request and informed Mercado. Mercado then filed an action to annul the continuing
hold-out agreement and deed of assignment in the RTC of Manila. Thereafter, SMC filed its answer with counterclaim against
Mercado. It contended that Mercado delivered only two CBC certificates of deposit.  SMC also filed a third-party complaint with
EASCO. However, Mercado filed an urgent manifestation and motion seeking the dismissal of the complaint. He claimed that he was
no longer interested in annulling the continuing hold-out agreement and deed of assignment. The RTC, however, denied the motion. 

During trial, Mercado acknowledged the accuracy of SMCs computation of his outstanding liability as of August 1991. Thus, the RTC
dismissed the complaint and ordered Mercado and EASCO to jointly and severally pay SMC. Aggrieved, Mercado and EASCO
appealed to the Court of Appeals (CA) insisting that Mercado did not default in the payment of his obligations to SMC. On appeal, the
CA affirmed the RTC decision in toto. EASCO filed a petition for review on certiorari before the Supreme Court but eventually
agreed to settle its liability with SMC. The petition was terminated. Meanwhile, Mercado passed away and was substituted by his
heirs. The petitioners subsequently filed a petition asserting that the decision of the CA was void on the ground that the SMC’s
counterclaim was permissive in nature and that inasmuch as SMC did not pay docket fees, the RTC never acquired jurisdiction over
the counterclaim.

Issue:

Whether or not SMC’s counterclaim is permissive in nature which requires payment of docket fees before the court could acquire
jurisdiction of the claim?

Held:

No. SMC’s counterclaim is compulsory in nature. A counterclaim that (1) arises out of (or is necessarily connected with) the
transaction or occurrence that is the subject matter of the opposing party’s claim; (2) falls within the jurisdiction of the court and (3)
does not require for its adjudication the presence of third parties over whom the court cannot acquire jurisdiction, is compulsory.

When Mercado sought to annul the continuing hold-out agreement and deed of assignment (which he executed as security for his
credit purchases, he in effect sought to be freed from them. While he admitted having outstanding obligations, he nevertheless asserted
that those were not covered by the assailed accessory contracts. For its part, aside from invoking the validity of the said agreements,
SMC therefore sought to collect the payment for the value of goods Mercado purchased on credit.  Thus, Mercado’s complaint and
SMCs counterclaim both touched the issues of whether the continuing hold-out agreement and deed of assignment were valid and
whether Mercado had outstanding liabilities to SMC. The same evidence would essentially support or refute Mercado’s claim and
SMCs counterclaim. Based on the foregoing, had these issues been tried separately, the efforts of the RTC and the parties would have
had to be duplicated. Clearly, SMCs counterclaim, being logically related to Mercado’s claim, was compulsory in nature.
Consequently, the payment of docket fees was not necessary for the RTC to acquire jurisdiction over the subject matter.
Remedial Law I| Page 30

Proton Pilipinas vs. Banque Nationale de Paris


G.R. No. 151242, June 15, 2005

Facts:

Proton Pilipinas Corporation (Proton) availed of the credit facilities of Banque Nationale de Paris (BNP). To guarantee the payment of
its obligation, its co-petitioners (Automotive, Asea and Autocorp) executed a corporate guarantee to the extent of US$2,000,000.00.
BNP and Proton subsequently entered into three trust receipt agreements, under which Proton would receive imported passenger
motor vehicles and hold them in trust for BNP. Proton would be free to sell the vehicles subject to the condition that it would deliver
the proceeds of the sale to BNP, to be applied to its obligations to it. In case the vehicles are not sold, Proton would return them to
BNP, together with all the accompanying documents of title. Proton failed to deliver the proceeds of the sale and return the unsold
motor vehicles. BNP then demanded from Automotive, Asea and Autocorp the payment of the amount of US$1,544,984.40
representing Proton's total outstanding obligations. These guarantors refused to pay. So, BNP filed before the Makati RTC a complaint
against petitioners praying that they be ordered to pay US$1,544,984.40 plus accrued interest and other related charges subsequent to
August 15, 1998 until fully paid.

The Makati RTC Clerk of Court assessed the docket fees which BNP paid at ₱352,116.30. The petitioners filed a Motion to Dismiss
on the ground that BNP failed to pay the correct docket fees and thus prevent the trial court from acquiring jurisdiction over the case.

The RTC denied petitioners' Motion to Dismiss. It ruled that the docket fees were properly paid since it is the Office of the Clerk of
Court that computes the correct docket fees, and it is their duty to assess the docket fees correctly, which they did. According to the
RTC, even granting arguendo that the docket fees were not properly paid, the court cannot just dismiss the case. The Court has not yet
ordered (and it will not in this case) to pay the correct docket fees.

The CA affirmed the RTC, ruling that Section 7(a) of Rule 141 of the Rules of Court excludes interest accruing from the principal
amount being claimed in the pleading in the computation of the prescribed filing fees.

Citing Administrative Circular No. 11-94, petitioners argue that BNP failed to pay the correct docket fees as the said circular provides
that in the assessment thereof, interest claimed should be included. There being an underpayment of the docket fees, the trial court did
not acquire jurisdiction over the case.

Issues:

(1) Whether or not BNP paid the correct docket fees?

(2) Whether or not the complaint filed by BNP should be dismissed on the ground that the trial court did not acquire jurisdiction over
the case since it failed to pay the correct docket fees?

Held:

(1) No. When the complaint in this case was filed in 1998, Rule 141 had been amended by Administrative Circular No. 11-94 which
provides for the inclusion of interest, damages of whatever kind, attorney's fees, litigation expenses, and costs in the total amount
claimed as basis for the computation of docket fees. The clerk of court should thus have assessed the filing fee by taking into
consideration "the total sum claimed, inclusive of interest, damages of whatever kind, attorney's fees, litigation expenses, and costs, or
the stated value of the property in litigation. (Additionally, the petitioners have adequately proven with documentary evidence that the
exchange rate when the complaint was filed was USD 1 = Php 43.21, and not Php 43.00 as determined by the Clerk of Court. This
overturned the disputable presumption of regularity of the Clerk’s application of the exchange rate.)
Remedial Law I| Page 31

(2) No. True, in Manchester Development Corporation v. Court of Appeals, this Court held that the court acquires jurisdiction over
any case only upon the payment of the prescribed docket fees, hence, it concluded that the trial court did not acquire jurisdiction over
the case. It bears emphasis, however, that the ruling in Manchester was clarified in Sun Insurance Office, Ltd. (SIOL) v. Asuncion
when this Court held that in the former, there was clearly an effort to defraud the government in avoiding to pay the correct docket
fees, whereas in the latter the plaintiff demonstrated his willingness to abide by paying the additional fees as required.

The principle in Manchester could very well be applied in the present case. However, in Manchester, petitioner did not pay any
additional docket fee until the case was decided by this Court. Thus, in Manchester, due to the fraud committed on the government,
this Court held that the court a quo did not acquire jurisdiction over the case and that the amended complaint could not have been
admitted inasmuch as the original complaint was null and void.

In the present case, a more liberal interpretation of the rules is called for considering that, unlike Manchester, private
respondent demonstrated his willingness to abide by the rules by paying the additional docket fees as required.

Discussion:

1. It is not simply the filing of the complaint or appropriate initiatory pleading, but the payment of the prescribed docket fee,
that vests a trial court with jurisdiction over the subject-matter or nature of the action. Where the filing of the initiatory
pleading is not accompanied by payment of the docket fee, the court may allow payment of the fee within a reasonable time
but in no case beyond the applicable prescriptive or reglementary period.

2. The same rule applies to permissive counterclaims, third-party claims and similar pleadings, which shall not be considered
filed until and unless the filing fee prescribed therefor is paid. The court may also allow payment of said fee within a
reasonable time but also in no case beyond its applicable prescriptive or reglementary period.

3. Where the trial court acquires jurisdiction over a claim by the filing of the appropriate pleading and payment of the
prescribed filing fee but, subsequently, the judgment awards a claim not specified in the pleading, or if specified the same has
been left for determination by the court, the additional filing fee therefor shall constitute a lien on the judgment. It shall be the
responsibility of the Clerk of Court or his duly authorized deputy to enforce said lien and assess and collect the additional fee.
Remedial Law I| Page 32

Ruby Shelter Builders vs. Formaran


G.R. No. 175914, February 10, 2009

Facts:

Petitioner Ruby Shelter Builders obtained a loan of more than Php 95M from respondents Tan and Obiedo, secured by real estate
mortgages over five parcels of land. When petitioner was unable to pay the loan when it became due and demandable, Tan and
Obiedo, through a Memorandum of Agreement, granted petitioner a period within which to settle its indebtedness, and condoned the
interests, penalties and surcharges accruing thereon amounting to more than Php 74M. The Memorandum required, in turn, that
petitioner execute simultaneously, "by way of dacion en pago," Deeds of Absolute Sale in favor of Tan and Obiedo, covering the same
parcels of land subject of the mortgages. The Deeds of Absolute Sale state that petitioner sold to Tan and Obiedo the parcels of land
for the stated purchase prices. The Memorandum further gives the petitioner an opportunity to redeem any of the parcels of land and
provides for the nullification of the Deeds of Absolute Sale. Otherwise, Tan and Obiedo could already present the Deeds to the Office
of the Register of Deeds to acquire TCTs in their names.

Petitioner defaulted in its obligation, so Tan and Obiedo presented the Deeds of Absolute Sale before the Register of Deeds of Naga
City and were able to secure TCTs over the five parcels of land in their names.

Petitioner filed before the RTC a Complaint against Tan, Obiedo, and Atty. Reyes (the notary public who notarized the MOA) for
declaration of nullity of deeds of sales and damages, with prayer for the issuance of a writ of preliminary injunction and/or TRO.
Among others, petitioner alleged that the Deeds of Absolute Sale over the five parcels of land were executed merely as security for the
payment of its loan to respondents Tan and Obiedo; that the Deeds of Absolute Sale, executed in accordance with the Memorandum of
Agreement, constituted pactum commisorium and as such, were null and void; and that the acknowledgment in the Deeds of Absolute
Sale were falsified. Upon filing its Complaint with the RTC, petitioner paid the sum of ₱13,644.25 for docket and other legal fees, as
assessed by the Office of the Clerk of Court. The Clerk of Court initially considered the case as an action incapable of pecuniary
estimation and computed the docket and other legal fees due thereon according to Section 7(b)(1), Rule 141 of the Rules of Court.

On the other hand, Tan contended that the civil case involved real properties, the docket fees for which should be computed in
accordance with Section 7(a), not Section 7(b)(1), of Rule 141 of the Rules of Court, as amended by A.M. No. 04-2-04-SC. Since
petitioner did not pay the appropriate docket fees, the RTC did not acquire jurisdiction over the said case.

The RTC granted Tan’s Omnibus Motion. It was held that both petitioner and respondent Tan must pay the docket fees in accordance
with Section 7(a), Rule 141 of the Rules of Court, as amended. Particularly, petitioner was ordered to pay additional filing fee.

The RTC Clerk of Court computed the additional docket fees petitioner must pay at ₱720,392.60. Petitioner filed a Petition for
Certiorari with the CA. The CA rendered a decision denying the petition for certiorari. According to the CA, the objectives of the
petitioner in filing the complaint were to cancel the deeds of sale and ultimately, to recover possession of the same. It is therefore a
real action. Consequently, the additional docket fees that must be paid cannot be assessed in accordance with Section 7(b), but must
comply with Section 7(a).

Issue:

Whether or not the subject case is a real action and therefore the petitioner should be assessed docket fees based on Section 7(a), Rule
141 of the Rules of Court as amended?

Held:

Yes. A real action is one in which the plaintiff seeks the recovery of real property; or, as indicated in what is now Section 1, Rule 4 of
the Rules of Court, a real action is an action affecting title to or recovery of possession of real property. While it is true that petitioner
Remedial Law I| Page 33

does not directly seek the recovery of title or possession of the property in question, his action for annulment of sale and his claim for
damages are closely intertwined with the issue of ownership of the building which, under the law, is considered immovable property,
the recovery of which is petitioner's primary objective. The prevalent doctrine is that an action for the annulment or rescission of a sale
of real property does not operate to efface the fundamental and prime objective and nature of the case, which is to recover said real
property. It is a real action. A careful examination of respondent’s complaint reveals that it is a real action involving not only the
recovery of real properties, but likewise the cancellation of the titles thereto.

The docket fees under Section 7(a), Rule 141, in cases involving real property, depend on the fair market value of the same:
the higher the value of the real property, the higher the docket fees due. In contrast, Section 7(b)(1), Rule 141 imposes a fixed
or flat rate of docket fees on actions incapable of pecuniary estimation. Considering that respondent’s complaint is a real
action, the Rule requires that "the assessed value of the property, or if there is none, the estimated value thereof shall be
alleged by the claimant and shall be the basis in computing the fees."
Remedial Law I| Page 34

St. Louis University vs. Cobarrubias


G.R. No. 187104, August 3, 2010

Facts:

Evangeline Cobarrubias is an associate professor of the SLU’s College of Human Sciences. She is an active member of the Union of
Faculty and Employees of Saint Louis University (UFESLU). The relevant Collective Bargaining Agreements (CBAs) between SLU
and UFESLU contain the common provision on forced leave that teaching employees in college who fail the yearly evaluation shall be
on forced leave for one (1) regular semester during which period all benefits due them shall be suspended.

SLU placed Cobarrubias on forced leave for the first semester of S.Y. 2007-2008 when she failed the evaluation for S.Y. 2002-2003,
2005-2006, and 2006-2007. Cobarrubias sought recourse from the CBA’s grievance machinery. The parties failed to settle their
dispute, prompting Cobarrubias to file a case for illegal forced leave or illegal suspension with the National Conciliation and
Mediation Board of the DOLE-CAR, Baguio City. When circulation and mediation again failed, the parties submitted the issues
between them for voluntary arbitration before Voluntary Arbitrator (VA) Fariñas. Cobarrubias argued that the CA already resolved the
forced leave issue in a prior case between the parties, ruling that the forced leave for teachers who fail their evaluation for three (3)
times within a five-year period should be coterminous with the CBA in force during the same five-year period.

VA Fariñas dismissed the case. He found that the CA decision is not yet final because of the pending appeal with the SC. He also
noted that the CBA clearly authorized SLU to place its teaching employees on forced leave.

Cobarrubias filed with the CA a petition for review under Rule 43 of the Rules of Court, but failed to pay the required filing fees and
to attach to the petition copies of the material portions of the record. The CA dismissed the petition outright for Cobarrubias’
procedural lapses.

Cobarrubias then filed her motion for reconsideration, arguing that the ground cited is technical. She, nonetheless, attached to her
motion copies of the material portions of the record and the postal money orders for ₱4,230.00. She maintained that the ends of justice
and fair play are better served if the case is decided on its merits.

The CA reinstated the petition. It found that Cobarrubias substantially complied with the rules by paying the appeal fee in full and
attaching the proper documents in her motion for reconsideration. The CA brushed aside SLU’s insistence on the finality of the VA
decision and annulled it.

SLU filed a Motion for Reconsideration but it was denied by the CA. Hence, the present petition for review on certiorari. SLU argues
that the CA should not have reinstated the appeal since Cobarrubias failed to pay the docket fees within the prescribed period, which
rendered the VA decision final and executory.

Issue:

Wether or not the CA erred in reinstating Cobarrubias’ petition despite her failure to pay the appeal fee within the reglementary
period?

Held:

Yes. Appeal is not a natural right but a mere statutory privilege, thus, appeal must be made strictly in accordance with the provision
set by law. Rule 43 of the Rules of Court provides that appeals from the judgment of the VA shall be taken to the CA, by filing a
petition for review within fifteen (15) days from the receipt of the notice of judgment. Furthermore, upon the filing of the petition,
the petitioner shall pay to the CA clerk of court the docketing and other lawful fees; non-compliance with the procedural
requirements shall be a sufficient ground for the petition’s dismissal. Thus, payment in full of docket fees within the
Remedial Law I| Page 35

prescribed period is not only mandatory, but also jurisdictional. It is an essential requirement, without which, the decision
appealed from would become final and executory as if no appeal has been filed.

The payment of the full amount of the docket fee is an indispensable step for the perfection of an appeal. Appeal is not perfected if
only a part of the docket fee is deposited within the reglementary period and the remainder is tendered after the expiration of the
period.

In the present case, Cobarrubias filed her petition for review on December 5, 2007, fifteen (15) days from receipt of the VA decision
on November 20, 2007, but paid her docket fees in full only after seventy-two (72) days, when she filed her motion for reconsideration
on February 15, 2008 and attached the postal money orders for ₱4,230.00. Undeniably, the docket fees were paid late, and without
payment of the full docket fees, Cobarrubias’ appeal was not perfected within the reglementary period.
Remedial Law I| Page 36

Gipa vs. Southern Luzon Institute


G.R. No. 177425, June 18, 2014

Facts:

SLI (Southern Luzon Institute, an educational institution in Bulan, Sorsogon) asserts absolute ownership of a 7,516-squaremeter
parcel of land situated in Brgy. Poblacion, Bulan, Sorsogon. However, petitioners (defendants in the court a quo) who were occupying
a portion of said land refused to vacate it despite demand prompting SLI to file a Complaint for Recovery of Ownership and
Possession with Damages against petitioners (defendants) Alonzo Gipa et.al., The defendants asserted in their answer that they did
not heed the demand as they believe that they have the right over the land they occupy and which was also occupied by their
predecessors-in-interest which, according to them, dates back to as early as 1950. Impugning SLI’s claims, the defendants averred that
SLI had not even for a single moment taken possession of the subject property and was merely able to procure a title over the same
thru fraud, bad faith and misrepresentation.

The RTC (court a quo) ruled in favor of SLI. The petitioners appealed to the CA. Their appeal was initially dismissed since it was not
shown that the appellate court docket fees and other lawful fees were paid. Through a Motion for Reconsideration (MR) with an
attached Certification that they already paid the appeal fee, the CA reinstated the appeal. Subsequently however, the CA required the
petitioners to pay within 10 days from receipt of minute resolution the amount of 30.00 pesos as legal research fund. It was received
by their counsel on March 13, 2006. Even after a lapse of 9 months, the petitioners still failed to comply with this directive resulting in
the dismissal of their appeal.

Petitioners then filed a Petition for Review on Certiorari and are invoking the principle of liberality in the application of technical
rules considering that they have paid the substantial amount of ₱3,000.00 for docket and other legal fees and fell short only by the
meager amount of ₱30.00. They assert that they come under the exception, in the name of substantial justice and fair play.

Issue:

Whether or not the principle of liberality in the application of the rules should be applied despite the non-payment of the 30-peso legal
research fund to perfect the appeal?

Held:

No. The procedural requirement under Section 4 of Rule 41 is not merely directory, as the payment of the docket and other legal fees
within the prescribed period is both mandatory and jurisdictional. It bears stressing that an appeal is not a right, but a mere statutory
privilege. An ordinary appeal from a decision or final order of the RTC to the CA must be made within 15 days from notice. And
within this period, the full amount of the appellate court docket and other lawful fees must be paid to the clerk of the court which
rendered the judgment or final order appealed from. Without such payment, the appeal is not perfected. The appellate court does not
acquire jurisdiction over the subject matter of the action and the Decision sought to be appealed from becomes final and executory.
Further, under Section 1 (c), Rule 50, an appeal may be dismissed by the CA, on its own motion or on that of the appellee, on the
ground of the non-payment of the docket and other lawful fees within the reglementary period as provided under Section 4 of Rule 41.
The payment of the full amount of the docket fee is an indispensable step for the perfection of an appeal.

The liberality which petitioners pray for has already been granted to them by the CA at the outset. Based on the premise that the
questioned Decision of the RTC has already become final and executory due to non-perfection, the CA could have dismissed the
appeal outright. But owing to the fact that only the meager amount of ₱30.00 was lacking and considering that the CA may opt not to
proceed with the case until the docket fees are paid, it still required petitioners, even if it was already beyond the reglementary period,
to complete their payment of the appeal fee within 10 days from notice. The CA’s leniency over petitioners’ cause did not end there.
Although they were given only 10 days to remit the ₱30.00 deficiency, the said court allowed an even longer period of nine months to
lapse. Moreover, petitioners’ failure to advance any explanation as to why they failed to pay the correct docket fees or to
complete payment of the same within the period allowed by the CA is fatal to their cause. Explaining this, the Court stated that
"[c]oncomitant to the liberal interpretation of the rules of procedure should be an effort on the part of the party invoking liberality to
Remedial Law I| Page 37

adequately explain his failure to abide by the rules." Those who seek exemption from the application of the rule have the burden
of proving the existence of exceptionally meritorious reason warranting such departure.
Remedial Law I| Page 38

Reyes vs. People


G.R. No. 193034, July 20, 2015

Facts:

Salud Gegato filed a Complaint against Rodging Reyes for grave threats in the MCTC of Bayugan and Sibagat, Bayugan, Agusan del
Sur. The information stated that Reyes uttered these words against Gegato: “Salud, stop your rumor against my wife because she will
be embarrassed. I’m warning you, don't mind our lives for I might kill you.” Before arraignment, petitioner filed a Motion to Quash
based on the ground of jurisdiction and that the crime is not Grave Threats under Article 282 of the Revised Penal Code, but Other
Light Threats under Article 285, paragraph 2 of the same Code which was denied by the MCTC. Reyes then filed a Motion to Inhibit
the Presiding Judge on the ground that Gegato is the Court Interpreter of the same court but it was also denied. The MCTC found
Reyes guilty beyond reasonable doubt of the crime charged.

On appeal to the RTC, Reyes was found guilty only of other light threats. Instead of filing an appeal to the CA within the reglementary
period, Reyes filed a Motion for Extension of time twice which was both denied by the CA. The first was denied for failure to pay
the full amount of the docket fees pursuant to Sec. 1, Rule 42 of the Rules of Court. The second was denied as no further extension
may be granted except for most compelling reason. After the filing of the second MotEx, Reyes filed this Petition. Accordingly, this
petition was NOTED but DISMISSED by the CA among the grounds for its dismissal is failure to pay the complete docket fees.
Reyes filed an MR but it was denied for failure of the petitioner to furnish copies to the Solicitor General and the private respondent.
Thus, petitioner filed a Second Motion for Reconsideration which was also denied by the CA. In this resolution, the CA set aside the
first resolution denying the 1st MR but also dismissed the Petition for Review. In the Third MR, the CA RESOLVED to merely NOTE
WITHOUT ACTION the petitioner's third Motion for Reconsideration, in view of the 23 November 2009 Resolution dismissing this
petition with finality. Now, Reyes insists that the CA erred in favoring procedural technicalities over his constitutional right to due
process.

Issue:

Whether or not the CA ruled correctly in dismissing the petition on the ground of failure to pay the complete docket fees?

Held:

Yes. Section 1, Rule 42 of the Rules of Court states the need to pay docket fees. The rule is that payment in full of the docket fees
within the prescribed period is mandatory. In Manchester v. Court of Appeals, it was held that a court acquires jurisdiction over any
case only upon the payment of the prescribed docket fee. The strict application of this rule was, however, relaxed two (2) years after in
the case of Sun Insurance Office, Ltd. v. Asuncion, wherein the Court decreed that where the initiatory pleading is not accompanied
by the payment of the docket fee, the court may allow payment of the fee within a reasonable period of time, but in no case beyond the
applicable prescriptive or reglementary period. This ruling was made on the premise that the plaintiff had demonstrated his
willingness to abide by the rules by paying the additional docket fees required.

Thus, in the more recent case of United Overseas Bank v. Ros, the Court explained that where the party does not deliberately intend to
defraud the court in payment of docket fees, and manifests its willingness to abide by the rules by paying additional docket fees when
required by the court, the liberal doctrine enunciated in Sun Insurance Office, Ltd., and not the strict regulations set in Manchester,
will apply.

Admittedly, this rule is not without recognized qualifications. The Court has declared that in appealed cases, failure to pay the
appellate court docket fee within the prescribed period warrants only discretionary as opposed to automatic dismissal of the appeal and
that the court shall exercise its power to dismiss in accordance with the tenets of justice and fair play, and with great deal of
circumspection considering all attendant circumstances.

In that connection, the CA, in its discretion, may grant an additional period of fifteen (15) days only within which to file the petition
for review upon proper motion and the payment of the full amount of the docket and other lawful fees and the deposit for costs before
the expiration of the reglementary period and that no further extension shall be granted except for the most compelling reason and in
no case to exceed fifteen (15) days. Therefore, the grant of any extensions for the filing of the petition is discretionary and subject to
the condition that the full amount of the docket and lawful fees are paid before the expiration of the reglementary period to file the
petition.
Remedial Law I| Page 39

It is only when persuasive reasons exist that the Rules may be relaxed to spare a litigant of an injustice not commensurate with his
failure to comply with the prescribed procedure.34 In the present case, petitioner failed to convince this Court of the need to relax the
rules and the eventual injustice that he will suffer if his prayer is not granted.
Remedial Law I| Page 40

NTC vs. Ebesa


G.R. No. 186102, February 24, 2016

Facts:

Early in 2005, NTC filed a case to expropriate the 1,479-square-meter portion of Lot No. 18470, situated in Quiot, Pardo, Cebu City.
It is declared under the co-ownership of the heirs of Teodulo Ebesa, namely, Porferia Ebesa, Efren Ebesa, Dante Ebesa and Cynthia
Ebesa Ramirez (Heirs of Ebesa), but is occupied by Atty. Fortunato Veloso (Veloso) (respondents), who allegedly purchased the
property, as evidenced by an unregistered Deed of Sale. NTC alleged that the acquisition of an easement right-of-way over a portion
of the subject property is necessary for the construction and maintenance of the Quiot (Pardo) 100MVA Substation Project in Cebu
City, an undertaking that partook of a public purpose.

In his Answer, Veloso, acting as his own counsel in collaboration with Atty. Nilo Ahat, conceded that the project was indeed intended
for a public purpose but disputed its necessity and urgency and alleged that the project will not only affect a portion of the property but
its entirety considering that the construction entails the installation of huge permanent steel towers and the air space directly above the
subject property will be permanently occupied with transmission lines. The RTC issued an Order of Expropriation. After settling the
amount of just compensation to be paid to Veloso, the RTC rendered a decision directing the NTC to pay Veloso the amount of 35M
pesos with interest and costs and or immediately return the land to Veloso and await the finality of the judgment before paying
Veloso. The MR filed by NTC was denied by the RTC prompting it to file an appeal before the CA. Thereafter, the CA directed the
NTC to submit official receipt or proof of payment of the appeal fees within 10 days from notice. T he NTC filed a Manifestation,
alleging that it cannot comply with the order of the CA as it did not pay appeal docket fees. It asseverated that the receiving
clerk of the RTC did not accept its payment for the appeal fees on the ground that it is exempted from doing so, being a
GOCC. The now respondents filed a Motion to Dismiss stating that the judgment has now attained finality since the payment of
docket fees is mandatory and jurisdictional and non-payment thereof will not toll the running of the appeal period. Afterwards, the
NTC filed another Manifestation informing the CA that it already filed on September 18, 2006 a Manifestation with Urgent Ex-Parte
Motion with the RTC and settled the payment of appeal fees also submitting the official receipts for the said payment

The CA granted the Motion to Dismiss. The CA held that the NTC's counsel should know that as a GOCC, it is not exempted from
the payment of docket and other legal fees.

Unyielding, the NTC, in its present appeal, contends that the failure to pay appeal docket fees does not automatically cause the
dismissal of the appeal, but lies on the discretion of the court. It asseverates that since its failure to pay the appeal fees was not willful
and deliberate, its omission could be excused in the interest of justice and equity. It reiterates that it was prepared to pay the docket
fees if not for the receiving clerk's advice that the same was not necessary as it is a GOCC. Even then, it eventually paid the appeal
fees, although past the reglementary period.

Issue:

Whether or not CA was correct in dismissing NTC’s appeal on the ground of non payment of docket fees resulting into the finality of
the RTC’s decision?

Held:

Yes. Verily, the payment of appeal docket fees is both mandatory and jurisdictional. It is mandatory as it is required in all appealed
cases, otherwise, the Court does not acquire the authority to hear and decide the appeal. The failure to pay or even the partial payment
of the appeal fees does not toll the running of the prescriptive period, hence, will not prevent the judgment from becoming final and
executory. Such was the circumstance in the instant appeal. The NTC failed to pay the appeal fees without justifiable excuse. That its
counsel or his representative was misled by the advice of the receiving clerk of the RTC is unacceptable as the exercise of ordinary
diligence could have avoided such a blunder. It is apparent from the records that the NTC had ample time to rectify the error or clarify
its reservation regarding the propriety of its supposed exemption from the appeal fees.

In the present case, the NTC failed to present any justifiable excuse for its failure to pay the docket fees like in the cases of  Mactan
Cebu International Airport Authority v. Manguhat and Yambao v. CA. In Mactan Cebu International Airport Authority, the petitioner
Remedial Law I| Page 41

took the initiative to verify the necessity of paying the docket fees and paid it outright, albeit six days after the lapse of the period to
appeal. Quite the opposite, the NTC in the present case never lifted a finger until it was required by the CA to present proof of its
payment of the docket fees and paid the same only six months after the period to appeal has prescribed.
Remedial Law I| Page 42

Vargas vs. Estate of Ogsos


G.R. No. 221062, October 5, 2016

Facts:

Ogsos, Sr. and the Heirs of Fermina Pepico (Fermina), represented by their Attorney-in-Fact, Catalino V. Noel, entered into a Contract
of Lease (lease contract) covering five (5) parcels of agricultural land owned by the latter. Based on the contract, Ogsos, Sr. agreed to
pay the Heirs of Fermina 230 piculs or 290.95 liquid-kilogram (lkg.) of centrifugal sugar every crop year, starting from crop year
1994-1995 to crop year 2000-2001, as lease rental. The contract was subsequently extended for three years and the lease rental was
modified. Petitioner and Kathryn, who are among the heirs of Fermina, claimed that the lease rentals from crop year 1994-1995 to
crop year 1998-1999 were not paid. Thus, they filed a Complaint for Specific Performance and Damages against respondents to
recover the unpaid lease rentals.
In their Answer, respondents alleged that they had faithfully complied with their obligations as embodied in the lease contract and its
subsequent amendments. They denied abandoning the leased premises and claimed that sometime in December 1998, petitioner and
Kathryn unlawfully took possession of the leased premises and appropriated for themselves the sugarcane ready for harvest under the
pretext that they would apply the proceeds thereof to the unpaid rent. Respondents filed a counterclaim for lost profits plus damages
that it had sustained when petitioner took over the possession of the leased premises and harvesting and appropriating respondents'
crops planted therein. Petitioners filed a motion to dismiss respondents' counterclaim arguing that the same were permissive and that
respondents had not paid the appropriate docket fees. The RTC, denied the said motion, declaring respondents' counterclaim as
compulsory; thus, holding that the payment of the required docket fees was no longer necessary. The CA affirmed the ruling of the
RTC.

Issue:

Whether or not the respondents' counterclaim for damages is compulsory and not permissive in nature, and thus, no payment of docket
fees is required?

Held:

No. In Spouses Mendiola v. CA, the Court had devised tests in determining whether or not a counterclaim is compulsory or
permissive:

(a) Are the issues of fact or law raised by the claim and the counterclaim largely the same?
(b) Would res judicata bar a subsequent suit on defendant's claims, absent the compulsory counterclaim rule?
(c) Will substantially the same evidence support or refute plaintiff's claim as well as the defendant's counterclaim?
(d) Is there any logical relation between the claim and the counterclaim, such that the conduct of separate trials of the respective
claims of the parties would entail a substantial duplication of effort and time by the parties and the court? 

If these tests result in affirmative answers, the counterclaim is compulsory.

Based on the abovementioned standards, the Court finds that the counterclaim of respondents is permissive in nature. This is because:
(a) the issue in the main case, i.e., whether or not respondents are liable to pay lease rentals, is entirely different from the issue in the
counterclaim, i.e., whether or not petitioner and Kathryn are liable for damages for taking over the possession of the leased premises
and harvesting and appropriating respondents' crops planted therein; (b) since petitioner and respondents' respective causes of action
arose from completely different occurrences, the latter would not be barred by res judicata had they opted to litigate its counterclaim in
a separate proceeding; (c) the evidence required to prove petitioner's claim that respondents failed to pay lease rentals is likewise
different from the evidence required to prove respondents' counterclaim that petitioner and Kathryn are liable for damages for
performing acts in bad faith; and (d) the recovery of petitioner's claim is not contingent or dependent upon proof of respondents'
counterclaim, such that conducting separate trials will not result in the substantial duplication of the time and effort of the court and
the parties.

In view of the finding that the counterclaim is permissive, and not compulsory as held by the courts a quo, respondents are required to
pay docket fees. However, it must be clarified that respondents' failure to pay the required docket fees,  per se, should not necessarily
lead to the dismissal of their counterclaim. It has long been settled that while the court acquires jurisdiction over any case only upon
the payment of the prescribed docket fees, its non-payment at the time of filing of the initiatory pleading does not automatically cause
its dismissal provided that: (a) the fees are paid within a reasonable period; and (b) there was no intention on the part of the claimant
to defraud the government. Here, respondents cannot be faulted for non-payment of docket fees in connection with their counterclaim,
primarily because RTC and CA had already found such counterclaim to be compulsory. Respondents' reliance on the findings of the
Remedial Law I| Page 43

courts a quo, albeit erroneous, exhibits their good faith in not paying the docket fees, much more their intention not to defraud the
government. Thus, the counterclaim should not be dismissed for nonpayment of docket fees. Instead, the docket fees required shall
constitute a judgment lien on the monetary awards in respondents’ favor.
Remedial Law I| Page 44

Camaso vs. TSM Shipping, Inc.


G.R. No. 223290, November 7, 2016

Facts:

A complaint for disability benefits was filed before the Labor Arbiter by seaman Woodrow Camacho who was suffering from tonsillar
cancer against his employer TSM Shipping (Phils.), Inc., Utkilen, and Jones Tulod who refused to shoulder his medical expenses. The
LA decided in his favor, but NLRC dismissed his complaint. On certiorari before the CA, latter dismissed his petition "for non-
payment of the required docketing fees as required under Section 3, Rule 46 of the Revised Rules of Court."

Camaso filed a Motion for Reconsideration arguing, inter alia, that a check representing the payment of the required docket fees was
attached to a copy of his petition filed before the CA. He further claimed that upon verification of his counsel’s messenger, the
Division Clerk of Court admitted that it was simply overlooked. CA denied Camaso's motion. Citing the presumption of regularity of
official duties, the CA gave credence to the explanation of the OIC of the CA Receiving Section, that there was no cash, postal money
order, or check attached to HIS petition when it was originally filed before the CA. In any event, the CA held that assuming that a
check was indeed attached to the petition, such personal check, i.e., Metrobank check dated July 6, 2015 under the personal account of
a certain Pedro L. Linsangan, is not a mode of payment sanctioned by the 2009 Internal Rules of the Court of Appeals (2009 IRCA),
which allows only payment in cash, postal money order, certified, manager's or cashier's checks payable to the CA.

Issue:

Whether or not the CA correctly dismissed Camaso's petition for certiorari despite the fact that a check representing payment of the
docket fees was attached to the copy of his petition?

Held:

No. The failure to pay the required docket fees per se should not necessarily lead to the dismissal of a case. It has long been settled
that while the court acquires jurisdiction over any case only upon the payment of the prescribed docket fees, its non-payment at the
time of filing of the initiatory pleading does not automatically cause its dismissal provided that: (a) the fees are paid within a
reasonable period; and there was no intention on the part of the claimant to defraud the government.

Here, it appears that when Camaso filed his certiorari petition through his counsel and via mail, a Metrobank check dated July 6, 2015
under the account name of Pedro L. Linsangan was attached thereto to serve as payment of docket fees.26  Although this was not an
authorized mode of payment under Section 6, Rule VIII27 of the 2009 IRCA, the attachment of such personal check shows that
Camaso exerted earnest efforts to pay the required docket fees. Clearly, this exhibits good faith and evinces his intention not to
defraud the government. In this relation, the assertion of the Officer-in-Charge of the CA Receiving Section that there was no check
attached to Camaso's certiorari petition is clearly belied by the fact that when it was examined at the Office of the Division Clerk of
Court, the check was found to be still stapled thereto.

In light of the foregoing circumstances, the Court deems it appropriate to relax the technical rules of procedure in the interest of
substantial justice and, hence, remands the instant case to the CA for the resolution of its substantial merits.
Remedial Law I| Page 45

Rule 2: Causes of Action (Splitting)

Dynamic Builders vs. Presbitero


G.R. No. 174202, April 7, 2015

Facts:

The Municipality of Valladolid, Negros Occidental published an invitation to bid for the construction of a 1,050-lineal-meter rubble
concrete seawall along the municipality’s shoreline. This infrastructure venture is known as the “Construction Shoreline Protection
Project.” The Bids and Awards Committee (BAC) conducted a pre-bid conference attended by six (6) prospective contractors
including Dynamic Builders. The BAC recommended the award in favor of HLJ Construction and Enterprise for a local infrastructure
project. Dynamic Builders questioned the declaration of its bid by the BAC as “substantially unresponsive”. Upon the denial of its
formal protest, it filed a petition for certiorari before the RTC. Simultaneously, it filed a petition for prohibition with application for
temporary restraining order and/or writ of preliminary injunction before the Supreme Court.

Dynamic Builders submits that Article XVII, Section 58 of Republic Act No. 9184 implicitly allowed it to simultaneously file a
Petition for Certiorari before the Regional Trial Court assailing the protest case on the merits, and another Petition before this court for
injunctive remedies. Dynamic Builders argues that the law prohibiting lower courts from issuing TRO, preliminary injunctions, or
preliminary mandatory injunctions in national infrastructure projects (R.A. 8975) also prohibits the same in local infrastructure
projects. As a result, only the SC may issue the preliminary injunction sought for in this petition.

Issue:

Whether or not Dynamic Builders violated the rule against splitting of a cause of action when it filed a petition for certiorari with the
RTC and at the same time a petition for prohibition with the SC?

Held:

Yes. Rule 2, Section 3 of the Rules of Court provides that “[a] party may not institute more than one suit for a single cause of action.”
Petitioner’s petition for prohibition seeks to enjoin the execution of public respondent’s Decision and Resolution on the protest — the
same Decision and Resolution sought to be set aside in the Petition before the Regional Trial Court. In essence, petitioner seeks the
same relief through two separate Petitions filed before separate courts. This violates the rule against forum shopping.

The petitioner is mistaken when it argued that only the Supreme Court can issue injunctive relief against infracture projects of local
governments. The Regional Trial Court can issue injunctive relief against government infrastructure projects, even those undertaken
by local governments, considering that the prohibition in Section 3 of Republic Act No. 8957 only mentions national government
projects. These courts can issue injunctive relief when there are compelling constitutional violations — only when the right is clear,
there is a need to prevent grave and irreparable injuries, and the public interest at stake in restraining or enjoining the project while the
action is pending far outweighs the inconvenience or costs to the party to whom the project is awarded.
Remedial Law I| Page 46

Rule 3: Parties
Relucio vs. Lopez
G.R. No. 138497, January 16, 2002

Facts:

Angelina Mejia Lopez filed a petition for Appointment as sole administratix of conjugal partnership of properties, forfeiture, etc.
against her husband, Alberto Lopez and petitioner Imelda Relucio. She alleged that sometime in 1968, her husband, who is legally
married to her, abandoned the latter and their four legitimate children; that he arrogated unto himself full and exclusive control and
administration of the conjugal properties, spending and using the same for his sole gain and benefit to the total exclusion of her and
their four children; and after abandoning his family, maintained an illicit relationship and cohabited with Relucio since 1976.

A motion to dismiss was filed by Relucio on the ground that Angelina Lopez has no cause of action against her. The RTC denied her
motion. A Motion for Reconsideration was also denied by the RTC. She filed a petition for certiorari with the CA which also denied.
Hence, this petition.

Issue:

Whether or not Relucio is an indispensable party in this case?

Held:

No. A real party in interest is one who stands “to be benefited or injured by the judgment of the suit.” In this case, petitioner would not
be affected by any judgment in Special Proceedings M-3630. If petitioner is not a real party-in-interest, she cannot be an indispensable
party. An indispensable party is one without whom there can be no final determination of an action. Petitioner’s participation in
Special Proceedings M-3630 is not indispensable. Certainly, the trial court can issue a judgment ordering Alberto J. Lopez to make an
accounting of his conjugal partnership with respondent, and give support to respondent and their children, and dissolve Alberto J.
Lopez’ conjugal partnership with respondent, and forfeit Alberto J. Lopez’ share in property co-owned by him and petitioner. Such
judgment would be perfectly valid and enforceable against Alberto J. Lopez. Nor can petitioner be a necessary party in Special
Proceedings M-3630. A necessary party as one who is not indispensable but who ought to be joined as party if complete relief is to be
accorded those already parties, or for a complete determination or settlement of the claim subject of the action. In the context of her
petition in the lower court, respondent would be accorded complete relief if Alberto J. Lopez were ordered to account for his alleged
conjugal partnership property with respondent, give support to respondent and her children, turn over his share in the co-ownership
with petitioner and dissolve his conjugal partnership or absolute community property with respondent.
Remedial Law I| Page 47

De Castro vs. Court of Appeals


July 18, 2002

Facts:

Petitioners Constante A. De Castro and Corazon A. De Castro were co-owners of four lots. Francisco Antigo was authorized by the De
Castros to act as real estate broker in the sale of these lands. 5% of which will be given to him as commission. Later on, two lots were
bought by Times Transit Corporation. However, Artigo sued the De Castros to collect the unpaid balance of his broker’s commission.
One of the defenses advanced by the De Castros is that the complaint failed to implead their other siblings who were co-owners as
well. The De Castros claim that Artigo always knew that the two lots were co-owned by Constante and Corazon with their other
siblings Jose and Carmela whom Constante merely represented. The De Castros contend that failure to implead such indispensable
parties is fatal to the complaint since Artigo, as agent of all the four coowners, would be paid with funds co-owned by the four co-
owners. RTC held that Artigo is entitled to 5% commission on the purchase price as provided in the contract of agency. CA affirmed
the decision of RTC and held that it is not necessary to implead the other co-owners since the action is based on contract of agency.

Issue:

Whether or not the other co-owners (other De Castro siblings) are indispensable parties?

Held:

No. An indispensable party is one whose interest will be affected by the courts action in the litigation and without whom no final
determination of the case can be. Joinder of indispensable parties is mandatory and courts cannot proceed without their presence.
However, the rule on mandatory joinder of indispensable parties is not applicable to the instant case.

There is no dispute that Constante appointed Artigo as agent. Whether Constante appointed Artigo as agent, in Constante’s individual
or representative capacity, or both, the De Castros cannot seek the dismissal of the case for failure to implead the other co-owners as
indispensable parties. The De Castros admit that the other co-owners are solidarily liable under the contract of agency, citing Article
1915 of the Civil Code, which reads:

Art. 1915. If two or more persons have appointed an agent for a common transaction or undertaking, they shall be solidarily liable to
the agent for all the consequences of the agency.

When the law expressly provides for solidarity of the obligation, as in the liability of co-principals in a contract of agency, each
obligor may be compelled to pay the entire obligation. The agent may recover the whole compensation from any one of the co-
principals, as in this case.
Remedial Law I| Page 48

Urquiola vs. Court of Appeals


G.R. No. 141463. August 6, 2002

Facts:

Pura Kalaw Ledesma was the registered owner of Lot 689, covered by TCT Nos. 111267 and 111266, in Tandang Sora, Quezon City.
This parcel of land was adjacent to certain portions of Lot 707 of the Piedad Estates, namely, Lot 707-A and 707-B, registered in the
name of Herminigilda Pedro under TCT Nos. 16951 and 16952, respectively. On October 29, 1964, Herminigilda sold Lot 707-A and
707-B to Mariano Lising who then registered both lots and Lot 707- C in the name of M.B. Lising Realty and subdivided them into
smaller lots.

Certain portions of the subdivided lots were sold to third persons including herein petitioners, spouses Victor and Honorata Orquiola,
who purchased a portion of Lot 707-A-2, Lot 5, Block 1 of the subdivision plan (LRC), Psd-42965.

Sometime in 1969, Ledesma filed a complaint, docketed as Civil Case No. Q-12918, with the Regional Trial Court of Quezon City
against Herminigilda Pedro and Mariano Lising for allegedly encroaching upon Lot 689. During the pendency of the action, Tandang
Sora Development Corporation replaced Ledesma as plaintiff by virtue of an assignment of Lot 689 made by Ledesma in favor of said
corporation. The RTC adjudged defendants Pedro and Lising jointly and severally liable for encroaching on plaintiff’s land.

As a result, the Deputy Sheriff of Quezon City directed petitioners, through an alias writ of execution, to remove the house they
constructed on the land they were occupying.

To prohibit Judge Baclig of the RTC-QC from issuing a writ of demolition and the Quezon City sheriff from implementing the alias
writ of execution, petitioners filed with the CA a petition for prohibition with prayer for a restraining order and preliminary injunction
on the ground that they bought the subject parcel of land in good faith and for value, and since they were not impleaded in Civil Case
No. Q-12918, the writ of demolition issued in connection therewith cannot be enforced against them because to do so would amount
to deprivation of property without due process of law. CA dismissed the petition and held that the petitioners were considered privies
who derived their rights from Lising by virtue of the sale and could be reached by the execution order.

Issue:

Whether or not the decision can be enforced against petitioners even if they were not impleaded as parties?

Held:

No. The petitioners are fully entitled to the legal protection of their lot by the Torrens system. P etitioners acquired the lot before the
commencement of Civil Case No. Q-12918 and acquired the registered title in their own names. They are not successors-in-interest of
Mariano Lising, and as such, they cannot be reached by the order of execution in Civil Case No. Q-12918.

As builders in good faith and innocent purchasers for value, petitioners have rights over the subject property and hence they are proper
parties in interest in any case thereon. Consequently, private respondents should have impleaded them in Civil Case No. Q-12918.
Since they failed to do so, petitioners cannot be reached by the decision in said case. No man shall be affected by any proceeding to
which he is a stranger, and strangers to a case are not bound by any judgment rendered by the court. In the same manner, a writ of
execution can be issued only against a party and not against one who did not have his day in court. Only real parties in interest in an
action are bound by the judgment therein and by writs of execution and demolition issued pursuant thereto.
Remedial Law I| Page 49

China Banking Corp. vs. Oliver


G.R. No. 135796, October 3, 2002.

Facts:

In August 1995, Pangan Lim, Jr. and a certain Mercedes M. Oliver opened a joint account in China Banking Corporation at EDSA
Balintawak Branch.Thereafter, Lim and Oliver applied for a P17 million loan and offered a collateral. On November 17, 1995, Lim
and Oliver executed in favor of Chinabank a promissory note for P16,650,000, as well as a Real Estate Mortgage on the property. The
mortgage document showed Mercedes Oliver’s (Oliver One) address to be No. 95 Malakas Street, Diliman, Quezon City.

Later, respondent claiming that she is Mercedes M. Oliver with postal office address at No. 40 J.P. Rizal St., San Pedro, Laguna, filed
an action for annulment of mortgage and cancellation of title with damages against Chinabank and the Register of Deeds. Respondent
(Oliver Two), claimed that she was the registered and lawful owner of the land subject of the real estate mortgage; that the owners
duplicate copy of the title had always been in her possession; and that she did not apply for a loan or surrender her title to Chinabank.
She prayed that: (1) the owners duplicate copy surrendered to Chinabank as well as the original title with the Registry of Deeds be
cancelled; (2) the mortgage be declared null and void; and (3) the Registry of Deeds be ordered to issue a new and clean title in her
name.

Chinabank moved to dismiss the case for lack of cause of action and non-joinder of an indispensable party, the mortgagor. The RTC
denied the Motion.

Issue:

Whether or not the mortgagor Mercedes Oliver (Oliver One) is an indispensable party to the case?

Held:

No. An indispensable party is a party in interest, without whom no final determination can be had of an action. It is true that
mortgagor Oliver One is a party in interest, for she will be affected by the outcome of the case. She stands to be benefited in case the
mortgage is declared valid, or injured in case her title is declared fake. However, mortgagor Oliver One’s absence from the case does
not hamper the trial court in resolving the dispute between respondent Oliver Two and petitioner.

Oliver Two’s Complaint was for annulment of mortgage due to petitioner’s negligence in not determining the actual ownership of the
property, resulting in the mortgage’s annotation on TCT No. S-50195 in the Registry of Deeds custody. This, respondent Oliver Two
can do in her complaint without necessarily impleading the mortgagor Oliver One. Hence, Oliver One is not an indispensable party in
the case filed by Oliver Two. Chinabank has interest in the loan which, however, is distinct and divisible from the mortgagor’s
interest, which involves the land used as collateral for the loan.

A party is also not indispensable if his presence would merely permit complete relief between him and those already parties to the
action, or will simply avoid multiple litigation, as in the case of Chinabank and mortgagor Oliver One. The latter’s participation in this
case will simply enable petitioner Chinabank to make its claim against her in this case, and hence, avoid the institution of another
action. Thus, it was the bank who should have filed a third-party complaint or other action versus the mortgagor Oliver One.
Remedial Law I| Page 50

Lotte Phils. Co. Inc. vs Dela Cruz


G.R. No. 166302, July 28, 2005

Facts:

Erlinda Dela Cruz, et al. filed an illegal dismissal case against Lotte Phils. Co. Inc and 7J Maintenance and Janitorial Services. They
alleged that they were hired by 7J Maintenance and Janitorial Services, and was assigned with Lotte Phil, Co., Inc., herein petitioner to
perform piece works and as repackers or sealers. They further aver that due to the expiration of the service contract between 7J and
herein petitioner, they were dispensed with their services, and that they were told by 7J that the latter will call them if they will be
needed for work. But they were never called back. The Labor Arbiter, rendered judgment declaring 7J being the respondents’
employer, and that the former was guilty of illegal dismissal. The respondents appealed to the NLRC, praying that herein petitioner be
declared as their direct employer because 7J is merely a labor-only contractor. In its decision, NLRC affirmed the decision of the
Labor Arbiter. MR was also denied. Thus, the respondents filed a petition for certiorari before the CA. Lotte Phils. Co moved to
dismiss the petition on the ground that the respondents failed to implead 7J which was the party-in-interest to the said case. The Court
of Appeals reversed and set aside the rulings of the Labor Arbiter and the NLRC. In its decision, the Court of Appeals declared Lotte
as the real employer of respondents and that 7J who engaged in labor-only contracting was merely the agent of Lotte. Hence, this
petition for review on certiorari.

Issue:

Whether or not 7J should have been impleaded in the petition before the CA as an indispensable party?

Held:
Yes. An indispensable party is a party in interest without whom no final determination can be had of an action, and who shall be joined
either as plaintiffs or defendants. The joinder of indispensable parties is mandatory. The presence of indispensable parties is necessary
to vest the court with jurisdiction, which is the authority to hear and determine a cause, the right to act in a case.  Thus, without the
presence of indispensable parties to a suit or proceeding, judgment of a court cannot attain real finality. The absence of an
indispensable party renders all subsequent actions of the court null and void for want of authority to act, not only as to the
absent parties but even as to those present.
In this case, 7J is an indispensable party. It is a party in interest because it will be affected by the outcome of the case. The Labor
Arbiter and the NLRC found 7J to be solely liable as the employer of respondents. The Court of Appeals however rendered Lotte
jointly and severally liable with 7J who was not impleaded by holding that the former is the real employer of respondents. Plainly, its
decision directly affected 7J.
However, non-joinder of indispensable parties is not a ground for the dismissal of an action and the remedy is to implead the non-
party claimed to be indispensable. Parties may be added by order of the court on motion of the party or on its own initiative at any
stage of the action and/or such times as are just. If the petitioner refuses to implead an indispensable party despite the order of the
court, the latter may dismiss the complaint/petition for the petitioner/plaintiffs failure to comply therefor.
Although 7J was a co-party in the case before the Labor Arbiter and the NLRC, respondents failed to include it in their petition
for certiorari in the Court of Appeals.
Remedial Law I| Page 51

Carabeo vs. Dingco


G.R. No. 190823, April 4, 2011

Facts:

Domingo Carabeo, herein petitioner, entered into a contract denominated as "Kasunduan sa Bilihan ng Karapatan sa
Lupa" (Kasunduan) with Spouses Norberto and Susan Dingco, herein respondents, whereby petitioner agreed to sell his rights on a
parcel of unregistered land situated in Purok III, Tugatog, Orani, Bataan to the respondents. The respondents tendered the initial
payment, while the remaining balance was refused by the petitioner, proffering as reason therefor that he would register the land first.
Subsequently, it came to know to the respondents that the land was already registered to the petitioner’s name, thus they offered to pay
the balance, but the petitioner declined. They filed a complaint for specific performance before the Regional Trial Court (RTC) of
Balanga, Bataan. The RTC rendered its decision in favor of the respondents. On appeal, the CA affirmed such decision. MR was
denied as well. Hence this petition for review on certiorari. The petitioner’s counsel, avers that the case should be dismissed due to the
death of Domingo Carabeo, being an action personam.

Issue:

Whether or not the case should be dismissed on the ground that the petitioner died?

Held:

No. The question as to whether an action survives or not depends on the nature of the action and the damage sued for. In the causes of
action which survive, the wrong complained [of] affects primarily and principally property and property rights, the injuries to the
person being merely incidental, while in the causes of action which do not survive, the injury complained of is  to the person, the
property and rights of property affected being incidental.

In the present case, respondents are pursuing a property right arising from the kasunduan, whereas petitioner is invoking nullity of the
kasunduan to protect his proprietary interest. Assuming arguendo, however, that the kasunduan is deemed void, there is a corollary
obligation of petitioner to return the money paid by respondents, and since the action involves property rights, it survives.

It bears noting that trial on the merits was already concluded before petitioner died. Since the trial court was not informed of
petitioner’s death, it may not be faulted for proceeding to render judgment without ordering his substitution. Its judgment is thus valid
and binding upon petitioner’s legal representatives or successors-in-interest, insofar as his interest in the property subject of the action
is concerned.

In another vein, the death of a client immediately divests the counsel of authority. Thus, in filing a Notice of Appeal, petitioner’s
counsel of record had no personality to act on behalf of the already deceased client who, it bears reiteration, had not been substituted
as a party after his death. The trial court’s decision had thereby become final and executory, no appeal having been perfected.
Remedial Law I| Page 52

Dela Cruz vs. Joaquin


G.R. No. 162788, July 28, 2005

Facts:

Pedro Joaquin filed a Complaint for the recovery of possession and ownership, the cancellation of title, and damages, against
petitioners in the Regional Trial Court of Baloc, Sto. Domingo, Nueva Ecija. Respondent alleged that he had obtained a loan from
them in the amount of ₱9,000, payable after 5 years. To secure the payment of the obligation, he supposedly executed a Deed of Sale
in favor of petitioners. The Deed was for a parcel of land in Pinagpanaan, Talavera, Nueva Ecija. The parties also executed another
document entitled "Kasunduan." 

Respondent claimed that the Kasunduan showed the Deed of Sale to be actually an equitable mortgage. Spouses De la Cruz contended
that this document was merely an accommodation to allow the repurchase of the property, a right that he failed to exercise.

RTC issued a Decision in favor of the respondent. The trial court declared that the parties had entered into a sale with a right of
repurchase. It further held that respondent had made a valid tender of payment on two separate occasions to exercise his right of
repurchase. 

Sustaining the trial court, the CA noted that petitioners had given respondent the right to repurchase the property within five (5) years
from the date of the sale. The appellate court also found no reason to overturn the finding that respondent had validly exercised his
right to repurchase the land.

In 2004, the CA denied reconsideration and ordered a substitution by legal representatives, in view of respondent’s death on
December 24, 1988. Petitioners assert that the RTC’s Decision was invalid for lack of jurisdiction. They claim that respondent died
during the pendency of the case. There being no substitution by the heirs, the trial court allegedly lacked jurisdiction over the
litigation.

Issue:

Whether or not the RTC Decision should be declared void because there was no substitution made despite the death of the respondent?

Held:

No. This general rule notwithstanding, a formal substitution by heirs is not necessary when they themselves voluntarily appear,
participate in the case, and present evidence in defense of the deceased. The rule on the substitution by heirs is not a matter of
jurisdiction, but a requirement of due process. Thus, when due process is not violated, as when the right of the representative or heir is
recognized and protected, noncompliance or belated formal compliance with the Rules cannot affect the validity of a promulgated
decision. Mere failure to substitute for a deceased plaintiff is not a sufficient ground to nullify a trial court’s decision. The alleging
party must prove that there was an undeniable violation of due process.

The records of the present case contain a "Motion for Substitution of Party Plaintiff" dated February 15, 2002, filed before the CA.

Evidently, the heirs of Pedro Joaquin voluntary appeared and participated in the case. The court stress that the appellate court had
ordered his legal representatives to appear and substitute for him. The substitution even on appeal had been ordered correctly. In all
proceedings, the legal representatives must appear to protect the interests of the deceased. After the rendition of judgment, further
proceedings may be held, such as a motion for reconsideration or a new trial, an appeal, or an execution.

Considering the foregoing circumstances, the Motion for Substitution may be deemed to have been granted; and the heirs, to have
substituted for the deceased, Pedro Joaquin. There being no violation of due process, the issue of substitution cannot be upheld as a
ground to nullify the trial court’s Decision.

Navarro vs. Escobido


G.R. No. 153788, November 27, 2009

Facts:
Remedial Law I| Page 53

Karen Go, doing business under the trade name KARGO Enterprises, entered into a Lease Agreement with Option to Repurchase.
KARGO Enterprises was represented by Glenn Go (Karen Go’s husband). Subsequently, Karen Go filed two complaints before the
RTC for replevin and/or sum of money with damages against Roger Navarro. In these complaints, Karen Go prayed that the RTC
issue writs of replevin for the seizure of two (2) motor vehicles in Navarro’s possession. In his Answers, Navarro alleged as a special
affirmative defense that the two complaints stated no cause of action, since Karen Go was not a party to the Lease Agreements with
Option to Purchase. RTC dismissed the case on the ground that the complaints did not state a cause of action.

The RTC issued another order setting aside the order of dismissal. Acting on the presumption that Glenn Go’s leasing business is a
conjugal property, the RTC held that Karen Go had sufficient interest in his leasing business to file the action against Navarro.
However, the RTC held that Karen Go should have included her husband, Glenn Go, in the complaint based on Section 4, Rule 3 of
the Rules of Court (Rules). Thus, the lower court ordered Karen Go to file a motion for the inclusion of Glenn Go as co-plaintiff.

When the RTC denied Navarro’s motion for reconsideration, Navarro filed a petition for certiorari with the CA, contending that the
RTC committed grave abuse of discretion when it reconsidered the dismissal of the case and directed Karen Go to amend her
complaints by including her husband Glenn Go as co-plaintiff. Navarro alleges that even if the lease agreements were in the name of
KARGO Enterprises, since it did not have the requisite juridical personality to sue, the actual parties to the agreement are himself and
Glenn Go. Since it was Karen Go who filed the complaints and not Glenn Go, she was not a real party-in-interest and the complaints
failed to state a cause of action. Navarro posits that the RTC erred when it ordered the amendment of the complaint to include Glenn
Go as a co-plaintiff, instead of dismissing the complaint outright because a complaint which does not state a cause of action cannot be
converted into one with a cause of action by a mere amendment or a supplemental pleading. In effect, the lower court created a cause
of action for Karen Go when there was none at the time she filed the complaints.

Issue:

Whether or not Karen Go is a real party in interest and hence, she has a cause of action against Navarro?

Held:

Yes. The 1997 Rules of Civil Procedure requires that every action must be prosecuted or defended in the name of the real party-in-
interest, i.e., the party who stands to be benefited or injured by the judgment in the suit, or the party entitled to the avails of the suit.

As the registered owner of Kargo Enterprises, Karen Go is the party who will directly benefit from or be injured by a judgment in this
case. Thus, contrary to Navarro’s contention, Karen Go is the real party-in-interest, and it is legally incorrect to say that her Complaint
does not state a cause of action because her name did not appear in the Lease Agreement that her husband signed in behalf of Kargo
Enterprises. Whether Glenn Go can legally sign the Lease Agreement in his capacity as a manager of Kargo Enterprises, a sole
proprietorship, is a question we do not decide, as this is a matter for the trial court to consider in a trial on the merits.

In a co-ownership, co-owners may bring actions for the recovery of co-owned property without the necessity of joining all the other
co-owners as co-plaintiffs because the suit is presumed to have been filed for the benefit of his co-owners. Article 487 of the Civil
Code, which provides that any of the co-owners may bring an action for ejectment, covers all kinds of action for the recovery of
possession. In sum, in suits to recover properties, all co-owners are real parties in interest. However, pursuant to Article 487 of the
Civil Code and relevant jurisprudence, any one of them may bring an action, any kind of action, for the recovery of co-owned
properties. Therefore, only one of the co-owners, namely the co-owner who filed the suit for the recovery of the co-owned property, is
an indispensable party thereto. The other co-owners are not indispensable parties. They are not even necessary parties, for a complete
relief can be accorded in the suit even without their participation, since the suit is presumed to have been filed for the benefit of all co-
owners. Under this ruling, either of the spouses Go may bring an action against Navarro to recover possession of the Kargo
Enterprises-leased vehicles which they co-own.
Remedial Law I| Page 54

Landbank vs. Cacayuran


G.R. No. 191667, April 22, 2015

Facts:

The Sangguniang Bayan of the Municipality Agoo through two Resolutions authorized its Mayor to obtain loans from the LBP for the
implementation of a multi-phased plan (Re-development Plan) and construction of a commercial center on the Plaza Lot as part of
Phase II of the said plan. By virtue of such loan, the Municipality mortgaged the southern portion of the Agoo Plaza and authorized
the assignment of a portion of the Internal Revenue Allotment (IRA) in favor LBP. Consequently, respondent filed a suit against
petitioner including the Implicated Officers invoking his right as a taxpayer, after he did not get any response from the Municipality’s
officials on his request for the pertinent documents of the Redeveopment Plan. Respondent questioned the validity of the Subject
Loans on the ground that the Plaza Lot used as collateral thereof is property of public dominion and therefore, beyond the commerce
of man.

Petitioner filed a motion to dismiss, but to no avail. In its Answer, Land Bank claimed that it was not privy to the Implicated Officers’
acts of destroying the Agoo Plaza. It further asserted that Cacayuran did not have a cause of action against it since he was not privy to
any of the Subject Loans.

The RTC declared the Subject Loans null and void, finding that the resolutions approving the procurement of the same were passed in
a highly irregular manner and thus, ultra vires. As such, it pronounced that the Municipality was not bound by the Subject Loans and
that the municipal officers should, instead, be held personally liable for the same. Further, it ruled that since the Plaza Lot is a property
for public use, it cannot be used as collateral for the Subject Loans. The LBP and the municipal officers appealed to the CA. However,
the appeal of the municipal officers was deemed abandoned and dismissed for their failure to file an appellants' brief.

The Appellate court affirmed RTC’s decision with modification. Petitioner filed a Motion for Reconsideration when in the meantime,
the Municipality filed a Motion for Leave to Intervene and a Motion for Reconsideration in Intervention contending that as a
contracting party to the Subject Loans, it is an indispensable party to the action filed by Cacayuran. As such, there cannot be any "real
disposition" of the instant suit by reason of its exclusion from the same. However, Cacayuran insists that the Municipality was not a
real party-in-interest to the instant case as his complaint was against the municipal officers in their personal capacity for their ultra
vires acts which are not binding on the Municipality, hence, this case.

Issue:

Whether or not the Municipality of Agoo is an indispensable party?

Held:

Yes. The Municipality is an indispensable party under Sec 7, Rule 3 of the Rules of Court.

Sec 7, Rule 3 mandates that all indispensable parties are to be joined in a suit as it is the party whose interest will be affected by the
court’s action and without whom no final determination of the case can be had. His legal presence is an absolute necessity. Absence of
the indispensable party renders all subsequent actions of the court null and void for want of authority to act. Failure to implead any
indispensable party is not a ground for the dismissal of the complaint. The proper remedy is to implead them.

In this case, the records reveals that Cacayuran's complaint against LBP and the municipal officers primarily prays that the
commercialization of the Public Plaza be enjoined and also, that the Subject Loans be declared null and void for having been
unlawfully entered into by the said officers. However, Cacayuran failed to implead in his complaint the Municipality, a real party-in-
interest41and an indispensable party that stands to be directly affected by any judicial resolution on the case, considering that: (a) the
contracting parties to the Subject Loans are LBP and the Municipality; and (b) the Municipality owns the Public Plaza as well as the
improvements constructed thereon, including the Agoo People's Center.
Remedial Law I| Page 55

Divinagracia vs. Parilla


G.R. No. 196750, March 11, 2015

Facts:

Conrado Nobleza, Sr. owned a parcel of land in Iloilo City. During his lifetime, he contracted two marriages in which he had two
children in his first marriage and seven in his second in which two of his children predeceased him. Besides that, he also had three
illigitimate children.

According to petitioner (Santiago Divinagracia), upon the decedent’s death, he was able to acquire interests over the subject land
when some of the heirs including Felcon (in representation of his father who predeceased the decedent and his siblings) sold their
respective interests to him as embodied in a Deed of Extrajudicial Settlement or Adjudication with Deed of Sale, which, however, was
not signed by the other heirs who did not sell their respective shares, namely, Ceruleo, Celedonio, and Maude. Subsequently, the same
parties executed a Supplemental Contract whereby the vendors-heirs and Santiago agreed that Santiago would only pay the remaining
balance upon the partition of the subject land. However, Santiago was not able to have the Transfer of Certificate of Title (TCT)
cancelled and the subject document registered because of Ceruleo, Celedonio, and Maude’s refusal to surrender the said title
prompting the filing of a complaint for judicial partition and for receivership.

The RTC found that through the subject document, Santiago became a co-owner of the subject land and, as such, had the right to
demand the partition of the same. However, the RTC held that Santiago did not validly acquire Mateo, Sr.’s share over the subject
land, considering that Felcon admitted the lack of authority to bind his siblings with regard to Mateo, Sr.’s share thereon.

The CA set aside the RTC Rulings and, consequently, dismissed Santiago’s complaint for judicial partition.  It held that Felcon’s
siblings, as well as Maude’s children, are indispensable parties to the judicial partition of the subject land and, thus, their non-
inclusion as defendants in Santiago’s complaint would necessarily result in its dismissal. The heirs of Santiago moved for
reconsideration contending that Santiago had already bought the interests of the majority of the heirs and, thus, they should no longer
be regarded as indispensable parties, but to no avail, hence, this case.

Issue:

Whether or not all the co-owners are indispensable parties in an action for judicial partition?

Held:

Yes. An indispensable party is one whose interest will be affected by the court’s action in the litigation, and without whom no final
determination of the case can be had. The party’s interest in the subject matter of the suit and in the relief sought are so inextricably
intertwined with the other parties’ that his legal presence as a party to the proceeding is an absolute necessity. In his absence, there
cannot be a resolution of the dispute of the parties before the court which is effective, complete, or equitable. Thus, the absence of an
indispensable party renders all subsequent actions of the court null and void, for want of authority to act, not only as to the absent
parties but even as to those present.

With regard to actions for partition, Section 1, Rule 69 of the Rules of Court requires that all persons interested in the property shall be
joined as defendants, viz.:

SEC. 1. Complaint in action for partition of real estate. – A person having the right to compel the partition of real estate may do so as
provided in this Rule, setting forth in his complaint the nature and extent of his title and an adequate description of the real estate of
which partition is demanded and joining as defendants all other persons interested in the property.

Thus, all the co-heirs and persons having an interest in the property are indispensable parties; as such, an action for partition will not
lie without the joinder of the said parties.

In the instant case, however, a reading of Santiago’s complaint shows that as regards Mateo, Sr.’s interest, only Felcon was impleaded,
excluding therefrom his siblings and co-representatives. Similarly, with regard to Cebeleo, Sr.’s interest over the subject land, the
complaint impleaded his wife, Maude, when pursuant to Article 972 of the Civil Code; the proper representatives to his interest should
have been his children, Cebeleo, Jr. and Neobel. Verily, Santiago’s omission of the aforesaid heirs renders his complaint for partition
defective.
Remedial Law I| Page 56

In fine, the absence of the aforementioned indispensable parties in the instant complaint for judicial partition renders all subsequent
actions of the RTC null and void for want of authority to act, not only as to the absent parties, but even as to those present. Therefore,
the CA correctly set aside the November 29, 2002 Decision and the April 4, 2003 Order of the RTC.
Remedial Law I| Page 57

Rule 4: Venue

Pacific Consultants International Asia, Inc. vs. Schonfeld


G.R. No. 166920. February 19, 2007.
Facts:

Klaus Schonfeld, respondent, filed a Complaint for Illegal Dismissal against Pacific Consultants Asia, Inc. (Pacicon Philippines, Inc.)
before the Labor Arbiter. Petitioners filed a Motion to Dismiss the complaint on the ground that venue was improperly laid. It averred
that under the Employment Contract, complainant and Pacific Consultants International of Japan (PCIJ) had agreed that any
employment-related dispute should be brought before the London Court of Arbitration. The provision provides that:

Any question of interpretation, understanding or fulfillment of the conditions of employment, as well as any question arising between
the Employee and the Company which is in consequence of or connected with his employment with the Company and which can not
be settled amicably, is to be finally settled, binding to both parties through written submissions, by the Court of Arbitration in London.

The Labor Arbiter granted the Motion to Dismiss and ruled that since the parties had agreed that any differences regarding employer-
employee relationship should be submitted to the jurisdiction of the court of arbitration in London, this agreement is controlling. The
NLRC affirmed the LA’s decision. The CA reversed the LA and NLRC. It declared that under the employment contract, the parties
were not precluded from bringing a case related thereto in other venues. While there was, indeed, an agreement that issues between the
parties were to be resolved in the London Court of Arbitration, the venue is not exclusive, since there is no stipulation that the
complaint cannot be filed in any other forum other than in the Philippines. Hence, the present petition before the Supreme Court.

Issue:

Whether or not the venue stipulated in the Employment Contract precludes the filing of the case before other venue/forum?

Held:

No. The settled rule on stipulations regarding venue is that while they are considered valid and enforceable, venue stipulations in a
contract do not, as a rule, supersede the general rule set forth in Rule 4 of the Revised Rules of Court in the absence of qualifying or
restrictive words. They should be considered merely as an agreement or additional forum, not as limiting venue to the specified place.
They are not exclusive but, rather permissive. If the intention of the parties were to restrict venue, there must be accompanying
language clearly and categorically expressing their purpose and design that actions between them be litigated only at the place named
by them. In the instant case, no restrictive words like “only,” “solely,” “exclusively in this court,” “in no other court save—,”
“particularly,” “nowhere else but/except—,” or words of equal import were stated in the contract. It cannot be said that the court of
arbitration in London is an exclusive venue to bring forth any complaint arising out of the employment contract.

Note: Petitioner was also insisting on the application of the principle of forum non conveniens. The Court ruled that the mere fact that
respondent is a Canadian citizen and was a repatriate does not warrant the application of the principle because forum non
conveniens is not a ground for dismissal and the propriety of dismissing a case based on this principle requires a factual
determination; hence, it is properly considered as defense.
Remedial Law I| Page 58

Biaco vs. Philippine Countryside Rural Bank


G.R. No. 161417. February 8, 2007.

Topic: Action in personam, in rem and quasi in rem

Facts:

Ernesto Biaco failed to settle its loans (totaling P1,080,676.50) from Philippine Countryside Rural Bank. PCRB filed a complaint for
foreclosure of mortgage against the spouses Ernesto and Teresa Biaco before the RTC of Misamis Oriental. Summons was served to
the spouses Biaco through Ernesto at his office. Ernesto received the summons but for unknown reasons, he failed to file an answer.
Hence, the spouses Biaco were declared in default upon motion of the bank. PCRB was allowed to present its evidence ex parte.
Judgment was rendered in favor of PCRB. Spouses Biaco failed to appeal. A writ of execution was issued and the mortgaged property
was sold at public auction in favor of the PCRB in the amount of P150,000. Since the amount was insufficient, PCRB filed an “ex
parte motion for judgment” praying for the issuance of a writ of execution against the other properties of the spouses Biaco for the full
settlement of the remaining obligation. The trial court granted the motion. Ma. Teresa (the wife) then sought the annulment of the
RTC decision contending that extrinsic fraud prevented her from participating in the judicial foreclosure proceedings. According to
her, she came to know about the judgment in the case only after the lapse of more than six (6) months after its finality. She claimed the
that trial court failed to acquire jurisdiction because summons were served on her through her husband without any explanation as to
why personal service could not be made. The Court of Appeals denied Ma. Teresa’s petition and ruled that judicial foreclosure
proceedings are actions quasi in rem. As such, jurisdiction over the person of the defendant is not essential as long as the court
acquires jurisdiction over the res. Hence, the present petition.

Issue:

Whether or not the trial court acquired jurisdiction over Ma. Teresa?

Held:

No. The question of whether the trial court has jurisdiction depends on the nature of the action, i.e., whether the action is in personam,
in rem, or quasi in rem. The rules on service of summons under Rule 14 of the Rules of Court likewise apply according to the nature
of the action.

An action in personam is an action against a person on the basis of his personal liability. An action in rem is an action against the thing
itself instead of against the person. An action quasi in rem is one wherein an individual is named as defendant and the purpose of the
proceeding is to subject his interest therein to the obligation or lien burdening the property.

In an action in personam, jurisdiction over the person of the defendant is necessary for the court to validly try and decide the case. In a
proceeding in rem or quasi in rem, jurisdiction over the person of the defendant is not a prerequisite to confer jurisdiction on the court
provided that the court acquires jurisdiction over the res. Nonetheless, summons must be served upon the defendant not for the
purpose of vesting the court with jurisdiction but merely for satisfying the due process requirements.

In this case, the judicial foreclosure proceeding instituted by respondent PCRB undoubtedly vested the trial court with jurisdiction
over the res. A judicial foreclosure proceeding is an action quasi in rem. As such, jurisdiction over the person of petitioner is not
required, it being sufficient that the trial court is vested with jurisdiction over the subject matter.

While the trial court acquired jurisdiction over the res, its jurisdiction is limited to a rendition of judgment on the res. It cannot extend
its jurisdiction beyond the res and issue a judgment enforcing petitioner’s personal liability. In granting respondent PCRB’s ex parte
motion for deficiency judgment and ordering the issuance of a writ of execution against the spouses Biaco to satisfy the remaining
balance of the award, without first having acquired jurisdiction over the person of petitioner, as it did, the trial court violated her
constitutional right to due process, warranting the annulment of the judgment rendered in the case.

BPI Family Savings Bank, Inc. vs. Spouses Yujuico


G.R. No. 175796. July 22, 2015.

Topic: Venue; real vs. personal actions

Facts:
Remedial Law I| Page 59

BPI extrajudicially foreclosed the mortgage constituted on the two parcels of land subject of the Spouses Yujuico’s loan. Because
there was a deficiency, BPI sued the spouses to recover such deficiency in the Makati RTC (where the principal office of BPI is
located). The spouses filed a Motion to Dismiss the complaint on several grounds, namely: that the suit was barred by res judicata; that
the complaint stated no cause of action; and that the plaintiff’s claim had been waived, abandoned, or extinguished. Makati RTC
denied the Motion. The spouses filed a Motion for Reconsideration while BPI filed its comment/opposition to the Motion. The
respondents then filed their reply, in which they raised for the first time their objection on the ground of improper venue. They
contended that the action for the recovery of the deficiency, being a supplementary action of the extrajudicial foreclosure proceedings,
was a real action that should have been brought in the Manila RTC because Manila was the place where the properties were located.
Makati RTC denied the MR. The CA reversed the ruling of the RTC opining that a suit for recovery of the deficiency after the
foreclosure of a mortgage is in the nature of a mortgage action because its purpose is precisely to enforce the mortgage contract. As
such, the venue of an action for recovery of deficiency must necessarily be the same venue as that of the extrajudicial foreclosure of
mortgage. Thus, the suit for judgment on the deficiency filed by BPI against the spouses, being an action emanating from the
foreclosure of the real estate mortgage contract between them, must necessarily be filed also at the RTC of Manila, not at the RTC of
Makati. Hence, the present petition.

Issue:

Whether or not the venue was properly laid?

Held:

Yes. It is basic that the venue of an action depends on whether it is a real or a personal action.

Section 1, Rule 4 of the Rules of Court, a real action is one that affects title to or possession of real property, or an interest therein.
Real action is to be commenced and tried in the proper court having jurisdiction over the area wherein the real property involved, or a
portion thereof, is situated, which explains why the action is also referred to as a local action. In contrast, the venue of a personal
action is the place where the plaintiff or any of the principal plaintiffs resides, or where the defendant or any of the principal
defendants resides, or in the case of a nonresident defendant where he may be found, at the election of the plaintiff, for which reason
the action is considered a transitory one.

Based on the distinctions between real and personal actions, an action to recover the deficiency after the extrajudicial foreclosure
of the real property mortgage is a personal action, for it does not affect title to or possession of real property, or any interest
therein. Accordingly, the proper venue in this case is in Makati RTC because Makati was the place where the main office of BPI was
located.

Moreover, even assuming that the venue was improperly laid, it would be improper to dismiss the case considering that the spouses
had not raised such ground in their Motion to Dismiss. As earlier indicated, they came to raise the objection of improper venue for the
first time only in their reply to the petitioner’s comment on their Motion for Reconsideration. They did so belatedly. In civil
proceedings, venue is procedural, not jurisdictional, and may be waived by the defendant if not seasonably raised either in a motion to
dismiss or in the answer. In other words, unless the defendant seasonably objects, any action may be tried by a court despite its being
the improper venue.

Rule 6: Counterclaims

Alba vs. Malapajo


G.R. No. 198752. January 13, 2016

Facts:

Petitioner, Arturo Alba, filed a complaint against respondents, Raymund D. Malapajo, Ramil D. Malapajo and the Register of Deeds
of Roxas City for recovery of ownership and/or declaration of nullity or cancellation of title and damages alleging that the title
Remedial Law I| Page 60

of a parcel of land was transferred to the respondents by virtue of a Deed of Sale that was forged in which the respondents co-
authored.

In its Answer with counterclaim, the respondents contended that they were innocent purchasers for value and that the deed was a
unilateral document which was presented to them already prepared and notarized and that before the sale, petitioner had, on separate
occasions, obtained loans from them and their mother which were secured by separate real estate mortgages covering the subject
property and that the two real estate mortgages had never been discharged.

Petitioner in its Reply to the counterclaim, averred among others that, the court had not acquired jurisdiction over the nature of
respondents' permissive counterclaim, thus, there must be payment of docket fees and filing of a certification against forum shopping,
and, that the supposed loan extended by respondents’ mother to petitioner, must also be dismissed as respondents were not the real
parties-in-interest.

The RTC ruled that the counterclaims were compulsory prompting the petitioner to file a petition for certiorari before the CA. The
CA dismissed the petition for certiorari on technical grounds.

Issue:

Whether or not the respondent’s counterclaim was permissive in nature?

Held:

No. A counterclaim is any claim which a defending party may have against an opposing party. A compulsory counterclaim is one
which, being cognizable by the regular courts of justice, arises out of or is connected with the transaction or occurrence constituting
the subject matter of the opposing party's claim and does not require for its adjudication the presence of third parties of whom the
court cannot acquire jurisdiction. Such a counterclaim must be within the jurisdiction of the court both as to the amount and the nature
thereof, except that in an original action before the Regional Trial Court, necessarily connected with the subject matter of the opposing
party's claim or even where there is such a connection, the Court has no jurisdiction to entertain the claim or it requires for
adjudication the presence of third persons over whom the court acquire jurisdiction. A counterclaim is permissive if it does not arise
out of or is not necessarily connected with the subject matter of the opposing party's claim.

To determine whether a counterclaim is compulsory or permissive, the Court devised the following tests:
(a) Are the issues of fact and law raised by the claim and by the counterclaim largely the same?
(b) Would res judicata bar a subsequent suit on defendants’ claims, absent the compulsory counterclaim rule?;
(c) Will substantially the same evidence support or refute plaintiffs’ claim as well as the defendants’ counterclaim?; and
(d) Is there any logical relation between the claim and the counterclaim?

Here, petitioner seeks to recover the subject property by assailing the validity of the deed of sale on the subject property which he
allegedly executed in favor of respondents Malapajo on the ground of forgery. Respondent’s counterclaimed that, in case the deed of
sale is declared null and void, they be paid the loan petitioner obtained from them plus the agreed monthly interest which was covered
by a real estate mortgage on the subject property executed by petitioner in favor of respondents. There is a logical relationship
between the claim and the counterclaim, as the counterclaim is connected with the transaction or occurrence constituting the
subject matter of the opposing party's claim. Notably, the same evidence to sustain respondents' counterclaim would disprove
petitioner's case. In the event that respondents could convincingly establish that petitioner actually executed the promissory note and
the real estate mortgage over the subject property in their favor then petitioner's complaint might fail. Petitioner's claim is so related
logically to respondents' counterclaim, such that conducting separate trials for the claim and the counterclaim would result in the
substantial duplication of the time and effort of the court and the parties. Since respondents' counterclaim is compulsory, it must be set
up in the same action; otherwise, it would be barred forever. If it is filed concurrently with the main action but in a different
proceeding, it would be abated on the ground of litis pendentia; if filed subsequently, it would meet the same fate on the ground of res
judicata. There is, therefore, no need for respondents to pay docket fees and to file a certification against forum shopping for the court
to acquire jurisdiction over the said counterclaim.
Remedial Law I| Page 61

Lim Teck Chuan vs. Uy


G.R. No. 155701, March 11, 2015

Facts:

Antonio Lim Tanhu was the original owner of the lot which is the subject matter of the controversy. Allegedly, he sold the lot to
Spouses Cabansag; then Spouses Cabansag sold the same to Serafin Uy (respondent). Serafin then filed a petition before the RTC
praying for the issuance of a new owner’s duplicate TCT in his name. The petition was initially granted but subsequently nullified
because Lim Teck Chuan (petitioner) filed his Opposition alleging that he is one of the 6 legitimate descendants of Antonio and that
the original TCT was not lost and has always been in his custody. In the meantime, a certain Henry Lim sold the same lot to Leopolda
Cecilio by virtue of an Affidavit of Sole Adjudication/Settlement of the Estate of Antonio Lim Tanhu with Deed of Sale.

Serafin then filed a Complaint for Quieting of Title impleading Leopolda, Henry and the petitioner. Leopolda averred that she is a
buyer in good faith and for value. Petitioner set up a counterclaim against Serafin and a cross-claim against Lopeolda contending
that the property was never transferred and encumbered to any person during Antonio’s lifetime.

During the proceedings, both and Serafin and Leopolda entered into an amicable settlement and they both filed a Joint Motion to
Dismiss (MTD) on the main ground that the case had become moot and academic since Serafin’s title to the subject lot had been
allegedly quieted. Petitioner opposed the MTD on the ground that he was not included in the settlement. The RTC, however, granted
the MTD and it also dismissed petitioner’s counterclaim and cross-claim.

Petitioner filed directly with the SC a petition for review under Rule 45.

Issue:

Whether or not petitioner’s counterclaim or cross-claim could be prosecuted in the same action despite the dismissal of the main
complaint?

Held:

Yes. Section 2 of Rule 17 provides that:

xxx If a counterclaim has been pleaded by a defendant prior to the service upon him of the plaintiff’s motion for dismissal, the
dismissal shall be limited to the complaint. The dismissal shall be without prejudice to the right of the defendant to prosecute his
counterclaim in a separate action unless within fifteen (15) days from notice of the motion he manifests his preference to have his
counterclaim resolved in the same action….

The RTC erred when it dismissed the case when the present rules state that the dismissal shall be limited only to the complaint. A
dismissal of an action is different from a mere dismissal of the complaint. For this reason, since only the complaint and not the action
is dismissed, the defendant in spite of said dismissal may still prosecute his counterclaim in the same action. Citing Pinga v. Heirs of
German Santiago, the Court said that the dismissal of the complaint does not necessarily result to the dismissal of the counterclaim.

In the instant case, the petitioner’s preference to have his counterclaim (and cross-claims) be prosecuted in the same action was timely
manifested.
Remedial Law I| Page 62

Metrobank vs. CPR Promotions


G.R. No. 200567, June 22, 2015

Facts:

Because the respondents’ defaulted in their loan obligation to petitioner (MBTC), MBTC filed a petition for extra-judicial foreclosure
of the real estate mortgages securing the loan obligations. According to MBTC, despite the foreclosure sale, there remained a
deficiency balance of PhP2,628,520.73, plus interest and charges as stipulated and agreed upon in the promissory notes and deeds of
real estate mortgages. Despite petitioner’s repeated demands, however, respondents failed to settle the alleged deficiency. Thus,
petitioner filed an action for collection of sum of money against respondents before the RTC.

The RTC ruled in favor of petitioner that there, indeed, was a balance, and that respondents were liable for the said amount, as part of
their contractual obligation. It also denied the Motion for Reconsideration, prompting for petitioner to file an appeal. The CA reversed
the court a quo and ruled in favor of respondents and ordered that Metrobank to refund or return to the defendants-appellants the
amount representing the remainder of the proceeds of the foreclosure sale (because there was overpayment). Petitioner filed a Motion
for Reconsideration asserting among others that respondents never set up a counterclaim for refund of any amount, but the same was
denied, hence, this case.

Issue:

Whether or not the CA erred in ordering a refund to the respondent despite their failure to set it up as a counterclaim?

Held:

Yes. Rule 6 of the Rules of Court define a compulsory counterclaim as follows:

Section 7. Compulsory counterclaim. – A compulsory counterclaim is one which, being cognizable by the regular courts of justice,
arises out of or is connected with the transaction or occurrence constituting the subject matter of the opposing party’s claim and does
not require for its adjudication the presence of third parties of whom the court cannot acquire jurisdiction. Such a counterclaim must
be within the jurisdiction of the court both as to the amount and the nature thereof. Except that in an original action before the
Regional Trial Court, the counterclaim may be considered compulsory regardless of the amount. A compulsory counterclaim is
barred if not set up in the same action.

The court found out that such refund could have also been a compulsory counterclaim. However, it is elementary that a defending
party’s compulsory counterclaim should be interposed at the time he files his Answer, and that failure to do so shall effectively bar
such claim. As it appears from the records, what respondents initially claimed herein were moral and exemplary damages, as well as
attorney’s fees. Then, realizing, based on its computation, that it should have sought the recovery of the excess bid price, respondents
set up another counterclaim, this time in their Appellant’s Brief filed before the CA. Unfortunately, respondents’ belated assertion
proved fatal to their cause as it did not cure their failure to timely raise such claim in their Answer. Consequently, respondents’ claim
for the excess, if any, is already barred.
Remedial Law I| Page 63

Rule 6: Answers

Valdez vs. Dabon


A.C. No. 7353. November 16, 2015

Facts:

Complainant Nelson charged respondent Atty. Dabon, a Division Clerk of Court of the Court of Appeals (CA), with gross immorality
for allegedly carrying on an adulterous relationship with his wife, Sonia Romero Valdez, which was made possible by sexual assaults
and maintained through threat and intimidation. Respondent Atty. Dabon strongly refuted the accusation against him claiming that the
same was baseless and unfounded and that the complaint for disbarment was merely calculated to harass, annoy and besmirch his
reputation.

In his Comment, Atty. Dabon denied the charges of grossly immoral and unlawful acts through sexual assaults, abuses, threats and
intimidation. He posited that the allegations of spouses Nelson and Sonia in their respective affidavits were nothing but pure
fabrication solely intended to malign his name and honor.

The Court referred the case to the Integrated Bar of the Philippines (IBP) for investigation, report and recommendation. After the
parties had submitted their respective verified position papers, Investigating Commissioner of the IBP Commission on Bar
Discipline (IBP-CBD) rendered his Report and Recommendation, finding that the charge against respondent Atty. Dabon had been
sufficiently proven. The Board of Governors of the IBP adopted and approved the recommendation. Atty. Dabon filed a motion for
reconsideratio but it was denied by the IBP Board of Governors.

Issue:

Whether or not Atty. Dabon’s denial of the complaint against him constitutes negative pregnant?

Held:

Yes. Atty. Dabon interposed a blanket denial of the romantic involvement but at the same time, he seemed to have tacitly admitted the
illicit affair only that it was not attended by sexual assaults, threats and intimidations. The Court also observed that he devoted
considerable effort to demonstrate that the affair did not amount to gross immoral conduct and that no sexual abuse, threat or
intimidation was exerted upon the person of Sonia, but not once did he squarely deny the affair itself.

In other words, the respondent's denial is a negative pregnant, a denial coupled with the admission of substantial facts in the
pleading responded to which are not squarely denied. Stated otherwise, a negative pregnant is a form of negative expression which
carries with it an affirmation or at least an implication of some kind favorable to the adverse party. Where a fact is alleged with
qualifying or modifying language and the words of the allegation as so qualified or modified are literally denied, it has been held that
the qualifying circumstance alone is denied while the fact itself is admitted. It is clear from Atty. Dabon's Comment that his denial
only pertained as to the existence of a forced illicit relationship. Without a categorical denial thereof, he is deemed to have admitted
his consensual affair with Sonia.
Remedial Law I| Page 64

Republic vs. Sandiganbayan


G.R. No. 152154 July 15, 2003

Facts:

Petitioner Republic, through the Presidential Commission on Good Government (PCGG), represented by the Office of the Solicitor
General (OSG), filed a petition for forfeiture before the Sandiganbayan, entitled Republic of the Philippines vs. Ferdinand E. Marcos,
represented by his Estate/Heirs and Imelda R. Marcos.

In said case, petitioner sought the declaration of the aggregate amount of US$356 million deposited in escrow in the PNB, as ill-gotten
wealth. Petitioner filed a motion for summary judgment and/or judgment on the pleadings. Respondent Mrs. Marcos filed her
opposition thereto which was later adopted by respondents Mrs. Manotoc, Mrs. Araneta and Ferdinand, Jr.

In their Answer, particularly, in paragraph 22, they stated that:

22. Respondents specifically DENY paragraph 23 insofar as it alleges that Respondents clandestinely stashed the country’s wealth
in Switzerland and hid the same under layers and layers of foundations and corporate entities for being false, the truth being that
Respondents’ aforesaid properties were lawfully acquired.

Issue:
Whether or not paragraph 22 of the Answer constitutes negative pregnant?
Held:
Yes. Evidently, this particular denial had the earmark of what is called in the law on pleadings as a negative pregnant, that is, a denial
pregnant with the admission of the substantial facts in the pleading responded to which are not squarely denied.  It was in effect an
admission of the averments it was directed at. Stated otherwise, a negative pregnant is a form of negative expression which carries
with it an affirmation or at least an implication of some kind favorable to the adverse party. It is a denial pregnant with an admission
of the substantial facts alleged in the pleading. Where a fact is alleged with qualifying or modifying language and the words of the
allegation as so qualified or modified are literally denied, has been held that the qualifying circumstances alone are denied while the
fact itself is admitted.
In the instant case, the material allegations in paragraph 23 of the said petition were not specifically denied by respondents in
paragraph 22 of their answer. The denial contained in paragraph 22 of the answer was focused on the averment in paragraph 23 of the
petition for forfeiture that Respondents clandestinely stashed the country’s wealth in Switzerland and hid the same under layers and
layers of foundations and corporate entities.
Therefore, the allegations in the petition for forfeiture on the existence of the Swiss bank deposits in the sum of about US$356 million,
not having been specifically denied by respondents in their answer, were deemed admitted by them pursuant to Section 11, Rule 8 of
the 1997 Revised Rules on Civil Procedure.
Remedial Law I| Page 65

Caneland Sugar Corp. vs. Alon


G.R. No. 142896 September 12, 2007

Facts:

LBP foreclosed the mortgaged properties belonging to Caneland Sugar Corporation. There was another auction sale scheduled when
Caneland filed with the RTC a complaint for damages, injunction, and nullity of mortgage. RTC issued an Order holding in abeyance
the auction sale but it was rescheduled pursuant to PD 385. P.D. 385 provides that it shall be mandatory for government financial
institution to foreclose collaterals and/or securities for any loan, credit accommodations and/or guarantees granted by them whenever
the arrearages on such account, including accrued interest and other charges amount to at least 20% of the total outstanding obligation
as appearing in the books of the financial institution. Moreover, no restraining order, temporary or permanent injunction shall be
issued by the court against any government financial institution in any action taken by such institution in compliance with the
mandatory foreclosure provided by said law.

Before the CA, petitioner’s petition was also denied. Hence, this petition. Petitioner contends in the main that the RTC’s act of
authorizing the foreclosure of its property amounts to a prejudgment of the case since it amounts to a ruling that respondent has a valid
mortgage in its favor. Petitioner also argues, among others, that Presidential Decree (P.D.) No. 385 is not applicable inasmuch as at
the time of the lease to Sunnix, Inc., the management and control of its operations has already been virtually taken over by respondent.
On the other hand, respondent maintains that P.D. No. 385 prohibits the issuance of an injunctive order against government financial
institutions; the CA did not commit any grave abuse of discretion; the RTC Order merely dealt with the propriety of the injunctive
order and not the validity of the mortgage; and the issue of the propriety of the injunctive order has been rendered moot and academic
by the foreclosure sale conducted and the issuance of a certificate of sale by the sheriff.

Issue:
Whether or not the RTC was correct in finding the mortgage valid and not enjoining the foreclosure sale?
Held:
Yes. Petitioner does not dispute its loan obligation with respondent. Petitioner’s bone of contention before the RTC is that the
promissory notes are silent as to whether they were covered by the Mortgage Trust Indenture and Mortgage Participation on its
property covered by TCT No. T-11292. It does not categorically deny that these promissory notes are covered by the security
documents. These vague assertions are, in fact, negative pregnants, i.e., denials pregnant with the admission of the substantial
facts in the pleading responded to which are not squarely denied.

As defined in Republic of the Philippines v. Sandiganbayan, a negative pregnant is a "form of negative expression which carries with
it an affirmation or at least an implication of some kind favorable to the adverse party. It is a denial pregnant with an admission of the
substantial facts alleged in the pleading. Where a fact is alleged with qualifying or modifying language and the words of the allegation
as so qualified or modified are literally denied, has been held that the qualifying circumstances alone are denied while the fact itself is
admitted."

Petitioner’s allegations do not make out any justifiable basis for the granting of any injunctive relief. Even when the mortgagors were
disputing the amount being sought from them, upon the non-payment of the loan, which was secured by the mortgage, the mortgaged
property is properly subject to a foreclosure sale. This is in consonance with the doctrine that to authorize a temporary injunction, the
plaintiff must show, at least prima facie, a right to the final relief.
Remedial Law I| Page 66

Rule 7: Parts of a Pleading

Alma Jose vs. Javellana


G.R. No. 158239, January 25, 2012

Facts:

Margarita Marquez Alma Jose sold to respondent Ramon Javellana by deed of conditional sale two parcels of land. They agreed that
Javellana would pay P80,000.00 upon the execution of the deed and the balance of P80,000.00 upon the registration of the parcels of
land under the Torrens System (the registration being undertaken by Margarita within a reasonable period of time); and that should
Margarita become incapacitated, her son and attorney-in-fact, Juvenal M. Alma Jose (Juvenal), and her daughter, petitioner Priscilla
M. Alma Jose, would receive the payment of the balance and proceed with the application for registration. When Margarita and
Juvenal died, Priscilla did not comply with the undertaking to cause the registration of the properties under the Torrens System, and,
instead, began to improve the properties by dumping filling materials therein with the intention of converting the parcels of land into a
residential or industrial subdivision. Javellana then filed an action for specific performance, injunction, and damages against her in
the Regional Trial Court. Petitioner filed a motion to dismiss, stating that the complaint was already barred by prescription; and that
the complaint did not state a cause of action. The RTC denied said motion to dismiss, but subsequently in the MR, reversed its own
decision, and the complaint was dismissed. Javellana filed a MR which was denied by the RTC. Javellana filed a Notice of Appeal. It
appears that pending the appeal, Javellana also filed a petition for certiorari in the CA. The CA dismissed the certiorari petition finding
that the RTC did not commit grave abuse of discretion in issuing the orders, and holding that it only committed, at most, an error of
judgment correctible by appeal in issuing the challenged orders.

The CA in its decision on the appeal, reversed the RTC’s decision, it ruled that, the complaint sufficiently stated a cause of action; that
Priscilla, as sole heir, succeeded to the rights and obligations of Margarita with respect to the parcels of land; that Margaritas
undertaking under the contract was not a purely personal obligation but was transmissible to Priscilla, who was consequently bound to
comply with the obligation; that the action had not yet prescribed due to its being actually one for quieting of title that was
imprescriptible brought by Javellana who had actual possession of the properties. MR was also denied. Hence this petition.

Issue:

Whether or not Javellana committed forum shopping for filing in the CA a petition for certiorari to assail the orders of the RTC that
were the subject matter of their appeal pending in the CA?

Held:

No. Forum shopping happens when, in the two or more pending cases, there is identity of parties, identity of rights or causes of action,
and identity of reliefs sought. Where the elements of litis pendentia are present, and where a final judgment in one case will amount
to res judicata in the other, there is forum shopping. For litis pendentia to be a ground for the dismissal of an action, there must be: (a)
identity of the parties or at least such as to represent the same interest in both actions; (b) identity of rights asserted and relief prayed
for, the relief being founded on the same acts; and (c) the identity in the two cases should be such that the judgment which may be
rendered in one would, regardless of which party is successful, amount to res judicata in the other. For forum shopping to exist, both
actions must involve the same transaction, same essential facts and circumstances and must raise identical causes of action, subject
matter and issues. Clearly, it does not exist where different orders were questioned, two distinct causes of action and issues were
raised, and two objectives were sought. 

The appeal and the petition for certiorari actually sought different objectives. In his appeal in C.A.-G.R. CV No. 68259,
Javellana aimed to undo the RTCs erroneous dismissal of Civil Case No. 79-M-97 to clear the way for his judicial demand for
specific performance to be tried and determined in due course by the RTC; but his petition for certiorari had the ostensible
objective to prevent (Priscilla) from developing the subject property and from proceeding with the ejectment case until his
appeal is finally resolved, as the CA explicitly determined in its decision in C.A.-G.R. SP No. 60455. Nor were the dangers that
the adoption of the judicial policy against forum shopping designed to prevent or to eliminate attendant. The first danger,  i.e., the
multiplicity of suits upon one and the same cause of action, would not materialize considering that the appeal was a continuity of Civil
Case No. 79-M-97, whereas C.A.-G.R. SP No. 60455 dealt with an independent ground of alleged grave abuse of discretion
amounting to lack or excess of jurisdiction on the part of the RTC. The second danger, i.e., the unethical malpractice of shopping for a
friendly court or judge to ensure a favorable ruling or judgment after not getting it in the appeal, would not arise because the CA had
not yet decided C.A.-G.R. CV No. 68259 as of the filing of the petition for certiorari. Instead, the court sees the situation of resorting
to two inconsistent remedial approaches to be the result of the tactical misjudgment by Javellana’s counsel on the efficacy of the
appeal to stave off his caretaker’s eviction from the parcels of land and to prevent the development of them into a residential or
Remedial Law I| Page 67

commercial subdivision pending the appeal. In the petition for certiorari, Javellana explicitly averred that his appeal was inadequate
and not speedy to prevent private respondent Alma Jose and her transferee/assignee xxx from developing and disposing of the subject
property to other parties to the total deprivation of petitioners rights of possession and ownership over the subject property, and that
the dismissal by the RTC had emboldened private respondents to fully develop the property and for respondent Alma Jose to file an
ejectment case against petitioners overseer xxx. Thereby, it became far-fetched that Javellana brought the petition for certiorari in
violation of the policy against forum shopping.
Remedial Law I| Page 68

Medado vs. Heirs of Consing


G.R. No. 186720, February 8, 2012

Facts:

Petitioner Spouses Medado and the estate of the late Antonio Consing (Estate of Consing), as represented by Soledad Consing
(Soledad), executed Deeds of Sale with Assumption of Mortgage for the former's acquisition from the latter of the property
in Cadiz City identified as Hacienda Sol. As part of the deal, Spouses Medado undertook to assume the estate's loan with Philippine
National Bank (PNB). Subsequent to the sale, however, the Estate of Consing offered the subject lots to the government  via the
Department of Agrarian Reform's Voluntary Offer to Sell (VOS) program.

The Estate of Consing instituted with the RTC, Branch 44 of Bacolod City an action for rescission and damages, against Spouses
Medado, PNB and the Register of Deeds of Cadiz City, due to the alleged failure of the spouses to meet the conditions in their
agreement. In the meantime that Civil Case No. 00-11320 for rescission was pending, Land Bank of the Philippines (LBP) issued in
favor of the Estate of Consing a certificate of deposit of cash and agrarian reform bonds, as compensation for the lots covered by the
VOS. Spouses Medado feared that LBP would release the full proceeds thereof to the Estate of Consing. It prompted Spouses Medado
to file an action for injunction with prayer for the issuance of a temporary restraining order, with the RTC. They asked that the
following be issued by the trial court: (a) writ of prohibitory injunction to restrain LBP from releasing the remaining amount of the
VOS proceeds of the lots offered by the Estate of Consing, and restraining the Estate of Consing from receiving these proceeds; and
(b) writ of mandatory injunction to compel LBP to release the remaining amount of the VOS to the spouses. The RTC granted said
petition. Feeling aggrieved, the heirs of the late Antonio Consing questioned the RTC's order via a petition for certiorari filed with the
CA. They sought, among other reliefs, the dismissal of the complaint for injunction for violation of the rules on  litis pendetia. The CA
reversed the decision of the RTC. The MR was denied as well. Hence this petition. The petitioner contended that the consolidated
verification and certification against forum shopping of the petition filed with the CA was defective: first, for being signed only by
Soledad, instead of by all the petitioners, and second, its jurat cites a mere community tax certificate of Soledad, instead of a
government-issued identification card required under the 2004 Rules on Notarial Practice.

Issues:

(1) Whether or not Spouses Medado engaged in forum shopping when they filed an action for injunction despite the pendency of the
action for rescission?

(2) Whether or not the requirement for verification and certification against forum shopping have been substantially complied?

Held:

(1) Yes. There is forum shopping when the elements of litis pendencia are present, i.e., between actions pending before the courts,
there exist: (1) identity of parties, or at least such parties as represent the same interests in both actions; (2) identity of rights asserted
and relief prayed for, the relief being founded on the same facts; (3) and the identity of the two proceeding particulars is such that
judgment rendered in the other action will regardless of which party is successful, amount to res judicata in the action under
consideration; said requisites are also constitutive of the requisites for auter action pendant or lis pendens. Applying the foregoing,
there was clearly a violation of the rule against non-forum shopping when Spouses Medado instituted Civil Case No. 797-C for
injunction notwithstanding the pendency of Civil Case No. 00-11320 for rescission of contract and damages.

All elements of litis pendentia are present with the filing of the two cases. There is no dispute that there is identity of parties
representing the same interests in the two actions, both involving the estate and heirs of the late Consing on one hand, and Spouses
Medado on the other. The two other elements are likewise satisfied. There is an identity of rights asserted and reliefs prayed for in the
two cases, with the reliefs being founded on the same set of facts. In both cases, the parties claim their supposed right as owners of the
subject properties. They all anchor their claim of ownership on the deeds of absolute sale which they had executed, and the law
applicable thereto. They assert their respective rights, with Spouses Medado as buyers and the heirs as sellers, based on the same set of
facts that involve the deeds of sale’s contents and their validity. Both actions necessarily involve a ruling on the validity of the same
contract as against the same parties. Thus, the identity of the two cases is such as would render the decision in the rescission case res
judicata in the injunction case, and vice versa.
Remedial Law I| Page 69

(2) Yes. Records show that Soledad signed the verification and certification against forum shopping on behalf of her co-petitioners by
virtue of a Special Power of Attorney (SPA) attached to the petition filed with the CA. The SPA, signed by her co-heirs Ma. Josefa
Consing Saguitguit, Ma. Carmela Consing Lopez, Ma. Lourdes Consing Gonzales and Mary Rose Consing Tuason, provides that their
attorney-in-fact Soledad is authorized. The authority of Soledad includes the filing of an appeal before the CA, including the execution
of a verification and certification against forum shopping therefor, being acts necessary to protect, sue, prosecute, defend and adopt
whatever action necessary and proper in relation to their rights over the subject properties. In addition, the allegations and contentions
embodied in the CA petition do not deviate from the claims already made by the heirs in Civil Case Nos. 00-11320 and 797-C, both
specifically mentioned in the SPA.

In any case, the Court reiterate that where the petitioners are immediate relatives, who share a common interest in the property subject
of the action, the fact that only one of the petitioners executed the verification or certification of forum shopping will not deter the
court from proceeding with the action.
Remedial Law I| Page 70

Commission on Appointment vs. Paler


G.R. No. 172623, March 3, 2010

Facts:

Respondent Celso M. Paler was a Supervising Legislative Staff Officer II (SG-24) with the Technical Support Service of the
Commission on Appointments. Since he already had an approved leave from June 9 to July 30, 2003, Paler left for the United States
without verifying whether his application for leave (for August 1 November 4, 2003) was approved or denied. The Commission
Chairman informed Paler that he was being dropped from the roll of employees due to his continuous 30-day absence without leave
and in accordance with a CSC Memorandum Order. Paler moved for reconsideration but this was denied on the ground that it was
filed beyond the 15-day reglementary period. On appeal, the CSC reversed and set aside the Commission Chairman's decision. The
Commission filed a motion for reconsideration but this was denied by the CSC. This constrained petitioner to file with the CA a
petition for review under Rule 43. The CA affirmed the decision of the CSC. Hence, this petition. In his comment, Paler, aside from
arguing that the CA did not commit any error in sustaining the CSC resolutions, also assails Atty. Arturo L. Tiu's authority to file the
petition and sign the verification and certification of non-forum shopping on behalf of the Commission Chairman.

Issue:

Whether or not Atty. Arturo Tiu has the authority to sign the verification and certification of non-forum shopping?

Held:

As to the verification – YES. As to the certification against forum shopping – NO.

The petitioner in this case is the Commission on Appointments, a government entity created by the Constitution, and headed by its
Chairman. There was no need for the Chairman himself to sign the verification.  Its representative, lawyer or any person who
personally knew the truth of the facts alleged in the petition could sign the verification. 

With regard, however, to the certification of non-forum shopping, the established rule is that it must be executed by the
plaintiff or any of the principal parties and not by counsel. In this case, Atty. Tiu failed to show that he was specifically authorized
by the Chairman to sign the certification of non-forum shopping, much less file the petition in his behalf. There is nothing on record to
prove such authority. Atty. Tiu did not even bother to controvert Palers allegation of his lack of authority. This renders the petition
dismissible.
 
Remedial Law I| Page 71

Basan vs. Coca-Cola Bottlers, Phils.


G.R. Nos. 174365-66, February 4, 2015

Facts:

Petitioners Romeo Basan et al. filed a complaint for illegal dismissal and money claims against Coca-Cola Bottlers Philippines
alleging that the respondent dismissed them without just cause and prior written notice. Coca-Cola, on the other hand, argues that it
hired petitioners as temporary route helpers for a period in anticipation of the volume of work in their plants or sale offices. The Labor
Arbiter ruled in favor of petitioner finding that they are performing activities necessary and desirable to the usual business of the
petitioner for more than the period for regularization. The NLRC affirmed the Labor Arbiter’s decision only to be reversed by the
Court of Appeals on the ground that Basan, et al. were not regular employees. Basan et al. filed a petition for review before the
Supreme Court. Coca-Cola contends that the petition should be denied since the verification and forum shopping was signed by only
one of the petitioners.

Issue:

Whether or not the petition should fail on the ground that its verification and non-forum shopping has been signed by only one of the
petitioners?

Held:

No. On the procedural issue, we hold that while the general rule is that the verification and certification of non-forum shopping must
be signed by all the petitioners in a case, the signature of only one of them, petitioner Basan in this case, appearing thereon may be
deemed substantial compliance with the procedural requirement. Jurisprudence is replete with rulings that the rule on verification is
deemed substantially complied with when one who has ample knowledge to swear to the truth of the allegations in the complaint or
petition signs the verification, and when matters alleged in the petition have been made in good faith or are true and correct. Similarly,
this Court has consistently held that when under reasonable or justifiable circumstances, as when all the petitioners share a common
interest and invoke a common cause of action or defense, as in this case, the signature of only one of them in the certification
against forum shopping substantially complies with the certification requirement. Indeed, the application of technical rules of
procedure may be relaxed in labor cases to serve the demand of justice.
Remedial Law I| Page 72

Uy vs. Court of Appeals


G.R. No. 173186, September 16, 2015.

Facts:

Aniceto Uy previously obtained a favorable judgment for the recovery of possession of the subject land against Naval-Sai. In July
1999, Naval-Sai filed a complaint for Annulment of Deed with Damages before the same court against petitioner. Naval-Sai argued
that she never sold the lots and that her signature in the purported deed of sale is spurious. Naval-Sai filed an Amended Complaint
dated July 29, 1999. She asserted that the subject TCTs were already cancelled by virtue of the deed of sale. Unlike the original
complaint, however, the Amended Complaint was not signed by Naval-Sai, but by her counsel.

In his answer with counterclaim, petitioner raised the noncompliance with the requisite certification of non-forum shopping and
prescription. He asserted that jurisdiction has never been acquired over the parties and the subject matter because the certification
against forum shopping in the Amended Complaint was defective, for having been merely signed by Naval-Sai's counsel. He further
claimed that the action for annulment of deed of sale is already barred by the statute of limitations and that Naval-Sai is guilty of
estoppel and laches.

Issue:

Whether or not there was compliance with the requirements on certification for non-forum shopping?

Held:

Yes. A certification against forum shopping is a peculiar and personal responsibility of the party, an assurance given to the court or
other tribunal that there are no other pending cases involving basically the same parties, issues and causes of action. It must be
executed by the party-pleader, not by his counsel. If, however, for reasonable or justifiable reasons, the party-pleader is unable to sign,
he must execute a Special Power of Attorney (SPA) designating his counsel of record to sign on his behalf.

Here, the original complaint contained a proper verification and certification against forum shopping duly signed by Naval-Sai as
plaintiff. The verification and certification in the amended complaint, on the other hand, was only signed by her counsel, Atty.
Norberta L. Ela. Atty. Ela was not authorized to sign on behalf of Naval-Sai, as in fact, she assigned one Rodolfo Florentino as agent.
The Court of Appeals pointed out that in the certification in the amended complaint, Atty. Ela specified that it should be taken and
read together with the original complaint.

The Court of Appeals took this as a cautionary move on the part of Naval-Sai, justifying the relaxation of the rules on the ground of
substantial compliance. This cautionary move is ineffectual because under
the Rules of Civil Procedure, an amended complaint supersedes the original complaint. For all intents and purposes, therefore, the
original complaint and its verification and certification ceased to exist.

This, notwithstanding, there was still substantial compliance with the Rules. The general rule is that non-compliance or a defect in the
certification is not curable by its subsequent submission or correction. However, there are cases where we exercised leniency and
relaxed the rules on the ground of substantial compliance, the presence of special circumstances or compelling reasons. The rules on
forum-shopping are designed to promote and facilitate the orderly administration of justice and "should not be interpreted with such
absolute literalness as to subvert its own ultimate and legitimate objective or the goal of all rules of procedure which is to achieve
substantial justice as expeditiously as possible.
Remedial Law I| Page 73

Bandillon vs. La Filipina Uygongco Corporation (LFUC)


G.R. No. 202446. September 16, 2015

Facts:

Petitioners Eduardo Bandillon, et al. (employees) are truck drivers and employees of respondent La Filipina Uygongco Corporation
(LFUC). They filed a complaint for violation of labor standard laws against the latter before the DOLE Region VI. This case reached
the Supreme Court and the employees won the case.

Consequently, the employees filed a Motion for Execution before the DOLE Region VI to enforce the DOLE Secretary's Order of
June 4, 2003. After being served with the writ, LFUC filed a Petition for certiorari and injunction dated August 15, 2008 with the
Court of Appeals, seeking to set aside the writ of execution.

Meanwhile, the DOLE Regional Director had already issued an Order directing LFUC to pay the total amount of Php3,345,657.94 to
the employees. When LFUC received the Order, it filedDOLE Region VI a Motion for Reconsideration. This MR was, however,
denied.

In 2011, the CA rendered its decision remanding the case to DOLE VI Regional Director for re-computation of awards and reception
of evidence of the parties on the ground that that the office of DRD arrived at its computation without any evidence from the parties.

Before the Supreme Court, LFUC argued that the petition must fail alleging that several of the concerned employees did not sign the
Special Power of Attorney (SPA) authorizing their union president (Payda) to file this petition, and to sign the verification and
certification against forum shopping for such purpose.

Issue:

(1) Whether or not the petition must fail on the ground that several employees concerned did not sign the SPA authorizing their union
president to sign the verification and certification against forum shopping?

(2) Whether or not LFUC committed forum shopping by filing a petition for certiorari with the CA and subsequently filing a Motion
for Reconsideration of the DOLE-Regional Director’s writ of execution?

Held:

(1) No. According to prevailing jurisprudence, neither the fact that the union president alone signed the petition's verification and
certification against forum shopping, nor the fact that the SPA authorizing the filing of the petition was not signed by all petitioners,
invalidate nor render the petition defective, as the present case is one of those instances when the rules are interpreted more liberally in
order to attain substantial justice. Payda’s lone signature and the SPA signed by most of the petitioners already substantially comply
with the requirements for a properly and validly filed petition.

Indeed, Payda alone signed the verification and certification against forum shopping - as the person authorized in the SPA to do so -
but instead of rendering the petition defective or invalid, this Court, as it has previously ruled regards the same as already in
substantial compliance with the rule.

Rules on non-compliance with the requirements on, or submission of defective, verification and certification against forum
shopping:

1) A distinction must be made between non-compliance with the requirement on or submission of defective verification, and non-
compliance with the requirement on or submission of defective certification against forum shopping.
2) As to verification, non-compliance therewith or a defect therein does not necessarily render the pleading fatally defective. The court
may order its submission or correction or act on the pleading if the attending circumstances are such that strict compliance with the
Rule may be dispensed with in order that the ends of justice may be served thereby.
3) Verification is deemed substantially complied with when one who has ample knowledge to swear to the truth of the allegations in
the complaint or petition signs the verification, and when matters alleged in the petition have been made in good faith or are true and
correct.

4) As to certification against forum shopping, non-compliance therewith or a defect therein, unlike in verification, is generally not
curable by its subsequent submission or correction thereof, unless there is a need to relax the Rule on the ground of "substantial
compliance" or presence of "special circumstances or compelling reasons."
Remedial Law I| Page 74

5) The certification against forum shopping must be signed by all the plaintiffs or petitioners in a case; otherwise, those who did not
sign will be dropped as parties to the case. Under reasonable or justifiable circumstances, however, as when all the plaintiffs or
petitioners share a common interest and invoke a common cause of action or defense, the signature of only one of them in the
certification against forum shopping substantially complies with the Rule.
6) Finally, the certification against forum shopping must be executed by the party-pleader, not by his counsel. If, however, for
reasonable or justifiable reasons, the party- pleader is unable to sign, he must execute a Special Power of Attorney designating his
counsel of record to sign on his behalf.

(2) Yes. LFUC’s filing of a motion for reconsideration before the DOLE-VI Regional Director rendered moot and academic its
petition for certiorari then pending with the Court of Appeals; as such, LFUC’s failure to withdraw the petition or to even notify the
appellate court of the motion for reconsideration filed before the DOLE amounts to a violation of the rules against forum shopping.

There is no question that as a result of LFUC’s pursuit of the two simultaneous remedies, the rulings of the Court of Appeals on the
petition for certiorari and the DOLE Secretary on LFUC’s motion for reconsideration are now essentially conflicting, as the former
bars any execution and instead directs a further hearing of certain evidence, while the latter states that such evidence had the chance to
be heard and execution should now proceed as a matter of course. Such conflict is exactly the scenario that the rules against forum
shopping try to avert.
Remedial Law I| Page 75

People vs. Arojado


G.R. No. 207041. November 9, 2015

Facts:

In an Information dated March 23, 2009, Jesus Arojado was charged with the crime of murder by the Office of the City Prosecutor of
Roxas City, Capiz. The case was docketed as Criminal Case No. C7509 and was raffled off to Branch 16 of the Regional Trial Court
of Roxas City, Iloilo (RTC of Roxas City). Arojado filed a Motion to Dismiss the Information filed against him on the ground that the
investigating prosecutor who filed the said Information failed to indicate therein the number and date of issue of her Mandatory
Continuing Legal Education (MCLE) Certificate of Compliance, as required by Bar Matter No.1922 (B.M. No. 1922).

On the other hand, the petitioner contended that: (1) the Information sought to be dismissed is sufficient in form and substance; (2) the
lack of proof of MCLE compliance by the prosecutor who prepared and signed the Information should not prejudice the interest of the
State in filing charges against persons who have violated the law; and (3) an administrative edict cannot prevail over substantive or
procedural law, by imposing additional requirements for the sufficiency of a criminal information.

The RTC of Roxas City, RTC of Roxas City issued an Order dismissing the subject Information without prejudice. Respondent filed a
Motion for Reconsideration, but the trial court denied it. In its presently assailed Decision, the CA denied respondent’s petition and
affirmed the questioned RTC Orders. Respondent’s Motion for Reconsideration was likewise denied by the CA. Petitioner contends
that: (1) the term “pleadings” as used in B.M. No. 1922 does not include criminal Informations filed in court; (2) the failure of the
investigating prosecutor to indicate in the Information the number and date of issue of her MCLE Certificate of Compliance is a mere
formal defect and is not a valid ground to dismiss the subject Information which is otherwise complete in form and substance.

Issue:

(1) Whether or not a criminal information is considered as a pleading?

(2) Whether or not the failure of the prosecutor to incidate his MCLE Certificate of Compliance in the Information renders it
defective?

Held:

Yes. An information is, for all intents and purposes, considered an initiatory pleading because it is a written statement that contains the
cause of action of a party, which in criminal cases is the State as represented by the prosecutor, against the accused. Like a pleading,
the Information is also filed in court for appropriate judgment. Undoubtedly then, an Information falls squarely within the ambit of
Bar Matter No. 1922.

An information is a pleading since the allegations therein, which charge a person with an offense, is basically the same as a
complaint in a civil action which alleges a plaintiff’s cause or cause of action. Section 1, Rule 6 of the Rules of Court, as amended,
defines pleadings as the written statements of the respective claims and defenses of the parties submitted to the court for appropriate
judgment.

(2) Yes. However, the Court said that the rule has been revised to the effect that instead of dismissing the complaint, the lawyer would
now be subject to disciplinary action. Failure of a lawyer to indicate in his or her pleadings the number and date of issue of his or her
MCLE Certificate of Compliance will no longer result in the dismissal of the case and expunction of the pleadings from the records.
The trial court should not have dismissed the information but it should have simply required the investigating prosecutor to indicate
therein the number and date of issue of her MCLE Certificate of Compliance.
Remedial Law I| Page 76

Rule 8: Manner of Making Allegations in Pleadings

Fernando Medical Enterprises, Inc. vs. Wesleyan University


G.R. No. 207970. January 20, 2016
Facts:

Fernando Medical Enterprises, a domestic corporation dealing with medical equipment and supplies, delivered to and installed
medical equipment and supplies at the Wesleyan University's hospital. According to the petitioner, the Wesleyan University paid only
P67,3 57,683.23 of its total obligation of P123,901,650.00, leaving unpaid the sum of P54,654,195.54. However, on February 11,
2009, petitioner and respondent entered into an agreement whereby the former agreed to reduce its claim to only P50,400,000.00, and
allowed the latter to pay the adjusted obligation on installment.

Wesleyan University expressly admitted the following (a) the four transactions for the delivery and installation of various hospital
equipment; (b) the total liability of the Wesleyan University; (c) the payments made by the Wesleyan University; (d) the balance still
due to the petitioner; and (e) the execution of the February 11, 2009 agreement. The Wesleyan University denied the rest of the
complaint "for lack of knowledge or information sufficient to form a belief as to the truth or falsity thereof”, inasmuch as the alleged
transactions were undertaken during the term of office of the past officers of defendant Wesleyan University-Philippines.

Fernando Medical then filed its Motion for Judgment Based on the Pleadings, stating that the Wesleyan University had admitted the
material allegations of its complaint and thus did not tender any issue as to such allegations. The Wesleyan University opposed the
Motion for Judgment Based on the Pleadings, arguing that it had specifically denied the material allegations in the complaint. The
RTC issued the order denying the Motion for Judgment Based on the Pleadings of the petitioner. The CA affirmed the lower court’s
decision.

Issue:

Whether or not the respondent sufficiently denied the material allegations in the petitioner’s complaint?

Held:

No. The answer admits the material allegations of ultimate facts of the adverse party’s pleadings not only when it expressly confesses
the truth of such allegations but also when it omits to deal with them at all. The controversion of the ultimate facts must only be by
specific denial.

Section 10, Rule 8 of the Rules of Court recognizes only three modes by which the denial in the answer raises an issue of fact. The
first is by the defending party specifying each material allegation of fact the truth of which he does not admit and, whenever
practicable, setting forth the substance of the matters upon which he relies to support his denial. The second applies to the defending
party who desires to deny only a part of an averment, and the denial is done by the defending party specifying so much of the material
allegation of ultimate facts as is true and material and denying only the remainder. The third is done by the defending party who is
without knowledge or information sufficient to form a belief as to the truth of a material averment made in the complaint by stating so
in the answer. Any material averment in the complaint not so specifically denied are deemed admitted except an averment of the
amount of unliquidated damages.

In Civil Case No. 09122116, the respondent expressly admitted paragraph nos. 2, 3, 4, 5, 9 and 10 of the complaint. The admission
related to the petitioner’s allegations on: (a) the four transactions for the delivery and installation of various hospital equipment; (b)
the total liability of the respondent; (c) the payments made by the respondents; (d) the balance still due to the petitioner; and (e) the
execution of the February 11, 2009 agreement. The admission of the various agreements, especially the February 11, 2009 agreement,
significantly admitted the petitioner’s complaint

The respondent denied paragraph nos. 6, 7 and 8 of the complaint “for lack of knowledge or information sufficient to form a belief as
to the truth or falsity thereof, inasmuch as the alleged transactions were undertaken during the term of office of the past officers of
defendant Wesleyan University Philippines.” But the manner of denial effective was not a specific denial.
Remedial Law I| Page 77

Go Tong Electrical Supply vs. BPI Family Savings Bank


G.R. No. 187487. June 29, 2015

Facts:

Go Tong Electrical applied and was granted a loan by DBS Bank of the Philippines (formerly BSA) which is covered by a Promissory
Note. Go also executed a Comprehensive Surety Agreement (CSA) covering any and all obligations undertaken by Go Tong
Electrical. Because of its failure to pay the loan, respondent file a complaint for sum of money against it. In their Answer with
Counterclaim (Answer), petitioners merely stated that they "specifically deny" the allegations under the complaint. The RTC ruled in
favor of respondent. The CA sustained the RTC's ruling.

Issue:

Whether or not petitioner was able to sufficiently deny the allegations in the respondent’s complaint?

Held:

No. The genuineness and due execution of the loan documents in this case were deemed admitted by petitioners under the parameters
of Section 8, Rule 8 of the Rules which provides:

SEC. 8. How to contest such documents. - When an action or defense is founded upon a written instrument, copied in or attached to
the corresponding pleading as provided in the preceding Section, the genuineness and due execution of the instrument shall be
deemed admitted unless the adverse party, under oath, specifically denies them, and sets forth what he claims to be the facts; but the
requirement of an oath does not apply when the adverse party does not appear to be a party to the instrument or when compliance
with an order for an inspection of the original instrument is refused.

A general denial does not become specific by the use of the word “specifically.” A reading of the Answer shows that petitioners
failed to specifically deny the execution of the Credit Agreement, PN, and CSA under the auspices of the above quoted rule. The mere
statement in paragraph 4 of their Answer, i.e., that they “specifically deny” the pertinent allegations of the Complaint “for being self-
serving and pure conclusions intended to suit plaintiff’s purposes,” does not constitute an effective specific denial as contemplated by
law. Verily, a denial is not specific simply because it is so qualified by the defendant. Stated otherwise, a general denial does not
become specific by the use of the word “specifically.” Neither does it become so by the simple expedient of coupling the same with a
broad conclusion of law that the allegations contested are “self-serving” or are intended “to suit plaintiff’s purposes.
Remedial Law I| Page 78

Benguet Exploration, Inc. vs. Court of Appeals


G.R. No. 117434 February 9, 2001

Facts:

Benguet Exploration, Inc. (Benguet) filed a complaint for damages against Seawood Shipping, Inc. (Seawood Shipping) and
Switzerland General Insurance, Co., Ltd. (Switzerland Insurance) in the RTC. The cases were jointly tried, during which Benguet
presented its employees, Rogelio Lumibao and Ernesto Cayabyab, as witnesses.

Rogelio Lumibao, marketing assistant of Benguet, was in charge of exportation. He explained that Seawood Shipping was chartered
by Benguet to transport copper concentrates. The bill of lading stated that the cargo, consisting of 2,243.496 wet metric tons of copper
concentrates, was loaded on board Sangkulirang No. 3 at Poro Point, San Fernando, La Union. It was insured by Switzerland
Insurance. When the cargo was unloaded in Japan, however, Rogelio Lumibao received a report from a surveyor in Japan stating that
the cargo was 355 metric tons short of the amount stated in the bill of lading. For this reason, petitioner Benguet made a claim of the
loss to Seawood Shipping and Switzerland Insurance.In a letter, Benguet made a formal demand for the value of the alleged
shortage. As both Seawood Shipping and Switzerland Insurance refused the demand, petitioner Benguet brought these cases against
Seawood Shipping and Switzerland Insurance. On cross-examination, Lumibao admitted that he did not see the actual loading of the
cargo at Poro Point and that his knowledge was limited to what was contained in the bill of lading which he received about two days
after the loading. 

Ernesto Cayabyab also testified that he was the secretary of Nil Alejandre, manager of Benguet. According to Cayabyab, he was sent
to the warehouse (bodega) at Poro Point, La Union to assist in the loading of the copper concentrates. Cayabyab said he was present
when the cargo was loaded on the ship, as evidenced by the Certificate of Loading, Certificate of Weight, and the Mates Receipt, all
dated July 28, 1985. According to Cayabyab, the Marine Surveyor and the Chief Mate would go around the boat to determine how
much was loaded on the ship. Cayabyab stated that he saw petitioner Benguet’s representative and his immediate superior, Mr.
Alejandre, and the Inspector of Customs, Mr. Cardenas, sign the Certificate of Weight. Cayabyab also witnessed the ship captain sign
the Certificate of Weight, which stated therein that 2,243.496 wet metric tons of copper concentrates were loaded on the
ship. Cayabyab likewise confirmed the authenticity of the Mates Receipt, saying that he witnessed the Chief Mate sign the document.

The RTC, however, dismissed Benguet’s complaint. On appeal, the CA affirmed the decision of the RTC. Benguet moved for
reconsideration but was denied. Hence this petition. Benguet contends, among others, that the genuineness and due execution of the
documents presented, i.e., Bill of Lading, Certificate of Loading, Certificate of Weight, Mates Receipt, were properly established by
the testimony of its witness, Ernesto Cayabyab, and that as a result, there is a prima facie presumption that their contents are true.

Issue:

Whether the genuineness and due execution of the documents presented were properly established by the testimony of the plaintiff’s
witness, resulting to prima facie presumption that their contents are true?

Held:

No. The genuineness and due execution of the documents were not properly established by Benguet’s witnesses. The court ruled that
the admission of the due execution and genuineness of a document simply means that the party whose signature it bears admits that he
signed it or that it was signed by another for him with his authority; that at the time it was signed it was in words and figures exactly as
set out in the pleading of the party relying upon it; that the document was delivered; and that any formal requisites required by law,
such as a seal, an acknowledgment, or revenue stamp, which it lacks, are waived by him. In another case, the Court held that when the
law makes use of the phrase genuineness and due execution of the instrument it means nothing more than that the instrument is not
spurious, counterfeit, or of different import on its face from the one executed.

It is equally true, however, that execution can only refer to the actual making and delivery, but it cannot involve other matters without
enlarging its meaning beyond reason. The only object of the rule was to enable a plaintiff to make out a prima facie, not a conclusive
case, and it cannot preclude a defendant from introducing any defense on the merits which does not contradict the execution of the
instrument introduced in evidence.

In this case, respondents presented evidence which casts doubt on the veracity of these documents. Respondent Switzerland Insurance
presented Export Declaration No. 1131/85 which petitioners own witness, Rogelio Lumibao, prepared, in which it was stated that the
copper concentrates to be transported to Japan had a gross weight of only 2,050 wet metric tons or 1,845 dry metric tons, 10 percent
Remedial Law I| Page 79

more or less. On the other hand, Certified Adjusters, Inc., to which Switzerland Insurance had referred petitioners claim, prepared a
report which showed that a total of 2,451.630 wet metric tons of copper concentrates were delivered at Poro Point.

Considering the discrepancies in the various documents showing the actual amount of copper concentrates transported to Poro Point
and loaded in the vessel, there is no evidence of the exact amount of copper concentrates shipped. Thus, whatever presumption of
regularity in the transactions might have risen from the genuineness and due execution of the Bill of Lading, Certificate of Weight,
Certificate of Loading, and Mates Receipt was successfully rebutted by the evidence presented by respondent Switzerland Insurance
which showed disparities in the actual weight of the cargo transported to Poro Point and loaded on the vessel. This fact is compounded
by the admissions made by Lumibao and Cayabyab that they had no personal knowledge of the actual amount of copper concentrates
loaded on the vessel.
Remedial Law I| Page 80

Asian Construction Development Corp. vs. Court of Appeals


G.R. No. 160242May 17, 2005

Facts:

Monark Equipment Corporation (MEC) filed a Complaint  for a sum of money with damages against the Asian Construction and
Development Corporation (ACDC) with the Regional Trial Court (RTC) of QC for non-payment of rentals of several equipments
amounting to P5,071,335.86 despite demand. ACDC filed a motion to file and admit answer with third-party complaint against
Becthel Overseas Corporation (Becthel). In its answer, ACDC admitted its indebtedness to MEC in the amount of P5,071,335.86 but
alleged the various special and affirmative defenses that the equipment leased were used in the construction project of Becthel and it
was not paid of its services that resulted to the non-payment of rentals of the leased equipments. In its third-party complaint, ACDC
prayed to dismiss the complaint against it and to order that Becthel pay the amount of  P456,666.67 plus interest thereon and attorneys
fees. Later on, MEC filed a motion for summary judgment, alleging therein that there was no genuine issue as to the obligation of
ACDC to MEC, which motion was opposed by ACDC. The trial court issued a Resolution denying the motion of ACDC for leave to
file a third-party complaint and granting the motion of MEC, which the trial court considered as a motion for a judgment on the
pleadings. ACDC appealed the resolution to the Court of Appeals (CA), but was denied. Its MR having denied, petitioner filed the
present case.

Issue:

Whether or not ACDC should be allowed to file a third-party complaint against Becthel?

Held:

No. The third-party complaint is improper. The Court, ruled that the defendant may implead another as third-party defendant (a) on an
allegation of liability of the latter to the defendant for contribution, indemnity, subrogation or any other relief; (b) on the ground of
direct liability of the third-party defendant to the plaintiff; or (c) the liability of the third-party defendant to both the plaintiff and the
defendant. There must be a causal connection between the claim of the plaintiff in his complaint and a claim for contribution,
indemnity or other relief of the defendant against the third-party defendant.

The third-party complaint does not have to show with certainty that there will be recovery against the third-party defendant, and it is
sufficient that pleadings show possibility of recovery.  In determining the sufficiency of the third-party complaint, the allegations in
the original complaint and the third-party complaint must be examined. A third-party complaint must allege facts which prima
facie show that the defendant is entitled to contribution, indemnity, subrogation or other relief from the third-party defendant.

In this case, the claims of the respondent, as plaintiff in the RTC, against the petitioner as defendant therein, arose out of the contracts
of lease and sale; such transactions are different and separate from those between Becthel and the petitioner as third-party plaintiff for
the construction of the latters project in Mauban, Quezon, where the equipment leased from the respondent was used by the petitioner.
The controversy between the respondent and the petitioner, on one hand, and that between the petitioner and Becthel, on the other, are
thus entirely distinct from each other. There is no showing in the proposed third-party complaint that the respondent knew or approved
the use of the leased equipment by the petitioner for the said project in Quezon. Becthel cannot invoke any defense the petitioner had
or may have against the claims of the respondent in its complaint, because the petitioner admitted its liabilities to the respondent for
the amount of P5,075,335.86. The barefaced fact that the petitioner used the equipment it leased from the respondent in connection
with its project with Becthel does not provide a substantive basis for the filing of a third-party complaint against the latter. There is no
causal connection between the claim of the respondent for the rental and the balance of the purchase price of the equipment and parts
sold and leased to the petitioner, and the failure of Becthel to pay the balance of its account to the petitioner after the completion of the
project in Quezon. Thus, the denial of the third-party complaint is proper.
Remedial Law I| Page 81

Rule 9: Effect of Failure to Plead

Salvador vs. Rabaja


G.R. No. 199990, February 4, 2015

Facts:

Spouses Rabaja were leasing an apartment in a subject lot registered under the names of the Spouses Salvador. Later on, the subject
property was sold by Spouses Salvador to Spouses Rabaja and personally introduced one Gonzales as the administrator of the
property. Later on, Spouses Rabaja made an initial payment to Gonzales in the presence of Herminia Salvador. Gonzales also
presented to Spouses Rabaja an SPA executed by Rolado Salvador. On the same day, the parties executed the Contract to Sell  which
stipulated that for a consideration of ₱5,000,000.00, Spouses Salvador sold, transferred and conveyed in favor of Spouses Rabaja the
subject property. Spouses Rabaja made several payments totalling ₱950,000.00, which were received by Gonzales pursuant to the
SPA provided earlier as evidenced by the check vouchers signed by Gonzales and the improvised receipts signed by Herminia.

Sometime in June 1999, however, Spouses Salvador complained to Spouses Rabaja that they did not receive any payment from
Gonzales. This prompted Spouses Rabaja to suspend further payment of the purchase price; and as a consequence, they received a
notice to vacate the subject property from Spouses Salvador for non-payment of rentals. Thereafter, Spouses Salvador instituted an
action for ejectment against Spouses Rabaja. In turn, Spouses Rabaja filed an action for rescission of contract against Spouses
Salvador and Gonzales, the subject matter of the present petition.

In the rescission case, Spouses Rabaja demanded the rescission of the contract to sell praying that the amount of ₱950,000.00 they
previously paid to Spouses Salvador be returned to them. Spouses Salvador filed their answer with counterclaim and cross-claim
contending that there was no meeting of the minds between the parties and that the SPA in favor of Gonzales was falsified. Gonzales
filed her answer stating that the SPA was not falsified and that the payments of Spouses Rabaja amounting to ₱950,000.00 were all
handed over to Spouses Salvador. Spouses Salvador were declared in default for non-appearance in the pre-trial conference and the
RTC allowed the respondents to present evidence ex parte. The RTC ruled in favor of Spouses Rabaja, stating that the contract to sell
was actually a contract of sale and that Gonzales was undoubtedly the attorney-in-fact of Spouses Salvador absent any taint of
irregularity. Spouses Rabaja could not be faulted in dealing with Gonzales who was duly equipped with the SPA from Spouses
Salvador. On appeal, the CA affirmed the decision of the RTC but modifying the same that Gonzales was not solidarily liable with
Sps. Salvador. Their MR was denied, hence, this petition. Spouses Salvador contend that the order of default must be lifted because
reasonable grounds exist to justify their failure to attend the pre-trial conference

Issue:

Whether or not Spouses Salvador can be validly declared in default for non-appearance in the pre-trial conference?

Held:

No. The failure to attend the pre-trial conference does not result in the default of an absent party. Under the 1997 Rules of Civil
Procedure, a defendant is only declared in default if he fails to file his Answer within the reglementary period. On the other hand, if a
defendant fails to attend the pre-trial conference, the plaintiff can present his evidence ex parte. Still, the non-appearance of the
defendant bears the same effect as that of the declaration of default for failure to file an answer within the prescribed period. The case
of Philippine American Life & General Insurance Company v. Joseph Enario discussed the difference between the non-appearance of
a defendant in a pre-trial conference and the declaration of a defendant in default in the present Rules of Civil Procedure. The decision
instructs:

“Prior to the 1997 Revised Rules of Civil Procedure, the phrase "as in default" was initially included in Rule 20 of the old
rules, and which read as follows:

Sec. 2. A party who fails to appear at a pre-trial conference may be non-suited or considered as in default.”

It was, however, amended in the 1997 Revised Rules of Civil Procedure. Justice Regalado, in his book, REMEDIAL LAW
COMPENDIUM, explained the rationale for the deletion of the phrase "as in default" in the amended provision, to wit:

“1. This is a substantial reproduction of Section 2 of the former Rule 20 with the change that, instead of defendant being
declared "as in default" by reason of his non-appearance, this section now spells out that the procedure will be to allow the ex
parte presentation of plaintiff’s evidence and the rendition of judgment on the basis thereof.”
Remedial Law I| Page 82

Still, in the same book, Justice Regalado clarified that while the order of default no longer obtained, its effects were retained, thus:

“Failure to file a responsive pleading within the reglementary period, and not failure to appear at the hearing, is the sole
ground for an order of default, except the failure to appear at a pre-trial conference wherein the effects of a default on the part
of the defendant are followed, that is, the plaintiff shall be allowed to present evidence ex parte and a judgment based thereon
may be rendered against defendant.”

From the foregoing, the failure of a party to appear at the pre-trial has indeed adverse consequences. If it is the defendant who fails to
appear, then the plaintiff is allowed to present his evidence ex parte and the court shall render judgment based on the evidence
presented. Thus, the plaintiff is given the privilege to present his evidence without objection from the defendant, the likelihood being
that the court will decide in favor of the plaintiff, the defendant having forfeited the opportunity to rebut or present its own evidence.

There is no dispute that Spouses Salvador and their counsel failed to attend the pre-trial conference set on February 4, 2005 despite
proper notice. Spouses Salvador aver that their non-attendance was due to the fault of their counsel as he forgot to update his calendar.
This excuse smacks of carelessness, and indifference to the pre-trial stage. It simply cannot be considered as a justifiable excuse by the
Court. As a result of their inattentiveness, Spouses Salvador could no longer present any evidence in their favor. Spouses Rabaja, as
plaintiffs, were properly allowed by the RTC to present evidence ex parte against Spouses Salvador as defendants. Considering that
Gonzales as co-defendant was able to attend the pre-trial conference, she was allowed to present her evidence. The RTC could only
render judgment based on the evidence presented during the trial.
Remedial Law I| Page 83

BDO vs. Tansipek


G.R. No. 181235 July 22, 2009

Facts:

J. O. Construction, Inc. (JOCI) filed a complaint for sum of money against Philippine Commercial and Industrial Bank (PCIB). PCIB,
in turn, filed a third-party complaint against John Tansipek (respondent). Tansipek failed to answer the third-party complaint and he
was declared in default for failure to do so. He filed a Motion for Reconsideration from the Order of Default but was denied by the
RTC. Tansipek then filed a Petition for Certiorari with the CA assailing the Default Order and the denial of the Motion for
Reconsideration. The CA dismissed the Petition for failure to attach the assailed Orders. Later, the CA denied respondent Tansipek’s
Motion for Reconsideration for having been filed out of time. He did not appeal said denial to the Supreme Court.

On the third party complaint, Tansipek is ordered to pay PCIB all amounts that the latter shall have to pay to JOCI on account of this
case. Tansipek appealed the Decision to the CA. The CA ruled that it was an error for the trial court to have acted on PCIB’s motion to
declare Tansipek in default. The CA, thus, remanded the case to the RTC for further proceedings.

Issue:

Whether or not Tansipek’s remedy against the Order of Default is correct?

Held:

No. Tansipek’s remedy against the Order of Default was erroneous from the very beginning. He should have filed a Motion to Lift
Order of Default, and not a Motion for Reconsideration, pursuant to Section 3(b), Rule 9 of the Rules of Court:

(b) Relief from order of default.—A party declared in default may at any time after notice thereof and before judgment file a motion
under oath to set aside the order of default upon proper showing that his failure to answer was due to fraud, accident, mistake or
excusable negligence and that he has a meritorious defense. In such case, the order of default may be set aside on such terms and
conditions as the judge may impose in the interest of justice.

A Motion to Lift Order of Default is different from an ordinary motion in that the Motion should be verified; and must show fraud,
accident, mistake or excusable neglect, and meritorious defenses. The allegations of (1) fraud, accident, mistake or excusable neglect,
and (2) of meritorious defenses must concur.

Assuming for the sake of argument, however, that Tansipek’s Motion for Reconsideration may be treated as a Motion to Lift Order of
Default, his Petition for Certiorari on the denial thereof has already been dismissed with finality by the CA. Tansipek did not appeal
said ruling of the CA to the Supreme Court. The dismissal of the Petition for Certiorari assailing the denial of Tansipek’s Motion,
thus, constitutes a bar to the retrial of the same issue of default under the doctrine of the law of the case.

It is important to note that a party declared in default is not barred from appealing from the judgment on the main case, whether or not
he had previously filed a Motion to Set Aside Order of Default, and regardless of the result of the latter and the appeals therefrom.
However, the appeal should be based on the Decision’s being contrary to law or the evidence already presented, and not on the alleged
invalidity of the default order.
Remedial Law I| Page 84

Bitte vs. Jonas


GR No. 212256, December 9, 2015

Facts:

This controversy stemmed from the two civil cases filed by the parties Bitte and Spouses Jonas against each other relative to a
purported contract of sale involving a property in Davao City, registered in the name of Rosa Elsa Serrano Jonas. Before Rosa Elsa
went to Australia, she had executed a Special Power of Attorney (SPA) authorizing her mother, Andrea, to sell the property. Cipriano,
brother of Rosa Elsa, then offered the property for sale to Spouses Bitte, showing them the authority of Andrea. Spouses Bitte later
sought a meeting for final negotiation with Rosa Elsa. However, shortly after her arrival in the Philippines, Rosa Elsa revoked the SPA
and handed a copy thereof to Andrea. The parties met but no final agreement was reached. Rosa Elsa eventually withdrew from the
transaction.

Spouses Bitte then filed before the RTC a Complaint for Specific Performance with Damages. While the case was pending, Andrea
sold the subject property to Spouses Bitte, through a deed of absolute sale. In response to the complaint, Rosa Elsa countered that
despite her appointment of her mother as her attorney-in-fact/agent, she later gave her instructions not to sell the property.

During the pre-trial conference, Spouses Bitte failed to appear. So, the RTC dismissed their complaint and set the reception of Rosa
Elsa's counterclaim for hearing. Later on, Benjamin Bitte manifested the withdrawal of their counsel. The RTC then cancelled the
reception of Rosa Elsa's evidence without reconsidering the dismissal of the complaint.

Spouses Jonas thereafter filed before the RTC a complaint for Annulment of Deed of Absolute Sale, Cancellation of TCT and
Recovery of Possession, Injunction and Damages against Spouses Bitte. The two cases were consolidated. Spouses Bitte were again
declared in default for their failure to attend the pre-trial. The counsel of Spouses Bitte withdrew and a new one entered his
appearance and then filed a verified motion for reconsideration. For the third time, Spouses Bitte once again failed to appear in the
pre-trial and were, thus, declared non-suited. Rosa Elsa then presented her evidence ex parte.

When the case went up to the Supreme Court, Spouses Jonas claim that the door to any reliefs for Spouses Bitte, be it through a
motion for reconsideration or this subject petition, was closed by the finality and immutability of the RTC declaration of their default.
In other words, it is their stand that the petitioners do not have the right to obtain recourse from this Court.

Issue:

Whether or not petitioners Bitte no longer have the personality to file the present petition on the basis of the RTC’s declaration of their
default?

Held:

No. The rule is that "right to appeal from the judgment by default is not lost and can be done on grounds that the amount of the
judgment is excessive or is different in kind from that prayed for, or that the plaintiff failed to prove the material allegations of his
complaint, or that the decision is contrary to law."

If a party who has been declared in default has in his arsenal the remedy of appeal from the judgment of default on the basis of the
decision having been issued against the evidence or the law, that person cannot be denied the remedy and opportunity to assail the
judgment in the appellate court. Despite being burdened by the circumstances of default, the petitioners may still use all other
remedies available to question not only the judgment of default but also the judgment on appeal before this Court. Those remedies
necessarily include an appeal by certiorari under Rule 45 of the Rules of Court.
Remedial Law I| Page 85

Rule 10: Amendment of Pleadings

Yujuico vs. United Resources Asset Management Corp.


GR No. 211113, June 29, 2015

Facts:

To secure the loan of SRADEC from United Resources Asset Management, Inc (URAMI), several stockholders of STRADEC
executed Pledge Agreements  whereby they pledged a certain amount of their stocks in the said company in favor of URAMI. Yujuico
is one of the said stockholders. Not having been able to comply with its loan obligations, a Notice, signed by Atty. Nethercott,
informing them about the impending auction sale were sent to STRADEC and to its stockholders. Nethercott claims that he is
URAMI’s attorney-in-fact. As response, Yujuico filed before the RTC of Pasig an injunction complaint seeking to enjoin the sale at
public auction mentioned in Atty. Nethercott's notice. In his complaint, Yujuico asserts that the planned auction sale is void as
Nethercott was not authorized to initiate the sale in behalf of URAMI.

As the RTC did not issue the injunction, the auction sale pushed through with URAMI emerging as the winning bidder. Thereafter, the
RTC issued a writ of preliminary injunction, which effectively prevented URAMI from appropriating the stocks it had purchased
during the auction sale. Nethercott filed his answer denying the material allegations of the injucntion complaint. However, URAMI, in
its answer, agreed with Yujuico that the sale was VOID and that it never authorized Nethercott to cause the sale but it pointed out that
since URAMI never sanctioned the sale, it should not be liable for the prejudice caused by the sale and hence, overall it asked for the
dismissal of the case.

Yujuico then filed a Motion for Summary Judgment in view of the admissions made by URAMI in its answer regarding Atty.
Nethercott's lack of authority to cause the auction sale of pledged stocks, there was no longer any genuine issue left to be resolved in
trial.

The resolution of such Motion was however, deferred due to the TRO issued by the SC which remained in effect for more than a year.
URAMI then changed its counsel and then filed an answer with compulsory counterclaim. In its amended answer, URAMI still
vouched for the dismissal of the injunction complaint but reneged from its previous admissions under the original answer. It
now claims that Nethercott was DULY AUTHORIZED by the Board to cause the sale. The RTC allowed the said amended
answer against the protest of Yujuico. But on MR of Yujuico, the RTC set aside its previous resolution, explaining that it cannot
admit the amended answer because URAMI did not first secure a leave of court. In compliance, URAMI filed the pertinent motion
which was granted by the RTC.

In a petition for certiorari in the CA, the higher court sustained the allowance by the RTC of the amended answer. Hence, this appeal.

Issue:

Whether or not the amended answer of URAMI may still be allowed?

Held:

Yes. Our rules of procedure allow a party in a civil action to amend his pleading as a  matter of right, so long as the pleading is
amended only once and before a responsive pleading is served (or, if the pleading sought to be amended is a reply, within ten days
after it is served). Otherwise, a party can only amend his pleading upon prior leave of court.

As a matter of judicial policy, courts are impelled to treat motions for leave to file amended pleadings with  liberality.  Hence, as long
as it does not appear that the motion for leave was made with bad faith or with intent to delay the proceedings,  courts are justified to
grant leave and allow the filing of an amended pleading. Once a court grants leave to file an amended pleading, the same becomes
binding and will not be disturbed on appeal unless it appears that the court had abused its discretion.

In this case, URAMI filed its motion for leave seeking the admission of its amended answer more than two (2) years after it filed its
original answer. Despite the considerable lapse of time between the filing of the original answer and the motion for leave, the RTC
still granted the said motion. Such grant was later affirmed on appeal by the Court of Appeals.

Matters involving the amendment of pleadings are primarily governed by the pertinent provisions of Rule 10 and not by Section 4 of
Rule 129 of the Rule of Court. Hence, allegations (and admissions) in a pleading—even if not shown to be made through "palpable
mistake"—can still be corrected or amended provided that the amendment is sanctioned under Rule 10 of the Rules of Court.
Remedial Law I| Page 86

Nevertheless, even if we are to apply Section 4 of Rule 129 to the present case, we still find the allowance of URAMI's  amended
answer to be in order. To our mind, a consideration of the evidence that URAMI plans to present during trial indubitably reveals that
the admissions made by URAMI under its original answer were a product of clear and patent mistake.

One of the annexes attached by URAMI is a board resolution authorizing Nethercott to cause the sale. With the existence of such
board resolution, the statement in URAMI's original answer pertaining to the lack of authority of Atty. Nethercott to initiate the 23
June 2004 auction sale thus appears mistaken, if not entirely baseless and unfounded.

Also the mere fact that it was filed only after 2 years after the original answer was filed does not discredit the amended answer as a
mere dilatory measure for it should be remembered that the SC issued a TRO which was in effect for more than a year so even if it
wants to, URAMI could not have filed said amended answer.
Remedial Law I| Page 87

Lisam Enterprises vs. Banco De Oro


G.R. No. 143264, April 23, 2012

Facts:

LISAM, in the course of its business transaction, acquired a parcel of residential land with improvement in Legazpi City. Sometime on
March 1996, Lilian and Leandro Soriano obtained a loan from PCIB (now BDO) in their personal capacity and executed a real estate
mortgage as security for such loan. This real estate mortgage, it is being alleged, was executed, without authority and consent of the
board of LISAM and with the use of a falsified board resolution. The spouses specifically falsified the signature of Lolita Soriano as
corporate secretary and director of LISAM. Later on, Spouses Soriano signed, through instigation of PCIB, a Deed of Assumption of
Loans and Mortgage Obligations, wherein LISAM was made to assume the liabilities of said spouses. After realizing this, Lolita
demanded from the Spouses the compliance of their obligation and to free LISAM from such but it went unanswered. Thereafter,
Soriano commenced a derivative suit against the Spouses before the SEC for Fraudulent Scheme and Unlawful Machination with
Damages. Moreover, the plaintiffs instituted before the RTC a case for the nullification of the Real Estate Mortgage. The RTC issued
a writ of preliminary injunction to enjoin PCIB from disposing the property. In their Answer, Lilian and the Estate of Leandro stated
that they were authorized by LISAM to mortgage the said property and that the proceeds of the loan were for the benefit and use of
LISAM.

PCIB filed a Motion to Dismiss the Complaint on grounds of lack of legal capacity to sue, failure to state cause of action, and  litis
pendencia. The RTC then issued an order dismissing the case. Pending resolution of its MR, Lisam filed a Motion to Admit
Amended complaint. Its amended complaint now stating that Lolita exhausted intra-corporate remedies and that she had asked
LISAM’s board to take action but to no avail.

The Trial court issued the questioned Order denying both the Motion for Reconsideration and the Motion to Admit Amended
Complaint. It held that no new argument had been raised by petitioners in their motion for reconsideration to address the fact of
plaintiffs' failure to allege in the complaint that petitioner Lolita A. Soriano made demands upon the Board of Directors of Lisam
Enterprises, Inc. to take steps to protect the interest of the corporation against the fraudulent acts of the Spouses Soriano and
PCIB. The trial court further ruled that the Amended Complaint can no longer be admitted, because the same absolutely changed
petitioners' cause of action.

The petitioner went straight to the SC alleging that the case involves pure questions of law.

Issue:

Whether or not the amended complaint should have been admitted?

Held:

Yes. A responsive pleading having been filed, amendments to the complaint may, therefore, be made only by leave of court and no
longer as a matter of right. The courts should be liberal in allowing amendments to pleadings to avoid a multiplicity of suits and in
order that the real controversies between the parties are presented, their rights determined, and the case decided on the merits without
unnecessary delay. This liberality is greatest in the early stages of a lawsuit, especially in this case where the amendment was made
before the trial of the case, thereby giving the petitioners all the time allowed by law to answer and to prepare for trial.

Furthermore, amendments to pleadings are generally favored and should be liberally allowed in furtherance of justice in order that
every case, may so far as possible, be determined on its real facts and in order to speed up the trial of the case or prevent the circuitry
of action and unnecessary expense. That is, unless there are circumstances such as inexcusable delay or the taking of the adverse party
by surprise or the like, which might justify a refusal of permission to amend.

A reading of the amended complaint will reveal that all the foregoing requisites had been alleged therein. Hence, the amended
complaint remedied the defect in the original complaint and now sufficiently states a cause of action.
Remedial Law I| Page 88

Tiu vs. Philippine Bank of Communications


G.R. No. 151932. August 19, 2009.

Facts:

Asian Water Resources, Inc. (AWRI), represented by its officers applied for a real estate loan with Philippine Bank of
Communications (PBCOM). One of its loans was not guaranteed by a collateral, so the bank required all the members of the Board of
Directors of the Corp. to become sureties. A Surety Agreement was executed by its Directors and acknowledged by a notary public on
the same date. All copies of the Surety Agreement, except two, were kept by bank. (One copy was retained by the notary public for his
notarial file and the other was sent to the Records Management and Archives Office). The bank then demanded payment. Unable to
collect, the bank’s counsel filed a complaint before the RTC for collection against the directors. On their answer, the directors alleged
among other things, that they were not personally liable on the promissory notes, because they signed the Surety Agreement in their
capacities as officers of the corp. They claimed that the Surety Agreement attached to the complaint were falsified, considering that
when they signed the same, the words in his personal capacity did not yet appear in the document. They attached a copy from the
Records Management and Archives Office). Because of this development, the bank’s counsel searched for and retrieved the file copy
of the Surety Agreement. It was discovered that the insertion was ordered by the bank auditor, in accordance with bank standard
operating procedures. However, the notary public was never informed of the insertion. The bank then filed a Reply and Answer to
Counterclaim with Motion for Leave of Court to Substitute Annex A of the Complaint wherein it attached the duplicate original
copy retrieved from the file of the notary public. RTC issued an Order allowing the substitution of the altered document with the
original Surety Agreement. Aggrieved, petitioners sought recourse before the CA via a petition for certiorari under Rule 65 of the
Rules of Court which dismissed the petition for lack of merit Hence, the current petition.
 
Issue:

Whether or not the substitution of the document should have been allowed?

Held:

Yes. With respect to the bank’s right to amend its complaint, including the documents annexed thereto, after petitioners have filed
their answer, Section 3, Rule 10 of the Rules of Court specifically allows amendment by leave of court. 

The granting of leave to file amended pleading is a matter particularly addressed to the sound discretion of the trial court; and that
discretion is broad, subject only to the limitations that the amendments should not substantially change the cause of action or alter the
theory of the case, or that it was not made to delay the action. Nevertheless, as enunciated in Valenzuela, even if the amendment
substantially alters the cause of action or defense, such amendment could still be allowed when it is sought to serve the higher interest
of substantial justice; prevent delay; and secure a just, speedy and inexpensive disposition of actions and proceedings.
 
Amendments to pleadings are generally favored and should be liberally allowed in furtherance of justice. That is, unless there are
circumstances such as inexcusable delay or the taking of the adverse party by surprise or the like, which might justify a refusal of
permission to amend.
 
In the present case, there was no fraudulent intent on the part of PBCOM in submitting the altered surety agreement. In fact, the bank
admitted that it was a mistake on their part to have submitted it in the first place instead of the original agreement.  It also admitted
that, through inadvertence, the copy that was attached to the complaint was the copy wherein the words IN HIS PERSONAL
CAPACITY were inserted to conform to the banks standard practice. This alteration was made without the knowledge of the notary
public. The Bank’s counsel had no idea that what it submitted was the altered document, thereby necessitating the substitution of the
surety agreement with the original thereof, in order that the case would be judiciously resolved.

Moreover, the opposing party could not be prejudiced by the substitution since they can still present the substituted documents, as part
of the evidence of their affirmative defenses. Besides, they are not precluded from filing the appropriate criminal action against the
bank for attaching the altered copy of the surety agreement to the complaint. The substitution of the documents would not, in any way,
erase the existence of falsification, if any. 

Remington Industrial Sales Corp. vs. Court of Appeals


G.R. No. 133657. May 29, 2002
Remedial Law I| Page 89

Facts:

Remington filed a complaint for sum of money and damages arising from breach of contract before the RTC. Impleaded as principal
defendant therein was Industrial Steels, Ltd. (ISL), with Ferro Trading GMBH (Ferro) and respondent British Steel as alternative
defendants. ISL and respondent British Steel separately moved for the dismissal of the complaint on the ground that it failed to state a
cause of action against them. RTC denied the motions to dismiss, as well as the ensuing motion for reconsideration. ISL then filed its
answer to the complaint.
On the other hand, respondent British Steel filed a petition for certiorari and prohibition before the Court of Appeals and claimed that
the complaint failed to state a cause of action against it.
Meanwhile, petitioner sought to amend its complaint. Pursuant to Section 2, Rule 10 of the Rules of Court, petitioner maintained that it
can amend the complaint as a matter of right because respondent has not yet filed a responsive pleading thereto. Subsequently,
petitioner prayed that the proceedings in the special civil action be suspended. The trial court ruled on petitioner’s favor.
Meanwhile, CA granted the writ of certiorari and ordered the respondent judge to dismiss without prejudice the Complaint against
petitioner British Steel (Asia) Ltd. 
Petitioner filed a motion for reconsideration of the appellate courts decision, which was denied, Hence this present petition for review
under rule 45.
Issue:
Whether or not the Court of Appeals, by granting the extraordinary writ of certiorari, correctly ordered the dismissal of the complaint
for failure to state a cause of action, despite the fact that petitioner exercised its right to amend the defective complaint under Section
2, Rule 10 of the Rules of Court?
Held:
No. Section 2, Rule 10 of the Revised Rules of Court explicitly states that a pleading may be amended as a matter of right before a
responsive pleading is served. This only means that prior to the filing of an answer, the plaintiff has the absolute right to amend the
complaint whether a new cause of action or change in theory is introduced. The reason for this rule is implied in the subsequent
Section 3 of Rule 10. Under this provision, substantial amendment of the complaint is not allowed without leave of court after an
answer has been served, because any material change in the allegations contained in the complaint could prejudice the rights of the
defendant who has already set up his defense in the answer. Conversely, it cannot be said that the defendant’s rights have been
violated by changes made in the complaint if he has yet to file an answer thereto. In such an event, the defendant has not presented any
defense that can be altered or affected by the amendment of the complaint in accordance with Section 2 of Rule 10. 
The right granted to the plaintiff under procedural law to amend the complaint before an answer has been served is not precluded by
the filing of a motion to dismiss or any other proceeding contesting its sufficiency. Were we to conclude otherwise, the right to amend
a pleading under Section 2, Rule 10 will be rendered nugatory and ineffectual, since all that a defendant has to do to foreclose this
remedial right is to challenge the adequacy of the complaint before he files an answer.
Moreover, amendment of pleadings is favored and should be liberally allowed in the furtherance of justice in order to determine every
case as far as possible on its merits without regard to technicalities. 
Here, the fact that the other defendants below has filed their answers to the complaint does not bar petitioner’s right to amend the
complaint as against respondent. Indeed, where some but not all the defendants have answered, the plaintiff may still amend its
complaint once, as a matter of right, in respect to claims asserted solely against the non-answering defendant, but not as to claims
asserted against the other defendants.
Remedial Law I| Page 90

Rule 13: Filing and Service of Pleadings

Palileo vs. Planters Development Bank


G.R. No. 193650. October 8, 2014.
Facts:

George Philip Palileo and Jose Dela Cruz (petitioners) filed before the RTC a complaint for specific performance and/or sum of
money and damages with prayer for the issuance of writs of preliminary attachment and preliminary injunction against Planters
Development Bank. Summons was served to PDP and it filed its answer. However, PDP failed to attend the last pre-trial hearing and it
failed to file its pre-trial brief. Thus, Palileo and Dela Cruz were allowed to present their evidence ex parte. The RTC ruled in favor of
Palileo and Dela Cruz. PDP received a copy of the RTC Decision on July 17, 2006.

On July 31, 2006, PDB filed by private courier service — specifically LBC — an Omnibus Motion for Reconsideration and for New
Trial. Petitioners’ copy of the Omnibus Motion for Reconsideration and for New Trial was likewise sent on July 31, 2006 by courier
service through LBC, but in their address of record — Tupi, South Cotabato — there was no LBC service at the time.

On August 2, 2006, PDB filed with the RTC another copy of the Omnibus Motion for Reconsideration and for New Trial via
registered mail; another copy thereof was simultaneously sent to petitioners by registered mail as well.

The RTC denied the Omnibus Motion on the ground that it violated the Rule on Motions as it set the hearing on August 18, 2006 or 16
days after filing (under the Rules; it must not be later than 10 days). Since the Motion was pro-forma, it did not toll the running of the
reglementary period thus making the RTC’s decision final and executory.

The Court of Appeals initially affirmed the decision of the RTC but on MR, the CA decided to relax the rules and held that the Motion
was not pro-forma considering that the address of the trial court as well as that of the opposing counsel is too distant from the office of
the counsel.

Issue:

Whether or not PDP’s Omninus Motion for Reconsideration/New Trial was filed late?

Held:

Yes. PDB’s Omnibus Motion for Reconsideration and for New Trial was filed one day too late. The bank received a copy of the trial
court’s June 15, 2006 Decision on July 17, 2006; thus, it had 15 days — or up to August 1, 2006 — within which to file a notice of
appeal, motion for reconsideration, or a motion for new trial, pursuant to the Rules of Court. Yet, it filed the omnibus motion for
reconsideration and new trial only on August 2, 2006.

Its filing or service of a copy thereof to petitioners by courier service cannot be trivialized. Service and filing of pleadings by courier
service is a mode not provided in the Rules. This is not to mention that PDB sent a copy of its omnibus motion to an address or area
which was not covered by LBC courier service at the time. Realizing its mistake, PDB refiled and resent the omnibus motion by
registered mail, which is the proper mode of service under the circumstances. By then, however, the 15-day period had expired. Thus,
the RTC’s decision had become final and executory by the failure of PDP to file a timely appeal.
Remedial Law I| Page 91

Heirs of Numeriano Miranda, Sr. vs. Miranda


G.R. No. 179638. July 8, 2013
Facts:

In 1994, the heirs of Numeriano Miranda (petitioners) filed a Complaint for Annulment of Titles and Specific Performance against the
heirs of Pedro Miranda. The RTC ruled in favor of the heirs of Pedro Miranda and sustain the validity of their titles. Petitioners did not
file an appeal and the Decision became final and executory. In 2001, the RTC issued a Writ of Execution which was not implemented.
In 2005, Pablo Miranda (respondent) filed with the RTC a Petition for Revival of Judgment. The RTC granted the petition on June 20,
2006. On July 13, 2006, petitioners filed a Notice of Appeal via LBC. The RTC denied the notice of appeal. Aggrieved, petitioners
filed a Petition for Mandamus with the CA. The CA denied the petition on the ground that the Notice of Appeal was filed out of time.
Before the Supreme Court, the petitioners contend that their appeal was perfected on time. They insist that the Notice of Appeal,
which they filed on the 15th day via LBC, was seasonably filed since the law does not require a specific mode of service for filing a
notice of appeal.

Issue:

Whether or not the filing of the Notice of Appeal thru a courier is a valid mode of service for filing a notice of appeal?

Held:

No. Under Section 3, Rule 13 of the Rules of Court, pleadings may be filed in court either personally or by registered mail. In the first
case, the date of filing is the date of receipt. In the second case, the date of mailing is the date of receipt.

In this case, however, the counsel for petitioners filed the Notice of Appeal via a private courier, a mode of filing not provided in the
Rules. Though not prohibited by the Rules, it cannot be considered the filing of petitioners’ Notice of Appeal via LBC timely filed. It
is established jurisprudence that “the date of delivery of pleadings to a private letter-forwarding agency is not to be considered as the
date of filing thereof in court”; instead, “the date of actual receipt by the court x x x is deemed the date of filing of that pleading.”
Records show that the Notice of Appeal was mailed on the 15th day and was received by the court on the 16th day or one day beyond
the reglementary period. Thus, the CA correctly ruled that the Notice of Appeal was filed out of time.
Remedial Law I| Page 92

Fortune Life Insurance Company, Inc. vs. Commission on Audit (COA) Proper
G.R. No. 213525. January 27, 2015.

Facts:

The Provincial Government of Antique and Fortune Life Insurance executed a memorandum of agreement concerning the life
insurance coverage of qualified barangay secretaries, treasurers and tanod, the former obligating P4,393,593.60 for the premium
payment, and subsequently submitting the corresponding disbursement voucher to COA-Antique for pre-audit. The latter office
disallowed the payment for lack of legal basis under Republic Act No. 7160 (Local Government Code).

Subsequently, Fortune Insurance filed a petition for money claim in the Commission on Audit (COA).

COA denied its petition holding that under Section 447 and Section 458 of the Local Government Code only municipal or city
governments are expressly vested with the power to secure group insurance coverage for barangay workers; and for the LGU’s failure
to comply with the requirement of publication under Section 21 of Republic Act No. 9184 (Government Procurement Reform Act).

Fortune Insurance filed a Motion for Reconsideration which was denied on July 14, 2014.

Fortune Insurance filed a petition for certiorari before the Supreme Court on August 12, 2014. The Supreme Court dismissed the
petition for (a) the late filing of the petition; (b) the nonsubmission of the proof of service and verified declaration; and (c) the failure
to show grave abuse of discretion on the part of the respondents. Fortune Insurance then filed a Motion for Reconsideration.

Issue:

Whether or not Fortune Insurance complied with the rule on proof of service (Section 13, Rule 13)?

Held:

No. Section 13, Rule 13 of the Rules of Court concerns two types of proof of service, namely: the affidavit and the registry receipt,
viz.:

Section 13. Proof of Service.—x x x. If service is made by registered mail, proof shall be made by such affidavit and the registry
receipt issued by the mailing office. The registry return card shall be filed immediately upon its receipt by the sender, or in lieu
thereof the unclaimed letter together with the certified or sworn copy of the notice given by the postmaster to the addressee.

Section 13 thus requires that if the service is done by registered mail, proof of service shall consist of the affidavit of the person
effecting the mailing and the registry receipt, both of which must be appended to the paper being served. A compliance with
the rule is mandatory, such that there is no proof of service if either or both are not submitted.

Here, the petition for certiorari only carried the affidavit of service executed by one Marcelino T. Pascua, Jr., who declared that he had
served copies of the petition by registered mail “under Registry Receipt Nos. 70449, 70453, 70458, 70498 and 70524 attached to the
appropriate spaces found on pages 64-65 of the petition.” The petition only bore, however, the cut print-outs of what appeared to be
the registry receipt numbers of the registered matters, not the registry receipts themselves. The rule requires to be appended the
registry receipts, not their reproductions. Hence, the cut print-outs did not substantially comply with the rule.
Remedial Law I| Page 93

Rule 14: Summons

Valmonte vs. Court of Appeals


G.R. No. 131724, February 28, 2000

Facts:

Petitioners Lourdes and Alfredo are husband and wife. They are both residents of the USA. Petitioner Alfredo, however, is a
practicing lawyer in the Philippines with office address at Ermita, Manila. A complaint for partition of real property and accounting of
rentals was filed by private respondent Rosita, who is the sister of petitioner Lourdes. The subject of the action is a three-door
apartment located in Paco, Manila. In her Complaint, Rosita alleged that in a letter previously sent by petitioner Lourdes to Rosita, the
former referred to her husband as the party to whom all communications intended for her should be sent, whose office address is in the
Philippines.

Service of summons was then made upon petitioner Alfredo, who at the time, was at his office in  Manila. Alfredo accepted the
summons, insofar as he was concerned, but refused to accept the summons for his wife, on the ground that he was not authorized to
accept the process on her behalf. Accordingly, the process server left without leaving a copy of the summons and complaint for
petitioner Lourdes. Alfredo thereafter filed his Answer with Counterclaim. Petitioner Lourdes, however, did not file her Answer. For
this reason, private respondent moved to declare her in default. Petitioner Alfredo entered a special appearance in behalf of his wife
and opposed the private respondent’s motion. The RTC, denied the motion of Rosita to declare Petitioner Lourdes in default and
subsequently, her Motion for Reconsideration was denied. On appeal, the CA granted the petition and declared Petitioner Lourdes in
default. The CA also ruled that the Petitioner Lourdes was validly served with summons. Hence, this petition. Petitioner Lourdes
contends, among others, that there was no valid substituted service.

Issue:

Whether or not the summons intended to Petitioner Lourdes was validly served to her husband Alfredo?

Held:

No. There was no valid service of process on Lourdes.

It will be helpful to determine first the nature of the action filed against petitioners Lourdes and Alfredo by private respondent,
whether it is an action in personam, in rem or quasi in rem. This is because the rules on service of summons embodied in Rule 14
apply according to whether an action is one or the other of these actions.

In an action in personam, personal service of summons or, if this is not possible and he cannot be personally served, substituted
service, as provided in Rule 14, 7-8 is essential for the acquisition by the court of jurisdiction over the person of a defendant who does
not voluntarily submit himself to the authority of the court. If defendant cannot be served with summons because he is temporarily
abroad, but otherwise he is a Philippine resident, service of summons may, by leave of court, be made by publication. Otherwise
stated, a resident defendant in an action in personam, who cannot be personally served with summons, may be summoned either by
means of substituted service in accordance with Rule 14, 8 or by publication as provided in 17 and 18 of the same Rule.

If the action is in rem or quasi in rem, jurisdiction over the person of the defendant is not essential for giving the court jurisdiction so
long as the court acquires jurisdiction over the res. If the defendant is a nonresident and he is not found in the country, summons may
be served extraterritorially in accordance with Rule 14, Section 17. Service of summons in the manner provided in Sec. 17 is not for
the purpose of vesting it with jurisdiction but for complying with the requirements of fair play or due process, so that he will be
informed of the pendency of the action against him and the possibility that property in the Philippines belonging to him or in which he
has an interest may be subjected to a judgment in favor of the plaintiff and he can thereby take steps to protect his interest if he is so
minded.

Applying the foregoing rules to this case, private respondent’s action, which is for partition and accounting under Rule 69, is in the
nature of an action quasi in rem. Such an action is essentially for the purpose of affecting the defendant’s interest in a specific
property and not to render a judgment against him. As petitioner Lourdes A. Valmonte is a nonresident who is not found in the
Philippines, service of summons on her must be in accordance with Rule 14, Section 17. Such service, to be effective outside the
Philippines, must be made either (1) by personal service; (2) by publication in a newspaper of general circulation in such places and
for such time as the court may order, in which case a copy of the summons and order of the court should be sent by registered mail to
the last known address of the defendant; or (3) in any other manner which the court may deem sufficient.
Remedial Law I| Page 94

Since the service of summons upon petitioner Lourdes was not done by means of any of the first two modes, the question is whether
the service on her attorney, petitioner Alfredo, can be justified under the third mode, namely, in any . . . manner the court may deem
sufficient. The Court holds it cannot. This mode of service, like the first two, must be made outside the  Philippines, such as through
the Philippine Embassy in the foreign country where the defendant resides. Moreover, there are several reasons why the service of
summons on Atty. Alfredo D. Valmonte cannot be considered a valid service of summons on petitioner Lourdes. In the first place,
service of summons on petitioner Alfredo D. Valmonte was not made upon the order of the court as required by Rule 14, Sec. 17 and
certainly was not a mode deemed sufficient by the court which in fact refused to consider the service to be valid and, on that basis,
declare petitioner Lourdes in default for her failure to file an answer. In the second place, service in the attempted manner on
petitioner was not made upon prior leave of the trial court as required also in Rule 14, Section 17. As provided in Section 19, such
leave must be applied for by motion in writing, supported by affidavit of the plaintiff or some person on his behalf and setting forth
the grounds for the application. Finally, and most importantly, because there was no order granting such leave, petitioner Lourdes was
not given ample time to file her Answer which, according to the rules, shall be not less than sixty (60) days after notice.
Remedial Law I| Page 95

Note: The doctrine of substantial compliance is no longer controlling.

Millenium Industrial Commercial Corp. vs. Tan


G.R. No. 131724, February 28, 2000

Facts:
Millenium Industrial Commercial Corporation, executed a Deed of Real Estate Mortgage over its real property in favor of
respondent Tan. The mortgage was executed to secure payment of petitioner's indebtedness to respondent. Thereafter, respondent filed
against petitioner a complaint for foreclosure of mortgage in the Regional Trial Court. The summons and a copy of the complaint were
served upon petitioner through a certain Lynverd Cinches, described in the sheriff's return, as "a Draftsman, a person of sufficient age
and (discretion) working therein, he is the highest ranking officer or Officer-in-Charge of defendant's Corporation, to receive
processes of the Court."

Petitioner moved for the dismissal of the complaint on the ground that there was no valid service of summons upon it, as a result of
which the trial court did not acquire jurisdiction over it. Petitioner invoked Rule 14, Section13 of the 1964 Rules of Court and
contended that service on Lynverd Cinches, as alleged in the sheriff's return, was invalid as he is not one of the authorized persons on
whom summons may be served and that, in fact, he was not even its employee. The trial court denied its motion to dismiss on the
ground that the petitioner is deemed to have voluntarily submitted itself to the jurisdiction of the Court by interposing an affirmative
defense that the obligation sought to be collected was already paid and extinguished. Later on, Petitioner then filed a petition
for certiorari in the Court of Appeals but was dismissed. The appellate court ruled that although petitioner denied Lynverd Cinches'
authority to receive summons for it, its actual receipt of the summons could be inferred from its filing of a motion to dismiss, hence,
the purpose for issuing summons had been substantially achieved. Hence, this petition. Petitioner alleges that the enumeration of
persons on whom service of summons on a corporation may be effected in Rule 14, Section 13, is exclusive and mandatory, thus the
doctrine of substantial compliance does not apply.

Issue:

Whether or not the service of summons to the Draftsman when the defendant is a corporation is valid?

Held:

No. Petitioner contends that the enumeration in Rule 14, Sec. 13 is exclusive and that service of summons upon one who is not
enumerated therein is invalid. This is the general rule. However, it is settled that substantial compliance by serving summons on
persons other than those mentioned in the above rule may be justified. In G & G Trading Corporation v. Court of Appeals, the Court
ruled that although the service of summons was made on a person not enumerated in Rule 14, Section 13, if it appears that the
summons and complaint were in fact received by the corporation, there is substantial compliance with the rule as its purpose has been
attained.

In Porac Trucking, Inc. v. Court of Appeals, the Court enumerated the requisites for the application of the doctrine of substantial
compliance, to wit: (a) there must be actual receipt of the summons by the person served, i.e., transferring possession of the copy of
the summons from the Sheriff to the person served; (b) the person served must sign a receipt or the sheriff's return; and (c) there must
be actual receipt of the summons by the corporation through the person on whom the summons was actually served. The third
requisite is the most important for it is through such receipt that the purpose of the rule on service of summons is attained.

In this case, there is no dispute that the first and second requisites were fulfilled. With respect to the third, the appellate court held
that petitioner's filing of a motion to dismiss the foreclosure suit is proof that it received the copy of the summons and the complaint.
There is, however, no direct proof of this or that Lynverd Cinches actually turned over the summons to any of the officers of the
corporation. For there to be substantial compliance, actual receipt of summons by the corporation through the person served must be
shown. Where a corporation only learns of the service of summons and the filing of the complaint against it through some
person or means other than the person actually served, the service of summons becomes meaningless. This is particularly true in
the present case where there is serious doubt if Lynverd Cinches, the person on whom service of summons was effected, is in fact an
employee of the corporation. Except for the sheriff's return, there is nothing to show that Lynverd Cinches was really a draftsman
employed by the corporation. The question is whether it is allowable to merely infer actual receipt of summons by the corporation
through the person on whom summons was served.
Remedial Law I| Page 96

E.B. Villarosa vs. Benito


G.R. No. 136426August 6, 1999

Facts:

Petitioner E.B. Villarosa & Partner Co., Ltd. is a limited partnership with principal office address at 102 Juan Luna St., Davao City
and with branch offices at 2492 Bay View Drive, Tambo, Parañaque, Metro Manila and Kolambog, Lapasan, Cagayan de Oro City.
Imperial Development, as plaintiff, filed a Complaint for Breach of Contract and Damages against petitioner, as defendant, before the
Regional Trial Court of Makati allegedly for failure of the latter to comply with its contractual obligation. Summons, together with the
complaint, were served upon E.B. Villarosa, through its Branch Manager Engr. Wendell Sabulbero at the stated address at Kolambog,
Lapasan, Cagayan de Oro City, but the Sheriffs Return of Service stated that the summons was duly served upon defendant E. B.
Villarosa & Partner Co., Ltd. thru its Branch Manager Engr. WENDELL SALBULBERO at their new office Villa Gonzalo, Nazareth,
Cagayan de Oro City, and evidenced by the signature on the face of the original copy of the summons.

E.B. Villarosa filed a Motion to Dismiss, which it prayed for the dismissal of the complaint on the ground of improper service of
summons and for lack of jurisdiction over the person of the defendant. E.B. Villarosa contends that the trial court did not acquire
jurisdiction over its person since the summons was improperly served upon its employee in its branch office at Cagayan de Oro City
who is not one of those persons named in Section 11, Rule 14 of the 1997 Rules of Civil Procedure upon whom service of summons
may be made. Imperial Development filed an Opposition to Defendants Motion to Dismiss alleging that the records show that E.B.
Villarosa, through its branch manager, actually received the summons and the complaint as evidenced by the signature appearing on
the copy of the summons, among others. The trial court issued an Order denying E.B. Villarosa’s Motion to Dismiss. The trial court
stated that since the summons and copy of the complaint were in fact received by the corporation through its branch manager, there
was substantial compliance with the rule on service of summons and consequently, it validly acquired jurisdiction over the person of
the E.B. Villarosa.

E.B. Villarosa, by Special Appearance, filed a Motion for Reconsideration alleging that Section 11, Rule 14 of the new Rules did not
liberalize but, on the contrary, restricted the service of summons on persons enumerated therein; and that the new provision is very
specific and clear in that the word manager was changed to general manager, secretary to corporate secretary, and excluding therefrom
agent and director. Further, in its Reply, it contends that the changes in the new rules are substantial and not just general semantics.
E.B. Villarosa’s Motion for Reconsideration was denied, hence, the present petition alleging that, respondent court gravely abused its
discretion tantamount to lack or in excess of jurisdiction in denying petitioners motions to dismiss and for reconsideration, despite the
fact that the trial court did not acquire jurisdiction over the person of petitioner because the summons intended for it was improperly
served. Petitioner invokes Section 11 of Rule 14 of the 1997 Rules of Civil Procedure.

Issue:

Whether or not the trial court acquired jurisdiction over the person of the corporation upon service of summons on its Branch
Manager?

Held:

No. The service of summons to the branch manager of the Petitioner is invalid. The Court ruled that when the complaint was filed by
Petitioner on April 3, 1998, the 1997 Rules of Civil Procedure was already in force.

Section 11, Rule 14 of the 1997 Rules of Civil Procedure provides that:

“When the defendant is a corporation, partnership or association organized under the laws of the Philippines with
a juridical personality, service may be made on the president, managing partner, general manager, corporate secretary,
treasurer, or in-house counsel.”

Petitioner contends that the enumeration of persons to whom summons may be served is restricted, limited and exclusive following the
rule on statutory construction expressio unios est exclusio alterius and argues that if the Rules of Court Revision Committee intended
to liberalize the rule on service of summons, it could have easily done so by clear and concise language. The Court agrees with
petitioner. Notably, under the new Rules, service of summons upon an agent of the corporation is no longer authorized.

The designation of persons or officers who are authorized to accept summons for a domestic corporation or partnership is now limited
and more clearly specified in Section 11, Rule 14 of the 1997 Rules of Civil Procedure.  The rule now states general manager instead
Remedial Law I| Page 97

of only manager; corporate secretary instead of secretary; and treasurer instead of cashier. The phrase agent, or any of its directors is
conspicuously deleted in the new rule.

Retired Justice Oscar Herrera, who is also a consultant of the Rules of Court Revision Committee, stated that (T)he rule must be
strictly observed. Service must be made to one named in (the) statute x x x.

It should be noted that even prior to the effectivity of the 1997 Rules of Civil Procedure, strict compliance with the rules has been
enjoined. In the case of Delta Motor Sales Corporation vs. Mangosing, the Court held:

“A strict compliance with the mode of service is necessary to confer jurisdiction of the court over a corporation.  The
officer upon whom service is made must be one who is named in the statute; otherwise the service is insufficient. x x x.”

Accordingly, the Court ruled that the service of summons upon the branch manager of petitioner at its branch office at Cagayan de
Oro, instead of upon the general manager at its principal office at Davao City is improper.  Consequently, the trial court did not acquire
jurisdiction over the person of the petitioner.
Remedial Law I| Page 98

Mason vs. Court of Appeals


G.R. No. 144662 October 13, 2003

Facts:

Spouses Mason owned two parcels of land located along EDSA in Pasay City. They entered into a lease contract with Columbus
Philippines Bus Corporation, where Columbus undertook to construct a building worth P10 million at the end of the third year of the
lease. For failure to comply with this stipulation, the Spouses filed a complaint for rescission of contract with damages against
Columbus before the RTC. Summons was served upon Columbus through a certain Ayreen Rejalde. While the receiving copy of the
summons described her as a secretary of Columbus, the sheriff’s return indicated that she is a secretary to the corporate president, duly
authorized to receive legal processes.

Columbus failed to file its answer, hence, spouses Mason filed a motion to declare it in default. The motion was granted and the
spouses were allowed to present evidence ex-parte. Thereafter, the case was submitted for decision. The trial court ruled in favor of
the spouses. That decision became final on May 12, 1999. The following day, Columbus filed a motion to lift order of default, which
was opposed by the spouses. The trial court ordered the parties to submit their respective memoranda. However, without waiting for
the same, the trial court denied the motion to lift order of default on the ground that the decision has already become final and
executory. Consequently, the spouses’ Motion for Execution was granted. Columbus filed a motion for reconsideration, which was
denied. Undaunted, it filed a manifestation and motion to lift the writ of execution but was also denied for being dilatory. It then
appealed to the CA, which ruled in its favor. The CA pointed out that Columbus was not properly served with summons, thus it cannot
be faulted if it failed to file an Answer. Spouses Mason filed a motion for reconsideration, but to no avail. Hence, this petition for
review.

Spouses Mason stressed that even though the summons was received by a mere filing clerk of Columbus, there was substantial
compliance with Section 11, Rule 14 because the summons actually reached said corporation. This can be gleaned from the motion to
lift order of default where Columbus did not question the validity of the service of summons but explained that its failure to answer
was due to its impression that the case would not be pursued by the spouses on the ground that payments were already made to them.
The spouses also cited Millenium Industrial Commercial Corporation v. Tan, and maintained that this Court, by referring to E.B
Villarosa & Partner Co., Ltd. v. Judge Benito, effectively ruled that said provision is the statement of the general rule on service of
summons upon corporation and the substantial compliance rule is the exception.

Issue:

Whether or not there was valid service of summons on Columbus for the trial court to acquire jurisdiction?

Held:

No. The question of whether the substantial compliance rule is still applicable under Section 11, Rule 14 of the 1997 Rules of Civil
Procedure has been settled in E.B. Villarosa which applies squarely to the instant case. In the said case, the Court held that there was
no valid service of summons on Villarosa as service was made through a person not included in the enumeration in Section 11, Rule
14 of the 1997 Rules of Civil Procedure, which revised Section 13, Rule 14 of the 1964 Rules of Court. Also in that case, the Court
discarded substantial compliance with the rule on service of summons, and ruled that the enumeration under the new rule is restricted,
limited and exclusive, following the rule in statutory construction that expressio unios est exclusio alterius. Had the Rules of Court
Revision Committee intended to liberalize the rule on service of summons, it could have easily done so by clear and concise language.
Absent a manifest intent to liberalize the rule, strict compliance with Section 11, Rule 14 of the 1997 Rules of Civil Procedure is
stressed. Neither can herein petitioners invoke the ruling in Millenium to support their position for said case is not on all fours with the
instant case. It must be stressed that Millenium was decided when the 1964 Rules of Court were still in force and effect, unlike the
instant case which falls under the new rule. Hence, the cases cited by spouses Mason where the doctrine of substantial compliance was
upheld must be deemed overturned by Villarosa, which is the later case.

At this juncture, it is worth emphasizing that notice to enable the other party to be heard and to present evidence is not a mere
technicality or a trivial matter in any administrative or judicial proceedings. The service of summons is a vital and indispensable
ingredient of due process. The Court will deprive Columbus of its right to present its defense in this multi-million peso suit, if
compliance with the rules on service of summons will be disregarded.

Jose vs. Boyon


Remedial Law I| Page 99

G.R. No. 147369 October 23, 2003

Facts:

Spouses Jose lodged a complaint for specific performance against Spouses Boyon to compel them to facilitate the transfer of
ownership of a parcel of land subject of a controverted sale. Public respondent Judge Perello issued summons to Spouses Boyon. As
per return of the summons, substituted service was resorted to by the process server allegedly because efforts to serve the summons
personally to Spouses Boyon failed. Spouses Jose filed an Ex-parte Motion for Leave of Court to Effect Summons by Publication
which the RTC granted. Thereafter, the RTC judge, without a written motion, issued an Order declaring Spouses Boyon in default for
failure to file their respective answers. Spouses Jose were then allowed to submit their evidence ex-parte. Ultimately, the RTC ruled in
favor of Spouses Jose.

Helen Boyon, who was then residing in USA, was surprised to learn from her sister of the resolution issued by the RTC. Spouses
Boyon then filed an Ad Cautelam motion questioning, among others, the validity of the service of summons effected by the court  a
quo. The RTC denied the said motion, stating that the defaulted spouses already lost their standing in court. Spouses Boyon filed a
motion for reconsideration but it was denied. Meanwhile, Spouses Jose moved for the execution of the RTC judgment and the same
was granted.

Spouses Boyon filed before the CA a Petition for certiorari questioning the jurisdiction of the RTC. The CA ruled that the RTC never
acquired jurisdiction over Spouses Boyon because of the invalid service of summons upon them. First, the sheriff failed to comply
with the requirements of substituted service of summons because he did not specify in the Return of Summons the prior efforts he had
made to locate them and the impossibility of promptly serving the summons upon them by personal service. Second, the subsequent
summons by publication was equally infirm, because the Complaint was a suit for specific performance and therefore an action in
personam.

Thus, the present Petition for Review on Certiorari under Rule 45. Spouses Jose aver, among others, that the CA erred in ruling that
the service of summons was invalid. They submit that although the case filed before the trial court was denominated as an action for
specific performance, it was actually an action quasi in rem, because it involved a piece of real property located in the Philippines.

Issue:

Whether or not the service of summons upon Spouses Boyon is valid?

Held:

No.In general, trial courts acquire jurisdiction over the person of the defendant by the service of summons. Where the action is in
personam and the defendant is in the Philippines, such service may be done by personal or substituted service, following the
procedures laid out in Sections 6 and 7 of Rule 14 of the Revised Rules of Court, which read:

"Section 6. Service in person on defendant. - Whenever practicable, the summons shall be served by handing a copy thereof to the
defendant in person, or, if he refuses to receive and sign for it, by tendering it to him.

"Section 7. Substituted service. - If, for justifiable causes, the defendant cannot be served within a reasonable time as provided in the
preceding section, service may be effected (a) by leaving copies of the summons at the defendant's residence with some person of
suitable age and discretion then residing therein, or (b) by leaving the copies at defendant’s office or regular place of business with
some competent person in charge thereof."

As can be gleaned from the above-quoted Sections, personal service of summons is preferred to substituted service. Only if the former
cannot be made promptly can the process server resort to the latter. Moreover, the proof of service of summons must:

(a) indicate the impossibility of service of summons within a reasonable time;

(b) specify the efforts exerted to locate the defendant; and,

(c) state that the summons was served upon a person of sufficient age and discretion who is residing in the address, or
who is in charge of the office or regular place of business, of the defendant.
Remedial Law I| Page 100

It is likewise required that the pertinent facts proving these circumstances be stated in the proof of service or in the officer’s return.
The failure to comply faithfully, strictly and fully with all the foregoing requirements of substituted service renders the service of
summons ineffective.

In the instant case, it appears that the process server hastily and capriciously resorted to substituted service of summons without
actually exerting any genuine effort to locate Spouses Boyon. The only effort he exerted was to go to No. 32 Ariza Drive, Camella
Homes, Alabang, to try to serve the summons personally. While the Return of Summons states that efforts to do so were ineffectual
and unavailing because Helen Boyon was in the United States and Romeo Boyon was in Bicol, it did not mention exactly what efforts
-- if any -- were undertaken to find respondents. Furthermore, it did not specify where or from whom the process server obtained the
information on their whereabouts.

As to the Service of Summons by Publication or Extraterritorial Service of Summons, it applies only when the action is  in
rem or quasi in rem. In the instant case, what was filed before the trial court was an action for specific performance directed against
respondents. While the suit incidentally involved a piece of land, the ownership or possession thereof was not put in issue, since they
did not assert any interest or right over it. Moreover, this Court has consistently declared that an action for specific performance is
an action in personam.
Remedial Law I| Page 101

Manotoc vs. Court of Appeals


G.R. No. 130974 August 16, 2006

Facts:

The Estate of Trajano seeks the enforcement of a foreign court’s judgment rendered by the United States District Court of Honolulu,
Hawaii, USA in a case against Imee Marcos-Manotoc for the wrongful death of deceased Archimedes Trajano committed by military
intelligence officials of the Philippines allegedly under the command, direction, authority, supervision, tolerance, sufferance and/or
influence of Manotoc, pursuant to the provisions of Rule 39 of the then Revised Rules of Court. The trial court issued a
Summons addressed to Manotoc at Alexandra Condominium Corporation or Alexandra Homes, E2 Room 104, at No. 29 Meralco
Avenue, Pasig City. The Summons and a copy of the Complaint were allegedly served upon (Mr.) Macky de la Cruz, an alleged
caretaker of Manotoc at that condominium unit. When Manotoc failed to file her Answer, the trial court declared her in default
through an Order.

Manotoc, by special appearance of counsel, filed a Motion to Dismiss on the ground of lack of jurisdiction of the trial court over her
person due to an invalid substituted service of summons, averring that: (1) the address of defendant indicated in the Complaint was not
her dwelling, residence, or regular place of business; (2) the party (de la Cruz), who was found in the unit, was neither a
representative, employee, nor a resident of the place; (3) the procedure prescribed by the Rules on personal and substituted service of
summons was ignored; (4) defendant was a resident of Singapore; and (5) whatever judgment rendered in this case would be
ineffective and futile.

During the hearing on the Motion to Dismiss, Manotoc presented Carlos Gonzales, who testified that he saw defendant Manotoc as a
visitor in Alexandra Homes only two times. She also presented her Philippine passport and the Disembarkation/Embarkation
Card issued by the Immigration Service of Singapore to show that she was a resident of Singapore. She claimed that the person
referred to as "Mrs. Manotoc" may not even be her, but the mother of one Tommy Manotoc. On the other hand, Trajano presented the
lead counsel in a case involving the Marcoses who testified that he participated in the deposition taking of Marcos, Jr. The counsel
confirmed that Marcos, Jr. testified that Manotoc’s residence was at the Alexandra Apartment, Greenhills.

The trial court rejected Manotoc’s Motion to Dismiss, relying on the presumption that the sheriff’s substituted service was made in the
regular performance of official duty, and that such presumption stood in the absence of proof to the contrary.   The trial court likewise
discarded Manotoc’s plea for reconsideration for lack of merit.  Manotoc then filed a Petition for Certiorari and Prohibition before the
CA. The CA adopted the findings of the trial court. Manotoc filed a Motion for Reconsideration but was denied. Hence, the present
petition for review on certiorari.

Issue:

Whether or not there is a valid substituted service of summons for the trial court to acquire jurisdiction over Manotoc?

Held:

No. A meticulous scrutiny of the Sheriff’s Return readily reveals the absence of material data on the serious efforts to serve the
Summons on Manotoc in person. There is no clear valid reason cited in the Return why those efforts proved inadequate, to
reach the conclusion that personal service has become impossible or unattainable outside the generally couched phrases of "on
many occasions several attempts were made to serve the summons x x x personally," "at reasonable hours during the day," and "to no
avail for the reason that the said defendant is usually out of her place and/or residence or premises."

Besides, apart from the allegation of Manotoc’s address in the Complaint, it has not been shown that respondent Trajano or Sheriff
Cañelas, who served such summons, exerted extraordinary efforts to locate Mantooc. Certainly, the second paragraph of the
Complaint only states that respondents were "informed, and so [they] allege" about the address and whereabouts of Manotoc. Before
resorting to substituted service, a plaintiff must demonstrate an effort in good faith to locate the defendant through more direct means.
More so, in the case in hand, when the alleged petitioner’s residence or house is doubtful or has not been clearly ascertained, it would
have been better for personal service to have been pursued persistently.

Also, there is a serious nonconformity from the requirement that the summons must be left with a "person of suitable age and
discretion" residing in defendant’s house or residence.
Remedial Law I| Page 102

There are two (2) requirements under the Rules: (1) recipient must be a person of suitable age and discretion; and (2) recipient must
reside in the house or residence of defendant. Both requirements were not met. In this case, the Sheriff’s Return lacks information as
to residence, age, and discretion of Mr. De la Cruz, aside from the sheriff’s general assertion that de la Cruz is the "resident caretaker"
of Manotoc as pointed out by the alleged receptionist and telephone operator of Alexandra Homes. It is doubtful if Mr. De la Cruz is
residing with Manotoc in the condominium unit considering that a married woman of her stature in society would unlikely hire a male
caretaker to reside in her dwelling. Besides, Mr. De la Cruz’s refusal to sign the Receipt for the summons is a strong indication that he
did not have the necessary "relation of confidence" with Manotoc. To protect Manotoc’s right to due process by being accorded
proper notice of a case against her, the substituted service of summons must be shown to clearly comply with the rules. Due to non-
compliance with the prerequisites for valid substituted service, the proceedings held before the trial court perforce must be annulled.

Discussion:

Requirements for Substituted Service

Section 8 of Rule 14 of the old Revised Rules of Court which applies to this case provides:

SEC. 8.  Substituted service. – If the defendant cannot be served within a reasonable time as provided in the preceding section
[personal service on defendant], service may be effected (a) by leaving copies of the summons at the defendant’s residence with some
person of suitable age and discretion then residing therein, or (b) by leaving the copies at defendant’s office or regular place of
business with some competent person in charge thereof.

We can break down this section into the following requirements to effect a valid substituted service:

(1) Impossibility of Prompt Personal Service

The party relying on substituted service or the sheriff must show that defendant cannot be served promptly or there is impossibility of
prompt service. Section 8, Rule 14 provides that the plaintiff or the sheriff is given a "reasonable time" to serve the summons to the
defendant in person, but no specific time frame is mentioned.

"Reasonable time" is defined as "so much time as is necessary under the circumstances for a reasonably prudent and diligent man to
do, conveniently, what the contract or duty requires that should be done, having a regard for the rights and possibility of loss, if any[,]
to the other party."

To the plaintiff, "reasonable time" means no more than seven (7) days since an expeditious processing of a complaint is what a
plaintiff wants. To the sheriff, "reasonable time" means 15 to 30 days because at the end of the month, it is a practice for the branch
clerk of court to require the sheriff to submit a return of the summons assigned to the sheriff for service. The Sheriff’s Return provides
data to the Clerk of Court, which the clerk uses in the Monthly Report of Cases to be submitted to the Office of the Court
Administrator within the first ten (10) days of the succeeding month. Thus, one month from the issuance of summons can be
considered "reasonable time" with regard to personal service on the defendant.

Sheriffs are asked to discharge their duties on the service of summons with due care, utmost diligence, and reasonable promptness and
speed so as not to prejudice the expeditious dispensation of justice. Thus, they are enjoined to try their best efforts to accomplish
personal service on defendant. On the other hand, since the defendant is expected to try to avoid and evade service of summons, the
sheriff must be resourceful, persevering, canny, and diligent in serving the process on the defendant . For substituted service of
summons to be available, there must be several attempts by the sheriff to personally serve the summons within a reasonable period [of
one month] which eventually resulted in failure to prove impossibility of prompt service.

"Several attempts" means at least three (3) tries, preferably on at least two different dates. In addition, the sheriff must cite why such
efforts were unsuccessful. It is only then that impossibility of service can be confirmed or accepted.

(2) Specific Details in the Return

The sheriff must describe in the Return of Summons the facts and circumstances surrounding the attempted personal service. The
efforts made to find the defendant and the reasons behind the failure must be clearly narrated in detail in the Return. The date and
time of the attempts on personal service, the inquiries made to locate the defendant, the name/s of the occupants of the alleged
residence or house of defendant and all other acts done, though futile, to serve the summons on defendant must be specified in the
Return to justify substituted service. The form on Sheriff’s Return of Summons on Substituted Service prescribed in the Handbook for
Remedial Law I| Page 103

Sheriffs published by the Philippine Judicial Academy and the SC Administrative Circular No. 5 (1989) require a narration of the
efforts made to find the defendant personally and the fact of failure.

(3) A Person of Suitable Age and Discretion

If the substituted service will be effected at defendant’s house or residence, it should be left with a person of "suitable age and
discretion then residing therein."

A person of suitable age and discretion is one who has attained the age of full legal capacity (18 years old) and is considered to have
enough discernment to understand the importance of a summons.

"Discretion" is defined as "the ability to make decisions which represent a responsible choice and for which an understanding of
what is lawful, right or wise may be presupposed".

Thus, to be of sufficient discretion, such person must know how to read and understand English to comprehend the import of the
summons, and fully realize the need to deliver the summons and complaint to the defendant at the earliest possible time for the person
to take appropriate action. Thus, the person must have the "relation of confidence" to the defendant, ensuring that the latter would
receive or at least be notified of the receipt of the summons. The sheriff must therefore determine if the person found in the alleged
dwelling or residence of defendant is of legal age, what the recipient’s relationship with the defendant is, and whether said person
comprehends the significance of the receipt of the summons and his duty to immediately deliver it to the defendant or at least notify
the defendant of said receipt of summons. These matters must be clearly and specifically described in the Return of Summons.

(4) A Competent Person in Charge

If the substituted service will be done at defendant’s office or regular place of business, then it should be served on a competent person
in charge of the place. Thus, the person on whom the substituted service will be made must be the one managing the office or
business of defendant, such as the president or manager; and such individual must have sufficient knowledge to understand the
obligation of the defendant in the summons, its importance, and the prejudicial effects arising from inaction on the summons. Again,
these details must be contained in the Return.
Remedial Law I| Page 104

DOLE Philippines vs. Quilala


G.R. No. 168723, July 9, 2008

Facts:

All Season Farm Corporation sought the recovery of a sum of money, accounting and damages Dole Philippines, Inc. (Tropifresh
Division) (DOLE) and several of its officers in Makati City RTC. According to DOLE, an alias summons was served upon it through
a certain Marifa Dela Cruz, a legal assistant employed by DOLE Pacific General Services, Ltd., which is an entity separate from
Dole. Thus, DOLE filed a Motion to Dismiss the complaint on the following grounds: (a) the RTC lacked jurisdiction over the person
of Dole due to improper service of summons; (b) the complaint failed to state a cause of action; (c) All Season was not the real party
in interest; and (d) the officers of Dole cannot be sued in their personal capacities for alleged acts performed in their official capacities
as corporate officers of Dole. This was denied by the RTC.

In the CA, DOLE filed a petition for certiorari contending that the alias summons was not properly served. The appellate court,
however, ruled otherwise. It reasoned that DOLE’s president had known of the service of the alias summons although he did not
personally receive and sign it. It also held that in today’s corporate setup, documents addressed to corporate officers are received in
their behalf by their staff.

In this Petition for Review, DOLE assails the validity of the service of summons on it consequently questioning whether or not
jurisdiction over it was acquired by the RTC. nt All Season, for its part, contends that the trial court had acquired jurisdiction over
petitioner, since petitioner received the alias summons through its president. More so, petitioner had admitted that it received the alias
summons in its Entry of Appearance with Motion for Time filed on May 5, 2003.

Issue:

Whether or not the service of summons was valid?

Held:

No. Considering that the service of summons was made on a legal assistant, not employed by herein petitioner and who is not one of
the designated persons under Section 11, Rule 14, the trial court did not validly acquire jurisdiction over petitioner. Well-settled is the
rule that service of summons on a domestic corporation is restricted, limited and exclusive to the persons enumerated in Section 11,
Rule 14 of the 1997 Rules of Civil Procedure, following the rule in statutory construction that expressio unios est exclusio
alterius. Service must therefore be made on the president, managing partner, general manager, corporate secretary, treasurer, or in-
house counsel.

Contrary to private respondents claim that it was received upon instruction of the president of the corporation as indicated in the
Officers Return, such fact does not appear in the receiving copy of the alias summons which Marifa Dela Cruz signed.  There was no
evidence that she was authorized to receive court processes in behalf of the president.
HOWEVER, under Section 20 of the same Rule, a defendants voluntary appearance in the action is equivalent to service of
summons. Note that on May 5, 2003, petitioner filed an Entry of Appearance with Motion for Time. It was not a conditional
appearance entered to question the regularity of the service of summons, but an appearance submitting to the jurisdiction of the court
by acknowledging the receipt of the alias summons and praying for additional time to file responsive pleading. Consequently,
petitioner having acknowledged the receipt of the summons and also having invoked the jurisdiction of the RTC to secure affirmative
relief in its motion for additional time, petitioner effectively submitted voluntarily to the jurisdiction of the RTC.
Remedial Law I| Page 105

Santos vs. PNOC


G. R. No. 170943, September 23, 2008

Facts:

PNOC Exploration Corporation filed a complaint for a sum of money against petitioner Pedro T. Santos, Jr. in the Regional Trial
Court of Pasig City. It sought to collect the amount of P698,502.10 representing petitioner’s unpaid balance of the car loan advanced
to him by respondent when he was still a member of its board of directors. Personal service of summons to petitioner failed because he
could not be located in his last known address despite earnest efforts to do so. Subsequently, on respondent’s motion, the trial court
allowed service of summons by publication which was published in REMATE, a newspaper of general circulation.

When petitioner failed to file his answer within the prescribed period, respondent moved that the case be set for the reception of its
evidence ex parte which the trial court granted. After it has presented evidence ex parte, the case was submitted for decision. Days
after such submission, Santos filed an Omnibus Motion for Reconsideration and to Admit Attached Answer. He sought
reconsideration of the order stating that alleging that the affidavit of service submitted by respondent failed to comply with Section 19,
Rule 14 of the Rules of Court as it was not executed by the clerk of court. The motion was denied by the RTC. It held that the rules
did not require the affidavit of complementary service by registered mail to be executed by the clerk of court.

Santos elevated the matter to the CA via petition for certiorari. During the pendency of such petition, the RTC rendered its decision
ordering Santos to pay PNOC. Meanwhile, the CA dismissed the petition. Now in this petition for review, Santos maintains that the
RTC did not acquire jurisdiction over his person due to improper service of summons.

Issue:

Whether or not service by publication is proper in actions in personam?

Held:

Yes. Petitioner invokes the distinction between an action in rem and an action in personam and claims that substituted service may be
availed of only in an action in rem. Petitioner is wrong. The in rem/in personam distinction was significant under the old rule because
it was silent as to the kind of action to which the rule was applicable. Because of this silence, the Court limited the application of the
old rule to in rem actions only. This has been changed. The present rule expressly states that it applies [i]n any action where the
defendant is designated as an unknown owner, or the like, or whenever his whereabouts are unknown and cannot be ascertained by
diligent inquiry. Thus, it now applies to any action, whether in personam, in rem or quasi in rem.

Service of summons by publication is proved by the affidavit of the printer, his foreman or principal clerk, or of the editor, business or
advertising manager of the newspaper which published the summons. The service of summons by publication is complemented by
service of summons by registered mail to the defendants last known address. This complementary service is evidenced by an affidavit
showing the deposit of a copy of the summons and order for publication in the post office, postage prepaid, directed to the defendant
by registered mail to his last known address. The rules, however, do not require that the affidavit of complementary service be
executed by the clerk of court. While the trial court ordinarily does the mailing of copies of its orders and processes, the duty to make
the complementary service by registered mail is imposed on the party who resorts to service by publication.

Moreover, even assuming that the service of summons was defective, the trial court acquired jurisdiction over the person of petitioner
by his own voluntary appearance in the action against him. Petitioner voluntarily appeared in the action when he filed the Omnibus
Motion for Reconsideration and to Admit Attached Answer. This was equivalent to service of summons and vested the trial court with
jurisdiction over the person of petitioner.
Remedial Law I| Page 106

Ong vs. Co
G.R. No. 206653, February 25, 2015

Facts:

Petitioner Yuk Ling Ong, a British-Hong Kong national, and respondent Benjamin Co a Filipino citizen, were married in the
Philippines. Respondent filed a petition for declaration of nullity on the ground of psychological incapacity before the RTC.
Respondent indicated that petitioner’s address was 23 Sta. Rosa Street, Unit B-2 Manresa Garden Homes, Quezon City. The RTC
issued summons. In his Server’s Return, process server Rodolfo Torres, Jr. stated that, on August 1, 2002, substituted service of
summons with the copy of the petition was effected after several futile attempts to serve the same personally on petitioner. The said
documents were received by Mr. Roly Espinosa, a security officer. The RTC found respondent’s marriage with petitioner as void  ab
initio on the ground of psychological incapacity under Article 36 of the Family Code. It stated that summons was served on petitioner
on August 1, 2002, but she failed to file her responsive pleading within the reglementary period. The public prosecutor also stated that
there were no indicative facts to manifest collusion. Thus, the RTC concluded that petitioner was psychologically incapacitated to
perform her essential marital obligations.

Consequently, petitioner filed a petition for annulment of judgment under Rule 47 of the Rules of Court before the CA claiming that
she was never notified of the cases filed against her. She prayed that the RTC decision be nullified on the grounds of extrinsic fraud
and lack of jurisdiction. CA found the petition for annulment of judgment to be devoid of merit. It held that there was no sufficient
proof to establish that respondent employed fraud to insure petitioner’s non-participation in the trial of the aforementioned case.
Petitioner moved for reconsideration, but her motion was denied by the CA. Hence, this petition.

Issue:

Whether or not substituted service of summons was validly availed of?

Held:

No. Jurisdiction over the defendant is acquired either upon a valid service of summons or the defendant's voluntary appearance in
court. If the defendant does not voluntarily appear in court, jurisdiction can be acquired by personal or substituted service of summons
as laid out under Sections 6 and 7 of Rule 14 of the Rules of Court,

The landmark case of Manotoc v. CA (Manotoc) thoroughly discussed the rigorous requirements of a substituted service of summons,
to wit: xxx
(1) Impossibility of Prompt Personal Service
For substituted service of summons to be available, there must be several attempts by the sheriff to personally serve the summons
within a reasonable period of one month which eventually resulted in failure to prove impossibility of prompt service. "Several
attempts" means at least three (3) tries, preferably on at least two different dates. In addition, the sheriff must cite why such efforts
were unsuccessful. 

(2) Specific Details in the Return


The sheriff must describe in the Return of Summons the facts and circumstances surrounding the attempted personal service. The
efforts made to find the defendant and the reasons behind the failure must be clearly narrated in detail in the Return. The date and
time of the attempts on personal service, the inquiries made to locate the defendant, the name/s of the occupants of the alleged
residence or house of defendant and all other acts done, though futile, to serve the summons on defendant must be specified in the
Return to justify substituted service.

(3) Person of Suitable Age and Discretion]


The sheriff must therefore determine if the person found in the alleged dwelling or residence of defendant is of legal age, what the
recipient's relationship with the defendant is, and whether said person comprehends the significance of the receipt of the summons
and his duty to immediately deliver it to the defendant or at least notify the defendant of said receipt of summons. These matters must
be clearly and specifically described in the Return of Summons. (Emphases and underscoring supplied)

Here, the the process server immediately opted for substituted service of summons after only two (2) days from the issuance of the
summons. The server’s return utterly lacks sufficient detail of the attempts undertaken by the process server to personally serve the
summons on petitioner. The server simply made a general statement that summons was effected after several futile attempts to serve
the same personally. The server did not state the specific number of attempts made to perform the personal service of summons; the
Remedial Law I| Page 107

dates and the corresponding time the attempts were made; and the underlying reason for each unsuccessful service. He did not explain
either if there were inquiries made to locate the petitioner, who was the defendant in the case. These important acts to serve the
summons on petitioner, though futile, must be specified in the return to justify substituted service. The server’s return did not describe
in detail the person who received the summons, on behalf of petitioner. It simply stated that the summons was received by Mr. Roly
Espinosa of sufficient age and discretion, the Security Officer thereat. It did not expound on the competence of the security officer to
receive the summons.

Also, aside from the server’s return, respondent failed to indicate any portion of the records which would describe the specific
attempts to personally serve the summons. Respondent did not even claim that petitioner made any voluntary appearance and actively
participated in Civil Case No. 02-0306.

Given that the meticulous requirements in Manotoc were not met, the Court is not inclined to uphold the CA’s denial of the petition
for annulment of judgment for lack of jurisdiction over the person of petitioner because there was an invalid substituted service of
summons. Accordingly, the decision in Civil Case No. 02-0306 must be declared null and void.
Remedial Law I| Page 108

Nation Petroleum Gas vs. RCBC


G.R. No. 183370. August 17, 2015

Facts:

Rizal Commercial Banking Corporation filed against Nation Petroleum Gas and its directors/officers a Complaint for civil damages
arising from estafa in relation to violations of the Trust Receipts Law. Thereafter, Sheriff Leodel N. Roxas served upon petitioners a
copy of the summons, as well as other pertinent documents.

Petitioners filed through counsel a Special Appearance with Motion to Dismiss . They asserted that the trial court did not acquire
jurisdiction over the corporation since the summons was improperly served upon Abante, who is a mere liaison officer and not one of
the corporate officers specifically enumerated in Section 11, Rule 14 of the Rules. Likewise, the individual petitioners argued that the
sheriff and/or process server did not personally approach them at their respective address as stated in the Complaint. Neither did he
resort to substituted service of summons, and that, even if he did, there was no strict compliance with Section 7, Rule 14 of the Rules.

Respondent countered in its Opposition with Motion to Declare Defendants in Default that there was valid service of summons upon
petitioners. With respect to the corporation, Abante received the summons upon the express authority and instruction of the corporate
secretary, petitioner Melinda Ang. As regards the individual petitioners, the Sheriffs Report reflects that they were served "at their
given addresses, but they refused to acknowledge receipt thereof."

RTC denied petitioners' motion to dismiss and respondent's motion to declare them in default. Petitioner elevated the jurisdictional
issue to the CA via petition for certiorari and prohibition. The appellate court later dismissed the petition and denied the motion for
reconsideration; hence, this petition.

Issue:

Whether or not the court acquired jurisdiction over the person of the (a) corporation and (b) individual officers of the corporation?
Held:

(a) As to the corporation – YES. When the defendant is a domestic corporation like herein petitioner, service of summons may be
made only upon the persons enumerated in Section 11, Rule 14 of the Rules. The foregoing notwithstanding, there was a valid and
effective service of summons upon petitioner corporation through its liaison officer who acted as the agent of the corporate secretary.
Abante, in so receiving the summons, did so in representation of Ang who, as corporate secretary, is one of the officers competent
under the Rules of Court to receive summons on behalf of a private juridical person. Thus, while it may be true that there was no
direct, physical handing of the summons to Ang, the latter could at least be charged with having constructively received the same,
which in Our view, amounts to a valid service of summons.

(b) As to the officers – YES. Despite improper service of summons upon their persons, the individual petitioners are deemed to have
submitted to the jurisdiction of the court through their voluntary appearance. The second sentence of Section 20 Rule 14 of the Rules
that "[t]he inclusion in a motion to dismiss of other grounds aside from lack of jurisdiction over the person of the defendant shall not
be deemed a voluntary appearance" clearly refers to affirmative defenses, not affirmative reliefs.

In the present case, the individual petitioners prayed, among others, for the following: (1) discharge of the writ of attachment on their
properties; (2) denial of the motion to declare them in default; (3) admission of the Comment/Opposition (to the motion to declare
them in default) filed on December 19, 2006; and (4) denial of respondent’s motion to strike off from the records (their opposition to
the motion to declare them in default). By seeking affirmative reliefs from the trial court, the individual petitioners are deemed to have
voluntarily submitted to the jurisdiction of said court. A party cannot invoke the jurisdiction of a court to secure affirmative relief
against his opponent and after obtaining or failing to obtain such relief, repudiate or question that same jurisdiction.
Remedial Law I| Page 109

Green Star Express vs. Nissin Universal Robina Corp.


G.R. No. 181517 July 6, 2015

Facts:

A Mitsubishi L-300 van which Universal Robina Corporation (URC) owned figured in a vehicular accident with petitioner Green Star
Express, Inc.' s (Green Star) passenger bus, resulting in the death of the van's driver. Thus, the bus driver, petitioner Fruto Sayson, Jr.,
was charged with the crime of reckless imprudence resulting in homicide.

Thereafter, Green Star sent a demand letter to respondent URC for the repair of its passenger bus amounting. URC denied any liability
therefore and argued that the criminal case shall determine the ultimate liabilities of the parties. Thereafter, the criminal case was
dismissed without prejudice, due to insufficiency of evidence. Sayson and Green Star then filed a complaint for damages against URC
before the RTC of San Pedro, Laguna. Francis Tinio, one of URC's employees, was the one who received the summons. URC filed a
Motion to Dismiss claiming lack of jurisdiction due to improper service. RTC denied URC's motion to dismiss. It ruled that there was
substantial compliance because there was actual receipt of the summons by URC. CA reversed the RTC ruling. Aggrieved, Green Star
and Sayson moved for reconsideration, but the same was denied. Hence, this petition.

Issue:
Whether or not the summons was properly served on URC, vesting the trial court with jurisdiction?

Held:

No. It is a well-established rule that the rules on service of summons upon a domestic private juridical entity must be strictly complied
with. Otherwise, the court cannot be said to have acquired jurisdiction over the person of the defendant.

Section 11. Service upon domestic private juridical entity. – When the defendant is a corporation, partnership or association
organized under the laws of the Philippines with a juridical personality, service may be made on the president, managing partner,
general manager, corporate secretary, treasurer, or in-house counsel.

Service must, be made only on the person expressly listed in the rules. Here, Tinio, a, member of URC’s accounting staff, received the
summons on January 22, 2004. Green star claims that it was received upon instruction of Junadette Avedillo. The general manager of
the corporation. Such fact, however, does not appear in the Sheriff’s Return. The Return did not even state whether Avedillo was
present at the time the summons was received by Tinio, the supposed assistant manager. Green Star further avers that the sheriff
tendered the summons, but Avedillo simply refused to sign and receive the same. She then allegedly instructed Tinio to just receive it
in her behalf. However, Green Star never presented said sheriff as witness during the hearing of URC’s motion to dismiss to attest to
said claim. And while the sheriff executed an affidavit which appears to support such allegation, the same was likewise not presented
as evidence. It was only when the case was already before the CA that said affidavit first surfaced. Since the service of summons was
made on a cost accountant, which is not one of the designated persons under Section 11 of Rule 14, the trial court did not validly
acquire jurisdiction over URC, although the corporation may have actually received the summons. To rule otherwise will be an
outright circumvention of the rules, aggravating further the delay in the administration of justice.
Remedial Law I| Page 110

Guy vs. Gacott


G.R. No. 206147. January 13, 2016
Facts:

Atty. Glenn Gacott purchased two brand new transreceivers from Quantech Systems Corporation (QSC) in Manila through its
employee Rey Medestomas. Due to major defects, Gacott personally returned the transreceivers to QSC and requested that they be
replaced. Medestomas received the returned transreceivers and promised to send him the replacement units within two (2) weeks.
Time passed and Gacott did not receive the replacement units as promised. QSC informed him that there were no available units and
that it could not refund the purchased price. Gacott then filed a Complaint for Damages before the RTC against QSC. Summons was
served upon QSC and Medestomas, after which they filed their Answer, verified by Medestomas himself and a certain Elton Ong. The
RTC ruled in favor of Gacott. The decision became final. Subsequently, a Writ of Execution was issued. During the execution stage,
Gacott learned that QSC was not a corporation, but was in fact a general partnership registered with the SEC. In the articles of
partnership, Michael Guy was appointed as General Manager of QSC. The Sheriff then went to DOTC-LTO to check whether QCS,
Medestomas and Guy had personal properties registered therein. Upon learning that Guy had vehicles registered in his name, Gacott
instructed the sheriff to proceed with the attachment of one of the motor vehicles of Guy. The sheriff attached Guy’s vehicle. Guy then
filed a Motion to Lift Attachment Upon Personalty, arguing that he was not a judgment debtor and, therefore, his vehicle could not be
attached. The RTC denied Guy’s motion citing Section 21 of the Corporation Code. Since QSC is an ostensible corporation, Guy is
considered as a general partner and he may be held jointly and severally liable with QSC. Guy filed a Motion for Reconsideration
arguing that he was neither impleaded as a defendant nor validly served with summons and, thus, the trial court did not acquire
jurisdiction over his person. On appeal to the CA, the CA ruled that Guy, being a partner in QSC, was bound by the summons served
upon QSC.

Issue:

Whether or not there was a valid service of summons over QSC?

Held:

No. Under Section 11, Rule 14 of the 1997 Revised Rules of Civil Procedure, when the defendant is a corporation, partnership or
association organized under the laws of the Philippines with a juridical personality, the service of summons may be made on the
president, managing partner, general manager, corporate secretary, treasurer, or in-house counsel. Jurisprudence is replete with
pronouncements that such provision provides an exclusive enumeration of the persons authorized to receive summons for juridical
entities. The records of this case reveal that QSC was never shown to have been served with the summons through any of the
enumerated authorized persons to receive such, namely: president, managing partner, general manager, corporate secretary, treasurer
or in-house counsel. Service of summons upon persons other than those officers enumerated in Section 11 is invalid. Even substantial
compliance is not sufficient service of summons.

Nevertheless, while proper service of summons is necessary to vest the court jurisdiction over the defendant, the same is merely
procedural in nature and the lack of or defect in the service of summons may be cured by the defendant’s subsequent voluntary
submission to the court’s jurisdiction through his filing a responsive pleading such as an answer. In this case, it is not disputed that
QSC filed its Answer despite the defective summons. Thus, jurisdiction over its person was acquired through voluntary appearance.

Note: Although the trial court acquired jurisdiction over QSC, this did not extend to the person of Guy insofar as holding him
solidarily liable with the partnership. A suit against the partnership is not necessarily a suit impleading each and every
partner A partnership is a juridical entity that has a distinct and separate personality from the persons composing it. Since
Guy was not impleaded in the case, the judgment rendered cannot bind nor prejudice him. Therefore, it was improper to levy
upon his property. Moreover, even if he was properly impleaded, his liability is only subsidiary pursuant to Art. 1816 of the
NCC.
Remedial Law I| Page 111

Sunrise Garden Corporation vs. Court of Appeals


G.R. No. 158836. September 30, 2015.

Facts:
Sunrise Garden and the Sangguniang Panlungsod of Antipolo City agreed on a joint undertaking to construct a city road connecting
four barangays in Antipolo City. Sunrise Garden’s contractor began to position its construction equipment. However, armed guards,
allegedly hired by Hardrock Aggregates, Inc., prevented Sunrise Garden’s contractor from using an access road to move the
construction equipment. Sunrise Garden filed a Complaint for damages with prayer for temporary restraining order and writ of
preliminary injunction against Hardrock Aggregates, Inc. The RTC issued the Writ of Preliminary Injunction. While the Complaint
was pending, informal settlers started to encroach on the area of the proposed city road. Sunrise Garden then filed a Manifestation
seeking the amendment of the Writ to include any and all persons or group of persons from interfering, preventing or obstructing all of
its contractors, equipment personnel and representatives in proceeding with the construction of the city road. The RTC then issued an
Amended Writ of Preliminary Injunction. The construction of the city road then continued. However, armed guards of K-9 Security
Agency, allegedly hired by First Alliance Real Estate Development, Inc., blocked Sunrise Garden’s contractor’s employees and
prevented them from proceeding with the construction. Sunrise Garden then filed a Motion to cite K-9 Security Agency in contempt.
K-9 Security Agency, joined by First Alliance opposed the Motion to cite them in contempt, raising the defense of lack of jurisdiction
over their persons, since they were not bound by the Amended Writ of Preliminary Injunction. Sunrise Garden filed an Ex parte
Motion to require K-9 Security Agency and First Alliance Real Estate Development, Inc. to comply with the Amended Writ of
Preliminary Injunction. K-9 and First Alliance filed a Motion for Reconsideration reiterating their arguments that since the trial court
did not acquire jurisdiction over them, the Writ of Preliminary Injunction could not be enforced against them. The MR was denied
which prompted First Alliance to file a Petition for Certiorari with a prayer for temporary restraining order with the Court of Appeals.
The CA granted the TRO and it also issued a Writ of Preliminary Injunction against the RTC Order (for First Alliance to comply with
the Amended Writ of Injunction). Sunrise Garden and the Republic filed a petition before the SC to assail the decision of the CA.
While Sunrise Garden and the Republic’s petitions before the SC were pending, the CA also annulled the Amended Writ of
Preliminary Injunction issued by the trial court.
Issue:
Whether or not the trial court acquired jurisdiction over First Alliance Real Estate Development, Inc. to be bound by the Amended
Writ of Preliminary Injunction?
Held:
No. First Alliance was not a party to the Complaint for Damages filed by Sunrise Garden before the RTC. Both Sunrise Garden and
the Republic did not refute the primary issue of lack of jurisdiction over First Alliance; this is an admission that the trial court did not
acquire jurisdiction over the latter.
On the contention that the trial court acquired jurisdiction because of First Alliance’s voluntary appearance, the Supreme Court ruled
that voluntary appearance in court may not always result in submission to the jurisdiction of a court. When a party makes a special
appearance to challenge, among others, the court’s jurisdiction over his person, he cannot be considered to have submitted to its
authority.
(1) Special appearance operates as an exception to the general rule on voluntary appearance;
(2) Accordingly, objections to the jurisdiction of the court over the person of the defendant must be explicitly made, i.e., set forth in an
unequivocal manner; and
(3) Failure to do so constitutes voluntary submission to the jurisdiction of the court, especially in instances where a pleading or motion
seeking affirmative relief is filed and submitted to the court for resolution.
The appearance of First Alliance and K-9 Security Agency should not be deemed as a voluntary appearance because it was for the
purpose of questioning the jurisdiction of the trial court. The records of this case show that the defense of lack of jurisdiction was
raised at the first instance and repeatedly argued by K-9 Security Agency and respondent First Alliance Real Estate Development, Inc.
in their pleadings.
Remedial Law I| Page 112

Rule 15: Motions

Acampado vs. Cosmilla


G.R. No. 198531, September 28, 2015

Facts:

Spouses Cosmilla filed a Petition for the Declaration of the Nullity of Document against petitioners (Ethel Acampado, Emmie
Acampado, Elvie Acampado, Earlyn Acampado and Evelyn Acampado) before the RTC of Kalibo, Aklan, Branch 6. Spouses
Cosmilla alleged that the sale of their share on the subject property was effected thru a forged Special Power of Attorney (SPA) and is
therefore null and void. After trial on the merits, the RTC rendered a Decision dismissing the complaint of the respondents for failure
to prove by preponderance of evidence that the signatures of the respondents in the SPA were forged. Respondents filed a Motion for
Reconsideration. The respondents, however, failed to comply with the requirement of notice of hearing as required under Sections 4
and 5 of Rule 15 of the Revised Rules of Court, the court a quo denied the Motion for Reconsideration. The CA affirmed the decision
but later on reversed its decision in relaxing the rules on notice and hearing required in the motions.

Issue:

Whether or not the Motion for Reconsideration is a contentious motion that needs to comply with the required notice and hearing and
service to the adverse party?

Held:

Yes. The Motion for Reconsideration is a contentious motion that needs to comply with the required notice and hearing and service to
the adverse party as mandated by Sections 4 and 6, Rule 15 of Revised Rules of Court:

It is important, however, to note that these doctrines refer exclusively to a motion, since a motion invariably contains a prayer, which
the movant makes to the court, which is to repeat usually in the interest of the adverse party to oppose and in the observance of due
process, the other party must be given the opportunity to oppose the motion. In keeping with the principles of due process, therefore, a
motion which does not afford the adverse party the chance to oppose it should simply be disregarded. Failure to comply with the
required notice and hearing is a fatal defect that is deleterious to respondent’s cause.

As born by the records, no notice of hearing was appended to the Motion for Reconsideration of the respondent. As discussed above, a
motion for reconsideration is a litigated motion where the right of the adverse party will be affected by its admission. The adverse
party in this case had the right to resist the motion because it may result to the reversal of a prior favorable decision. The proof of
service was therefore indispensable in order to avoid surprises on the opposite party. The absence thereof is fatal to the motion.

It bears stressing that a motion without notice and hearing, is pro forma, a mere scrap of paper that cannot be acted by the court. It
presents no question that the court can decide. The court has no reason to consider it and the clerk has no right to receive it.
Indisputably, any motion that does not contain proof of service and notice to the adverse party is not entitled to judicial cognizance.
Remedial Law I| Page 113

Laude vs. Ginez-Jabalde


G.R. No. 217456. November 24, 2015

Facts:

Jeffrey “Jennifer” Laude (Jennifer) was killed at the Celzone Lodge on Ramon Magsaysay Drive in Olongapo City allegedly by 19-
year-old US Marine Joseph Pemberton. A Complaint for murder was filed by Jennifer’s sibling, Marilou S. Laude, against Pemberton
before the Olongapo City Office of the City Prosecutor. Pemberton was detained in Camp Aguinaldo, the general headquarters of the
Armed Forces of the Philippines. Public Prosecutor filed an Information for murder against Pemberton before the Regional Trial Court
in Olongapo City. A warrant of arrest against Pemberton. Pemberton surrendered personally to Judge Roline M. Ginez-Jabalde and he
was then arraigned.

On the same day, Marilou S. Laude filed an Urgent Motion to Compel the Armed Forces of the Philippines to Surrender Custody of
Accused to the Olongapo City Jail and a Motion to Allow Media Coverage. The [M]otion was [scheduled] for hearing on December
22, 2014, at 2 p.m. According to petitioners, they were only able to serve the Motion on Pemberton’s counsel through registered mail.
In any case, they claim to have also “furnished a copy of the [M]otion personally . . . at the hearing of the [M]otion. On December 23,
2014, Judge Ginez-Jabalde denied petitioners’ Urgent Motion for lack of merit and the Motion for Reconsideration was also denied.

Petitioners argue that respondent Judge committed grave abuse of discretion tantamount to an excess or absence of jurisdiction when
she dismissed the Urgent Motion to Compel the Armed Forces of the Philippines to Surrender Custody o[f] Accused to the Olongapo
City Jail [based] on mere technicalities. In particular, they argue that the three-day rule on motions under 1997 Rules of Court is not
absolute, and should be liberally interpreted when a case is attended by exigent circumstances. Petitioners advance that the rationale
behind the three-day notice rule is satisfied when there is an opportunity to be heard, which was present in this case since Pemberton’s
counsel and the Public Prosecutor were present in the hearing of the two Motions filed by petitioners. Petitioners allege that the court
noted their attendance, and were able to make comments during the December 22, 2014 Motion hearing. They assert that the rights of
Pemberton were not compromised in any way earliest possible date.

Issue:

Whether or not the RTC is correct in dismissing the urgent motion filed by the petitioner on the ground of non-compliance with the
three-day notice rule?

Held:

Yes. The failure of petitioners to comply with the three-day notice rule is unjustified. Rule 15, Section 4 of the Rules of Court clearly
makes it a mandatory rule that the adverse party be given notice of hearing on the motion at least three days prior. Failure to comply
with this notice requirement renders the motion defective consistent with protecting the adverse party’s right to procedural due
process. As an integral component of procedural due process, the three-day notice required by the Rules is not intended for the benefit
of the movant. Rather, the requirement is for the purpose of avoiding surprises that may be sprung upon the adverse party, who must
be given time to study and meet the arguments in the motion before a resolution by the court. Principles of natural justice demand that
the right of a party should not be affected without giving it an opportunity to be heard.
Remedial Law I| Page 114

Rule 16: Motion to Dismiss (Omnibus Motion)

De Guzman vs. Ochoa


G.R. No. 169292. April 13, 2011

Facts:

Respondent spouses Cesar Ochoa and Sylvia Ochoa, through respondent Araceli Azores, ostensibly acting as attorney in fact,
commenced in the Regional Trial Court (RTC) in Pasig City an action seeking the annulment of contract of mortgage, foreclosure
sale, certificate of sale and damages. Petitioners, as defendants in Civil Case No. 68896, filed a motion to dismiss, alleging the sole
ground that the complaint did not state a cause of action. The petitioners’ motion to dismiss was formally opposed by the private
respondents. On December 16, 2002, the respondent RTC Judge denied petitioners’ motion to dismiss and at the same time set for
pretrial conference, directing the parties to submit their respective pre-trial briefs. The petitioners filed a second motion to dismiss,
alleging that the certification against forum shopping attached to the complaint was not executed by the principal parties (plaintiffs) in
violation of Sec. 5, Rule 7, 1997 Rules of Civil Procedure, rendering the complaint fatally defective and thus dismissible.

The private respondents opposed the second motion to dismiss. The respondent RTC Judge issued her first assailed order, denying the
second motion to dismiss. The petitioners filed their motion for reconsideration, but the respondent RTC Judge denied the motion
through her second assailed order. Aggrieved, petitioners elevated the order of denial to the CA via a petition for certiorari contending
that the RTC should have dismissed the complaint motu proprio since it was fatally defective. The CA denied the petition for lack of
merit.

Issues:
(1) Whether or not the petitioner may be allowed to file second motion to dismiss for defective verification and certification of forum
shopping?

(2) Whether or not an order denying a second motion to dismiss may be assailed by a petition for certiorari?

Held:

(1) No. Section 8, Rule 15 of the Rules of Court defines an omnibus motion as a motion attacking a pleading, judgment or proceeding.
A motion to dismiss is an omnibus motion because it attacks a pleading, that is, the complaint. For this reason, a motion to dismiss,
like any other omnibus motion, must raise and include all objections available at the time of the filing of the motion because under
Section 8, “all objections not so included shall be deemed waived.” As inferred from the provision, only the following defenses under
Section 1, Rule 9, are excepted from its application: [a] lack of jurisdiction over the subject matter; [b] there is another action pending
between the same parties for the same cause (litis pendentia); [c] the action is barred by prior judgment (res judicata); and [d] the
action is barred by the statute of limitations or prescription. In the case at bench, the petitioners raised the ground of defective
verification and certification of forum shopping only when they filed their second motion to dismiss, despite the fact that this ground
was existent and available to them at the time of the filing of their first motion to dismiss.

Absent any justifiable reason to explain this fatal omission, the ground of defective verification and certification of forum shopping
was deemed waived and could no longer be questioned by the petitioners in their second motion to dismiss.

(2) No. An order denying a motion to dismiss is an interlocutory order which neither terminates the case nor finally disposes of it, as it
leaves something to be done by the court before the case is finally decided on the merits. As such, the general rule is that the denial of
a motion to dismiss cannot be questioned in a special civil action for certiorari which is a remedy designed to correct errors of
jurisdiction and not errors of judgment. Therefore, an order denying a motion to dismiss may only be reviewed in the ordinary course
of law by an appeal from the judgment after trial. The ordinary procedure to be followed in such cases is to file an answer, go to trial,
and if the decision is adverse, reiterate the issue on appeal from the final judgment. Only in exceptional cases where the denial of the
motion to dismiss is tainted with grave abuse of discretion that the Court allows the extraordinary remedy of certiorari.
Remedial Law I| Page 115

Rule 17: Dismissal of Actions

Lim Teck Chuan vs. Uy


G.R. No. 155701. March 11, 2015
Facts:
Antonio Lim Tanhu was the original owner of the lot which is the subject matter of the controversy. Allegedly, he sold the lot to
Spouses Cabansag; then Spouses Cabansag sold the same to Serafin Uy (respondent). Serafin then filed a petition before the RTC
praying for the issuance of a new owner’s duplicate TCT in his name. The petition was initially granted but subsequently nullified
because Lim Teck Chuan (petitioner) filed his Opposition alleging that he is one of the 6 legitimate descendants of Antonio and that
the original TCT was not lost and has always been in his custody. In the meantime, a certain Henry Lim sold the same lot to Leopolda
Cecilio by virtue of an an Affidavit of Sole Adjudication/Settlement of the Estate of Antonio Lim Tanhu with Deed of Sale.
Serafin then filed a Complaint for Quieting of Title impleading Leopolda, Henry and the petitioner. Leopolda averred that she is a
buyer in good faith and for value. Petitioner set up a counterclaim against Serafin and a cross-claim against Lopeolda contending
that the property was never transferred and encumbered to any person during Antonio’s lifetime.
During the proceedings, both and Serafin and Leopolda entered into an amicable settlement and they both filed a Joint Motion to
Dismiss (MTD) on the main ground that the case had become moot and academic since Serafin’s title to to the subject lot had been
allegedly quieted. Petitioner opposed the MTD on the ground that he was not included in the settlement. The RTC, however, granted
the MTD and it also dismissed petitioner’s counterclaim and cross-claim.
Petitioner filed directly with the SC a petition for review under Rule 45.
Issue:
Whether or not petitioner’s counterclaim or cross-claim could be prosecuted in the same action despite the dismissal of the main
complaint?
Held:
Yes. Section 2 of Rule 17 provides that:
xxx If a counterclaim has been pleaded by a defendant prior to the service upon him of the plaintiff’s motion for dismissal, the
dismissal shall be limited to the complaint. The dismissal shall be without prejudice to the right of the defendant to prosecute his
counterclaim in a separate action unless within fifteen (15) days from notice of the motion he manifests his preference to have his
counterclaim resolved in the same action.
The RTC erred when it dismissed the case when the present rules state that the dismissal shall be limited only to the complaint. A
dismissal of an action is different from a mere dismissal of the complaint. For this reason, since only the complaint and not the action
is dismissed, the defendant in spite of said dismissal may still prosecute his counterclaim in the same action. Citing Pinga v. Heirs of
German Santiago, the Court said that the dismissal of the complaint does not necessarily result to the dismissal of the counterclaim.
In the instant case, the petitioner’s preference to have his counterclaim (and cross-claims) be prosecuted in the same action was timely
manifested.
Remedial Law I| Page 116

Ching vs. Cheng


G.R. No. 175507. October 8, 2014

Facts:

Antonio Ching owned several businesses and properties, among which was Po Wing Properties, Inc. It is also alleged that while he
was unmarried, he had children from two women. Ramon Ching alleged that he was the only child of Antonio Ching with his
common-law wife, Lucina Santos. Joseph Cheng and Jaime Cheng, on the other hand, claim to be Antonio Ching’s illegitimate
children with his housemaid, Mercedes Igne. When Antonio Ching was murdered, Ramon Ching allegedly induced Mercedes Igne and
her children, Joseph and Jaime, to sign an agreement and waiver to Antonio Ching’s estate in consideration of ₱22.5 million.
Mercedes Igne’s children alleged that Ramon Ching never paid them.  Ramon Ching allegedly executed an affidavit of settlement of
estate, naming himself as the sole heir and adjudicating upon himself the entirety of Antonio Ching’s estate. After a year of
investigating Antonio Ching’s death, the police found Ramon Ching to be its primary suspect. Information was filed against him, and
a warrant of arrest was issued.

First Case: Joseph, Jaime and Mercedes (the Chengs) filed a complaint for declaration of nullity of titles against Ramon Ching before
the Regional Trial Court of Manila. Po Wing Properties was also impleaded in an amended complaint and Lucina was also allowed to
intervene. After the responsive pleadings had been filed, Po Wing Properties filed a motion to dismiss on the ground of lack of
jurisdiction of the subject matter. The RTC granted the Motion to Dismiss.

Second Case: The Chengs and Lucina Santos filed a complaint for "Annulment of Agreement, Waiver, Extra-Judicial Settlement of
Estate and the Certificates of Title Issued by Virtue of Said Documents with Prayer for Temporary Restraining Order and Writ of
Preliminary Injunction" against Ramon Ching and Po Wing Properties. The Chengs and Lucina Santos filed a motion to dismiss their
complaint in the second case, praying that it be dismissed without prejudice. The RTC issued an order granting the motion to
dismiss. The dismissal was made without prejudice.

Ching and Po Wing Properties filed a motion for reconsideration. They argue that the dismissal should have been with prejudice under
the "two dismissal rule" of Rule 17, Section 1 of the 1997 Rules of Civil Procedure, in view of the previous dismissal of the first case.

Third Case: During the pendency of the MR, the Chengs and Lucina Santos filed a complaint for "Disinheritance and Declaration of
Nullity of Agreement and Waiver, Affidavit of Extra-Judicial Agreement, Deed of Absolute Sale, and Transfer Certificates of Title
with Prayer for TRO and Writ of Preliminary Injunction" against Ramon Ching and Po Wing Properties.

Branch 6 issued an omnibus order resolving both the motion for reconsideration in the second case and the motion to dismiss in the
third case. The trial court denied the motion for reconsideration and the motion to dismiss, holding that the dismissal of the second
case was without prejudice and, hence, would not bar the filing of the third case. 

The Court of Appeals rendered the decision in the first certiorari case dismissing the petition. The appellate court ruled that Ramon
Ching and Po Wing Properties’ reliance on the "two-dismissal rule" was misplaced since the rule involves two motions for dismissals
filed by the plaintiff only. In this case, it found that the dismissal of the first case was upon the motion of the defendants, while the
dismissal of the second case was at the instance of the plaintiffs.

Upon the denial of their MR, Ramon Ching and Po Wing Properties filed this present petition for review.

Issue:

Whether or not the two-dismissal rule applies in this case?

Held:

No. As a general rule, dismissals under Section 1 of Rule 17 are without prejudice except when it is the second time that the plaintiff
caused its dismissal. Accordingly, for a dismissal to operate as an adjudication upon the merits, i.e, with prejudice to the re-filing of
the same claim, the following requisites must be present:
(1) There was a previous case that was dismissed by a competent court;
(2) Both cases were based on or include the same claim;
Remedial Law I| Page 117

(3) Both notices for dismissal were filed by the plaintiff; and
(4) When the motion to dismiss filed by the plaintiff was consented to by
the defendant on the ground that the latter paid and satisfied all the claims of the former.

The purpose of the "two-dismissal rule" is "to avoid vexatious litigation." When a complaint is dismissed a second time, the plaintiff is
now barred from seeking relief on the same claim.

The dismissal of the second case was without prejudice in view of the "two-dismissal rule". Here, the first case was filed as an
ordinary civil action. It was later amended to include not only new defendants but new causes of action that should have been
adjudicated in a special proceeding. A motion to dismiss was inevitably filed by the defendants on the ground of lack of jurisdiction.
The dismissal of the first case was done at the instance of the defendant under Rule 16, Section 1(b) of the Rules of Civil Procedure.

Thus, when respondents filed the second case, they were merely refiling the same claim that had been previously dismissed on the
basis of lack of jurisdiction. When they moved to dismiss the second case, the motion to dismiss can be considered as the first
dismissal is at the plaintiff’s instance.
Remedial Law I| Page 118

Rule 18: Pre-Trial


People vs. Perez
G.R. No. 142556. February 5, 2003
Facts:
The prosecution alleges that, on January 17, 1997, about noontime, in Sitio Baco, Barangay Macarang, Palauig, Zambales, six-year
old Mayia Ponseca(victim) was walking along Sulok on her way to her house in Sitio Camiling when appellant Jesus Sebunga Perez
approached her. Appellant introduced himself as Johnny and immediately afterwards, strangled her neck and boxed her abdomen. Still
in shock, Mayia fell down. At that point, a dog arrived and barked at them. Thereafter, the appellant raped the victim. The accused-
appellant after being arrested was charged for the crime of rape. During pre-trial the prosecution and defense stipulated on the
following facts:
1. The identity of the accused;
2. The accused was at the time of the incident in the vicinity thereof;
3. The victim in this case, Mayia P. Ponseca, was born on 23 May 1990 as evidenced by her birth certificate;
4. That after the incident, the child was subjected to a medico-legal examination to which a medico-legal certificate was issued by Dr.
Editha Divino.

The prosecution marked in evidence the birth certificate of the victim Mayia O. Ponseca as Exhibit A, and the medico-legal certificate
issued by Dr. Editha Divino as Exhibit B. As a defense, appellant avers during the incident, he left the fishpond and walked home to
Barangay Alwa which was about thirty meters from the fishpond. After trial, the trial Court convicted him for the crime of statutory
rape. Hence, this automatic review.

In his Reply Brief, appellant contends that even assuming that the guilt of appellant has been proven beyond reasonable doubt, the trial
court erred in imposing the death penalty. Appellant maintains that the death penalty cannot be imposed on him for failure of the
prosecution to prove Mayia’s age by independent evidence. Appellant points out that while Mayia’s birth certificate was duly marked
during the pre-trial, it was not presented and identified during the trial. Appellant asserts that Mayia’s minority must not only be
specifically alleged in the Information but must also be established beyond reasonable doubt during the trial.

Issue:

Whether or not the stipulation during the pre-trial on the victim’s age is binding on the accused?

Held:
Yes. The Court ruled that, during the pre-trial, the prosecution marked in evidence Mayia’s birth certificate as Exhibit A. The
prosecution submitted its Offer of Evidence which included Exhibit A, a certified true copy of Mayia’s birth certificate.  The trial court
admitted Exhibit A without any objection from the defense.
The purpose of pre-trial is to consider the following: (a) plea bargaining; (b) stipulation of facts; (c) marking for identification of
evidence of the parties; (d) waiver of objections to admissibility of evidence; (e) modification of the order of trial if the accused admits
the charge but interposes lawful defenses; and (f) such matters as will promote a fair and expeditious trial of the criminal and civil
aspects of the case. Facts stipulated and evidence admitted during pre-trial bind the parties. Section 4, Rule 118 of the Revised Rules
of Criminal Procedure provides:

SEC. 4. Pre-trial order. - After the pre-trial conference, the court shall issue an order reciting the actions taken, the facts stipulated,
and evidence marked. Such order shall bind the parties, limit the trial to matters not disposed of, and control the course of the action
during the trial, unless modified by the court to prevent manifest injustice.

Moreover, Mayia herself testified in open court as to her age. During the trial on December 15, 1998, which was about twenty-three
(23) months after the rape incident occurred on January 17, 1997, Mayia testified on cross-examination that she was 8 years old last
May 23. Thus, by deduction, since Mayia was born on May 23, 1990 as shown in her birth certificate, she was about six (6) years and
seven (7) months old on January 17, 1997, the day the crime took place. The Court ruled that the prosecution has indisputably proven
that Mayia was below seven years old at the time appellant raped her. Thus, the trial court was correct in imposing the death penalty
on appellant. Under Article 335 of the Revised Penal Code, as amended by Section 11 of Republic Act No. 7659,  the death penalty
shall be imposed if the crime of rape is committed against a child below seven (7) years old. Mayia was six (6) years and seven (7)
months old when appellant raped her.
Remedial Law I| Page 119

Rule 19: Intervention

Office of the Ombudsman vs. Sison


G.R. No. 185954 February 16, 2010
Facts:

The Isog Han Samar Movement, represented by Fr. Noel Labendia of the Diocese of Calbayog, Catbalogan, Samar, filed a letter-
complaint addressed to Ombudsman accusing Governor Tan and other local public officials of the Province of Samar, including Sison,
the Provincial Budget Officer, of highly anomalous transactions entered into by them amounting to several millions of pesos.   The
letter-complaint stemmed from the audit conducted by the Legal and Adjudication Office (LAO), Commission on Audit (COA),
which found, that various purchases totaling PhP 29.34 million went without proper bidding procedures and documentations; that
calamity funds were expended without a State of Calamity having been declared by the President; and that purchases for rice,
medicines, electric fans, and cement were substantially overpriced.mThe Office of the Ombudsman found basis to proceed with the
administrative case against the impleaded provincial officials of Samar. In his counter-affidavit, Sison denied the accusations
contained in the letter-complaint and claimed his innocence on the charges. He asserted that his function is limited to the issuance of a
certification that an appropriation for the requisition exists, that the corresponding amount has been obligated, and that funds are
available. He did not, in any way, vouch for the truthfulness of the certification issued by the requesting parties.

The Office of the Ombudsman rendered a Decision, finding Sison and several other local officials of the Province of Samar guilty of
grave misconduct, dishonesty, and conduct prejudicial to the best interest of the service and dismissing him from service. Sison
appealed to the CA via a Petition for Review under Rule 43. CA rendered a decision reversing and setting aside the decision of the
Office of the Ombudsman against Sison. It held that the Office of the Ombudsman failed to adduce substantial evidence in order to
convict Sison. Moreover, it reasoned that Sison’s responsibility as Provincial Budget Officer was to ensure that appropriations exist in
relation to the emergency purchase being made and that he had no hand or discretion in characterizing a particular purchase as
emergency in nature. Hence, he cannot be held administratively liable for simply attesting to the existence of appropriations for a
certain purpose, save if such certification is proved to be false.

The Office of the Ombudsman filed an Omnibus Motion for Intervention and to Admit Attached Motion for Reconsideration, which
was subsequently denied by the CA. Hence, this petition.

Issue:

Whether or not the Office of the Ombudsman may be allowed to intervene and seek reconsideration of the adverse decision rendered
by the CA?

Held:

No. To warrant intervention under Rule 19 of the Rules of Court, two requisites must concur: (1) the movant has a legal interest in the
matter in litigation; and (2) intervention must not unduly delay or prejudice the adjudication of the rights of the parties, nor should the
claim of the intervenor be capable of being properly decided in a separate proceeding. The interest, which entitles one to intervene,
must involve the matter in litigation and of such direct and immediate character that the intervenor will either gain or lose by the direct
legal operation and effect of the judgment.

In support of its argument that it has legal interest, the Office of the Ombudsman cites Philippine National Bank v. Garcia, Jr. In the
said case, PNB imposed upon its employee, Garcia, the penalty of forced resignation for gross neglect of duty. On appeal, the CSC
exonerated Garcia from the administrative charges against him. However, Office of the Ombudsman cannot use Garcia to support its
intervention in the appellate court for the following reasons:

First, Sison was not exonerated from the administrative charges against him, and was, in fact, dismissed for grave misconduct,
dishonesty, and conduct prejudicial to the best interest of the service by the Office of the Ombudsman in the administrative case,
OMB-C-A-05-0051-B. Thus, it was Sison who appealed to the CA being, unquestionably, the party aggrieved by the judgment on
appeal.
Remedial Law I| Page 120

Second, the issue here is the right of the Office of the Ombudsman to intervene in the appeal of its decision, not its right to appeal.

And third, Garcia should be read along with Mathay, Jr. v. Court of Appeals and National Appellate Board of the National Police
Commission v. Mamauag (Mamauag), in which this Court qualified and clarified the exercise of the right of a government agency to
actively participate in the appeal of decisions in administrative cases.

Clearly, the Office of the Ombudsman is not an appropriate party to intervene in the instant case. It must remain partial and detached.
More importantly, it must be mindful of its role as an adjudicator, not an advocate.

Furthermore, the Rules provides explicitly that a motion to intervene may be filed at any time before rendition of judgment by the trial
court. In the instant case, the Omnibus Motion for Intervention was filed only on July 22, 2008, after the Decision of the CA was
promulgated on June 26, 2008. The Office of the Ombudsman was aware of the appeal filed by Sison. The Rules of Court provides
that the appeal shall be taken by filing a verified petition for review with the CA, with proof of service of a copy on the court or
agency a quo. Clearly, the Office of the Ombudsman had sufficient time within which to file a motion to intervene. As such, its failure
to do so should not now be countenanced.
Remedial Law I| Page 121

Ombudsman vs. Chavez


G.R. No. 172206 July 3, 2013

Facts:

The Batangas State University Board of Regents (BSU-BOR) received an Order from Deputy Ombudsman directing the former to
enforce the Office of the Ombudsman's Joint Decision and Supplemental Resolution finding herein respondents guilty of dishonesty
and grave misconduct and imposing the penalty of dismissal from service. Pursuant to said Order, the BSU-BOR issued Resolution
resolving to implement the Order of the Office of the Ombudsman. Thus, herein respondents filed a petition for injunction with prayer
for issuance of a temporary restraining order or preliminary injunction before the RTC. The gist of the petition before the RTC is that
the BSU-BOR should be enjoined from enforcing the Ombudsman's Joint Decision and Supplemental Resolution because the same are
still on appeal and, therefore, are not yet final and executory.

The RTC ordered the dismissal of herein respondents' petition for injunction on the ground of lack of cause of action. Respondents
filed their notice of appeal and promptly filed a Motion for Issuance of a Temporary Restraining Order and/or Injunction with the CA.
CA granted respondents' prayer for a temporary restraining order enjoining the BSU-BOR from enforcing its Resolution No. 18, series
of 2005.

Thereafter, the Office of the Ombudsman filed a Motion to Intervene and to Admit Attached Motion to Recall Temporary Restraining
Order, with the Motion to Recall Temporary Restraining Order attached thereto. Respondents opposed said motion and then filed an
Urgent Motion for Issuance of a Writ of Preliminary Injunction. CA denied the Office of the Ombudsman’s motion. Hence, this
petition.

Issue:

Whether or not the Office of the Ombudsman has legal personality to intervene in the appealed case before the CA?

Held:

Yes. The CA should have allowed the Office of the Ombudsman to intervene in the appeal pending with the lower court. As held in
the case of Office of the Ombudsman v. Samaniego, the Ombudsman is in a league of its own. It is different from other investigatory
and prosecutory agencies of the government because the people under its jurisdiction are public officials who, through pressure and
influence, can quash, delay or dismiss investigations directed against them. Its function is critical because public interest (in the
accountability of public officers and employees) is at stake. It is true that under our rule on intervention, the allowance or disallowance
of a motion to intervene is left to the sound discretion of the court after a consideration of the appropriate circumstances. However,
such discretion is not without limitations. One of the limits in the exercise of such discretion is that it must not be exercised in
disregard of law and the Constitution. The CA should have considered the nature of the Ombudsman's powers as provided in the
Constitution and RA 6770.

Here, since its power to ensure enforcement of its Joint Decision and Supplemental Resolution is in danger of being impaired, the
Office of the Ombudsman had a clear legal interest in defending its right to have its judgment carried out. The CA patently erred in
denying the Office of the Ombudsman's motion for intervention.
Remedial Law I| Page 122

Anonuevo vs. Intestrate of Jalandoni


G.R. No. 178221 December 1, 2010

Facts:

Rodolfo Jalandoni died intestate. Bernardino, the brother of Rodolfo, filed petition for issuance of letters of administration with the
Court of First Instance of Negros Occidental to commence the judicial settlement of the estate. May Anonuevo and their siblings
introduced themselves as children of Sylvia Blee Desantis, who is the daughter of Isabel Blee with one John Desantis. Isabel (their
grandmother) was at the time of Rodolfo’s death is the legal spouse of the latter. For this reason, Isabel is entitled to a share in estate
of Rodolfo.

Petitioners pray that they be allowed to intervene in the intestate proceedings because Sylvia and Isabel have already passed away.
Respondents opposed because the evidence showed by the Petitioners reveal that Isabel has a subsisting marriage with John Desantis
at the time she was purportedly married to Rodolfo. Thus, her marriage with Rodolfo was void ab initio.

The intestate court issued an order allowing petitioners to take part in settlement proceedings. The CA reversed the decision of the
intestate court and sided with the respondents.

Issue:

Whether or the not the CA erred when it nullified the orders of intestate court allowing petitioners to intervene in settlement
proceedings?

Held:

No. A court’s power to allow or deny intervention, albeit discretionary in nature, is circumscribed by the basic demand of sound
judicial procedure that only a person with interest in an action or proceeding may be allowed to intervene. Otherwise stated, a court
has no authority to allow a person, who has no interest in an action or proceeding, to intervene therein.

Petitioners and their siblings failed to offer sufficient evidence to establish that Isabel was the legal spouse of Rodolfo. The very
evidence of the petitioners and their siblings negates their claim that Isabel has interest in Rodolfo’s estate. The birth certificate of
Sylvia precisely serves as the competent evidence of marriage between Isabel and John Desantis. The inability of the petitioners and
their siblings to present evidence to prove that Isabel’s prior marriage was dissolved results in a failure to establish that she has interest
in the estate of Rodolfo. Clearly, an intervention by the petitioners and their siblings in the settlement proceedings cannot be justified.
Remedial Law I| Page 123

Fernandez vs. Court of Appeals


A.M. OCA IPI No. 12-201-CA-J. February 19, 2013

Facts:

Complainants Ethelwoldo Fernandez and Antonio Henson were elected to the board of directors of NADECOR. In a regular
stockholder’s meeting where two groups were vying for control over the company, Calalang, De Jesus, Romulo, Ayala, Lazatin,
Fernandez, Nitorreda, Engle were elected. Gatmaitan was also elected as Corporate Secretary.

Thereafter, the Ricaforts, claiming to be stockholders of record, sought to annul the said meeting held. They filed a complaint before
the RTC of Pasig. The Ricaforts alleged that they were not given due notice of the said meeting thus they were not present and were
not able to exercise their right. The RTC agreed with the Ricaforts.

Four separate Petition for Certiorari were filed by the members of the board with the CA, all with application for a TRO and/or
preliminary injunction. The CA denied such applications, but on the same day nevertheless, the 11th division issued a TRO. During
the effectivity of the TRO, the old Board of Directors assumed the functions of the new one in order to prevent any hiatus and not to
prejudice the corporation. All the CA petitions were consolidated as well as the other cases.

The respondents Ricafort filed their Comment Ad Cautelam to the petition in CA-G.R. No. 122784. The petitioners therein thereafter
filed three (3) urgent motions to resolve their application for writ of preliminary injunction, on March 8, on May 22, and again on June
6, 2012. The Writ of Preliminary Injunction was granted by the CA 14th Division, which not for long was questioned.

Complainants filed with the Supreme Court a Petition for Certiorari and Prohibition, seeking to annul the writ of preliminary
injunction issued by the CA’s Special 14th Division. Complainants also filed an Administrative case against the Justices of the 14th
Division of the CA.

Issue:

Whether or not the some of the petitioners who were not parties in the cases in the Court of Appeals have a legal personality to assail
the writ of preliminary injunction issued by the CA 14th Division?

Held:

No. Section 1 of Rule 19 of the Rules of Court provides that a person who has a legal interest in the matter in litigation, or in the
success of either of the parties, or an interest against both, or is so situated as to be adversely affected by a distribution or other
disposition of property in the custody of the court or of an officer thereof may, with leave of court, be allowed to intervene in the
action. Conversely, a person who is not a party in the main suit cannot be bound by an ancillary writ, such as a preliminary injunction.
Indeed, he cannot be affected by any proceeding to which he is a stranger. Moreover, a person not an aggrieved party in the original
proceedings that gave rise to the petition for certiorari, will not be permitted to bring the said action to annul or stay the injurious writ.
Such is the clear import of Sections 1 and 2 of Rule 65 of the Rules of Court. Thus, a person not a party to the proceedings in the trial
court or in the CA cannot maintain an action for certiorari in the Supreme Court to have the judgment reviewed. Stated differently, if a
petition for certiorari or prohibition is filed by one who was not a party in the lower court, he has no standing to question the assailed
order.

The complainants, who at various times served as elected members of the Board of NADECOR, did not bother to intervene in the CA
petitions, hence, they are not entitled to the service of pleadings and motions therein. Complainant Fernandez was himself a defendant
in SEC Case No. 11-164 in the RTC, but he cose not to join any of the four CA petitions.

Persons who are not parties to any of the consolidated petitions have no personality to assail the said injunctive writ. They should first
have intervened below, and then filed a motion for reconsideration from the questioned CA order.
Remedial Law I| Page 124

Rodriguez vs. Court of Appeals


G.R. No. 184589, June 13, 2013

Facts:

Purita Landicho by virtue of November 16, 1965 decision of CFI of Rizal was able to register to her name the property subject of the
case. However, a TCT was issued in her name instead of OCT. The subject property in this case, after being sold several times, was
brought by Philippine Chinese Charitable Association, Inc. (PCCAI), herein respondent from WPFI on November 13, 1973 and was
issued TCT on July 15, 1975. But it appeared that on November 14, 1996, Landicho executed a DOAS in favor of Deogenes O.
Rodriguez, herein petitioner. Seven years after, Rodriguez filed with Omnibus Motion with the RTC, praying for the Land
Registration Authority(LRA) to issue OCT in his name. During trial, TCT which was issued in the name of PCCAI was presented by
the petitioner, and alleged that it was fictitious. PCCAI now filed a motion for leave to intervene, alleging that it has a substantial legal
interest over the said land. PCCAI filed an MR. Both motion for leave to intervene, and MR was denied by the RTC, it ruled that
intervention would not be allowed after the decision has become final and executory. The LRA upon receipt of the order, filed a
manifestation to the RTC that it cannot comply with the said order, since two TCT were issued covering the same land. At the same
time, PCCAI filed a petition for Certiorari and Prohibition before the CA, it argued that even though the issuance of execution for final
and executory order is a ministerial duty, it should refrain from issuing such if it will be unjust and inequitable, and that since it was an
indispensable party, they should have been allowed to intervene by the RTC. The CA decided in favor of PCCAI, MR was denied as
well. Hence, this petition.

Issue:

Whether or not the petitioner should be allowed to intervene even the November 16, 1965 judgment CFI of Rizal was already final
and executory?

Held:

Yes. The subject property is presently covered by TCT No. 482970 in the name of PCCAI.  As the registered owner, PCCAI clearly
has a legal interest in the subject property. The issuance of another certificate of title to Rodriguez will adversely affect PCCAI,
constituting a cloud on its TCT No. 482970.

Although Rule 19 is explicit on the period when a motion to intervene may be filed, the Court allowed exceptions in several cases, viz:

This rule, however, is not inflexible. Interventions have been allowed even beyond the period prescribed in the Rule, when demanded
by the higher interest of justice. Interventions have also been granted to afford indispensable parties, who have not been impleaded,
the right to be heard even after a decision has been rendered by the trial court, when the petition for review of the judgment has
already been submitted for decision before the Supreme Court, and even where the assailed order has already become final and
executory.  In Lim v. Pacquing, the motion for intervention filed by the Republic of the Philippines was allowed by this Court to avoid
grave injustice and injury and to settle once and for all the substantive issues raised by the parties.

In fine, the allowance or disallowance of a motion for intervention rests on the sound discretion of the court after consideration of the
appropriate circumstances.  Rule 19 of the Rules of Court is a rule of procedure whose object is to make the powers of the court fully
and completely available for justice.  Its purpose is not to hinder or delay, but to facilitate and promote the administration of justice.

The particular circumstances of this case similarly justify the relaxation of the rules of procedure on intervention.  First, the interests
of both PCCAI and Rodriguez in the subject property arose only after the CFI Decision dated November 16, 1965 in Land Reg. Case
No. N-5098 became final and executory. PCCAI bought the subject property from WPFI on November 13, 1973 and was issued TCT
No. 482970 for the same on July 15, 1975; while Rodriguez bought the subject property from Landicho on November 14, 1996.  
Second, as previously discussed herein, both PCCAI and Rodriguez trace their titles back to Landicho.  Hence, the intervention of
PCCAI could not unduly delay or prejudice the adjudication of the rights of Landicho, the original party in Land Reg. Case No. N-
5098.  Third, the latest proceedings in Land Reg. Case No. N-5098 involved Rodriguez's Omnibus Motion, filed before the RTC on
May 18, 2005, in which he prayed for the execution of the November 16, 1965 Decision of the CFI.  PCCAI moved to intervene in the
case only to oppose Rodriguez's Omnibus Motion on the ground that the subject property is already registered in its name under TCT
No. 482970, which originated from Landicho's TCT No. 167681.  And fourth, after learning of Rodriguez's Omnibus Motion in Land
Reg. Case No. N-5098 via the November 3, 2006 subpoena issued by the RTC, PCCAI was reasonably expected to oppose the same.
Remedial Law I| Page 125

Such action was the most opportune and expedient remedy available to PCCAI to prevent the RTC from ordering the issuance of a
decree of registration and OCT in Rodriguez's name.  For this reason, the RTC should have allowed the intervention of PCCAI.
Remedial Law I| Page 126

Yao vs. Perello


G.R. No. 153828, October 24, 2003

Facts:

HLURB rendered a decision rescinding the contract to sell between petitioner (Yao) and PR Builders, and ordering PR Builders to
refund petitioner the amount of ₱2,116,103.31, as well as to pay damages in the amount of ₱250,000. Thereafter, the HLURB issued a
writ of execution against PR Builders and its managers, and referred the writ to the office of the Clerk of Court of Muntinlupa for
enforcement. Pursuant to the writ, the deputy sheriff levied on a parcel of land in Canlubang, Calamba, Laguna, registered in the
names of spouses Pablito Villarin and private respondent, Bernadine Villarin. The property was scheduled for public auction on March
20, 2002.

Private respondent filed before the RTC of Parañaque City, a petition for prohibition with prayer for temporary restraining order
and/or writ of preliminary injunction, seeking to enjoin Sheriff Melvin T. Bagabaldo from proceeding with the public auction. Private
respondent alleged that she co-owned the property subject of the execution sale; that the property regime between private respondent
and her husband was complete separation of property, and that she was not a party in the HLURB case, hence, the subject property
could not be levied on to answer for the separate liability of her husband.

On April 25, 2002, or more than a month after public respondent judge issued the resolution of March 22, 2002, petitioner filed a
motion for intervention. The RTC denied the motion. Aggrieved, petitioner filed the instant petition for certiorari imputing grave
abuse of discretion to public respondent judge in: (a) declaring the subject property exempt from execution and therefore could not be
sold to satisfy the obligation of private respondent’s husband, and (b) denying petitioner’s motion for intervention on the ground that
the same was filed late.

Issue:

Whether or not petitioner can validly intervene to question the resolution of the RTC exempting the property of Bernadine Villarin
from execution?

Held:

No. Petitioner’s claim that he had the right to intervene is without basis. Nothing in the Rules require the inclusion of a private party as
respondent in petitions for prohibition. On the other hand, to allow intervention, it must be shown that (a) the movant has a legal
interest in the matter in litigation or otherwise qualified, and (b) consideration must be given as to whether the adjudication of the
rights of the original parties may be delayed or prejudiced, or whether the intervenor’s rights may be protected in a separate
proceeding or not. Both requirements must concur as the first is not more important than the second.

In the case at bar, it cannot be said that petitioner’s right as a judgment creditor was adversely affected by the lifting of the levy on the
subject real property. Records reveal that there are other pieces of property exclusively owned by the defendants in the HLURB case
that can be levied upon.

Moreover, even granting for the sake of argument that petitioner indeed had the right to intervene, he must exercise said right in
accordance with the rules and within the period prescribed therefor.

As provided in the Rules of Court, the motion for intervention may be filed at any time before rendition of judgment by the trial court.
Petitioner filed his motion only on April 25, 2002, way beyond the period set forth in the rules. The court resolution granting private
respondent’s petition for prohibition and lifting the levy on the subject property was issued on March 22, 2002. By April 6, 2002, after
the lapse of 15 days, the said resolution had already become final and executory.

Pinlac vs. Court of Appeals


Remedial Law I| Page 127

G.R. No. 91486. September 10, 2003

Facts:

Petitioners filed a Petition for Quieting of Title over three vast parcels of land known as Lot Nos. 1, 2 & 3. Lot No. 1 was covered by
TCT No. 5690, while Lot Nos. 2 and 3 were originally covered by OCT No. 614 and OCT No. 333, respectively. The trial court
rendered a Partial Decision in favor of petitioners and against the defendants who were declared in default, including respondent
owners of Vilmar-Maloles (Vilma) Subdivision whose properties were within Lot No. 2. The defaulted title owners of Vilma filed
with the Court of Appeals a Petition to Annul the Partial Decision of the trial court, which was granted.

The appellate court ruled that the court a quo did not acquire jurisdiction over the person of respondents because of defective service
of summons by publication. Petitioners motion for reconsideration of the said decision was also denied. Consequently, it filed a
petition for certiorari before the Supreme Court, but the Court only affirmed CA’s decision. Aggrieved, petitioner then, filed a Motion
for Reconsideration contending among others that the disposition of the trial court with respect to Lot No. 3, should not have been
annulled by the Court of Appeals because the petition for annulment of judgment filed by the respondents concerned only Lot No. 2.
The Court then, issued a Resolution partially granting petitioners motion for reconsideration by reinstating paragraphs 4 and 5 of the
dispositive portion of the trial court’s Partial Decision pertaining to Lot No. 3. However, the Republic of the Philippines, represented
by the Land Registration Authority (LRA), thru the Office of the Solicitor General (OSG), filed a motion for intervention and a
Petition-In-Intervention praying that judgment be rendered declaring among other that, the OCT No. 333 was a valid and existing title
in line with the decisions this Honorable Court had already rendered, hence, this case.

Issue:

Whether a Motion for intervention and a Petition-In-Intervention filed by the Republic were proper taking into account that a partial
Resolution by the Supreme Court has already been rendered?

Held:
Yes. The rule on intervention, like all other rules of procedure is intended to make the powers of the Court fully and completely
available for justice. It is aimed to facilitate a comprehensive adjudication of rival claims overriding technicalities on the timeliness of
the filing thereof. Indeed, in exceptional cases, the Court has allowed intervention notwithstanding the rendition of judgment by the
trial court. In one case, intervention was allowed even when the petition for review of the assailed judgment was already submitted for
decision in the Supreme Court. In Mago v. Court of Appeals intervention was granted even after the decision became final and
executory, thus:

The permissive tenor of the provision on intervention shows the intention of the Rules to give to the court the full measure of
discretion in permitting or disallowing the same. But needless to say, this discretion should be exercised judiciously and only after
consideration of all the circumstances obtaining in the case.

The intervention of the Republic is necessary to protect public interest as well as government properties located and projects
undertaken on Lot No. 3. The Constitutional mandate that no person shall be deprived of life, liberty, or property without due process
of law can certainly be invoked by the Republic which is an indispensable party to the case at bar.  As correctly pointed out by the
Solicitor General, while the provision is intended as a protection of individuals against arbitrary action of the State, it may also be
invoked by the Republic to protect its properties.
Remedial Law I| Page 128

Chipongian vs. Benitez-Lirio


G.R. No. 162692. August 26, 2015

Facts:
The late Vicente Benitez was married to Isabel Chipongian, the petitioner's sister. Isabel had predeceased Vicente. The couple had no
offspring that was why Vicente and the petitioner had executed a deed of extrajudicial settlement respecting the estate of Isabel,
whereby the latter waived all his rights to the estate of Isabel in favor of Vicente. According to the petitioner, however, Vicente
executed an affidavit on the same date whereby he affirmed that the waiver did not extend to the paraphernal properties of Isabel.
Upon the death of Vicente, Victoria Benitez Lirio (Victoria), a sister of Vicente, and Feodor Benitez Aguilar (Feodor), a nephew of
Vicente, initiated proceedings for the settlement of the estate of Vicente in the Regional Trial Court and it issued the letters of
administration to Feodor.
The petitioner intervened in the Special Proceedings and sought the partial revocation of the issuance of the Letters of Administration
order for the exclusion of the paraphernal properties of Isabel from the estate of Vicente. He cited the affidavit of Vicente in support of
the partial revocation. However, Feodor countered with the request that he be allowed to continue to administer all the properties left
by Vicente, including the paraphernal properties of Isabel. Consequently, petitioner specifically moved for the exclusion of the
paraphernal properties of Isabel from Vicente's estate, but he withdrew the motion even before the RTC could rule on it. Instead, he
filed a Motion for Leave to Intervene and to Admit Complaint-in-Intervention. Respondents Victoria and Feodor opposed the
complaint-in-intervention, but the RTC granted the Motion for Leave to Intervene and to Admit Complaint-in-Intervention, and
admitted the complaint-in-intervention of the petitioner.
The RTC rendered judgment dismissing the complaint-in-intervention, for it took petitioner 12 years to assert the purported affidavit
allegedly executed in his favor by Vicente O. Benitez. The Petitioner filed a Motion for Reconsideration for the judgment, but the
same was denied. A notice of appeal was also filed, but was denied for it was filed beyond the reglementary period. Another Motion
for Reconsideration on the denial of notice of appeal was again denied due to petitioner’s failure to pay the court docket fees, although
the Court conceded that the notice of appeal was filed on time. As his last resort, petitioner filed a Motion to Set Aside the order
denying his Motion for Reconsideration, but the same was denied, prompting him to file a petition for Certiorari under Rule 65 before
the CA.
The CA dismissed the petition for certiorari opining that petitioner paid the appeal fees only on March 31, 1999, but as admitted by
him in his Motion for Reconsideration, the last day to perfect his appeal was on August 21, 1998. This prompted petitioner to file a
petition for certiorari before the Supreme Court. While in the Supreme Court, respondents Victoria and Feodor sought the denial of the
petition for review because the petitioner did not file a record on appeal, as mandated under Section 2(a) Rule 41 of the Rules of Court.
However, in his reply to respondent’s comment, petitioner submitted that it had to be noted that the appeal was from the decision of
the trial court to dismiss petitioner's complaint-in-intervention and not “the final order or judgment rendered in the case”, obviously
referring to the main case, that was, the intestate estate case. Since the intervention was not an independent proceeding but only
ancillary or supplemental to the main case, the rule on multiple appeals does not apply and the filing of a record on appeal is not a pre-
requisite to the acceptance and consideration of the appeal by the appellate court.
Issue:
Whether or not the rule on multiple appeals (filing of a record on appeal) is applicable in Intervention?
Held:
Yes. Intervention is "a remedy by which a third party, not originally impleaded in the proceedings, becomes a litigant therein to enable
him, her or it to protect or preserve a right or interest which may be affected by such proceedings." If an intervention makes a third
party a litigant in the main proceedings, his pleading-in-intervention should form part of the main case. Accordingly, when the
petitioner intervened in Special Proceedings No. SP-797, his complaint-in-intervention, once admitted by the RTC, became part of the
main case, rendering any final disposition thereof subject to the rules specifically applicable to special proceedings, including Rule
109 of the Rules of Court, which deals with appeals in special proceedings.
The dismissal of the petitioner's intervention constituted "a final determination in the lower court of the rights of the party appealing,"
that is, his right in the paraphernal properties of his deceased sister. As such, it fell under paragraph (c) of Section 1 because it had
the effect of disallowing his claim against the estate of Vicente, as well as under paragraph (e) of Section 1 because it was a final
determination in the trial court of his intervention. Conformably with either or both paragraphs, the dismissal was the proper subject of
an appeal in due course by virtue of its nature of completely disposing of his intervention. The proper mode of appealing a
judgment or final order in special proceedings is by notice of appeal and record on appeal.

As the foregoing rules further indicate, a judgment or final order in special proceedings is appealed by record on appeal. A judgment
Remedial Law I| Page 129

or final order determining and terminating a particular part is usually appealable, because it completely disposes of a particular matter
in the proceeding, unless otherwise declared by the Rules of Court. The ostensible reason for requiring a record on appeal instead of
only a notice of appeal is the multi-part nature of nearly all special proceedings, with each part susceptible of being finally determined
and terminated independently of the other parts. An appeal by notice of appeal is a mode that envisions the elevation of the original
records to the appellate court as to thereby obstruct the trial court in its further proceedings regarding the other parts of the case. In
contrast, the record on appeal enables the trial court to continue with the rest of the case because the original records remain with the
trial court even as it affords to the appellate court the full opportunity to review and decide the appealed matter. Considering that the
petitioner did not submit a record on appeal in accordance with Section 3 of Rule 41, he did not perfect his appeal of the judgment
dismissing his intervention. As a result, the dismissal became final and immutable. He now has no one to blame but himself. The right
to appeal, being statutory in nature, required strict compliance with the rules regulating the exercise of the right. As such, his
perfection of his appeal within the prescribed period was mandatory and jurisdictional, and his failure to perfect the appeal within the
prescribed time rendered the judgment final and beyond review on appeal.
Remedial Law I| Page 130

Rules 23-29: Modes of Discovery

Dasmariñas Garments vs. Reyes


G.R. No. 108229 August 24, 1993

Facts:

American President Lines (APL) instituted an action against Dasmariñas Garments to recover the sum of US $53,228.45 as well as
an amount equivalent to twenty-five percent (25%) thereof as attorney's fees and litigation expenses. Dasmariñas Garments filed an
Answer with Counterclaim. APL on the other hand, filed a motion during the hearing praying that it intended to take the depositions
of Kenneth H. Lee and Yeong Fang Yeh in Taipei, Taiwan and prayed that for this purpose, a commission or letters rogatory be
issued addressed to the consul, vice-consul or consular agent of the Republic of the Philippines in Taipei.

Five days later, APL filed an amended motion stating that since the Philippine Government has no consulate office in Taiwan in view
of its “one China policy,” there being in lieu thereof an office set up by the President “presently occupied by Director Joaquin Roces
which is the Asian Exchange Center, Inc.,” it was necessary—and it therefore prayed—“that commission or letters rogatory be issued
addressed to Director Joaquin Roces, Executive Director, Asian Executive Exchange Center, Inc., Room 901,112 Chunghsiao, E.
Road, Section 1, Taipei, Republic of China, to hear and take the oral deposition of the forenamed persons.

Dasmariñas Garments opposed the motion on the grounds that the motion was fatally defective in that it does not seek that a foreign
court examine a person within its jurisdiction, the Issuance of letters rogatory was unnecessary because the witnesses can be
examined before the Philippine Court and the Rules of Court expressly require that the testimony of a witness must be taken orally in
open court and not by deposition

The RTC ruled in favor of the motion of the APL and affirmed by the CA.

Issue:

Whether or not the motion to take the testimonies of plaintiffs Taiwanese witnesses, Kenneth H. Lee and Yeong Fah Yeh, by
deposition (upon written interrogatories) should be allowed?

Held:

Yes. Depositions are chiefly a mode of discovery. They are intended as a means to compel disclosure of facts resting in the
knowledge of a party or other person which are relevant in some suit or proceeding in court. Depositions, and the other modes of
discovery (interrogatories to parties; requests for admission by adverse party; production or inspection of documents or things;
physical and mental examination of persons) are meant to enable a party to learn all the material and relevant facts, not only known
to him and his witnesses but also those known to the adverse party and the latter’s own witnesses. In fine, the object of discovery is
to make it possible for all the parties to a case to learn all the material and relevant facts, from whoever may have knowledge thereof,
to the end that their pleadings or motions may not suffer from inadequacy of factual foundation, and all the relevant facts may be
clearly and completely laid before the Court, without omission or suppression.

It is apparent then that the deposition of any person may be taken wherever he may be, in the Philippines or abroad. If the party or
witness is in the Philippines, his deposition “shall be taken before any judge, municipal or notary public” (Sec. 10, Rule 24, Rules of
Court). If in a foreign state or country, the deposition “shall be taken: (a) on notice before a secretary or embassy or legation, consul
general, consul, vice consul, or consular agent of the Republic of the Philippines, or (b) before such person or officer as may be
appointed by commission or under letters rogatory”.

Where the deposition is to be taken in a foreign country where the Philippines has no “secretary or embassy or legation, consul
general, consul, vice-consul, or consular agent,” then obviously it may be taken only “before such person or officer as may be
appointed by commission or under letters rogatory.”

A commission may be defined as “(a)n instrument issued by a court of justice, or other competent tribunal, to authorize a person to
take depositions, or do any other act by authority of such court or tribunal”. Letters rogatory, on the other hand, may be defined as
“(a)n instrument sent in the name and by the authority of a judge or court to another, requesting the latter to cause to be examined,
upon interrogatories filed in a cause pending before the former, a witness who is within the jurisdiction of the judge or court to whom
such letters are addressed”.
Remedial Law I| Page 131

Dasmariñas also contends that the “taking of deposition is a mode of pretrial discovery to be availed of before the action comes to
trial.” Not so. Depositions may be taken at any time after the institution of any action, whenever necessary or convenient. There is no
rule that limits deposition-taking only to the period of pre-trial or before it; no prohibition against the taking of depositions after pre-
trial. Indeed, the law authorizes the taking of depositions of witnesses before or after an appeal is taken from the judgment of a
Regional Trial Court “to perpetuate their testimony for use in the event of further proceedings in the said court” (Rule 134, Rules of
Court), and even during the process of execution of a final and executory judgment.
Remedial Law I| Page 132

Go vs. People
G.R. No. 185527. July 18, 2012

Facts:

Petitioners Harry Go, Tonny Ngo, Jerry Ngo and Jane Go were charged before the MeTC of Manila for Other Deceits under Article
318 of the RPC. Upon arraignment, petitioners pleaded not guilty to the charge. The prosecution’s complaining witness, Li Luen
Ping, a frail old businessman from Laos, Cambodia, traveled from his home country back to the Philippines in order to attend the
hearing held on September 9, 2004. However, trial dates were subsequently postponed due to his unavailability.

The private prosecutor filed with the MeTC a Motion to Take Oral Deposition of Li Luen Ping, alleging that he was being treated for
lung infection at the Cambodia Charity Hospital in Laos, Cambodia and that, upon doctor’s advice, he could not make the long travel
to the Philippines by reason of ill health. Notwithstanding petitioners’ Opposition, the MeTC granted the motion after the prosecution
complied with the directive to submit a Medical Certificate of Li Luen Ping. The RTC reversed the MeTC. The RTC held that
Section 17, Rule 23 on the taking of depositions of witnesses in civil cases cannot apply suppletorily to the case since there is a
specific provision in the Rules of Court with respect to the taking of depositions of prosecution witnesses in criminal cases, which is
primarily intended to safeguard the constitutional rights of the accused to meet the witness against him face to face.

The CA reversed the RTC and affirmed the MeTC’s decision ruling that no rule of procedure expressly disallows the taking of
depositions in criminal cases and that, in any case, petitioners would still have every opportunity to cross examine the complaining
witness and make timely objections during the taking of the oral deposition either through counsel or through the consular officer
who would be taking the deposition of the witness.

Issue:

Whether or not the deposition of a prosecution’s main witness outside the country should be allowed?

Held:

No. The procedure under Rule 23 to 28 of the Rules of Court allows the taking of depositions in civil cases, either upon oral
examination or written interrogatories, before any judge, notary public or person authorized to administer oaths at any time or place
within the Philippines; or before any Philippine consular official, commissioned officer or person authorized to administer oaths in a
foreign state or country, with no additional requirement except reasonable notice in writing to the other party. But for purposes of
taking the deposition in criminal cases, more particularly of a prosecution witness who would forseeably be unavailable for trial, the
testimonial examination should be made before the court, or at least before the judge, where the case is pending as required by the
clear mandate of Section 15, Rule 119 of the Revised Rules of Criminal Procedure.

Since the conditional examination of a prosecution witness must take place at no other place than the court where the case is pending,
the RTC properly nullified the MeTC’s orders granting the motion to take the deposition of Li Luen Ping before the Philippine
consular official in Laos, Cambodia.

The condition of the private complainant being sick and of advanced age falls within the provision of Section 15 Rule 119 of the
Rules of Court. However, said rule substantially provides that he should be conditionally examined before the court where the case is
pending. Thus, this Court concludes that the language of Section 15 Rule 119 must be interpreted to require the parties to present
testimony at the hearing through live witnesses, whose demeanor and credibility can be evaluated by the judge presiding at the
hearing, rather than by means of deposition. No where in the said rule permits the taking of deposition outside the Philippines
whether the deponent is sick or not.
Remedial Law I| Page 133

Vda. de Manguerra vs. Risos


G.R. No. 152643, August 28, 2008

Facts:

Respondents (Risos, et. al.) were charged with Estafa Through Falsification of Public Document before the RTC. The case, arose
from the falsification of a deed of real estate mortgage allegedly committed by respondents where they made it appear that petitioner
(Concepcion De Manguerra), the owner of the mortgaged property known as the Gorordo property, affixed her signature to the
document. Concepcion who was a resident of Cebu City, while on vacation in Manila, was unexpectedly confined at the Makati
Medical Center due to upper gastro-intestinal bleeding; and was advised to stay in Manila for further treatment. The counsel of
Concepcion filed a motion to take the latter’s deposition due to her weak physical condition and old age, which limited her freedom
of mobility. The RTC of Cebu granted the motion and directed that Concepcion’s deposition be taken before the Clerk of Court of
Makati City. After several motions for change of venue of the deposition-taking, Concepcion’s deposition was finally taken at her
residence.
Aggrieved, respondents assailed the RTC orders in a special civil action for certiorari before the CA. The CA nullified the RTC
orders, declaring that the examination of prosecution witnesses, as in the present case, is governed by Section 15, Rule 119 of the
Revised Rules of Criminal Procedure and not Rule 23 of the Rules of Court. The latter provision, said the appellate court, only
applies to civil cases. Pursuant to the specific provision of Section 15, Rule 119, Concepcion’s deposition should have been taken
before the judge or the court where the case is pending, which is the RTC of Cebu, and not before the Clerk of Court of Makati City;
and thus, in issuing the assailed order, the RTC clearly committed grave abuse of discretion. MR was denied. Hence, this petition.
Issue:
Whether or not the deposition of a prosecution witness not before the court where the action is pending may be allowed?
Held:
No. Rule 119 specifically states that a witness may be conditionally examined: 1) if the witness is too sick or infirm to appear at the
trial; or 2) if the witness has to leave the Philippines with no definite date of returning. Undoubtedly, the procedure set forth in Rule
119 applies to the case at bar. It is thus required that the conditional examination be made before the court where the case is pending.
It is also necessary that the accused be notified, so that he can attend the examination, subject to his right to waive the same after
reasonable notice. As to the manner of examination, the Rules mandate that it be conducted in the same manner as an examination
during trial, that is, through question and answer.
Petitioners further insist that Rule 23 applies to the instant case, because the rules on civil procedure apply suppletorily to criminal
cases. It is true that Section 3, Rule 1 of the Rules of Court provides that the rules of civil procedure apply to all actions, civil or
criminal, and special proceedings. In effect, it says that the rules of civil procedure have suppletory application to criminal cases.
However, it is likewise true that the criminal proceedings are primarily governed by the Revised Rules of Criminal Procedure.
Considering that Rule 119 adequately and squarely covers the situation in the instant case, we find no cogent reason to apply Rule 23
suppletorily or otherwise.
Rule 119 categorically states that the conditional examination of a prosecution witness shall be made before the court where the case
is pending. Contrary to petitioner’s contention, there is nothing in the rule which may remotely be interpreted to mean that such
requirement applies only to cases where the witness is within the jurisdiction of said court and not when he is kilometers away, as in
the present case. Therefore, the court may not introduce exceptions or conditions.
Remedial Law I| Page 134

People vs. Webb


G.R. No. 132577. August 17, 1999

Facts:

Hubert Webb, an accused in the crime of Rape with Homicide, filed a Motion to Take Testimony by Oral Deposition, to take the
testimonies of some vital witnesses residing in the U.S., before the proper Philippine consular authorities since the Philippine court
had no jurisdiction over them and may not therefore be compelled by subpoena to testify. Respondent further alleged that the taking
of the oral depositions of the aforementioned individuals whose testimonies are allegedly ‘material and indispensable’ to establish his
innocence of the crime charged is sanctioned by Section 4, Rule 24 of the Revised Rules of Court. The prosecution thereafter filed an
opposition to the said motion averring that Rule 24, Section 4 of the Rules of Court has no application in criminal cases. The trial
court denied the motion but was thereafter reversed by the CA on appeal.

Issue:

Whether or not deposition of the defense’s witnesses outside the country may be allowed in this case?

Held:

No. It need not be overemphasized that the factual circumstances only serves to underscore the immutable fact that the depositions
proposed to be taken from the five U.S. based witnesses would be merely corroborative or cumulative in nature and in denying
respondent’s motion to take them, the trial court was but exercising its judgment on what it perceived to be a superfluous exercise on
the belief that the introduction thereof will not reasonably add to the persuasiveness of the evidence already on record.

It is pointed out that the defense has already presented at least fifty-seven (57) witnesses and four hundred sixty four (464)
documentary exhibits, many of them of the exact nature as those to be produced or testified to by the proposed foreign deponents.
Under the circumstances, we sustain the proposition that the trial judge commits no grave abuse of discretion if she decides that the
evidence on the matter sought to be proved in the United States could not possibly add anything substantial to the defense evidence
involved.

The use of discovery procedures is directed to the sound discretion of the trial judge. The deposition taking can not be based nor can
it be denied on flimsy reasons. Discretion has to be exercised in a reasonable manner and in consonance with the spirit of the law.
Remedial Law I| Page 135

Disini vs. Sandiganbayan


G.R. No. 175730, July 5, 2010

Facts:

Disini, herein petitioner, was charged for two information for corruption of public officials, Art 212 in relation to Art 210 (RPC) and
violation of RA 3019. For failure to file a responsive pleading, petitioner was declared in default. Subsequently, petitioner filed a
Motion to Lift Order of Default and for Leave to File and Admit Attached Answer, together with an Answer to Amended Complaint
with Compulsory Counterclaims. He maintained that he was unaware of the civil case pending against him because he never received
summons or other processes from the court, nor any pleadings from the parties of the case. His only fault, he averred, was that he was
ignorant of the proceedings in the case because of the absence of a proper notice. Petitioner asked the respondent court to look at his
meritorious defenses. He then invoked the liberality of the courts in lifting default orders to give both parties every opportunity to
defend their cases, and pointed out that the proceedings, being in their pre-trial stage, would not be delayed by petitioner’s
participation therein. Sandiganbayan denied his motion to lift order of default. Thus, petitioner filed a Petition for Certiorari before
the Supreme Court.

In his Petition, petitioner originally sought the nullification of the proceedings before the Sandiganbayan on the theory of lack of
jurisdiction over his person, premised on the alleged impropriety in the service of summons. However, petitioner subsequently filed
several motions with the Sandiganbayan which sought various affirmative reliefs from that court, sans any qualification of the nature
of its appearance and without reserving or reiterating its previous objection on the ground of lack of jurisdiction over the person. One
of the motions is a Motion for Leave To Take Deposition.

Issue:

Whether or not by filing a Motion for Leave To Take Deposition, the petitioner is deemed to have waived his objections to the lack
of jurisdiction of the Sandiganbayan?

Held:

Yes. It is important to note that there are two instances when the defendant can take depositions under Section 1 of Rule 23: (1) after
the court has acquired jurisdiction over the defendant or the property subject of the action; and (2) after an answer has been served.
Both instances presuppose that the court has already acquired jurisdiction over the defendant. By seeking this relief (Motion for
Leave To Take Deposition), petitioner is deemed to have voluntarily submitted himself to the jurisdiction of the Sandiganbayan.
Thus, petitioner may be held to have waived his objections regarding the lack of jurisdiction over his person by seeking affirmative
relief through the said provision.

While petitioner bewailed the mode of service of summons on him and questioned the Sandiganbayan’s jurisdiction over his person,
he has rendered his own arguments moot by his voluntary appearance or submission to the jurisdiction of the  Sandiganbayan.
Jurisprudence holds that an objection based on lack of jurisdiction over the person is waived when the defendant files a motion or
pleading which seeks affirmative relief other than the dismissal of the case.
Remedial Law I| Page 136

Afulugencia vs. Metro Bank


G.R. No. 185145. February 5, 2014.

Facts:

Petitioners, Spouses Vicente and Leticia Afulugencia, filed a Complaint for nullification of mortgage, foreclosure, auction sale,
certificate of sale and other documents, with damages, against respondents Metropolitan Bank & Trust Co. (Metrobank) and
Emmanuel L. Ortega (Ortega) before the RTC of Malolos City. With the conclusion of pre-trial, petitioners filed a Motion for
Issuance of Subpoena Duces Tecum Ad Testificandum to require Metrobank’s officers to appear and testify as the petitioners’ initial
witnesses during the August 31, 2006 hearing for the presentation of their evidence-in-chief, and to bring the documents relative to
their loan with Metrobank, as well as those covering the extrajudicial foreclosure and sale of petitioners’ 200- square meter land in
Meycauayan, Bulacan.

Metrobank filed an Opposition arguing that for lack of a proper notice of hearing, the Motion must be denied; that being a litigated
motion, the failure of petitioners to set a date and time for the hearing renders the Motion ineffective and pro forma; that pursuant to
Sections 1 and 6 of Rule 25 of the Rules, Metrobank’s officers – who are considered adverse parties – may not be compelled to
appear and testify in court for the petitioners since they were not initially served with written interrogatories; that petitioners have not
shown the materiality and relevance of the documents sought to be produced in court; and that petitioners were merely fishing for
evidence.

Issue:

Whether or not a party not served with written interrogatories may be compelled by the adverse party to give testimony in open
court?

Held:

No. As a rule, in civil cases, the procedure of calling the adverse party to the witness stand is not allowed, unless written
interrogatories are first served upon the latter. This is embodied in Section 6, Rule 25 of the Rules, which provides – Sec. 6. Effect of
failure to serve written interrogatories. Unless thereafter allowed by the court for good cause shown and to prevent a failure of
justice, a party not served with written interrogatories may not be compelled by the adverse party to give testimony in open court, or
to give a deposition pending appeal.

One of the purposes of the above rule is to prevent fishing expeditions and needless delays; it is there to maintain order and facilitate
the conduct of trial. It will be presumed that a party who does not serve written interrogatories on the adverse party beforehand will
most likely be unable to elicit FACTS useful to its case if it later opts to call the adverse party to the witness stand as its witness.

In the present case, petitioners seek to call Metrobank’s officers to the witness stand as their initial and main witnesses, and to present
documents in Metrobank’s possession as part of their principal documentary evidence. This is tantamount to building their whole
case from the evidence of their opponent. The burden of proof and evidence falls on petitioners, not on Metrobank; if petitioners
cannot prove their claim using their own evidence, then the adverse party Metrobank may not be pressured to hang itself from its
own defense.
Remedial Law I| Page 137

Allied Agri-Business Development Co. vs. Court of Appeals


G.R. No. 118438. December 4, 1998

Facts:

Cherry Valley Farms Limited (Cherry Valley), a foreign company based in England, filed against petitioner Allied Agri-Business
Development Co. Inc. (Allied) a complaint with the Regional Trial Court of Makati City for collection of sum of money for failure of
Allied to pay several duck hatching eggs and ducklings which in value totaled 51,245.12. Allied filed an answer  denying the material
allegations of the complaint. Cherry Valley served on Allied’s counsel a Request for Admission. Allied filed
its Comments/Objections alleging that: (a) the admissions requested were matters which the private respondent had the burden to
prove through its own witness during the trial and thus petitioner need not answer; and, (b) the request for admission regarding the
ownership set-up of petitioner corporation was immaterial and improper for not having been pleaded in the complaint. In its
Reply to Comments/Objections to Request for Admission, Cherry Valley maintained that there was no need on its part to produce a
witness to testify on the matters requested for admission, for these pertained to incidents personal to and within the knowledge of
petitioner alone. Thereafter, Cherry Valley filed a motion with the trial court to resolve the objections of Allied to the request for
admission. Trial court issued an Order disregarding Allied’s Comments/Objections to Request for Admission in view of its non-
compliance with Sec. 2, Rule 26, of the Rules of Court and directing Allied to answer the request for admission within ten (10) days
from receipt of the order, otherwise, the matters contained in the request would be deemed admitted. Allied moved to reconsider the
order but was denied. Allied’s motion for reconsideration and directed the latter to answer the request for admission within a non-
extendible period of five (5) days from receipt of the order. Allied failed to submit a sworn answer to the request for admission
within the additional period of five (5) days granted by the trial court. Hence, Cherry Valley filed a motion for summary judgment
alleging that there was already an implied admission on the matters requested for admission pursuant to Rule 26 of the Rules of
Court. Trial court rendered judgment against petitioner. Allied appealed to the Court of Appeals which  affirmed the summary
judgment. Hence, this petition.
Issue:
Whether or not Allied’s failure to answer the request for admission resulted in its admission of the matters stated in the request?

Held:

Yes. The purpose of the rule governing requests for admission of facts and genuineness of documents is to expedite trial and to relieve
parties of the costs of proving facts which will not be disputed on trial and the truth of which can be ascertained by reasonable
inquiry. Each of the matters of which an admission is requested shall be deemed admitted unless within a period designated in the
request which shall not be less than fifteen (15) days after service thereof, or within such further time as the court may allow on
motion, the party to whom the request is directed files and serves upon the party requesting the admission a sworn statement either
denying specifically the matters of which an admission is requested or setting forth in detail the reasons why he cannot truthfully
either admit or deny those matters. Upon service of request for admission, the party served may do any of the following acts:  (a) he
may admit each of the matters of which an admission is requested, in which case, he need not file an answer; (b) he may admit the
truth of the matters of which admission is requested by serving upon the party requesting a written admission of such matters within
the period stated in the request, which must not be less than ten (10) days after service, or within such further time as the court may
allow on motion and notice; (c) he may file a sworn statement denying specifically the matter of which an admission is requested; or,
(d) he may file a sworn statement setting forth in detail the reasons why he cannot truthfully either admit or deny the matters of which
an admission is requested.
The records show that although petitioner filed with the trial court its comments and objections to the request for admission served on
it by private respondent, the trial court disregarded the objections and directed petitioner after denying its motion for reconsideration,
to answer the request within five (5) days from receipt of the directive; otherwise, the matters of which the admission was requested
would be deemed admitted. Petitioner failed to submit the required answer within the period. The matter set forth in the request were
therefore deemed admitted by petitioner. The burden of affirmative action is on the party upon whom notice is served to avoid the
admission rather than upon the party seeking the admission. Hence, when petitioner failed to reply to a request to admit, it may not
argue that the adverse party has the burden of proving the facts sought to be admitted. Petitioners’ silence is an admission of the facts
stated in the request.
Remedial Law I| Page 138

Philippine Health Insurance Corporation vs. Our Lady of Lourdes Hospital


G.R. No. 193158, November 11, 2015

Facts:

Petitioner Philippine Health Insurance Corporation (PHIC) filed a complaint with its Legal Sector-Prosecution Department against
Our Lady of Lourdes Hospital (OLLH) for the administrative offense of filing multiple claims, which was penalized under one of the
provisions of the Implementing Rules and Regulations (IRR) of R.A. No. 7875. Allegedly, OLLH filed two claims of the same
amount of PhilHealth benefits involving the same patient for the same diagnosis and covering the same period of confinement.
Summons was duly served, then, an answer was filed by OLLH. The parties were then, directed to file their respective Position
Papers, but only PHIC complied with the order. OLLH moved to defer the submission of its position paper pending the answer of
the PHIC President and CEO to the written interrogatories as well as the inspection and copying of the original transmittal
letter and all other claims that accompanied Annex B11 of the Complaint. According to OLLH, these modes of discovery were
availed of because its representatives were denied and/or not given access to documents and were not allowed to talk to PHIC
personnel with regard to the charge. PHIC filed its Comment on OLLH’s motion. In its defense, PHIC asserted that OLLH’s overt
acts clearly revealed its intent to delay the administrative proceedings. It stressed that the material points which OLLH sought to
establish in its resort to modes of discovery were already presented in the pleadings and documents it submitted for consideration of
the Arbiter. Specifically, the subject information and documents sought to be examined were the same information and documents
which OLLH itself prepared, produced, and submitted to the PHIC. Likewise, the PHIC procedure subject of the interrogatories,
apart from being publicly accessible and already known to OLLH, was immaterial to the case given OLLH’s sole defense that it
inadvertently attached the wrong document that led to the processing of two separate claims.

PHIC Arbitration Department ruled that the interrogatories and motion for production and inspection of documents filed by OLLH
could not be given be given due course. The Court of Appeals reversed the Resolution of the PHIC Arbiter and ruled that petitioner
OLLH has shown good cause for its resort to the modes of discovery as the same was anchored on its being able to intelligently
prepare a position paper considering that it was not allowed access to some pertinent documents or talk to PHIC personnel with
regard the charge of filing multiple claims.

Issue:

Whether or not OLLH’s resort to modes of discovery, specifically the written interrogatories and production and inspection of
documents, was proper?

Held:

No. Through written interrogatories, a party may elicit from the adverse party or parties any facts or matter that are not privileged and
are material and relevant to the subject of the pending action. Like other modes of discovery authorized by the Rules, the purpose of
written interrogatories is to assist the parties in clarifying the issues and in ascertaining the facts involved in a case. On the other
hand, the provision on production and inspection of documents is to enable not only the parties but also the court (in this case, the
PHIC Arbitration Department) to discover all the relevant and material facts in connection with the case pending before it. It must be
shown, therefore, that the documents sought to be produced, inspected and/or copied/photographed are material or contain evidence
relevant to an issue involved in the action.

In this case, the questions contained in the written interrogatories filed and received on July 28, 2009 sought to elicit facts that could
already be seen from the allegations as well as attachments of the Complaint and the Verified Answer. Specifically, the entries in the
three (3) Validation Report that OLLH sought to be identified and/or explained by PHIC are either immaterial or irrelevant (to the
issue of whether OLLH is guilty of filing multiple claims and OLLH’s defense that it inadvertently attached a second copy of the
subject PhilHealth Claim Form 2 to the Transmittal Letter filed on June 19, 2007) or, even if material or relevant, are self-
explanatory and need no further elaboration from PHIC. Thus, the interrogatories were frivolous and need not be answered. Aside
from this, the PHIC Arbitration Department correctly observed that the written interrogatories were mistakenly addressed to the
President and CEO of PHIC, who could not competently answer, either based on his job description or firsthand experience, issues
that arose from and related to the filing and processing of claims. We likewise find as self-serving the allegation of OLLH that its
representatives were denied access to the documents pertaining to the subject PhilHealth claim and, at the same time, were not
allowed to talk to any of the PhilHealth personnel. No iota of evidence, documentary or testimonial, was submitted to substantiate
this convenient excuse.
Remedial Law I| Page 139

Rule 30: Trial


People vs. Bustamante
G.R. No. 189836, June 5, 2013

Note: The case illustrated the order in the examination of witness (direct, cross, redirect, recross) under Rule 132. This is different
in the order of trial (in civil cases, Section 5, Rule 30 and in criminal cases, Sec. 11, Rule 119)

Facts:

Romeo Bustamante was charged with the crime of rape by means of threat and intimidation for having sexual intercourse with his own
daughter, the herein offended party, AAA for several times, starting from the time that the offended party was only eleven (11) years
of age.

[AAA] testified that she lived with his father, the [appellant] in this case, mother and younger siblings, 3 brothers and a sister, in x x x,
Alcala, Cagayan. At about lunch time or thereafter on February 17, 1997, she was alone in the second floor in their house when the
[appellant] arrived. Her younger brother Jayjun was playing outside while her mother went to clean their ginger garden. The
[appellant] laid her down on the floor and removed her shorts and panty. He then removed his pants, went on top of her and inserted
his penis into her vagina. [Appellant] removed his penis after he ejaculated and told her not to report what had happened. [Appellant]
forced her and she was not able to resist because she was still young during that time. She reported the incident to her mother and the
police.

On re-direct examination, [AAA] testified that she filed the case against the [appellant] so that the latter would no longer box and
maltreat her and because he raped her. On re-cross, it was revealed that [appellant] was neither armed during the incident nor covered
her mouth when he laid her down. She did not shout because she was afraid. [Appellant] threatened her before he raped her.

xxxx

[Appellant] testified that in the early morning on February 7, 1997, he went to Tuguegarao with his daughter, [AAA]. He went to Mrs.
Lolit Casauay, his employer, and Sgt. Poli to tell them his problem regarding [AAA] and her cousin having sex. Sgt. Poli advised
[appellant] to go to Alcala Police to have his problem entered in the blotter and to go back to him after. They stayed in Tuguegarao the
whole day and went back to x x x, Alcala, Cagayan about 7:00 o’clock in the evening. When they were approaching their house,
Purita Torrado called for [AAA] and told [appellant] that he was a traitor. Purita Torrado and brothers, Rogelio and Amador Torrado,
then entered his house, mauled him and tied his hands. Thereafter, policemen arrived and brought him to the Municipal Hall of Alcala,
Cagayan without informing him why. His daughter [AAA] charged him of the heinous crime of rape because his wife and brothers-in-
law harbored ill feelings against him, blaming him to have spread the rumor that Rogelio Torrado was the father of the child of his
own sister Purita Torrado. Before February 17, 1997, his daughter [AAA] admitted to him that she had sexual relations with her
cousin Randy Torrado for which reason he went to Tuguegarao to help [AAA] file a complaint against said Randy Torrado. It was
after they came from Tuguegarao that his daughter [AAA] charged him with rape.

On cross-examination, [appellant] testified that he did not report any barangay official that Randy Torrado sexually molested his
daughter x x x, but went to a person Ernie Fiesta who was not a barangay official. He admittedly told his problem to Sgt. Poli who
asked [AAA] questions but the same was not entered in the blotter of the Cagayan Police Provincial Office.

On re-direct, [appellant] further testified that it was his daughter [AAA] who told him that Randy Torrado molested her so he brought
her to Tuguegarao the following day, February 17, 1997. He was not able to enter it in the blotter of Alcala police as directed by Sgt.
Poli because when they arrived in Maraburab, Alcala from Tuguegarao, his brothers-in-law mauled him. He did not file any charges
against his brothers-in-law.

Police Inspector Carlos T. Poli testified as follows: He was then the Assistant Chief Investigator at the Cagayan Police Provincial
Headquarters on February 17, 1997. [Appellant], with his daughter [AAA] went to him. [Appellant] told him that his daughter was
molested by a nephew of his wife but he could not recall the name. He advised [appellant] to have the incident reported to, and entered
in the blotter of, the Alcala Police Station where the incident took place and to return for investigation. He talked to [AAA] who
admitted that there was truth to the report that she was molested and that there was a second occasion. He did not enter the report in
the blotter because they did not have a blotter so he advised [appellant] to have the case entered in the blotter of Alcala Police. The
report was not recorded because [appellant] only ought his advice and that he would first talk to his wife as the suspect was her
relative. Admittedly, he invited the wife of [appellant] to his office upon the request of her in-laws who pitied and considered the
[appellant] as their son. He asked the wife if she could help but the latter could not do it because her brother and sister were interested
to pursue the case.
Remedial Law I| Page 140

The trial court convicted him of the said crime. In his appeal, he claims that, as evidenced by the victim’s own testimony, AAA filed a
false complaint of rape against him mainly due to her ill feelings towards him brought about by his purported repeated physical
maltreatment of the victim.

Issue:

Whether or not the accused should be convicted of the crime of rape?

Held:

Yes. The Court rule that, it is settled in jurisprudence that in a prosecution for rape, the accused may be convicted solely on the basis
of the testimony of the victim that is credible, convincing, and consistent with human nature and the normal course of
things. Jurisprudence is likewise instructive that the factual findings of the trial court, especially on the credibility of the rape victim,
are accorded great weight and respect and will not be disturbed on appeal.

In the case at bar, both the trial court and the Court of Appeals found AAA to be a credible witness and her testimony worthy of full
faith and credit.

In his defense, appellant interposes denial while also ascribing ill motive on the part of the victim, his own biological daughter, for
accusing him of rape. However, it is well-settled that denial, if unsubstantiated by clear and convincing evidence, is a self-serving
assertion that deserves no weight in law because denial cannot prevail over the positive, candid and categorical testimony of the
complainant, and as between the positive declaration of the complainant and the negative statement of the appellant, the former
deserves more credence. Likewise, the testimonies of the witnesses presented by appellant failed to buttress his defense of denial as
they merely related to tangential matters which do not seriously affect the issue of AAA’s credibility.
Remedial Law I| Page 141

Rule 31: Consolidation and Severance

Metrobank vs. Sandoval


G.R. No. 169677 February 18, 2013

Facts:

On July 17, 1987, the Republic brought a complaint for reversion, reconveyance, restitution, accounting and damages in the
Sandiganbayan against Andres V. Genito, Jr., Ferdinand E. Marcos, Imelda R. Marcos and other defendants. The action was to
recover allegedly ill-gotten wealth of the Marcoses, their nominees, dummies and agents. Among the properties subject of the action
were 2 parcels of commercial land located in Tandang Sora (Old Balara), Quezon City, registered in the names of Spouses Andres V.
Genito, Jr. and Ludivina L. Genito. Republic later on amended its complaint to implead Asian Bank as an additional defendant since
the latter claimed ownership of the 2 parcels of land as the registered owner and it was also in possession of the properties by virtue of
the writ of possession issued by the RTC QC.

When the Republic was about to terminate its presentation of evidence against the original defendants, it moved to hold a separate trial
against Asian Bank. Asian Bank sought the deferment of any action on the motion and contended that it would be deprived of its day
in court if a separate trial were to be held against it without having been sufficiently apprised about the evidence the Republic had
adduced before it was brought in as an additional defendant. Republic maintained that a separate trial for Asian Bank was proper
because its cause of action against Asian Bank was entirely distinct and independent from its cause of action against the original
defendants; and that the issue with respect to Asian Bank was whether Asian Bank had actual or constructive knowledge at the time of
the issuance of the TCTs for the properties in its name that such properties were the subject of the complaint in Civil Case No. 0004,
while the issue as to the original defendants was whether they had "committed the acts complained of as constituting illegal or
unlawful accumulation of wealth which would, as a consequence, justify forfeiture of the said properties or the satisfaction from said
properties of the judgment that may be rendered in favor of the Republic." Asian Bank’s rejoinder to the reply asserted that the issue
in Civil Case No. 0004 was intimately related to the issue delving on the character of the properties as the ill-gotten wealth of the
original defendants; that it thus had a right to confront the evidence presented by the Republic as to the character of the properties; and
that the Sandiganbayan had no jurisdiction to decide Asian Bank’s ownership of the properties because the Sandiganbayan, being a
special court with limited jurisdiction, could only determine the issue of whether or not the properties were illegally acquired by the
original defendants. Sandiganbayan granted the Republic’s motion for separate trial. Asian Bank moved for the reconsideration of the
resolution, but the Sandiganbayan denied its motion.

Hence, Metrobank commenced this special civil action for certiorari as the successor-in-interest of Asian Bank and transferee of the
properties.

Issue:

Whether or not a separate trial against Asian Bank was proper?

Held:

No. The rule on separate trials in civil actions is found in Section 2, Rule 31 of the Rules of Court, which reads:

Section 2. Separate trials. – The court, in furtherance of convenience or to avoid prejudice, may order a separate trial of any claim,
cross-claim, counterclaim, or third-party complaint, or of any separate issue or of any number of claims, cross-claims, counterclaims,
third-party complaints or issues.

The text of the rule grants to the trial court the discretion to determine if a separate trial of any claim, cross-claim, counterclaim, or
third-party complaint, or of any separate issue or of any number of claims, cross-claims, counterclaims, third-party complaints or
issues should be held, provided that the exercise of such discretion is in furtherance of convenience or to avoid prejudice to any party.

Sandiganbayan committed grave abuse of its discretion in ordering a separate trial as to Asian Bank (Metrobank) on the ground that
the issue against Asian Bank was distinct and separate from that against the original defendants. Thereby, the Sandiganbayan veered
away from the general rule of having all the issues in every case tried at one time, unreasonably shunting aside the dictum in Corrigan,
supra, that a "single trial will generally lessen the delay, expense, and inconvenience to the parties and the courts." Exceptions to the
general rule are permitted only when there are extraordinary grounds for conducting separate trials on different issues raised in the
same case, or when separate trials of the issues will avoid prejudice, or when separate trials of the issues will further convenience, or
Remedial Law I| Page 142

when separate trials of the issues will promote justice, or when separate trials of the issues will give a fair trial to all parties.
Otherwise, the general rule must apply. The justification of the Sandiganbayan for allowing the separate trial did not constitute a
special or compelling reason like any of the exceptions.

To begin with, the issue relevant to Asian Bank was not complicated. In that context, the separate trial would not be in furtherance of
convenience. And, secondly, the cause of action against Asian Bank was necessarily connected with the cause of action against the
original defendants. Should the Sandiganbayan resolve the issue against Spouses Genito in a separate trial on the basis of the evidence
adduced against the original defendants, the properties would be thereby adjudged as ill-gotten and liable to forfeiture in favor of the
Republic without Metrobank being given the opportunity to rebut or explain its side. The outcome would surely be prejudicial
towards Metrobank.
Remedial Law I| Page 143

Republic vs. Heirs of Enrique Oribello


G.R. No. 199501 March 6, 2013

Facts:

This case involves a parcel of land situated in Nagbaculao, Kalaklan, Olongapo City, which was once classified as forest land by the
Bureau of Forest Development. The property was originally occupied by a certain Valentin Fernandez (Valentin) in 1968 by virtue of
a Residential Permit issued by the same government office. Upon Valentin’s death, his son Odillon continued to pay the property,
together with Spouses Apog. Odillon sold the property to Mrs. Balcita who later on sold the property to Oribello. Oribello then filed a
Miscellaneous Sales Application which as initially denied but on his second try, a sales patent was issued to him. Spouses Apog
protested with the DENR the grant of the patent and the DENR recommended the filing of a reversion proceeding suit. The OSG
instituted a complaint for reversion and cancellation of title before RTC Olongapo. The case was thereafter consolidated with the
complaint for recovery of possession filed by Spouses Apog.

On July 25,1997, the court issued an order stating that on several occasions when these cases were set for trial , neither of the
Republic’s counsels appeared., constraining the court to postpone the hearing. The actuations of both lawyers result to delay in the
early termination of these cases which have been pending since 1992.

The trial court dismissed the consolidated cases without prejudice for non-substitution of the deceased plaintiff (Oribello) and his
counsel. Republic moved for reconsideration, contending that the order applied exclusively to civil case for recovery of possession and
did not affect case for reversion of property. Petitioner prayed that it allowed to present its evidence. The Trial Court allowed the
continuation of the presentation of petitioner’s evidence in its Order dated June 2005. Heirs of Oribello filed then a Manifestation and
Motion. They pointed out that from the time the Republic received the Order in 1997, it did nothing to question the same, making the
order final. In its resolution in July 12, 2006, the RTC recalled its order and dismissed the case.

The Republic appealed to the CA. The CA agreed with respondents that petitioner has lost its right to participate in the proceedings of
the case when it failed to question the RTC’s September 12, 1997 order declaring it to have abandoned the case as a consequence of
petitioner’s inaction, such order inevitably became final. Republic, however, contends that the order, deeming it to have abandoned
the cases, is interlocutory in nature; thus is not appealable. Respondents argue otherwise, maintaining that such order is a dismissal of
the complaint on the ground of failure to prosecute which is under the Rules, considered an adjudication on the merits, and hence,
appealable.

Issue:

Whether or not the consolidated cases can be the subject of multiple appeals (or in other words, whether or not the judgment in one of
the consolidates cases can be appealed independently of the other case)?

Held:

Yes. Section 1, Rule 31 of the Rules of Court provides:

SECTION 1. Consolidation. When actions involving a common question of law or fact are pending before the court, it may order a
joint hearing or trial of any or all the matters in issue in the actions; it may order all the actions consolidated, and it may make such
orders concerning proceedings therein as may tend to avoid unnecessary costs or delay.

Consolidation is a procedural device to aid the court in deciding how cases in its docket are to be tried so that the business of the court
may be dispatched expeditiously and with economy while providing justice to the parties. To promote this end, the rule allows the
consolidation and a single trial of several cases in the court’s docket, or the consolidation of issues within those cases.

In the context of legal procedure, the term “consolidation” is used in three different senses:

(1) Where all except one of several actions are stayed until one is tried, in which case the judgment in the one trial is conclusive as to
the others. This is not actually consolidation but is referred to as such. (quasi-consolidation) (2) Where several actions are combined
into one, lose their separate identity, and become a single action in which a single judgment is rendered. This is illustrated by a
situation where several actions are pending between the same parties stating claims which might have been set out originally in one
complaint. (actual consolidation)
Remedial Law I| Page 144

(3) Where several actions are ordered to be tried together but each retains its separate character and requires the entry of a separate
judgment. This type of consolidation does not merge the suits into a single action, or cause the parties to one action to be parties to the
other. (consolidation for trial)

In the present case, the complaint for reversion filed by petitioner (Civil Case No. 225-0-92) was consolidated with the complaint for
recovery of possession filed by Oribello (Civil Case No. 223-0-91). While these two cases involve common questions of law and fact,
each action retains its separate and distinct character. The reversion suit settles whether the subject land will be reverted to the State,
while the recovery of possession case determines which private party has the better right of possession over the subject property.
These cases, involving different issues and seeking different remedies, require the rendition and entry of separate judgments. The
consolidation is merely for joint trial of the cases. Notably, the complaint for recovery of possession proceeded independently of the
reversion case, and was disposed of accordingly by the trial court.

Since each action does not lose its distinct character, severance of one action from the other is not necessary to appeal a judgment
already rendered in one action. There is no rule or law prohibiting the appeal of a judgment or part of a judgment in one case which is
consolidated with other cases. Further, severance is within the sound discretion of the court for convenience or to avoid prejudice. It is
not mandatory under the Rules of Court that the court sever one case from the other cases before a party can appeal an adverse ruling
on such case.
Remedial Law I| Page 145

Maraño vs. Pryce Gases, Inc.


G.R. No. 196592. April 6, 2015

Facts:

Petitioners (Spouses Juvy and Maria Luisa Maraño) filed for free patent application for a parcel of land in Damulaan, Albuera, Leyte
which was granted. Consequently, they then filed an ejectment complaint against respondent (Pryce Gases, Inc.) alleging that the latter
illegally entered the subject lot and constructed a building thereon. The MTC granted the petition, but was later on reversed by the
RTC. On appeal, the Court of Appeals remanded the case to the MTC for trial as a reivindicatory action under the ordinary rules of
civil procedure.

However, in the interim, the respondent filed a protest on the free patent application filed by the petitioners. The petitioners filed an
action to quiet title against the respondent with the RTC, 8th Judicial Region, Branch 14, Baybay City, Leyte and a month later, the
respondent also filed a complaint for reconveyance against the petitioners before the same RTC. The petitioners moved to dismiss the
respondent’s complaint, but the RTC denied their motion. The respondent later moved to amend its complaint from reconveyance to
the cancellation of the petitioners’ certificate of title. The petitioners again moved to dismiss the respondent’s amended complaint on
the ground of litis pendentia in view of the then pending reivindicatory action with the MTC.

The RTC dismissed the petitioners’ motion. The petitioners moved for reconsideration but their motion was likewise denied by the
RTC. The petitioners questioned the RTC’s resolution in a petition for certiorari with the CA.

The CA held that no litis pendentia existed between the reivindicatory action (then pending before the MTC) and the amended
complaint for cancellation of certificate of title filed by the respondent with the RTC. The petitioners moved to reconsider the CA’s
decision but their motion was denied, hence, this petition.

Issue:

Whether or not there was no litis pendentia existed between the reivindicatory action (then pending before the MTC) and the amended
complaint for cancellation of certificate of title filed by the respondent with the RTC?

Held:

Yes. Instead of ordering the dismissal of the respondent’s complaint for cancellation of certificate of title, the consolidation of the
reivindicatory action and the cancellation of certificate of title case is the appropriate remedy in the present situation. Consolidation is
proper when two or more actions pending, not necessarily, before the same court involve a common question of law or fact. In such
cases, the court may, order a joint hearing or trial of any or all the matters in issue in the actions, order all the actions consolidated, and
make such orders concerning the proceedings therein for the purpose of avoiding unnecessary costs and delay. Considering that the
validity of the petitioners’ certificate of title is the crucial issue in both the reivindicatory action pending appeal before the RTC and
the cancellation of certificate of title case filed by the respondent, these two cases should be consolidated in order to avoid the
possibility of rendering conflicting decisions and for the orderly administration of justice. And since the issue of validity of the
petitioners’ certificate of title has been subjected to a full-blown trial before the MTC and is now the subject of appeal before the RTC,
allowing the cancellation of certificate of title case to proceed independently and separately would be needlessly circuitous and would
necessarily delay the resolution of the present issue.
Remedial Law I| Page 146

Rule 33: Demurrer to Evidence

Republic vs. Gimenez


G.R. No. 174673, January 11, 2016

Facts:

The Republic, through the Presidential Commission on Good Government (PCGG), instituted a Complaint for Reconveyance,
Reversion, Accounting, Restitution and Damages against the Gimenez Spouses before the Sandiganbayan. The Complaint seeks to
recover ill-gotten wealth acquired by the Gimenez Spouses as dummies, agents, or nominees of former President Ferdinand E. Marcos
and Imelda Marcos. Documentary and testimonial evidence were presented by the Republic and accordingly, the Sandiganbayan gave
the Republic 30 days or until March 29, 2006 to file its formal offer of evidence. However, despite the repeated extensions and the
lapse of 75 days from the date of the termination of the presentation of evidence, Republic failed to submit its Formal Offer of
Evidence. The Sandiganbayan issued a Resolution declaring that the Republic waived the filing of its Formal Offer of Evidence.

Ignacio Gimenez then filed a Motion to Dismiss on Demurrer to Evidence. He argued that the Republic showed no right to relief as
there was no evidence to support its cause of action. Later, Fe Roa Gimenez filed a Motion to Dismiss on the ground of failure to
prosecute. Through her own Motion to Dismiss, she joined Ignacio Gimenez’s demurrer to evidence. Two days after, Fe Roa
Gimenez’s filing of the Motion to Dismiss, the Republic filed a Motion for Reconsideration [of the first assailed Resolution] and to
Admit Attached Formal Offer of Evidence. The Sandiganbayan on its second Resolution denied the Republic’s Motion for
Reconsideration and granted the Gimenez Spouses’ Motion to Dismiss. Hence, this petition. The Republic contends, among others,
that the Sandiganbayan erred when it granted the demurrer to evidence filed by respondents and dismissed the case despite a " prima
facie foundation (based on the pleadings and documents on record) that spouses Gimenezes amassed enormous wealth grossly
disproportionate to their lawful income or declared lawful assets."

Issue:

Whether or not the Sandiganbayan erred in granting the Demurrer to Evidence?

Held:

Yes. To erroneously grant a dismissal simply based on the delay to formally offer documentary evidence essentially deprives one
party of due process. Weighing the amount of time spent in litigating the case against the number of delays petitioner incurred in
submitting its Formal Offer of Evidence and the state’s policy on recovering ill-gotten wealth, this court is of the belief that it is but
only just that the Rules be relaxed and petitioner be allowed to submit its written Formal Offer of Evidence. The Sandiganbayan’s
Resolutions should be reversed.

Likewise, the court laid down the guidelines in resolving a demurrer to evidence:

A demurrer to evidence may be issued when, upon the facts and the law, the plaintiff has shown no right to
relief. Where the plaintiff’s evidence together with such inferences and conclusions as may reasonably be drawn
therefrom does not warrant recovery against the defendant, a demurrer to evidence should be sustained. A demurrer
to evidence is likewise sustainable when, admitting every proven fact favorable to the plaintiff and indulging in his
favor all conclusions fairly and reasonably inferable therefrom, the plaintiff has failed to make out one or more of
the material elements of his case, or when there is no evidence to support an allegation necessary to his claim. It
should be sustained where the plaintiff’s evidence is prima facie insufficient for a recovery.

Furthermore, this court already clarified what the trial court determines when acting on a motion to dismiss based on demurrer to
evidence:

“What should be resolved in a motion to dismiss based on a demurrer to evidence is whether the plaintiff is
entitled to the relief based on the facts and the law. The evidence contemplated by the rule on demurrer is that which
pertains to the merits of the case, excluding technical aspects such as capacity to sue. . . .”

What should be determined now by the Sandiganbayan is whether petitioner’s evidence is sufficient to entitle it to the relief it seeks
after the Sandiganbayan rested its case. The Court ruled that the evidences presented by the petitioner is sufficient to establish a
presumption that respondents acquired ill-gotten wealth during respondent Fe Roa Gimenez’s incumbency as public officer and which
Remedial Law I| Page 147

total amount or value was manifestly out of proportion to her and her husband’s salaries and to their other lawful income or properties
and that the Sandiganbayan brushed off the totality of evidence on which petitioner built its case.

Even assuming that no documentary evidence was properly offered, this court finds it clear from the second assailed Resolution that
the Sandiganbayan did not even consider other evidence presented by petitioner during the 19 years of trial. The Sandiganbayan erred
in ignoring petitioner’s testimonial evidence without any basis or justification. Numerous exhibits were offered as part of the
testimonies of petitioner’s witnesses.

A liberal application of the Rules is in line with the state’s policy to recover ill-gotten wealth. In case of doubt, courts should proceed
with caution in granting a motion to dismiss based on demurrer to evidence. An order granting demurrer to evidence is a judgment on
the merits. This is because while a demurrer "is an aid or instrument for the expeditious termination of an action," it specifically
"pertains to the merits of the case." To reiterate, "[d]emurrer to evidence authorizes a judgment on the merits of the case without the
defendant having to submit evidence on his [or her] part, as he [or she] would ordinarily have to do, if plaintiff’s evidence shows that
he [or she] is not entitled to the relief sought." The order of dismissal must be clearly supported by facts and law since an order
granting demurrer is a judgment on the merits.
Remedial Law I| Page 148

Doctrine: There is no oral motion for leave to file a demurrer. This doctrine may be deemed superseded by the New Rules on
Continuous Trial Act effective September 2017.

Bernardo vs. Court of Appeals


G.R. No. 119010 September 5, 1997

Facts:

Paz Bernardo was originally charged with four counts of violation of BP 22 before the RTC of Quezon City. Subsequently,
two of the cases were dismissed due to the Affidavit of Desistance executed by private respondent Florlita, thus the remaining two
cases were left to be disposed of by the trial court.

After presenting its last witness, the prosecution rested its case and formally offered its exhibits. After the prosecution had formally
offered its evidence, in open court, the counsel of the defense respectfully ask for leave of court to file demurrer to evidence on the
ground that the prosecutor failed to elicit facts where the checks were issued and where they were actually dishonored. However, the
prosecution objected and showed the properly marked exhibits and testimonial evidences for that matter. In view of such objections,
the court denied the grounds of the defense for demurrer. The defense, however, reiterated that they will file their demurrer to
evidence without leave of court and the court considered the motion dilatory. Further, the court considered that the defense waived
their right to present evidence and that the case is deemed submitted for decision.

Bernardo assailed the order of the respondent judge before the Court of Appeals by way of certiorari, prohibition and mandamus. She
argued that the trial court committed grave abuse of discretion in considering her to have waived her right to present evidence after the
denial of her motion for leave to file demurrer to evidence. The CA rendered a decision modifying in effect the order of the lower
court by directing the trial court to set the case for trial for reception of evidence for the petitioner. Petitioner moved for partial
reconsideration but was denied. Hence, this petition. The petitioner submits, among others, that when the trial court denied her motion,
it was in effect a denial only of the motion for leave to file demurrer to evidence and not the demurrer to evidence itself and, therefore,
the order of the respondent appellate court allowing petitioner to present evidence was premature.

Issue:

Whether or not the petitioner should be allowed to present evidence after the denial of her demurrer to evidence?

Held:

No. Under the new rule on demurrer to evidence, the accused has the right to file a demurrer to evidence after the prosecution has
rested its case. If the accused obtained prior leave of court before filing his demurrer, he can still present evidence if his demurrer is
denied. However, if he demurs without prior leave of court, or after his motion for leave is denied, he waives his right to present
evidence and submits the case for decision on the basis of the evidence for the prosecution. This power to grant leave to the accused to
file a demurrer is addressed to the sound discretion of the trial court. The purpose is to determine whether the accused in filing his
demurrer is merely stalling the proceedings.

In this case, petitioner admits that in the hearing, the trial court denied her motion for leave to file a demurrer to evidence. In such
case, the only right petitioner has under Section 15, Rule 119 of the Rules of Court after having been denied leave to submit a
demurrer is to adduce evidence in her defense. However, even without express leave of trial court, nay, after her motion for leave was
denied, petitioner insisted on a filing a demurrer instead of presenting evidence in her defense.

Once prior leave is denied and the accused still files his demurrer to evidence or motion to dismiss, the court no longer has discretion
to allow the accused to present evidence. The only recourse left for the court is to decide the case on the basis of the evidence
presented by the prosecution. And, unless there is grave abuse thereof amounting to lack or excess of jurisdiction, which is not present
in the instant case, the trial courts denial of prior leave to file demurrer to evidence or motion to dismiss may not be disturbed.
Remedial Law I| Page 149

Radiowealth Finance Co. vs. Del Rosario


G.R. No. 138739 July 6, 2000

Facts:

Spouses Del Rosario jointly and severally executed, signed and delivered in favor of Radiowealth Finance Company a Promissory
Note for P138,948. The spouses defaulted on the monthly installments and despite repeated demands, they failed to pay their
obligations. Radiowealth then filed a Complaint for the collection of a sum of money. During the trial, the credit and collection officer
of Radiowealth presented in evidence the check payments, the demand letters, the customer’s ledger card for the spouses and dishonor
slips. The officer admitted that he did not have personal knowledge of the transaction or the execution of any of these pieces of
documentary evidence, which had merely been endorsed to him. The trial court issued an Order terminating the presentation of
evidence for Radiowealth. Thus, Radiowealth formally offered its evidence and exhibits and rested its case. The spouses filed a
Demurrer to Evidence for alleged lack of cause of action. The RTC dismissed the complaint for failure of Radiowealth to substantiate
its claims, the evidence it had presented being merely hearsay. However, the CA reversed the trial court and remanded the case for
further proceedings.

Issue:
Whether or not remand of the case to the trial court was necessary when the CA reversed the RTC’s decision granting the demurrer to
evidence?
Held:
No. If the trial court denies the dismissal motion, i.e., finds that plaintiffs evidence is sufficient for an award of judgment in the
absence of contrary evidence, the case still remains before the trial court which should then proceed to hear and receive the
defendant’s evidence so that all the facts and evidence of the contending parties may be properly placed before it for adjudication as
well as before the appellate courts, in case of appeal. Nothing is lost. The doctrine is but in line with the established procedural
precepts in the conduct of trials that the trial court liberally receive all proffered evidence at the trial to enable it to render its decision
with all possibly relevant proofs in the record, thus assuring that the appellate courts upon appeal have all the material before them
necessary to make a correct judgment, and avoiding the need of remanding the case for retrial or reception of improperly excluded
evidence, with the possibility thereafter of still another appeal, with all the concomitant delays.  The rule, however, imposes the
condition by the same token that if his demurrer is  granted  by the trial court, and the order of dismissal is  reversed on appeal, the
movant loses his right to present evidence in his behalf and he shall have been deemed to have elected to stand on the insufficiency of
plaintiff’s case and evidence.  In such event, the appellate court which reverses the order of dismissal shall proceed to render judgment
on the merits on the basis of plaintiff’s evidence.
In other words, defendants who present a demurrer to the plaintiff’s evidence retain the right to present their own evidence,  if the trial
court disagrees with them; if the trial court agrees with them, but on appeal, the appellate court disagrees with both of them and
reverses the dismissal order, the defendants lose the right to present their own evidence. The appellate court shall, in addition, resolve
the case and render judgment on the merits, inasmuch as a demurrer aims to discourage prolonged litigations.
In the case at bar, the trial court, acting on the spouses’ demurrer to evidence, dismissed the Complaint on the ground that
Radiowealth had adduced mere hearsay evidence. However, on appeal, the appellate court reversed the trial court because the
genuineness and the due execution of the disputed pieces of evidence had in fact been admitted by the spouses. Applying Rule 33,
Section 1 of the 1997 Rules of Court, the CA should have rendered judgment on the basis of the evidence submitted by the
petitioner. In fact, the CA had sufficient evidence on record to decide the collection suit. A remand is not only frowned upon by the
Rules, it is also logically unnecessary on the basis of the facts on record.
Remedial Law I| Page 150

Cabador vs. People


G.R. No. 186001, October 2, 2009
Facts:
Antonio Cabador was charged before the RTC of Quezon City of murdering, in conspiracy with others, Atty. Valerio. After presenting
only five witnesses over five years of intermittent trial, the RTC declared at an end the prosecution’s presentation of evidence and
required the prosecution to make a written or formal offer of its documentary evidence within 15 days from notice. The public
prosecutor, however, asked for three extensions of time, the last of which was to end on July 28, 2006. Still, the prosecution did not
make the required written offer. Cabador then filed a motion to dismiss the case, complaining of a turtle-paced proceeding since his
arrest and detention in 2001 and invoking his right to a speedy trial. Unknown to Cabador, four days earlier or on July 28, 2006, the
prosecution asked the RTC for another extension of the period for its formal offer, which offer it eventually made on August 1, 2006,
the day Cabador filed his motion to dismiss. The RTC issued an Order treating Cabador’s motion to dismiss as a demurrer to
evidence. And, since he filed his motion without leave of court, the RTC declared him to have waived his right to present evidence in
his defense. The trial court deemed the case submitted for decision. Cabador filed a motion for reconsideration but it was denied. The
CA denied Cabador’s petition and affirmed the lower court’s actions. Hence, the present petition for review on certiorari.
Issue:
Whether or not Cabador’s motion to dismiss before the trial court was in fact a demurrer to evidence filed without leave of court?
Held:
No. In Enojas, Jr. v. Commission on Elections, the Court held that, to determine whether the pleading filed is a demurer to evidence or
a motion to dismiss, the Court must consider (1) the allegations in it made in good faith; (2) the stage of the proceeding at which it is
filed; and (3) the primary objective of the party filing it.
Here, the pertinent portions of Cabador’s motion to dismiss shows that he took pains to point how trial in the case had painfully
dragged on for years. The gaps between proceedings were long, with hearings often postponed because of the prosecutor’s
absence. This was further compounded, Cabador said, by the prosecutions repeated motions for extension of time to file its formal
offer and its failure to file it within such time.
In criminal cases, a motion to dismiss may be filed on the ground of denial of the accused’s right to speedy trial. This denial is
characterized by unreasonable, vexatious, and oppressive delays without fault of the accused, or by unjustified postponements that
unreasonably prolonged the trial. The fact is that Cabador did not even bother to do what is so fundamental in any demurrer. He did
not state what evidence the prosecution had presented against him to show in what respects such evidence failed to meet the elements
of the crime charged. His so-called demurrer did not touch on any particular testimony of even one witness. He cited no documentary
exhibit. Indeed, he could not because, he did not know that the prosecution finally made its formal offer of exhibits on the same date
he filed his motion to dismiss.
Besides, a demurrer to evidence assumes that the prosecution has already rested its case. Here, after the prosecution filed its formal
offer of exhibits, the same day Cabador filed his motion to dismiss, the trial court still needed to give him an opportunity to object to
the admission of those exhibits. It also needed to rule on the formal offer. And only after such a ruling could the prosecution be
deemed to have rested its case. Since Cabador filed his motion to dismiss before he could object to the prosecution’s formal offer,
before the trial court could act on the offer, and before the prosecution could rest its case, it could not be said that he had intended his
motion to dismiss to serve as a demurrer to evidence.
Remedial Law I| Page 151

Claudio vs. Saraza


G.R. No. 213286, August 26, 2015

Facts:

Mamerta Lopez et. al., filed an action for annulment of sale, power of attorney and mortgage with prayer for damages before the RTC
against Florentino Claudio and Spouses. Saraza. They allege that Florentino made it appear that his parents sold to him the lot subject
of the action through an Absolute Deed of Sale. Furthermore, they allege that the signatures of the vendors were forged. Later on,
Florentino executed a deed of real estate mortgage in favor of Spouses Saraza to secure his loan from them. That Spouses Saraza were
mortgagees in bad faith because they knew fully well that Florentino could not have acquired the subject property from his parents
because Porfirio had long been deceased while Mamerta was in the United States of America at the time of the alleged sale and lastly,
that it was only a few days after the execution of the mortgage that the TCT under Florentino’s name was issued.

Spouses Saraza filed a Demurrer to Evidence after the presentation of the evidence in chief of the prosecution which was granted. On
appeal, the Court of Appeals affirmed the RTC’s grant of the demurrer. However, the Supreme Court reversed both the RTC and the
CA.

Issue:

Whether or not remand of the proceedings to the RTC is proper when the Order grating the demurrer to evidence was subsequently
reversed by the Supreme Court?

Held:

Yes. Petitioners' evidence, standing alone and in the absence of controverting evidence, would afford sufficient basis for a judgment in
their favor and against Spouses Saraza. Despite the fact that Spouses Saraza are deemed to have waived their right to present evidence
before the RTC pursuant to Section 1, Rule 33 of the Rules of Court, still this disposition is without prejudice to the judgment on the
merits to be handed down by the RTC.
Remedial Law I| Page 152

Felipe vs. MGM Motor Trading Corporation


GR No. 191849, September 23, 2015

Facts:

Frederick Felipe filed a Complaint for Specific Performance and Damages against MGM Motors, and Ayala General Insurance
Corporation. He claims that he purchased on installment basis a Nissan Terrano Wagon through MGM Motors' authorized
representative Jane Sarmiento. He allegedly paid 200,000 pesos as downpayment and P5,000.00 reservation fee to Sarmiento. He
further issued seven (7) Allied Bank checks, each bearing the amount of P24,165.00 payable to MGM Motors. After it was delivered
to him, he insured it with Ayala Insurance. However, the vehicle was reportedly stolen while parked in Adriatico Street. As a result, he
tried to claim from Ayala Insurance but it was refused. On the other hand, despite his repeated demands, MGM Motors refused to
issue him the document of sale on installment.

In its Answer, MGM stated that it offered Felipe a discount of 200k if he would pay in cash but since the latter failed to pay in cash,
MGM did not give him the registration papers. Also, MGM Motors stated that petitioner was able to fraudulently register the vehicle
with the Land Transportation Office in his name and insure the same with Ayala Insurance. During a negotiation, MGM and Felipe
agreed to the amount of 1,020,000 as full payment and as a consequence, Felipe’s mother issued a post dated check in such amount
but it bounced. A case for BP 22 was filed against Felipe’s mother.

During the trial of this case, Only two pieces of evidence were admitted by the trial court: (1) the Official Receipt dated 7 May 1998
issued by MGM Motors wherein it acknowledged receipt of P200,000.00 from petitioner; and (2) the testimony of his father Alberto
that he was present when petitioner paid P200,000.00 to MGM Motors.

MGM Motors and Ayala Insurance filed their respective Motions to Dismiss on demurrer to evidence. The RTC granted it and
dismissed the case. This was likewise affirmed by the CA. Hence, this petition.

Issue:

Whether or not the demurrer to evidence was correctly granted?

Held:

Yes. The essential question to be resolved in a demurrer to evidence is whether the plaintiff has been able to show that he is entitled to
his claim, and it is incumbent upon the trial court judge to make such a determination.

Well-established is the rule that the burden of proof lies on the party who makes the allegations. There is no dispute that the only
pieces of evidence admitted in court are the testimony of Alberto and the receipt showing MGM Motors receiving P200,000.00 from
petitioner as partial payment of the subject car. The allegation that the purchase of the vehicle was on an installment basis was not
supported by any evidence. The receipt of a partial payment does not suffice to prove that the purchase was made on an installment
basis. Petitioner did not present any document to prove said allegation while MGM Motors produced a sales invoice wherein it was
stated that the mode of payment is "COD" or cash on delivery.

In the same vein, petitioner failed to substantiate his allegation against Ayala Insurance. Petitioner has the burden of proof to show
that a loss occurred and said loss was covered by his insurance policy. Considering that the trial court only admitted two pieces of
evidence in petitioner's favor and none of those tend to prove loss of the subject car and coverage thereof under the insurance policy,
petitioner is not entitled to the reliefs he had prayed for.
Remedial Law I| Page 153

Rule 34-35: Judgment on the Pleadings/Summary Judgment

Republic vs. Pilipinas Shell Petroleum Corp.


Facts:

Filipino Way Industries assigned Tax Credit Certificates (TCCs) to Pilipinas Shell Petroleum Corp. (PSPC). Believing that the TCCs
were actually good and valid, the Bureau of Customs (BOC) accepted and allowed PSPC to use the above TCCs to pay the customs
duties and taxes due on its oil importations. However, the TCCs were later on invalidated for having been fraudulently issued and
transferred. The Republic of the Philippines represented by the BOC then filed a collection suit in the RTC against PSPC. On the
other hand, PSPC filed with the Court of Tax Appeals a petition for review questioning the factual and legal bases of BOC's
collection efforts. However, upon reaching the SC, that case was eventually dismissed.

With the resumption of proceedings in the RTC, PSPC filed a motion for summary judgment arguing that there is no basis for the
Republic's claims considering that the subject TCCs were already fully utilized for the payment of PSPC's customs duties and taxes,
and that EXCOM Resolution No. 03-05-99, the basis of the cancellation of the TCCs, was declared void and invalid in Pilipinas Shell
Petroleum Corporation v. CIR, where the Supreme Court likewise ruled that the subject TCCs cannot be cancelled on the basis of
post-audit since a post-audit is not allowed and not a suspensive condition.

In its Comment/Opposition, BOC argued that rendition of summary judgment is inappropriate in this case in view of disputed facts
that necessitate a full-blown trial where both parties can present evidence on their respective claims.

RTC denied the motion for summary judgment. However, on MR by PSPC, the RTC reversed its earlier ruling and granted the motion
for summary judgment. With the denial of its motion for reconsideration, BOC appealed to the CA. CA denied the appeal and
affirmed the questioned orders of the RTC. BOC's motion for reconsideration was likewise denied by the CA. Hence, this petition for
review under Rules 45.

Issues:

(1) Whether or not summary judgment is proper?

(2) What is the proper remedy against a summary judgment?

Held:

(1) No. Under Rule 35 of the 1997 Rules of Civil Procedure, as amended, except as to the amount of damages, when there is no
genuine issue as to any material fact and the moving party is entitled to a judgment as a matter of law, summary judgment may be
allowed. Even if on their face the pleadings appear to raise issues, when the affidavits, depositions and admissions show that such
issues are not genuine, then summary judgment as prescribed by the Rules must ensue as a matter of law. The determinative factor,
therefore, in a motion for summary judgment, is the presence or absence of a genuine issue as to any material fact.

For a full-blown trial to be dispensed with, the party who moves for summary judgment has the burden of demonstrating clearly the
absence of genuine issues of fact, or that the issue posed is patently insubstantial as to constitute a genuine issue. Genuine issue means
an issue of fact which calls for the presentation of evidence as distinguished from an issue which is fictitious or contrived. Trial courts
have limited authority to render summary judgments and may do so only when there is clearly no genuine issue as to any material fact.
When the facts as pleaded by the parties are disputed or contested, proceedings for summary judgment cannot take the place of trial.
As certain facts pleaded are contested by the parties in this case, rendition of summary judgment is not proper.

(2) Petition for Review Under Rule 45. Any review by the appellate court of the propriety of the summary judgment rendered by the
trial court based on these pleadings would not involve an evaluation of the probative value of any evidence, but would only limit itself
to the inquiry of whether the law was properly applied given the facts and these supporting documents. Therefore, what would
inevitably arise from such a review are pure questions of law, and not questions of fact, which are not proper in an ordinary appeal
under Rule 41, but should be raised by way of a petition for review on certiorari under Rule 45.

Petitioner raised as sole issue in its brief filed with the CA the RTC's erroneous grant of summary judgment in favor of PSPC based on
its finding that there exists no genuine factual issue. Obviously, it availed of the wrong mode of appeal when it filed a notice of appeal
in the RTC under Section 2(a), Rule 41, instead of a petition for review on certiorari in this Court under Rule 45. Notwithstanding
petitioner's wrong mode of appeal, the CA should not have so easily dismissed the petition, considering that the parties involved are
local government units and that what is involved is the determination of their respective territorial jurisdictions.
Remedial Law I| Page 154

Comglas Corp. vs. Santos Car Check Center


G.R. No. 202989. March 25, 2015.

Facts:

Santos Car Check Center leased a space to Comglasco for a period of five years. However, the latter advised Santos through a letter
that it was pre-terminating their lease contract. Santos refused to accede to the pre-termination, reminding Comglasco that their
contract was for five years. Thereafter, Comglasco vacated the leased premises and stopped paying any further rentals. Santos sent
several demand letters, which Comglasco completely ignored. Eventually, Santos sent its final demand letter, which Comglasco again
ignored which led Santos to file a suit for breach of contract. Comglasco filed its Answer. Santos then moved for a judgment on the
pleadings, which the RTC granted. Judgment was rendered in favor of Santos and against Comglasco. The CA affirmed the RTC’s
Order. Hence, this petition. Comglasco maintains that the RTC was wrong to rule that its answer to Santos’ complaint tendered no
issue, or admitted the material allegations therein; that the court should have heard it out on the reason it invoked to justify its action to
pre-terminate the parties’ lease; that therefore a summary judgment would have been the proper recourse, after a hearing.

Issue:

Whether or not judgment on the pleadings is proper in this case?

Held:

Yes. Relying on Article 1267 of the Civil Code to justify its decision to pre-terminate its lease with Santos, Comglasco invokes the
1997 Asian currency crisis as causing it much difficulty in meeting its obligations. Article 1267 speaks of a prestation involving
service which has been rendered so difficult by unforeseen subsequent events as to be manifestly beyond the contemplation of the
parties. To be sure, the Asian currency crisis befell the region from July 1997 and for sometime thereafter, but Comglasco cannot be
permitted to blame its difficulties on the said regional economic phenomenon because it entered into the subject lease only on August
16, 2000, more than three years after it began, and by then Comglasco had known what business risks it assumed when it opened a
new shop in Iloilo City.

As found by the CA, Comglasco’s Answer admitted the material allegations in the complaint, to wit: a) that Santos holds absolute title
to a showroom space; b) that Comglasco leased the said showroom from Santos; c) that after a little over a year, Comglasco pre-
terminated the lease; d) that, disregarding Santos’ rejection of the pre-termination of their lease, Comglasco vacated the leased
premises on January 15, 2002; e) that Comglasco never denied the existence and validity of the parties’ lease contract. Specifically,
the CA noted that Paragraph 2 of the Answer admitted the allegations in Paragraphs 2, 3 and 4 of the complaint that the lease was for
five years, starting on August 16, 2000 and to expire on August 15, 2005, at a monthly rental of 60,000.00 on the first year, 66,000.00
on the second year, and 72,600.00 on the third up to the fifth year.

The RTC acted correctly in resorting to Section 1 of Rule 34, on Judgment on the Pleadings, to cut short a needless trial. Comglasco
cannot cite Article 1267 of the Civil Code, and that it must be deemed to have admitted the material allegations in the complaint.
Section 1, Rule 34 reads:

A judgment on the pleadings is a judgment on the facts as pleaded, and is based exclusively upon the allegations appearing in
the pleadings of the parties and the accompanying annexes. It is settled that the trial court has the discretion to grant a motion
for judgment on the pleadings filed by a party if there is no controverted matter in the case after the answer is filed. A genuine
issue of fact is that which requires the presentation of evidence, as distinguished from a sham, fictitious, contrived or false
issue.

Under Rule 35, on Summary Judgments, Comglasco had recourse to move for summary judgment, wherein it could have adduced
supporting evidence to justify its action on the parties' lease, but it did not do so. Section 2 of Rule 35 provides:

Sec. 2. Summary judgment for defending party. - A party against whom a claim, counterclaim, or cross-claim is asserted or a
declaratory relief is sought may, at any time, move with supporting affidavits, depositions or admissions for a summary
judgment in his favor as to all or any part thereof.
Remedial Law I| Page 155

Philippine Bank of Communications vs. Go


G.R. No. 175514. February 14, 2011.
Facts:
Jose Go obtained two loans from PBCom, evidenced by two promissory notes, embodying his commitment to pay P17,982,222.22 for
the first loan, and P80 million for the second loan, within a ten-year period from September 30, 1999 to September 30, 2009. To
secure the loans, he executed pledge agreements covering the shares of stock of Ever Gotesco Resources and Holdings, Inc. Two years
later, however, the market value of the said shares of stock plunged to less than P0.04 per share. Thus, PBCom, as pledgee, notified
Go in writing that it was renouncing the pledge agreements. Later, PBCom filed before the RTC a complaint for some of money
against the Spouses Go. PBCom alleged that Spouses Go defaulted on the two promissory notes, having paid only three installments
on interest payments—covering the months of September, November and December 1999. Consequently, the entire balance of the
obligations of Go became immediately due and demandable. Spouses Go filed their Answer with Counterclaim denying the material
allegations in the complaint and stating (1) the loan obligation is payable within the period of 10 years so it is not yet due and
demandable; (2) they made substantial payments so there is a need for accounting; and (3) there was no prior demand.
PBCom filed a Motion for Summary Judgment alleging that the Answer interposed no specific denials on the material averments in
paragraphs 8 to 11 of the complaint such as the fact of default, the entire amount being already due and demandable by reason of
default, and the fact that the bank had made repeated demands for the payment of the obligations. The RTC granted the Motion and
rendered its judgment ordering the Spouses Go to pay the obligation.
The Court of Appeals reversed the RTC. It found the supposed admission (as found by the RTC) to be insufficient to justify a
rendition of summary judgment in the case for sum of money, since there were other allegations and defenses put up by Spouses Go in
their Answer which raised genuine issues on the material facts in the action.
Hence, this petition.
Issue:
Whether or not a Summary Judgment was proper?
Held:
No. The CA correctly ruled that there exist genuine issues as to three material facts, which have to be addressed during trial: first, the
fact of default; second, the amount of the outstanding obligation, and third, the existence of prior demand.
Under Rule 35 of the 1997 Rules of Procedure, except as to the amount of damages, when there is no genuine issue as to any material
fact and the moving party is entitled to a judgment as a matter of law, summary judgment may be allowed. Summary or accelerated
judgment is a procedural technique aimed at weeding out sham claims or defenses at an early stage of litigation thereby avoiding the
expense and loss of time involved in a trial.
Under the Rules, summary judgment is appropriate when there are no genuine issues of fact which call for the presentation of
evidence in a full- blown trial. Even if on their face the pleadings appear to raise issues, when the affidavits, depositions and
admissions show that such issues are not genuine, then summary judgment as prescribed by the Rules must ensue as a matter of law.
The determinative factor, therefore, in a motion for summary judgment, is the presence or absence of a genuine issue as to any
material fact.
A “genuine issue” is an issue of fact which requires the presentation of evidence as distinguished from a sham, fictitious, contrived or
false claim. When the facts as pleaded appear uncontested or undisputed, then there is no real or genuine issue or question as to he
facts, and summary judgment is called for. The party who moves for summary judgment has the burden of demonstrating clearly the
absence of any genuine issue of fact, or that the issue posed in the complaint is patently unsubstantial so as not to constitute a genuine
issue for trial. Trial courts have limited authority to render summary judgments and may do so only when there is clearly no genuine
issue as to any material fact. When the facts as pleaded by the parties are disputed or contested, proceedings for summary judgment
cannot take the place of trial.
Juxtaposing the Complaint and the Answer discloses that the material facts here are not undisputed so as to call for the rendition of a
summary judgment. While the denials of Spouses Go could have been phrased more strongly or more emphatically, and the Answer
more coherently and logically structured in order to overthrow any shadow of doubt that such denials were indeed made, the pleadings
show that they did in fact raise material issues that have to be addressed and threshed out in a full-blown trial. The matters in
contention are, as the CA stated, whether or not respondents were in default, whether there was prior demand, and the amount of the
outstanding loan. These are the matters that the parties disagree on and by which reason they set forth vastly different allegations in
their pleadings which each will have to prove by presenting relevant and admissible evidence during trial.
Remedial Law I| Page 156

Adolfo vs. Adolfo


G.R. No. 201427. March 18, 2015.
Facts:
[MAIN CASE] Civil Case No. MAN-4821 – In 2004, Teofilo Adolfo filed with the RTC Mandaue a Petition for judicial separation of
property against his estranged wife, respondent Fe Adolfo. He alleged that the two are married and that during their marriage, they
acquired through conjugal funds Lot 1087-A-2-E, a 3,652-square-meter property (the subject property). However, due to
irreconcilable differences and since reunion is no longer possible, he prays for a decree of separation of conjugal property. In her
Answer, respondent contended that while she remained married to petitioner, she is the sole owner of the subject property, the same
being her paraphernal property which she inherited from her mother. She thus prayed for the petition to be dismissed.
[OTHER CASE] Civil Case No. MAN-2683 – In 1996, respondent’s sister, Florencia Tudtud and her husband (Spouses Gingoyons),
filed a case for partition with damages against respondent. The Complaint alleged that the respondent executed a deed of sale in favor
of Spouses Gingoyons over a 300-square-mater portion of the subject property but she refused to partition/subdivide the same despite
payment. For her defense, respondent claimed in her Answer that when the sale was made, the subject property constituted conjugal
property of her marriage with petitioner; since petitioner did not sign the deed of sale as he knew nothing about the sale, the sale was
null and void. The trial court ruled that the subject property constituted conjugal property of the marriage. Thus, it nullified the deed of
sale. This case was the subject of an appeal with the CA, which was docketed as C.A.-G.R. CV No. 78971.
[MAIN CASE] During the pretrial conference, petitioner submitted as evidence the copies of the Complaint, the respondent’s answer
and the Decision in Civil Case No. Man-2683. Petitioner then filed a Request for Admission of the genuineness and due execution of
the copies of the Complaint, Answer and Decision. Respondent failed to file her response to the request for admission. Consequently,
petitioner filed a Motion for Judgment Based on the Pleadings stating that since respondent failed to answer his request for admission,
the matters contained in the request are deemed admitted; that as a consequence of the application of the rule, respondent is in effect
considered to have admitted that the subject property is a conjugal asset of their subsisting marriage which may thus be the subject of
his petition for judicial separation of property; and that on account of said admission, a hearing on the merits becomes unnecessary.
The trial court then directed the transfer of the case to Branch 55 (the court which decided Civil Case No. MAN-2683). Branch 55
then granted petitioner’s Motion; it treated the Motion for Judgment on the Pleadings as one for Summary Judgment. It held that the
failure of respondent to respond to the Request for Admission was an admission that Lot 1087-A-2-E is a conjugal property, thus,
petitioner is entitled for partition. Respondent appealed the case to the CA docketed as C.A.-G.R. CV No. 01783.
[OTHER CASE] C.A.-G.R. CV No. 78971: The CA reversed the RTC and declared that the subject property was respondent’s
paraphernal property. It ordered the partition of the property. This decision became final and executory.
[MAIN CASE] C.A.-G.R. CV No. 01783: The CA held that the trial court cannot treat petitioner’s motion for judgment on the
pleadings as one for summary judgment. It then remanded the case back to the RTC. The petitioner then filed a petition for review
before the SC.
Issue:
Whether or not the trial court erred in rendering a summary judgment on Civil Case No. MAN-4821 despite the pendency of the
appeal in Civil Case No. MAN-2683?
Held:
Yes. Judgment on the pleadings is proper “where an answer fails to tender an issue, or otherwise admits the material allegations of the
adverse party’s pleading.” Summary judgment, on the other hand, will be granted “if the pleadings, supporting affidavits, depositions,
and admissions on file, show that, except as to the amount of damages, there is no genuine issue as to any material fact and that the
moving party is entitled to a judgment as a matter of law.
The existence or appearance of ostensible issues in the pleadings, on the one hand, and their sham or fictitious character, on the other,
are what distinguish a proper case for summary judgment from one for a judgment on the pleadings. In a proper case for judgment on
the pleadings, there is no ostensible issue at all because of the failure of the defending party’s answer to raise an issue. On the other
hand, in the case of a summary judgment, issues apparently exist — i.e., facts are asserted in the complaint regarding which there is as
yet no admission, disavowal or qualification; or specific denials or affirmative defenses are in truth set out in the answer — but the
issues thus arising from the pleadings are sham, fictitious or not genuine, as shown by affidavits, depositions, or admissions.
In rendering summary judgment, the trial court relied on respondent’s failure to reply to petitioner’s request for admission, her
admission in Civil Case No. MAN-2683, as well as its May 15, 2002 Decision declaring that the subject property is a conjugal asset. It
took judicial notice of the proceedings in said case. While there is nothing irregular with this — as courts may “take judicial notice of
a decision or the facts prevailing in another case sitting in the same court, the trial court however disregarded the fact that its decision
was then the subject of a pending appeal in C.A.- G.R. CV No. 78971. It should have known that until the appeal is resolved by the
appellate court, it would be premature to render judgment on petitioner’s motion for judgment on the pleadings; that it would be
Remedial Law I| Page 157

presumptuous to assume that its own decision would be affirmed on appeal. One of the issues raised in the appeal is precisely whether
the subject property is conjugal, or a paraphernal asset of the respondent. Thus, instead of resolving petitioner’s motion for judgment
on the pleadings, the trial court should have denied it or held it in abeyance.
On the part of petitioner, it must be said that he could not have validly resorted to a motion for judgment on the pleadings or summary
judgment. While it may appear that under Rules 34 and 35 of the 1997 Rules, he may file a motion for judgment on the pleadings or
summary judgment as a result of the consequent admission by respondent that the subject property is conjugal, this is not actually the
case. Quite the contrary, by invoking the proceedings and decision in Civil Case No. MAN-2683, petitioner is precluded from
obtaining judgment while the appeal in said case is pending, because the result thereof determines whether the subject property is
indeed conjugal or paraphernal. He may not preempt the appeal in C.A.-G.R. CV No. 78971.
Finally, the Court notes that the appellate court overlooked the May 30, 2007 Decision in C.A.-G.R. CV No. 78971, which became
final and executory on June 23, 2007. Because of this, petitioner’s case is left with no leg to stand on. There being no conjugal
property to be divided between the parties, Civil Case No. MAN-4821 must be dismissed.
Remedial Law I| Page 158

Rule 37: Motion for Reconsideration/New Trial

Mendezona vs. Ozamis


G.R. No. 143370. February 6, 2002

Facts:

The petitioners (Spouses Mario Mendezona and Teresita Mendezona, et al.) alleged that they own a parcel of land
in Lahug, Cebu City and that they traced their titles of ownership over their respective properties from a notarized Deed of
Absolute Sale executed in their favor by Carmen Ozamiz. They initiated a suit for quieting of title to remove a cloud on their
respective titles caused by the inscription thereon of a notice of lis pendens, which came about as a result of an incident in
special proceeding for guardianship over the person and properties of Carmen Ozamiz initiated by the respondents (Julio
Ozamis, et al.) in the RTC. Respondents instituted the petition for guardianship, alleging that Carmen Ozamiz, then 86 years old,
after an illness, had become disoriented and could no longer take care of herself nor manage her properties. Respondent Paz
O. Montalvan was designated as guardian over the person of Carmen Ozamiz while petitioner Mario J. Mendezona, respondents
Roberto J. Montalvan and Julio H. Ozamiz were designated as joint guardians over the properties of the said ward. As guardians,
respondents filed their inventories and accounts, listing therein Carmen Ozamiz’s properties, including the said Lahug property.
In their Answer, the respondents opposed the petitioners’ claim of ownership of the Lahug property and alleged that the titles
issued in the petitioners’ names are defective and illegal, and the ownership of the said property was acquired in bad faith and
without value. Respondents further alleged that at the time of the sale, Carmen Ozamiz was already ailing and not in full
possession of her mental faculties.

Trial on the merits ensued. The trial court rendered its decision in favor of the petitioners. On appeal to the CA, the appellate
court reversed the factual findings of the trial court. Petitioners filed a motion for reconsideration of the decision of the appellate
court. Subsequent thereto, the petitioners filed a motion for a new trial and/or for reception of evidence. They contended, among
other things, that the appellate court totally ignored the testimony of Judge Teodorico Durias regarding the mental condition of
Carmen Ozamiz a month before the execution of the Deed of Absolute Sale in question. The said testimony was taken in the
special proceeding. However, Judge Durias was not presented as a witness. Petitioners alleged that Judge Duriass testimony is a
newly-discovered evidence which could not have been discovered prior to the trial in the court below by the exercise of due
diligence. The appellate court denied both motions.

Issue:

Whether or not the testimony of Judge Durias in the Special Proceeding can be considered as newly discovered evidence?

Held:

No. A motion for new trial upon the ground of newly discovered evidence is properly granted only where there is concurrence of
the following requisites, namely: (a) the evidence had been discovered after trial; (b) the evidence could not have been
discovered and produced during trial even with the exercise of reasonable diligence; and (c) the evidence is material and not
merely corroborative, cumulative or impeaching and is of such weight that if admitted, would probably alter the result. All three
(3) requisites must characterize the evidence sought to be introduced at the new trial.

The requirement of reasonable diligence has not been met by the petitioners. As early as the pre-trial of the case, the name of
Judge Durias has already cropped up as a possible witness for the defendants, herein respondents. Judge Durias testimony in
another case cannot be considered as newly discovered evidence since the facts to be testified to by Judge  Durias which were
existing before and during the trial, could have been presented by the petitioners at the trial below. The testimony of
Judge Durias has been in existence waiting only to be elicited from him by questioning.

It has been held that a lack of diligence is exhibited where the newly discovered evidence was necessary or proper under the
pleadings, and its existence must have occurred to the party in the course of the preparation of the case, but no effort was made
to secure it; there is a failure to make inquiry of persons who were likely to know the facts in question, especially where
information was not sought from co-parties; there is a failure to seek evidence available through public records; there is a failure
to discover evidence that is within the control of the complaining party; there is a failure to follow leads contained in other
evidence; and, there is a failure to utilize available discovery procedures.  Thus, the testimony of Judge Durias cannot be
considered as newly discovered evidence to warrant a new trial.
Remedial Law I| Page 159

Chua vs. People


G.R. No. 196853. July 13, 2015

Facts:

Robert Chua issued several postdated PS Bank checks to Philip See pursuant to their rediscounting arrangement. However, See
claimed that when he deposited the checks, they were dishonored either due to insufficient funds or closed account. Despite
demands, Chua failed to make good the checks. See filed a complaint for violations of BP 22. He attached thereto a demand
letter. Chua was charged with 54 counts of violation of BP 22 before the MeTC. The prosecution formally offered as its
evidence the demand letter dated December 10, 1993. Chua, however, objected to its admissibility on the grounds that it is a
mere photocopy and that it does not bear any proof that he actually received it. Chua filed a Motion to Submit Demurrer to
Evidence. However, the MeTC failed to act on his motion since the judge of said court vacated his post.

Several years later, the prosecution filed a Motion to Re-Open Presentation of Prosecution's Evidence and Motion to Allow
Prosecution to Submit Additional Formal Offer of Evidence. It averred that while See was still trying to locate a demand letter
dated November 30, 1993 which it alleged to have been personally served upon Chua. However, See decided to have his house
rented out such that he emptied it with all his belongings and had it cleaned. It was during this time that he found the demand
letter dated November 30, 1993. The prosecution thus prayed that it be allowed to submit a supplemental offer of evidence to
include said demand letter dated November 30, 1993 as part of its evidence. Again, the records of the case bear no copy of an
MeTC Order or Resolution granting the aforesaid motion. Nevertheless, extant on records is a Formal Offer of Evidence  filed by
the private prosecutor submitting the demand letter dated November 30, 1993 as additional evidence. MeTC refused to take
cognizance of the supplemental formal offer on the ground that the same was filed by the private prosecutor without the
conformity of the public prosecutor. Be that as it may, the demand letter dated November 30, 1993 eventually found its way into
the records of this case. MeTC convicted Chua of 54 counts of violation of BP 22. The RTC affirmed the ruling of the MeTC.

Before the CA, Chua argued against the probative value of the demand letter dated November 30, 1993 by pointing out that: (1)
for more than 10 years from the time the case was filed, the prosecution never adverted to its existence. He thus surmised that
this was because the document was not really missing but in fact inexistent - a mere afterthought as to make it appear that the
second element of the offense is obtaining in the case; (2) the subject demand letter is not a newly discovered evidence as it
could have been discovered earlier through the exercise of due diligence; and, (3) his counsel's admission of the physical
existence of the subject demand letter and Chua's signature thereon does not carry with it the admission of its contents and his
receipt of the same. The CA likewise affirmed the decision of the lower court.

Issue:

Whether or not demand letter dated November 30, 1993 is a newly discovered evidence?

Held:

No. The question of whether evidence is newly discovered has two aspects: a temporal one, i.e., when was the evidence
discovered, and a predictive one, i.e., when should or could it have been discovered. It is to the latter that the requirement of due
diligence has relevance. In order that a particular piece of evidence may be properly regarded as newly discovered to justify new
trial, what is essential is not so much the time when the evidence offered first sprang into existence nor the time when it first
came to the knowledge of the party now submitting it; what is essential is that the offering party had exercised reasonable
diligence in seeking to locate such evidence before or during trial but had nonetheless failed to secure it.

The Rules do not give an exact definition of due diligence, and whether the movant has exercised due diligence depends upon
the particular circumstances of each case. Nonetheless, it has been observed that the phrase is often equated with "reasonable
promptness to avoid prejudice to the defendant." In other words, the concept of due diligence has both a time component and a
good faith component. The movant for a new trial must not only act in a timely fashion in gathering evidence in support of the
motion; he must act reasonably and in good faith as well. Due diligence contemplates that the defendant acts reasonably and in
good faith to obtain the evidence, in light of the totality of the circumstances and the facts known to him.

Under the Rules of Court, the requisites for newly discovered evidence are: (a) the evidence was discovered after trial; (b) such
evidence could not have been discovered and produced at the trial with reasonable diligence; and (c) it is material, not merely
cumulative, corroborative or impeaching, and is of such weight that, if admitted, will probably change the judgment."

In this case, the Court holds that the demand letter dated November 30, 1993 does not qualify as a newly discovered evidence within
the purview of the law. Per See's statements in his affidavit, the said evidence was already known to him at the time he filed his
Remedial Law I| Page 160

complaint against Chua. It was also apparently available considering that it was just kept in his house. Undeniably, had See exercised
reasonable diligence, he could have promptly located the said demand letter and presented it during trial.
Remedial Law I| Page 161

Padilla-Rumbaua vs. Rumbaua


G.R. No. 166738. August 14, 2009

Facts:

Petitioner (Rowena Padilla-Rumbaua) filed a petition for declaration of nullity of marriage against the respondent (Edward Rumbaua)
before the RTC on the ground of psychological incapacity. The Republic of the Philippines (Republic), through the office of the
Solicitor General (OSG), opposed the petition. The Regional Trial Court nullified the marriage of the petitioner but on appeal to the
CA by the OSG, the CA reversed the ruling of the RTC and denied the nullification of the parties’ marriage. The petitioner moved to
reconsider the decision, but the CA denied her motion. Hence, this petition. The petitioner prays, among others, that the RTC’s and the
CA’s decisions be reversed and set aside, and the case be remanded to the RTC for further proceedings; in the event the Court cannot
grant this prayer, that the CA’s decision be set aside and the RTC’s decision be reinstated. She argues that the inadequacy of her evidence
during the trial was the fault of her former counsel, and asserts that remanding the case to the RTC would allow her to cure the evidentiary insufficiencies.

Issue:

Whether or not the remand of the case for reception of additional evidence should be allowed?

Held:

No. A remand of the case to the RTC for further proceedings amounts to the grant of a new trial that is not procedurally proper at this stage. Section 1 of
Rule 37 provides that an aggrieved party may move the trial court to set aside a judgment or final order already rendered  and  to  grant a new
trial within the period for taking an appeal.  In addition, a motion for new trial may be filed only on the grounds of (1) fraud, accident,
mistake or excusable negligence that could not have been guarded against by ordinary prudence, and by reason of which the aggrieved
party’s rights have probably been impaired; or (2) newly discovered evidence that, with reasonable diligence, the aggrieved party
could not have discovered and produced at the trial, and that would probably alter the result if presented.

In the present case, the petitioner cites the inadequacy of the evidence presented by her former counsel as basis for a remand.  She did
not, however, specify the inadequacy.  That the RTC granted the petition for declaration of nullity prima facie shows that the
petitioner’s counsel had not been negligent in handling the case. Granting arguendo that the petitioner’s counsel had been negligent,
the negligence that would justify a new trial must be excusable, i.e. one that ordinary diligence and prudence could not have guarded
against.

As cited by the Court in Uy v. First Metro Integrated Steel Corporation, it explained:

“Blunders and mistakes in the conduct of the proceedings in the trial court as a result of the ignorance, inexperience or incompetence
of counsel do not qualify as a ground for new trial.  If such were to be admitted as valid reasons for re-opening cases, there would
never be an end to litigation so long as a new counsel could be employed to allege and show that the prior counsel had not been
sufficiently diligent, experienced or learned.”
Remedial Law I| Page 162

Senit vs. People


G.R. No. 192914. January 11, 2016

Facts:

Petitioner (Napoleon Senit) was charged with Reckless Imprudence Resulting to Multiple Serious Physical Injuries and Damage to
Property in an Amended Information which was filed with Branch 10 of the RTC in Malaybalay City. Such information stemmed
from a collision of the Super Five Nissan Bus driven by the petitioner and the Toyota pick-up vehicle owned and driven by private
respondent Toor, Sr., with his wife, son and housemaid riding with him. Petitioner pleaded not guilty upon being arraigned. Trial
thereafter ensued. However, after the initial presentation of evidence for the petitioner, he resigned from his employment and
transferred residence. His whereabouts allegedly became unknown so he was not presented as a witness by his new counsel. The RTC
rendered its Decision in absentia convicting the petitioner of the crime charged.

The RTC issued a Promulgation, which included an order for the arrest of the petitioner. The petitioner then filed a motion for new
trial via registered mail on the ground that errors of law or irregularities have been committed during trial that are allegedly prejudicial
to his substantial rights. He claimed that he was not able to present evidence during trial because he was not notified of the schedule.
Likewise, he mistakenly believed that the case against him has been dismissed as private complainant Toor, Sr. purportedly left the
country. The motion for new trial was denied by the lower court pronouncing that notices have been duly served the parties and that
the reason given by the petitioner was self-serving. On appeal to the CA, the CA affirmed the decision of the RTC and ruled, among
others, that the arguments of the petitioner to move for a new trial as baseless. The petitioner filed a motion for reconsideration but
was denied by the CA, hence this petition. The Petitioner theorizes that there was an error of law or irregularities committed when the
RTC promulgated a decision in absentia and deemed that he had waived his right to present evidence resulting to denial of due
process, a one-sided decision by the RTC, and a strict and rigid application of the Revised Rules of Criminal Procedure against him.

Issue:

Whether or not a motion for new trial or to re-open the case to allow the petitioner to present evidence should be allowed?

Held:

No. No errors of law or irregularities, prejudicial to the substantial rights of the petitioner, have been committed during trial. The
petitioner anchors his motion for new trial on Rule 121, Section 2(a) of the Revised Rules of Criminal Procedure.

First, it must be noted that the petitioner had already been arraigned and therefore, the court a quo had already acquired jurisdiction
over him. In fact, there was already an initial presentation of evidence for the defense when his whereabouts became unknown. Also,
the petitioner clearly had previous notice of the criminal case filed against him and was given the opportunity to present evidence in
his defense. The petitioner was not in any way deprived of his substantive and constitutional right to due process as he was duly
accorded all the opportunities to be heard and to present evidence to substantiate his defense, but he forfeited this right, through his
own negligence, by not appearing in court at the scheduled hearings. The negligence of the petitioner in believing that the case was
already terminated resulting to his failure to attend the hearings, is inexcusable. The Court finds that the negligence exhibited by the
petitioner, towards the criminal case against him in which his liberty is at risk, is not borne of ignorance of the law as claimed by his
counsel rather, lack of concern towards the incident, and the people who suffered from it. While there was no showing in the case at
bar that the counsel of the petitioner was grossly negligent in failing to inform him of the notices served, the Court cannot find anyone
to blame but the petitioner himself in not exercising diligence in informing his counsel of his whereabouts.

A motion for new trial based on newly-discovered evidence may be granted only if the following requisites are met: (a) that the
evidence was discovered after trial; (b) that said evidence could not have been discovered and produced at the trial even with the
exercise of reasonable diligence; (c) that it is material, not merely cumulative, corroborative or impeaching; and (d) that the evidence
is of such weight that, if admitted, it would probably change the judgment. It is essential that the offering party exercised reasonable
diligence in seeking to locate the evidence before or during trial but nonetheless failed to secure it."

In the instant case, the Court finds no reason to waive the procedural rules in order to grant the motion for new trial of the petitioner.
There is just no legal basis for the grant of the motion for new trial. The Court believes that the petitioner was given the opportunity to
be heard but he chose to put this opportunity into waste by not being diligent enough to ask about the status of the criminal case
against him and inform his counsel of his whereabouts.
Remedial Law I| Page 163

People vs. Li Ka Kim


G.R. No. 148586. May 25, 2004

Facts:

The Regional Intelligence and Investigation Division (RIID) of PNP-Region IV received a report from an informer, named "Boy," that
a certain alias "Ed," known to be a drug dealer operating in the southern part of Metro Manila, was looking for a buyer of shabu. PO2
Trambulo, an officer of RIID, made initial contact with Ed through a phone call using Boy’s cellular phone. Boy introduced PO2
Trambulo to Ed as "Rollie," a buyer of shabu. The parties agreed to meet at a mall in Parañaque City. The buy-bust operation was
successfully carried-out where Ed was arrested. An Information was then filed against Ed for violation of R.A. 6425, as amended.

Acting upon advice of counsel, Ed did not enter any plea during his arraignment; the court was thus constrained to enter a plea of "not
guilty" in his behalf. At the trial, PO2 Trambulo pointed to Ed as being the seller of the confiscated shabu and positively identified the
brown paper bag given to him by Ed containing the prohibited drug which he marked "CVT.” On the other hand, Ed denied the
allegations. The RTC convicted Ed and decreed the penalty of death. It debunked Ed’s defense of denial. The court found it hard to
believe that Ed would be singled out by the police officers from scores of people at the mall. The court likewise noted that the car, as
well as the license plate, used by Ed had been stolen, and that he was an undocumented alien as so shown by the letter of the
Commissioner on Immigration and Deportation to the State Prosecutor.

Ed then filed a motion to remand the case for new trial. In his motion, he asked for a new trial, citing Section 14, Rule 121, of the
Rules on Criminal Procedure, because of newly discovered evidence, i.e., his passport which would establish his true identity as
Huang Xiao Wei, a Chinese National, and as having entered the Philippines as a tourist.

Issue:

Whether or not the passport can be considered as newly discovered evidence as a ground for new trial?

Held:

No. The requisites of newly discovered evidence in order to justify a new trial are that - (a) the evidence is discovered after trial; (b)
such evidence could not have been discovered and produced at the trial even with the exercise of reasonable diligence; and (c) the
evidence is material, not merely cumulative, corroborative, or impeaching, and of such weight that, if admitted, would likely change
the judgment.

Not one of the requisites mentioned is attendant. Ed’s passport could have easily been presented and produced during the trial. Then,
too, the presentation of his passport, would hardly be material to the outcome of the case. Ed was positively identified by the
prosecution witnesses as being the perpetrator of the crime. Most importantly, Ed even identified himself as Li Ka Kim at the trial and
not as Huang Xiao Wei, that bolsters the conclusion that he deliberately concealed his true identity in the nefarious enterprise. Also,
the totality of evidence presented is convincing and points to Ed as being a person engaged in the sale of illegal drugs. The testimony
of the prosecution witnesses identifying him to be a seller of illegal drugs appears to be categorical and unfabricated.
Remedial Law I| Page 164

Rules 40-45

Heirs of Arturo Garcia I (In substitution of Heirs of Melecio Bueno) vs. Municipality of Iba, Zambales G.R. No. 162217,
July 22, 2015

Facts:

The late Melecio Bueno was the tenant-farmer beneficiary of an agricultural land located in Poblacion, Iba, Zambales. Bueno brought
an ejectment suit in the MTC of Iba against the Municipality of Iba, Province of Zambales, claiming that in 1983, the Municipality of
Iba had constructed the public market on a substantial portion of his land without his consent; and that his repeated demands for the
Municipality of Iba to vacate the property had remained unheeded. The MTC ruled in favor of Bueno. The Municipality of Iba then
filed its notice of appeal, but the MTC denied the same. The Municipality of Iba thus filed its petition for certiorari in the RTC of Iba,
Zambales to assail the denial of due course by the MTC. The RTC granted the petition for certiorari. The herein petitioners-heirs, who
meanwhile substituted Bueno upon his death, moved for the reconsideration of the judgment granting the petition for certiorari, but the
RTC denied the same.

Aggrieved, the Heirs appealed to the Court of Appeals by petition for review under Rule 42 of the Rules of Court. The CA dismissed
the petition for review for not being the proper mode of appeal. It observed that the assailed orders had been issued by the RTC in the
exercise of its original jurisdiction. The Heirs filed a motion for reconsideration but it was denied by the CA. Hence, the present case.
Although admitting that their petition for review under Rule 42 was inappropriate, the Heirs maintain that they substantially complied
with the requirements of an ordinary appeal under Rule 41, and pray that the Court exercise its equity jurisdiction.

Issue:

Whether or not a Petition for Review under Rule 42, despite being an improper remedy, may be treated as an Ordinary Appeal under
Rule 41?

Held:

No. An appeal brings up for review any error of judgment committed by a court with jurisdiction over the subject of the suit and over
the persons of the parties, or any error committed by the court in the exercise of its jurisdiction amounting to nothing more than an
error of judgment.  It was, therefore, very crucial for the Heirs and their counsel to have been cognizant of the different modes to
appeal the adverse decision of the RTC in the special civil action for certiorari brought by the Municipality of Iba. Such modes of
appeal were well delineated in the Rules of Court, and have been expressly stated in Section 2, Rule 41 of the Rules of Court since
July 1, 1997, to wit:

Section 2. Modes of appeal.

(a) Ordinary appeal.- The appeal to the Court of Appeals in cases decided by the Regional Trial Court in the exercise of its original
jurisdiction shall be taken by filing a notice of appeal with the court which rendered the judgment or final order appealed from and
serving a copy thereof upon the adverse party. No record on appeal shall be required except in special proceedings and other cases of
multiple or separate appeals where the law or these Rules so require. In such cases, the record on appeal shall be filed and served in
like manner.

(b) Petition for review.- The appeal to the Court of Appeals in cases decided by the Regional Trial Court in the exercise of its
appellate jurisdiction shall be by petition for review in accordance with Rule 42.

(c) Appeal by certiorari.-In all cases where only questions of law are raised or involved, the appeal shall be to the Supreme Court by
petition for review on certiorari in accordance with Rule 45.

Pursuant to this rule, in conjunction with Section 3 and Section 4 of Rule 41, the Heirs should have filed a notice of appeal in the RTC
within the period of 15 days from their notice of the judgment of the RTC, and within the same period, should have paid to the clerk of
the RTC the full amount of the appellate court docket and other lawful fees.

The filing of the notice of appeal within the period allowed by Section 3 sets in motion the remedy of ordinary appeal because the
appeal is deemed perfected as to the appealing party upon his timely filing of the notice of appeal. It is upon the perfection of the
Remedial Law I| Page 165

appeal filed in due time, and the expiration of the time to appeal of the other parties that the RTC shall lose jurisdiction over the case.
On the other hand, the non-payment of the appellate court docket fee within the reglementary period as required by Section 4, is both
mandatory and jurisdictional, the non-compliance with which is fatal to the appeal, and is a ground to dismiss the appeal under Section
1(c), Rule 50 of the Rules of Court. The compliance with these requirements was the only way by which the Heirs could have
perfected their appeal from the adverse judgment of the RTC.

In contrast, an appeal filed under Rule 42 is deemed perfected as to the petitioner upon the timely filing of the petition for review
before the CA, while the RTC shall lose jurisdiction upon perfection thereof and the expiration of the time to appeal of the other
parties.

The distinctions between the various modes of appeal cannot be taken for granted, or easily dismissed, or lightly treated. The appeal
by notice of appeal under Rule 41 is a matter or right, but the appeal by petition for review under Rule 42 is a matter of discretion. An
appeal as a matter of right, which refers to the right to seek the review by a superior court of the judgment rendered by the trial court,
exists after the trial in the first instance. In contrast, the discretionary appeal, which is taken from the decision or final order rendered
by a court in the exercise of its primary appellate jurisdiction, may be disallowed by the superior court in its discretion. Verily, the CA
has the discretion whether to give due course to the petition for review or not.

The procedure taken after the perfection of an appeal under Rule 41 also significantly differs from that taken under Rule 42. Under
Section 10 of Rule 41, the clerk of court of the RTC is burdened to immediately undertake the transmittal of the records by verifying
the correctness and completeness of the records of the case; the transmittal to the CA must be made within 30 days from the perfection
of the appeal.  This requirement of transmittal of the records does not arise under Rule 42, except upon order of the CA when deemed
necessary.

As borne out in the foregoing, the Heirs' resort to the petition for review under Rule 42 was wrong. Hence, the CA did not err in
denying due course to the petition for review. The Heirs’ plea for liberality is also unworthy of any sympathy from the Court, as the
latter always looked at appeal as not a matter of right but a mere statutory privilege. Moreover, the Heirs did not give any good reason
or cause that could warrant the relaxation of the rules in their favor.
Remedial Law I| Page 166

Estinozo vs. CA
G.R. No. 150276. February 12, 2008

Facts:

Cecilia Estinozo, a townmate of the complainants (Gaudencio Ang, et al), while in Sogod, Southern Leyte, represented to them that
she was one of the owners of Golden Overseas Employment and that she was recruiting workers to be sent abroad.  On the promised
date of their departure, however, private complainants never left the country. he then asked from the said complainants the payment of
placement and processing fees totaling P15,000.00. They were then informed by petitioner that there were no available plane tickets
and that they would leave by September of that year. After the failed promises of Estinozo, the complainants filed a case for estafa
against her which they eventually won, the trial court finding her guilty beyond reasonable doubt of estafa by false pretenses.

The CA affirmed the decision of the trial court. within the 15-day reglementary period to file a motion for reconsideration or a petition
for review, Estinozo filed with the appellate court a Motion for Extension of Time to File a Motion for Reconsideration.  On June 28,
2001, the CA, in the challenged Resolution,  denied the said motion pursuant to Rule 52, Section 1 of the Rules of Court and Rule 9,
Section 2 of the Revised Internal Rules of the Court of Appeals (RIRCA). Estinozo then filed a Motion for Reconsideration of the
June 28, 2001 Resolution of the CA. The appellate court denied the same, on August 17, 2001, in the other assailed Resolution.

Displeased with this series of denials,Estinozo instituted the instant Petition for Certiorari under Rule 65, arguing, among others, that:
(1) her previous counsel, by filing a prohibited pleading, foreclosed her right to file a motion for reconsideration of the CA’s decision,
and consequently an appeal therefrom; (2) she should not be bound by the mistake of her previous counsel especially when the latter’s
negligence and mistake would prejudice her substantial rights and would affect her life and liberty;

Issue:

Whether or not a petition for certiorari under Rule 65 was the proper remedy in assailing the CA’s decision denying her appeal?

Held:

No. Considering that, in this case, appeal by certiorari was available to Estinozo, she effectively foreclosed her right to resort to a
special civil action for certiorari, a limited form of review and a remedy of last recourse, which lies only where there is no appeal or
plain, speedy and adequate remedy in the ordinary course of law. A petition for review on certiorari under Rule 45 and a petition
for certiorari under Rule 65 are mutually exclusive remedies. Certiorari cannot co-exist with an appeal or any other adequate
remedy.

Even granting arguendo that the instant certiorari petition is an appropriate remedy, still this Court cannot grant the writ prayed for
because there was no grave abuse of discretion committed by the CA in the challenged issuances. The rule, as it stands now without
exception, is that the 15-day reglementary period for appealing or filing a motion for reconsideration or new trial cannot be extended,
except in cases before this Court, as one of last resort, which may, in its sound discretion grant the extension requested.

It is well to point out that with petitioner’s erroneous filing of a motion for extension of time and with her non-filing of a motion for
reconsideration or a petition for review from the CA’s decision, the challenged decision has already attained finality and may no
longer be reviewed by this Court. The instant Rule 65 petition cannot even substitute for the lost appeal— certiorari is not a procedural
device to deprive the winning party of the fruits of the judgment in his or her favor. Relief will not be granted to a party who seeks to
be relieved from the effects of the judgment when the loss of the remedy at law was due to his own negligence, or to a mistaken mode
of procedure.
Remedial Law I| Page 167

Heirs of Spouses Reterta vs. Spouses Mores


G.R. No. 159941. August 17, 2011.

Facts:

The Heirs of Reterta filed an action for quieting of title and reconveyance against the Spouses Mores in the RTC of Trece Martires
City averring that they were the true owners of the subject land having inherited the same from their father who has been granted the
land by virtue of his occupation and cultivation and for the continuous possession by him and his predecessors-in-interest of the land.
They also assert that they only discovered of an affidavit purportedly signed by their father waiving his rights, interests and
participation in the land in favor of one Lorenzo Mores. Spouses Mores filed a Motion to Dismiss against the complaint citing lack of
jurisdiction of the RTC over the land, the same being a friar land and not part of public domain, the rightful jurisdiction is with the
Director of Lands. The RTC granted such motion. The Heirs of Reterta timely filed their MR but it was denied. Therefore, the Heirs
assailed such dismissal via petition for certiorari under Rule 65.

Issue:

Whether or not a petition for certiorari under Rule 65 was the proper remedy in assailing the RTC’s Order granting a Motion to
Dismiss?

Held:

No. The order granting the respondents’ motion to dismiss was a final, as distinguished from an interlocutory, order against which the
proper remedy was an appeal in due course. Certiorari, as an extraordinary remedy, is not substitute for appeal due to its being availed
of only when there is no appeal, or plain, speedy and adequate remedy in the ordinary course of law. The order that the petitioners
really wanted to obtain relief from was the order granting the respondents motion to dismiss, not the denial of the motion for
reconsideration. The fact that the order granting the motion to dismiss was a final order for thereby completely disposing of the case,
leaving nothing more for the trial court to do in the action, truly called for an appeal, instead of certiorari, as the correct remedy.

Moreover, even Section 9 of Rule 37 of the Rules of Court, cited by the petitioners, indicates that the proper remedy against the denial
of the petitioner’s motion for reconsideration was an appeal from the final order dismissing the action upon the respondents motion to
dismiss.  The restriction against an appeal of a denial of a motion for reconsideration independently of a judgment or final order is
logical and reasonable. A motion for reconsideration is not putting forward a new issue, or presenting new evidence, or changing the
theory of the case, but is only seeking a reconsideration of the judgment or final order based on the same issues, contentions, and
evidence.

HOWEVER, despite availing of improper mode of remedy, the Court of Appeals should have given due course to the petition
because (a) the broader interest of justice demanded that certiorari be given due course to avoid the undeserved grossly unjust result
that would befall the petitioners otherwise; and (b) the order of the RTC granting the motion to dismiss on ground of lack of
jurisdiction over the subject matter evidently constituted grave abuse of discretion amounting to excess of jurisdiction.
 
On occasion, the Court has considered certiorari as the proper remedy despite the availability of appeal, or other remedy in the
ordinary course of law. In Francisco Motors Corporation v. Court of Appeals the Court has declared that the requirement that there
must be no appeal, or any plain speedy and adequate remedy in the ordinary course of law admits of exceptions, such as: (a) when it is
necessary to prevent irreparable damages and injury to a party; (b) where the trial judge capriciously and whimsically exercised his
judgment; (c) where there may be danger of a failure of justice; (d) where an appeal would be slow, inadequate, and insufficient; (e)
where the issue raised is one purely of law; (f) where public interest is involved; and (g) in case of urgency. Specifically, the Court has
held that the availability of appeal as a remedy does not constitute sufficient ground to prevent or preclude a party from making use
of certiorari if appeal is not an adequate remedy, or an equally beneficial, or speedy remedy. It is inadequacy, not the mere absence of
all other legal remedies and the danger of failure of justice without the writ, that must usually determine the propriety of  certiorari. A
remedy is plain, speedy and adequate if it will promptly relieve the petitioner from the injurious effects of the judgment, order, or
resolution of the lower court or agency.
Remedial Law I| Page 168

Manaloto vs. Veloso III


G.R. No. 171365. October 6, 2010

Facts:

[PRIOR CASE] Ermelinda Manaloto, et al. (petitioners) filed an unlawful detainer case against Ismael Veloso III (respondent). The
MeTC ruled in favor of the petitioners and ordered respondent to vacate. The RTC reversed the MeTC decision. After successive
appeals, the case went up to the Supreme Court which affirmed the decision of the RTC and that decision became final and executory.

Meanwhile, while respondent’s appeal of the MeTC’s decision was pending before RTC Branch 88, he filed before RTC Branch 227 a
Complaint for Breach of Contract (allegedly, the petitioners as lessors failed to make continuing repairs on the property and keep it
tenantable) and Damages (because he suffered embarrassment and humiliation when petitioners distributed copies of the MeTC
decision in the unlawful detainer case to the homeowners of Horseshoe Village) against petitioners.

Petitioners filed an Omnibus Motion to Dismiss respondent’s complaint. RTC Branch 227 dismissed respondent’s complaint for
violating the rule against splitting of cause of action, lack of jurisdiction, and failure to disclose the pendency of a related case.
Respondent received a copy of the RTC-Branch 227 decision on September 26, 2003. He filed a Motion for Reconsideration of said
judgment on October 10, 2003, which RTC-Branch 227 denied in an Order dated December 30, 2003. Respondent received a copy of
the RTC-Branch 227 order denying his Motion for Reconsideration on February 20, 2004, and he filed his Notice of Appeal on
March 1, 2004. However, the RTC- Branch 227, in an Order dated March 23, 2004, dismissed respondent’s appeal for being filed out
of time. Respondent received a copy of the RTC-Branch 27 order dismissing his appeal on April 30, 2004 and he filed a Motion for
Reconsideration of the same on May 3, 2004. The RTC-Branch 227, in another Order dated May 31, 2004, granted respondent’s latest
motion.

The CA then gave due course to the appeal and ruled in favor of respondent on his claim for damages and ordered the petitioners to
pay him moral damages. In a Petition for Review before the SC, petitioners now contend that respondent’s appeal was filed out of
time and the decision of the RTC which dismissed respondent’s complaint had already attained finality.

Issue:

Whether or not Ismael Veloso (respondent) timely filed his appeal?

Held:

Yes. Jurisprudence has settled the “fresh period rule,” according to which, an ordinary appeal from the RTC to the Court of Appeals,
under Section 3 of Rule 41 of the Rules of Court, shall be taken within fifteen (15) days either from receipt of the original judgment
of the trial court or from receipt of the final order of the trial court dismissing or denying the motion for new trial or motion for
reconsideration.

In this case, respondent received a copy of the Resolution dated September 2, 2003 of the RTC-Branch 227 dismissing his complaint
in Civil Case No. Q-02-48341 on September 26, 2003. Fourteen days thereafter, on October 10, 2003, respondent filed a Motion for
Reconsideration of said resolution. The RTC-Branch 227 denied respondent’s Motion for Reconsideration in an Order dated
December 30, 2003, which the respondent received on February 20, 2004. On March 1, 2004, just after nine days from receipt of the
order denying his Motion for Reconsideration, respondent already filed his Notice of Appeal. Clearly, under the fresh period rule,
respondent was able to file his appeal well-within the prescriptive period of 15 days, and the Court of Appeals did not err in giving
due course to said appeal.
Remedial Law I| Page 169

Latorre vs. Latorre


G.R. No. 183926. March 29, 2010

Facts:

Generosa Latorre filed before the RTC a Complaint for Collection and Declaration of Nullity of Deed of Absolute Sale with
application for Injunction against her own son, Luis Latorre and one Ifzal Ali. Luis Latorre immediately filed a Motion to Dismiss on
the sole ground that the venue of the case was improperly laid. He stressed that while the complaint was denominated as one for
Collection and Declaration of Nullity of Deed of Absolute Sale with application for Injunction, in truth the case was a real action
affecting title to and interest over the subject property. He insisted that all of petitioner's claims were anchored on her claim of
ownership over one-half portion of the subject property. Since the subject property is located in Makati City, respondent argued that
petitioner should have filed the case before the RTC of Makati City and not of Muntinlupa City. Ifzal also filed his motion to dismiss
on the ground of want of jurisdiction, asserting that he was immune from suit because he was an officer of ADB, an international
organization.

The RTC denied Luis’ motion to dismiss. He then filed an Answer Ad Cautelam. (RTC later on dismissed petitioner's claim against
Ifzal because the dispute was clearly between petitioner and respondent). The RTC thereafter ruled in favor of Luis.  Aggrieved,
Generosa filed her Motion for Reconsideration, which the RTC denied. Generosa then went up directly to the Supreme Court via a
petition for review on certiorari claiming that the RTC erred in treating the venue as jurisdiction and in treating her complaint as a real
action.
 
Issue:  

Whether or not a petition for review on certiorari under Rule 45 filed directly to the SC is proper?

Held:

No. Petitioner came directly to the Supreme Court on a Petition for Review on Certiorari under Rule 45, in relation to Rule 41, of the
Rules of Civil Procedure on alleged pure questions of law. There are three (3) modes of appeal from decisions of the RTC, namely:
(1) ordinary appeal or appeal by writ of error, where judgment was rendered in a civil or criminal action by the RTC in the exercise of
its original jurisdiction; (2) petition for review, where judgment was rendered by the RTC in the exercise of its appellate jurisdiction;
and (3) petition for review to the Supreme Court.
 
The first mode of appeal, governed by Rule 41, is brought to the Court of Appeals (CA) on questions of fact or mixed questions of
fact and law. The second mode of appeal, covered by Rule 42, is brought to the CA on questions of fact, of law, or mixed questions of
fact and law. The third mode of appeal, provided in Rule 45, is filed with the Supreme Court only on questions of law.
 
A question of law arises when there is doubt as to what the law is on a certain state of facts, while there is a question of fact
when the doubt arises as to the truth or falsity of the alleged facts.
 
In her Reply to respondents Comment, petitioner prayed that this Court decide the case on the merits. To do so, however, would
require the examination by this Court of the probative value of the evidence presented, taking into account the fact that the RTC failed
to adjudicate this controversy on the merits. This, unfortunately, the SC cannot do. It thus becomes exceedingly clear that the filing of
the case directly with this Court ran afoul of the doctrine of hierarchy of courts. Pursuant to this doctrine, direct resort from the lower
courts to the Supreme Court will not be entertained unless the appropriate remedy sought cannot be obtained in the lower tribunals.
This Court is a court of last resort, and must so remain if it is to satisfactorily perform the functions assigned to it by the Constitution
and by immemorial tradition.

Summary of other procedural lapses by the parties mentioned by the SC:

First. Petitioner filed her complaint with the RTC of Muntinlupa City instead of the RTC of Makati City, the latter being the proper
venue in this case. Actions affecting title to or possession of real property or an interest therein (real actions) shall be commenced and
tried in the proper court that has territorial jurisdiction over the area where the real property is situated. The action in the RTC, other
than for Collection, was for the Declaration of Nullity of the Deed of Absolute Sale involving the subject property, which is located
at No. 1366 Caballero St., Dasmarias Village, Makati City. The venue for such action is unquestionably the proper court
of Makati City, where the real property or part thereof lies, not the RTC of Muntinlupa City.

Second. The RTC also committed a procedural blunder when it denied respondent's motion to dismiss on the ground of improper
venue. The RTC insisted that trial on the merits be conducted even when it was awfully glaring that the venue was improperly laid, as
Remedial Law I| Page 170

pointed out by respondent in his motion to dismiss. After trial, the RTC eventually dismissed the case on the ground of lack of
jurisdiction, even as it invoked, as justification, the rules and jurisprudence on venue. Despite the conduct of trial, the RTC failed to
adjudicate this case on the merits. 

Third. When the RTC denied his Motion to Dismiss, respondent could have filed a petition for certiorari and/or prohibition inasmuch
as the denial of the motion was done without jurisdiction or in excess of jurisdiction or with grave abuse of discretion amounting to
lack of jurisdiction. However, despite this lapse, it is clear that respondent did not waive his objections to the fact of improper venue,
contrary to petitioner's assertion. Notably, after his motion to dismiss was denied, respondent filed a Motion for Reconsideration to
contest such denial. Even in his Answer Ad Cautelam, respondent stood his ground that the case ought to be dismissed on the basis of
improper venue.
Remedial Law I| Page 171

Alfredo vs. Borras


G.R. No. 144225. June 17, 2003

Facts:

The subject matter of the controversy is a parcel of land measuring 81,524 square meters in Bataan. The original owners are Spouses
Godofredo and Carmen Alfredo (Spouses Alfredo). They allegedly sold the land to Spouses Armando and Adelie Borras (Spouses
Borras) for P15,000: P7,000 of which for the mortgage debt to release the mortgage from Development Bank of the Philippines and
the remaining to be paid to Spouses Alfredo. Spouses Borras gave the money to Spouses Alfredo for the payment of the loan and
subsequently, the mortgage was released and the OCT was returned to Spouses Alfredo. Upon the payment of the balance of the
purchase price, Spouses Alfredo delivered to Spouses Borras the owner’s duplicate copy of OCT. Spouses Alfredo also introduced
Spouses Borras as the new owners of the land to their tenants.

In 1994, Spouses Borras learned that Spouses Alfredo had re-sold the portions of the subject land to several persons. They then filed
an adverse claim with the Register of Deeds in Bataan. They found out that Spouses Alfredo had secured an owner’s duplicate copy of
OCT No. 284 after filing a petition in court for the issuance of a new copy. They wrote to Spouses Alfredo and complained about their
acts but the latter did not reply. They then filed a Complaint for Specific Performance. They amended their complaint to include
additional defendants – the subsequent buyers.

In their Answer, Spouses Alfredo and the subsequent buyers argued that the action is unenforceable under the Statute of Frauds. They
pointed out that there is no written instrument evidencing the alleged contract of sale. They also asserted that the subsequent buyers
were buyers in good faith.

The RTC ruled in favor of Spouses Borras and found that there was a perfected contract of sale and the subsequent buyers were not
innocent purchasers.

The CA affirmed the ruling of the RTC and found the factual findings of the trial court well supported by evidence. Hence, this
petition.

Issue:

Whether or not the findings of the RTC and CA are binding on the Supreme Court?

Held:

Yes. In a petition for review on certiorari under Rule 45, the Court reviews only errors of law and not errors of facts. The factual
findings of the appellate court are generally binding on this Court. This applies with greater force when both the trial court and the
Court of Appeals are in complete agreement on their factual findings. In this case, there is no reason to deviate from the findings of the
lower courts. The facts relied upon by the trial and appellate courts are borne out by the record. We agree with the conclusions drawn
by the lower courts from these facts.

Valid and Enforceability of the Sale: The contract was consummated because the sellers and buyers have performed their respective
obligations under the contract: Spouses Borras had paid the price and the Spouses Alfredo delivered the subject land to the former.
The Statute of Frauds applies only to executory contracts and not to contracts either partially or totally performed. Thus, the Statute of
Frauds is not applicable in this case.

Note: There are many civil law issues under this case such as validity of alienation without the signature of the husband, without the
approval of the Secretary of Agriculture and Natural Resources, prescription, laches and whether or not the subsequent buyers were
buyer in good faith. The SC resolved them all in favor of Spouses Borras.

Validity of alienation without the signature of the husband: The sale is not void in accordance with the Civil Code (not Family Code),
it is merely voidable. And the husband had ratified the alienation anyway.
Remedial Law I| Page 172

Validity of alienation without the approval of the Secretary of Agriculture and Natural Resources: the approval after the 5 th year period
is merely directory not mandatory.

Prescription and laches: The action has not prescribed since the action is one for reconveyance based on an implied trust which does
not prescribe as long as possession of the land is with the plaintiff.

Subsequent buyers were not buyers in good faith: since the Spouses Borras had caused the notice of adverse claim prior to the sale and
their registration, they cannot be deemed buyers in good faith since they had notice of the adverse claim of the Spouses Borras.
Remedial Law I| Page 173

People vs. Corpuz


G.R. No. 148198. October 1, 2003

Facts:

Private complainants Belinda Cabantog, Concepcion San Diego, Erlinda Pascual and Restian Surio went to Alga-Moher International
Placement Services Corporation to apply for employment as factory workers in Taiwan. They were accompanied by a certain “Aling
Josie” who introduced them to the agency’s President and General Manager, Mrs. Evelyn Gloria Reyes. Mrs. Reyes asked them to
accomplish the application forms. Thereafter, they were told to return to the office with P10,000.00 each as processing fee. Private
complainants returned to the agency to pay the processing fees. Mrs. Reyes was not at the agency that time, but she called Elizabeth
Corpuz (appellant) on the telephone to ask her to receive the processing fees. Thereafter, appellant advised them to wait for the
contracts to arrive from the Taiwan employers. Nothing happened with their applications so they filed their complaint with the
National Bureau of Investigation which led to the arrest and detention of appellant. Appellant was then charged with the crime of
Illegal Recruitment in Large Scale. For her part, appellant resolutely denied having a hand in the illegal recruitment and claimed that
she merely received the money on behalf of Mrs. Reyes, where she had been working as secretary for three months prior to the
incident. In compliance with the order of her employer and since the cashier was absent, she received the processing fees of private
complainants, which she thereafter remitted to Mrs. Reyes. She had no knowledge that the agency’s license was suspended by the
POEA.

The trial court convicted the appellant. Hence, this appeal.

Issue:

Whether or not the trial court erred in convicting the appellant?

Held:

Yes. The trial court convicted appellant based on its findings that despite the suspension of the agency’s license, appellant still
convinced the applicants to give their money with the promise to land a job abroad. Moreover, as the registered secretary of the
agency she had management control of the recruitment business. It is axiomatic that findings of facts of the trial court, its
calibration of the collective testimonies of witnesses and probative weight thereof and its conclusions culled from said findings
are accorded by this Court great respect, if not conclusive effect, because of the unique advantage of the trial court in
observing and monitoring at close range, the conduct, deportment and demeanor of the witnesses as they testify before the
trial court. However, this principle does not apply if the trial court ignored, misunderstood or misconstrued cogent facts and
circumstances of substance which, if considered, would alter the outcome of the case. The exception obtains in this case.

On July 30, 1998, appellant received the processing fees of the private complainants since the cashier was absent that day. Her receipt
of the money was in compliance with the order of her employer, Mrs. Reyes. She did not convince the applicants to give her their
money since they went to the agency precisely to pay the processing fees upon the earlier advice of Mrs. Reyes. Moreover, as stated in
the last sentence of Section 6 of RA 8042, the persons who may be held liable for illegal recruitment are the principals, accomplices
and accessories. In case of juridical persons, the officers having control, management or direction of their business shall be liable.

In this case, the prosecution failed to establish that appellant, as secretary, had control, management or direction of the recruitment
agency. Appellant started her employment with the agency on May 1, 1998 and she was tasked to hold and document employment
contracts from the foreign employers. She did not entertain applicants and she had no discretion over how the business was managed.
The trial court’s finding that appellant, being the secretary of the agency, had control over its business, is not only non sequitur but has
no evidentiary basis. The culpability of the employee hinges on his knowledge of the offense and his active participation in its
commission. Where it is shown that the employee was merely acting under the direction of his superiors and was unaware that his acts
constituted a crime, he may not be held criminally liable for an act done for and in behalf of his employer.

In this case, the prosecution failed to adduce sufficient evidence to prove appellant’s active participation in the illegal recruitment
activities of the agency. As already established, appellant received the processing fees of the private complainants for and in behalf of
Mrs. Reyes who ordered her to receive the same. She neither gave an impression that she had the ability to deploy them abroad nor
convinced them to part with their money. More importantly, she had no knowledge that the license was suspended the day before she
received the money. Their failure to depart for Taiwan was due to the suspension of the license, an event which appellant did not have
control of. Her failure to refund their money immediately upon their demand was because the money had been remitted to Mrs. Reyes
on the same day she received it from them.
Remedial Law I| Page 174

Philippine Airlines, Inc. vs. Court of Appeals


G.R. No. 127473. December 8, 2003

Facts:

Private respondents (Judy Amor, et al.) filed with the RTC a complaint  for damages against petitioner (PAL) due to the latter’s failure
to honor their confirmed tickets. Private respondent Judy Amor purchased three plane tickets for her and her infant son, Gian Carlo as
well as her sister Jane Gamil for 7:10 a.m. flight bound for Manila from PAL. They arrived at the Legaspi Airport at 6:20 a.m. Carlo
Benitez was supposed to use the confirmed ticket of a certain Dra. Emily Chua. Judy went to the office of the station manager to
request that minor plaintiff Carlo Benitez be allowed to use the ticket of Dra. Chua. Lloyd Fojas, the check-in clerk on duty, wrote
something on the tickets which they later read as "late check-in 7:05" and told them to proceed to the cashier to make arrangements.
They pleaded with Fojas, pointing out that it is only 6:45 a.m., but the latter did not even look at him or utter any word. Private
respondents were not able to board said flight. The plane left at 7:30 a.m., twenty minutes behind the original schedule. They tried to
catch an afternoon flight.  Private respondents were told to wait for the 5:30 p.m. flight. They checked-in their bags and were told to
hand in their tickets. Later, a PAL employee at the check-in counter called out the name of private respondent minor Carlo Benitez
and was told by the PAL personnel that they cannot be accommodated. Fojas who was also at the counter then removed the boarding
passes inserted in private respondents’ tickets as well as the tags from their luggages.

PAL contends that private respondents are not entitled to their claim for damages because they were late in checking-in for the flight;
and that they were only chance or waitlisted passengers for the afternoon flight and were not accommodated because all confirmed
passengers of the flight had checked-in. The RTC rendered judgment upholding the evidence presented by private respondents.
Aggrieved, petitioner appealed to the CA which affirmed the judgment of the trial court. Hence, the present petition of PAL, raising
the following issues: (1) whether or not private respondents checked-in on time for the flight, and (2) whether or not the damages
awarded are excessive.

Issue:

Whether or not the case falls under the exemptions when the Supreme Court can review factual findings of the lower courts?

Held:

No. In petitions for review on certiorari under Rule 45 of the Rules of Court, the general rule is that only questions of law may be
raised by the parties and passed upon by this Court. Factual findings of the appellate court are generally binding on us especially when
in complete accord with the findings of the trial court. This is because it is not the Court’s function to analyze or weigh the evidence
all over again. However, this general rule admits of exceptions, to wit:

(a) where there is grave abuse of discretion; (b) when the finding is grounded entirely on speculations, surmises or conjectures; (c)
when the inference made is manifestly mistaken, absurd or impossible; (d) when the judgment of the Court of Appeals was based on a
misapprehension of facts; (e) when the factual findings are conflicting; (f) when the Court of Appeals, in making its findings, went
beyond the issues of the case and the same are contrary to the admissions of both appellant and appellee; (g) when the Court of
Appeals manifestly overlooked certain relevant facts not disputed by the parties and which, if properly considered, would justify a
different conclusion; and, (h) where the findings of fact of the Court of Appeals are contrary to those of the trial court, or are mere
conclusions without citation of specific evidence, or where the facts set forth by the petitioner are not disputed by the respondent, or
where the findings of fact of the Court of Appeals are premised on the absence of evidence and are contradicted by the evidence on
record.

Petitioner invokes exception (b). After review of the records, the Court found no reason to disturb the affirmance by the CA of the
findings of the trial court that the private respondents have checked-in on time; that they reached the airport at 6:20 a.m., based on the
testimonies of private respondent Judy Amor, and witnesses Salvador Gonzales and Atty. Owen Amor who were consistent in their
declarations on the witness stand and corroborated one another’s statements; and that the testimony of petitioner’s lone witness, Lloyd
Fojas is not sufficient to overcome private respondents’ evidence.

It is a well-entrenched principle that absent any showing of grave abuse of discretion or any palpable error in its findings, this Court
will not question the probative weight accorded by the lower courts to the various evidence presented by the parties. As we explained
in Superlines Transportation Co. Inc., vs. ICC Leasing & Financing Corporation:
Remedial Law I| Page 175

The Court is not tasked to calibrate and assess the probative weight of evidence adduced by the parties during trial all over again… So
long as the findings of facts of the Court of Appeals are consistent with or are not palpably contrary to the evidence on record, this
Court shall decline to embark on a review on the probative weight of the evidence of the parties.
Remedial Law I| Page 176

Augusto vs. Risos


G.R. No. 131794. December 10, 2003

Facts:

Felisa Augusto and her siblings, Jose Augusto, Magdalena Augusto and Alfonso Augusto, all married, were the co-owners of a parcel
of land located in Barrio Mactan, Opon, Cebu. They sold the property to Guillermo Omolon. They took possession of the property but
it was still registered in the names of the Augusto siblings, under the original certificate of title. Guillermo Omolon died intestate and
was survived by Cleofe Omolon. Cleofe Omolon filed a petition for the reconstitution of the OCT covering the lot before the RTC.
The RTC granted the petition. However, upon presentation of the aforesaid order to the Register of Deeds, Cleofe was informed that
the owner’s copy had already been issued to Ruben Augusto, pursuant to an order issued by the court and the same was in the
possession of Atty. Noel Archival. Cleofe filed a petition before the RTC alleging that as lawful co-owner and possessor of lot, she
had every right to have and hold the owner’s duplicate of the said OCT. She prayed that after due proceedings, the respondents Ruben
Augusto and Atty. Noel Archival be ordered to surrender the owner’s copy of the said title.

On the other hand, Ruben Augusto and Atty. Noel Archival alleged that the Deed of Absolute Sale executed by Felisa, Magdalena,
Alfonso and Jose, all surnamed Augusto, was falsified and fictitious, and, thus, null and void.

On October 22, 1997, the RTC issued an order directing Atty. Noel Archival to produce the owner’s copy of OCT to allow the
annotation of Cleofe’s interest, upon which the owner’s duplicate copy of the title may thereafter be returned. The respondents therein
filed a motion for a partial reconsideration of the order. However, the court issued an order denying the motion of the respondents
therein. The respondents filed a notice of appeal from the said order to the Court of Appeals. The RTC issued an order denying due
course therefor, on its perception that the orders subject thereof were interlocutory; hence, not appealable. Petitioners argue that
contrary to the ruling of the public respondent, its October 22, 1997 Order was final and appealable, as the same disposed of the case.

Issue:

Whether or not the October 22, 1997 order of the RTC directing Atty. Archival to produce the owner’s copy of OCT was a final order
and hence appealable?

Held:

No. Section 1, Rule 41 of the Rules of Court provides that an appeal may be taken only from a final order, and not from an
interlocutory one. A final order is one which disposes of the whole subject matter or terminates a particular proceeding or action,
leaving nothing to be done but to enforce by execution what has been determined. An order or judgment is deemed final if it finally
disposes of, adjudicates, or determines the rights, or some right or rights of the parties, either on the entire controversy or on some
definite and separate branch thereof, and concludes them until it is reversed or set aside. Where no issue is left for future
consideration, except the fact of compliance with the terms of the order, such order is final and appealable. 8 In contrast, an order is
interlocutory if it does not finally dispose of the case.

In this case, the order of the public respondent directing the petitioners to produce the owner’s copy of OCT No. 3560 in the Office of
the Register of Deeds for the annotation of the private respondent’s interest over the property is merely interlocutory and not final;
hence, not appealable by means of a writ of error. The public respondent had not fully disposed of the case as it had not yet ruled on
whether to grant the private respondent’s prayer for the surrender of the owner’s copy of OCT No. 3560. As gleaned from the order of
the respondent judge, he believed that he had no jurisdiction to delve into and resolve the issue of ownership over the property and
was disposed to dismiss the petition. Before so doing, he believed it was necessary that the petitioner’s claim over the property be
annotated at the dorsal portion of the title before the institution of an ordinary motion for the resolution of the conflicting claims of
ownership over the property.

Springfield Development Corporation, Inc. vs.


Presiding Judge, RTC, Misamis Oriental, Br. 40, Cagayan de Oro City
G.R. No. 142628. February 6, 2007
Remedial Law I| Page 177

Facts:

Petra Capistrano Piit previously owned Lot which measured 123,408 square. Springfield Development Corporation, Inc. (Springfield)
bought with an area of 68,732 square meters, and another with an area of 49,778 square meters. Springfield developed these properties
into a subdivision project called Mega Heights Subdivision.

The Department of Agrarian Reform (DAR), through its Municipal Agrarian Reform Officer, issued a Notice of Coverage, placing the
property under the coverage of Republic Act (R.A.) No. 6657 or the Comprehensive Agrarian Reform Law of 1988. DARAB
Provincial Adjudicator Abeto A. Salcedo, Jr. rendered a decision declaring the nature of the property as residential and not suitable for
agriculture. The Regional Director filed a notice of appeal, which the Provincial Adjudicator disallowed for being pro forma and
frivolous. The decision became final and executory and Springfield proceeded to develop the property.

The DAR Regional Director then filed a petition for relief from judgment of the DARAB Decision, docketed as DARAB. In its
Decision dated October 5, 1995, the DARAB granted the petition and gave due course to the Notice of Coverage. It also directed the
Municipal Agrarian Reform Office to proceed with the documentation, acquisition, and distribution of the property to the true and
lawful beneficiaries.

Springfield and the heirs of Piit (petitioners) filed with the RTC of Cagayan de Oro City, Branch 40, a petition for annulment of the
DARAB Decision dated October 5, 1995 and all its subsequent proceedings. Petitioners contend that the DARAB decision was
rendered without affording petitioners any notice and hearing.

On motion filed by the farmer-beneficiaries, the RTC issued an Order dismissing the case for lack of jurisdiction. Petitioners filed with
the Court of Appeals (CA) a special civil action for certiorari, mandamus, and prohibition with prayer for the issuance of writ of
preliminary injunction and/or temporary restraining order. Petitioners alleged that the RTC committed grave abuse of discretion when
it ruled that the annulment of judgment filed before it is actually an action for certiorari in a different color. According to petitioners,
what it sought before the RTC is an annulment of the DARAB Decision and not certiorari, as the DARAB Decision is void ab initio
for having been rendered without due process of law. The CA dismissed the petition for lack of merit, ruling that the RTC does not
have jurisdiction to annul the DARAB Decision because it is a co-equal body.

Issue:

Whether or not the RTC has jurisdiction over the decisions of DARAB?

Held:

No. The DARAB is a quasi-judicial body created by Executive Order Nos. 229 and 129-A. R.A. No. 6657 delineated its adjudicatory
powers and functions. The DARAB Revised Rules of Procedure adopted on December 26, 198827 specifically provides for the
manner of judicial review of its decisions, orders, rulings, or awards. Rule XIV, Section 1 states:

SECTION 1. Certiorari to the Court of Appeals. Any decision, order, award or ruling by the Board or its Adjudicators on any
agrarian dispute or on any matter pertaining to the application, implementation, enforcement or interpretation of agrarian reform
laws or rules and regulations promulgated thereunder, may be brought within fifteen (15) days from receipt of a copy thereof, to the
Court of Appeals by certiorari, except as provided in the next succeeding section. Notwithstanding an appeal to the Court of Appeals
the decision of the Board or Adjudicator appealed from, shall be immediately executory.

Further, the prevailing 1997 Rules of Civil Procedure, as amended, expressly provides for an appeal from the DARAB decisions to the
CA. The rule is that where legislation provides for an appeal from decisions of certain administrative bodies to the CA, it means that
such bodies are co-equal with the RTC, in terms of rank and stature, and logically, beyond the control of the latter. Given that
DARAB decisions are appealable to the CA, the inevitable conclusion is that the DARAB is a co-equal body with the RTC and its
decisions are beyond the RTC's control.

Note: There is no such thing as annulment of judgments of the decisions of quasi-judicial bodies. The remedy is a Petition for Review
under Rule 43.

De Vera vs. Santiago, Sr.


G.R. No. 179457. June 22, 2015

Facts:
Remedial Law I| Page 178

Wilfredo De Vera, et al. filed an action for reconveyance of ownership or possession with damages against Santiago, et al. before the
MTC alleging that they have allegedly been in actual and continuous possession and occupation of their respective portions of the land
since 1967, without disturbance from any third person. Later on, however, they discovered that their respective lots covered by Lot
7303 were already covered by Free Patent Titles in the names of Santiago which were acquired through manipulation,
misrepresentation, fraud and deceit. In their Answer, Santiago, et al. specifically denied the material allegations in petitioners'
complaint and countered that they are the owners of the land. MTC ruled in favor of Santiago. On appeal, RTC reversed the decision
of MTC.

Santiago filed with the CA a petition for review under Rule 42 of the Rules of Court. The CA granted the petition for review, and
annulled and set aside the Decisions of both the RTC and the MTC on the ground of lack of jurisdiction (the assessed value of the land
is more than 20,000 and therefore the CA ruled that the MTC had no jurisdiction when it took first cognizance of the case and
consequently, the RTC cannot render a valid judgment when the case was appealed to it). For the same reason, the CA declined to
resolve and deemed as moot and academic the other factual issues raised in the petition.

The CA also ruled that assuming arguendo that the RTC had jurisdiction over the case, it nonetheless has no authority to declare as
null and void the Original Certificates of Title (Free Patents) registered in the name of respondents because the said titles were issued
four (4) years prior to the filing of the petitioners' complaint for reconveyance. The CA denied De Vera's motion for reconsideration of
its Decision. Hence, the petition for review on certiorari.

Issue:

Whether or not the CA erred in annulling the decision of RTC for lack of jurisdiction?

Held:

Yes. Based on the Tax Declarations attached to their complaint, the disputed land has a total assessed value of ₱54,370.00. Therefore,
the RTC has jurisdiction over petitioners' civil action involving title to a real property outside Metro Manila with a total assessed value
in excess of ₱20,000.00.

Thus, while the CA is correct in ruling that the MTC has no jurisdiction over the case for reconveyance and recovery of ownership and
possession of a land with an assessed value over ₱20,000.00, the same cannot be said of its ruling with respect to the RTC. Under
Section 8, Rule 40 of the Rules of Court, if the MTC tried a case on the merits despite having no jurisdiction over the subject
matter, its decision may be reviewed on appeal by the RTC, to wit:

Sec. 8. Appeal from orders dismissing case without trial; lack of jurisdiction.

If an appeal is taken from an order of the lower court dismissing the case without a trial on the merits, the Regional Trial Court may
affirm or reverse it, as the case may be. In case of affirmance and the ground of dismissal is lack of jurisdiction over the subject
matter, the Regional Trial Court, if it has jurisdiction thereover, shall try the case on the merits as if the case was originally filed with
it. In case of reversal, the case shall be remanded for further proceedings.

If the case was tried on the merits by the lower court without jurisdiction over the subject matter, the Regional Trial Court on appeal
shall not dismiss the case if it has original jurisdiction thereof, but shall decide the case in accordance with the preceding section,
without prejudice to the admission of amended pleadings and additional evidence in the interest of justice.

In Serrano v. Spouses Gutierrez, the Court explained that the first paragraph of Section 8, Rule 40 contemplates an appeal from an
order of dismissal issued without trial of the case on the merits, while the second paragraph deals with an appeal from an order of
dismissal but the case was tried on the merits. Both paragraphs, however, involve the same ground for dismissal, i.e., lack of
jurisdiction. Verily, the second paragraph refutes respondents' contention that Section 8, Rule 40 refers solely to cases where the MTC
dismissed a case filed therein without a trial on the merits and an appeal to the RTC was taken from the order of dismissal. Therefore,
the RTC correctly proceeded to decide the case on the merits despite the MTC's lack of jurisdiction over the subject matter.

In contrast, the CA erroneously reversed and set aside the RTC Decision for lack of jurisdiction. Indeed, the RTC has appellate
jurisdiction over the case and its decision should be deemed promulgated in the exercise of that jurisdiction. The RTC’s appellate
jurisdiction, as contrasted to its original jurisdiction, is provided in Section 22 of B.P. Blg.129, as amended, thus:

SECTION 22. Appellate jurisdiction.–Regional Trial Courts shall exercise appellate jurisdiction over all cases decided by
Metropolitan Trial Courts, Municipal Trial Courts, and Municipal Circuit Trial Courts in their respective territorial jurisdictions.

The above-quoted provision vests upon the RTC the exercise of appellate jurisdiction over all cases decided by the Metropolitan Trial
Courts, Municipal Trial Courts, and Municipal Circuit Trial Courts in their respective territorial jurisdictions. Clearly then, the amount
involved is immaterial for purposes of the RTC’s appellate jurisdiction; all cases decided by the MTC are generally appealable to the
Remedial Law I| Page 179

RTC irrespective of the amount involved. Hence, the CA grossly erred in nullifying the RTC Decision for lack of jurisdiction, and in
declaring as moot and academic the factual issues raised in the respondents' petition for review when it should have proceeded to
review on appeal the factual findings of the RTC. This is because the RTC not only has exclusive original jurisdiction over petitioners'
action for reconveyance of ownership and possession with damages, but also appellate jurisdiction over the MTC Decision itself.
Remedial Law I| Page 180

Philippine National Construction Corporation vs. Asiavest Merchant Bankers (M) Berhad
G.R. No. 172301. August 19, 2015

Facts:

PNCC and Asiavest Holdings (M) Sdn. Bhd. (Asiavest Holdings) caused the incorporation of an associate company known as
Asiavest-CDCP Sdn. Bhd. (Asiavest-CDCP), through which they entered into contracts to construct rural roads and bridges for the
State of Pahang, Malaysia. In connection with this construction contract, PNCC obtained various guarantees and bonds from Asiavest
Merchant Bankers (M) Berhad to guarantee the due performance of its obligations. There was failure to perform the obligations under
the construction contract, prompting the State of Pahang to demand payment against Asiavest Merchant Bankers (M) Berhad's
performance bonds. It "entered into a compromise agreement with the State of Pahang by paying the reduced amount. Consequently,
the corporation demanded indemnity from PNCC by demanding the amount it paid to the State of Pahang.

Asiavest Merchant Bankers (M) Berhad filed a Complaint for recovery of sum of money against PNCC before the RTC of Pasig. It
based its action on Malaysian laws. PNCC filed Motions for extension of time to file its Answer. The trial court granted these motions,
with the last one set to expire on July 3, 1994. But PNCC filed a Motion for another five-day extension. The trial court denied this
Motion. On July 27, 1994, the trial court declared PNCC in default for failure to file any responsive pleading, and allowed Asiavest
Merchant Bankers (M) Berhad to present its evidence ex parte. The Regional Trial Court rendered judgment in favor of Asiavest
Merchant Bankers (M) Berhad.

The trial court also denied PNCC's Motion to Lift Order of Default. It also denied PNCC's Motion for Reconsideration Ad Cautelam.
PNCC brought its case before the Court of Appeals. The Court of Appeals dismissed PNCC's appeal for raising pure questions of law
exclusively cognizable by this court. It likewise denied the reconsideration.

Issue:

Whether or not the CA correctly ruled in dismissing the appeal on the ground that it raised pure questions of law?

Held:

Yes. Section 9(3) of Batas Pambansa Blg. 129 enumerates the appellate jurisdiction of the Court of Appeals. This section includes the
proviso: "except those falling within the appellate jurisdiction of the Supreme Court[.]" The Supreme Court's appellate jurisdiction is
found in Article VIII, Section 5(2)(e) of the Constitution: SECTION 5. The Supreme Court shall have the following powers: 2)
Review, revise, reverse, modify, or affirm on appeal or certiorari, as the law or the Rules of Court may provide, final judgments
and orders of lower courts in… e) All cases in which only an error or question of law is involved.

A question of law exists when the doubt or difference arises as to what the law is on a certain state of facts, while a question of fact
exists when the doubt or difference arises as to the truth or the falsehood of alleged facts. Questions of fact require the examination of
the probative value of the parties' evidence. This Petition originated from a default judgment against petitioner. Petitioner was not able
to present evidence before the trial court. Necessarily, the errors raised from the trial court involved only questions of law.
Remedial Law I| Page 181

Maravilla vs. Rios


G.R. No. 196875. August 19, 2015

Facts:

Joseph Rios filed a criminal case against Teddy Maravilla for reckless imprudence resulting in serious physical injuries before the
MTCC of Himamaylan City, Negros Occidental. Rios accused Maravilla of recklessly driving his jeep which caused it to collide with
the motorcycle he (respondent) was then driving; as a result, respondent was injured and incapacitated to work for more than ninety
days. After trial, the MTCC rendered judgment acquitting Maravilla of the crime charged against him. However, the court finds
preponderance of evidence to hold him liable for damages. Respondent interposed an appeal before the RTC which modified the
decision of the court by deleting the award of temperate damages in the amount of P20,000.00 and finding respondent liable to pay
Rios the amount of P256,386.25 as actual and compensatory damages.

Maravilla then filed a Petition for Review with the CA which the latter dismissed for failure of the petitioner to comply with pertinent
provisions of the Rules - "xxx b. Some relevant and pertinent pleadings and documents, which are necessary for a better understanding
and resolution of the instant petition, were not attached therein, in violation of Section 2(d), Rule 42 of the Revised Rules of Court, to
wit: (i) Copy of the information filed before the municipal trial court; ii. Copy of the appellant’s brief filed before the RTC; iii. Copy
of the appellee’s brief, if any; iv. ‘other pieces of evidence/documents adduced before the lower court.

Issue:

Whether or not the petition for review under Rule 42 should have been dismissed for failure of the petitioner to attached certain
documents?

Held:

Yes. Under Section 2, Rule 42 of the 1997 Rules of Civil Procedure (1997 Rules), a petition for review shall be accompanied by,
among others, copies of the pleadings and other material portions of the record as would support the allegations of the petition.
Section 3 of the same rule states that failure of the petitioner to comply with any of the requirements regarding the contents of and the
documents which should accompany the petition shall be sufficient ground for the dismissal thereof.

In Galvez v Court of Appeals, this Court held that there are three guideposts in determining the necessity of attaching pleadings and
portions of the record to petitions under Rules 42 and 65 of the 1997 Rules, to wit: First, not all pleadings and parts of case records are
required to be attached to the petition. Only those which are relevant and pertinent must accompany it. The test of relevancy is
whether the document in question will support the material allegations in the petition, whether said document will make out a prima
facie case of grave abuse of discretion as to convince the court to give due course to the petition. Second, even if a document is
relevant and pertinent to the petition, it need not be appended if it is shown that the contents thereof can also [be] found in another
document already attached to the petition. Thus, if the material allegations in a position paper are summarized in a questioned
judgment, it will suffice that only a certified true copy of the judgment is attached. Third, a petition lacking an essential pleading or
part of the case record may still be given due course or reinstated (if earlier dismissed) upon showing that petitioner later submitted the
documents required, or that it will serve the higher interest of justice that the case be decided on the merits. The guideposts, which
equally apply to a petition for review filed in the CA under Rule 42, reflect that the significant determinant of the sufficiency of the
attached documents is whether the accompanying documents support the allegations of the petition.

In this case, while the petitioner submitted additional necessary attachments along with his Motion for Reconsideration, he left out
important parts of the record — excerpts of the transcript of stenographic notes, the respondent’s formal offer of evidence, and the
trial court’s Order admitting said formal offer of evidence — that would support his claim that the trial court erred in awarding
damages to respondent since the latter failed to testify as to his hospital expenses and identify particular exhibits.
Remedial Law I| Page 182

Rule 38: Petition for Relief from Judgment

Escueta vs. Lim


G.R. No. 137162. January 24, 2007

Facts:

Respondent Rufina Lim filed an action to remove cloud on, or quiet title to, real property, with preliminary injunction and issuance of
[a hold-departure order] from the Philippines against Ignacio E. Rubio and the heirs of Baloloy. As to petitioner Corazon Escueta, in
spite of her knowledge that the disputed lots have already been sold by Ignacio Rubio to respondent, it is alleged that a simulated deed
of sale involving said lots was effected by Ignacio Rubio in her favor; and that the simulated deed of sale by Rubio to Escueta has
raised doubts and clouds over respondent’s title. In their separate amended answers, petitioners denied the material allegations of the
complaint.

The Baloloys failed to appear at the pre-trial. Upon motion of respondent, the trial court declared the Baloloys in default. They then
filed a motion to lift the order declaring them in default, which was denied by the trial court in an order. Consequently, respondent was
allowed to adduce evidence ex parte. Thereafter, the trial court rendered a partial decision. The judgment is rendered in favor of
[respondent] and against [petitioners, heirs] of Luz R. Baloloy. Atty. Arsenio Villalon, Jr., the former counsel of record of the
Baloloys received a copy of the partial decision on April 5, 1994. On July 4, 1994, the Baloloys, through their new counsel, filed a
petition for relief from judgment which was denied by the trial court for being filed out of time.

Issue:

Whether or not the petition for relief from judgment was filed beyond the reglementary period?

Held:

Yes. The 60-day period for filing a petition for annulment of judgment is reckoned from the time the party acquired knowledge of the
order, judgment or proceedings and not from the date he actually read the same. Section 3 of Rule 38 of the Rules of Court states:

SEC 3. TIME FOR FILING PETITION; CONTENTS AND VERIFICATION - a petition provided for in either of the preceding
sections of this Rule must be verified, filed within 60 days after the petitioner learns of the judgment, final order, or other proceeding
to be set aside, and not mroe that 6 months after such judgment or final order was entered, or such proceeding was taken; and must
be accompanied with affidavits showing the fraud, accident, mistake, or excusable negligence relied upon, and the facts constituting
the petitioner's good and substantial cause of action or defense as the case may be.

There is no reason for the Baloloys to ignore the effects of the above rule. the 60-day period is reckoned from the time the party
acquired knowledge of the order, judgment, or proceedings and not from the date he actually read the same.

The evidence on record as far as this issue is concerned shows that Atty. Arsenio Villalon, Jr., the former counsel of record of the
Baloloys received a copy of the partial decision dated June 23, 1993 on April 5, 1994. At that time, said former counsel is still their
counsel of record. The reckoning of the 60-day period therefore is the date when the said counsel of record received a copy of the
partial decision which was on April 5, 1994. The petition for relief was filed by the new counsel on July 4, 1994 which means that 90
days have already lapsed or 30 days beyond the 60-day period. Moreover, the records further show that the Baloloys received the
partial decision on September 13, 1993 as evidenced by Registry return cards which bear the numbers 02597 and 02598 signed by Mr.
Alejandrino Baloloy.

The Baloloys, apparently in an attempt to cure the lapse of the aforesaid reglementary period to file a petition for relief from judgment,
included in its petition the two Orders dated May 6, 1994 and June 29, 1994. The first Order denied Baloloys’ motion to fix the period
within which plaintiffs-appellants pay the balance of the purchase price. The second Order refers to the grant of partial execution, i.e.
on the aspect of damages. These Orders are only consequences of the partial decision subject of the petition for relief, and thus, cannot
be considered in the determination of the reglementary period within which to file the said petition for relief.

Further, no fraud, accident, mistake, or excusable negligence exists in order that the petition for relief may be granted.
Remedial Law I| Page 183

Rule 47: Annulment of Judgment


Diona vs. Balangue
G.R. No. 173559, January 7, 2013
Facts:

Respondents (Romeo Balangue, et al) obtained a loan secured by a real estate mortgage from petitioner (Leticia Diona). Because they
failed to pay, the petitioner filed a complaint against them. The RTC ruled in favor of the petitioner. Thereafter petitioner filed a
motion for execution, alleging that, the respondent failed to timely appealed. The respondent on the other hand filed a Motion to Set
Aside Judgment claiming that not all of them were served with summons. However, the RTC granted the motion for execution and
imposed 5% monthly interest. Respondent filed a motion to correct/amend judgment, alleging that in their agreement with the
petitioner, the latter only demands for 12% per annum, and not 5% monthly, which the RTC granted. The petitioner filed a petition for
certiorari, which was denied by the CA.

Issue:

Whether or not the decision of the RTC granting 5% monthly interest was properly annulled under Rule 47?

Held:

Yes. The award of 5% monthly interest violated their right to due process and, hence, the same may be set aside in a Petition for
Annulment of Judgment filed under Rule 47 of the Rules of Court.

A Petition for Annulment of Judgment under Rule 47 of the Rules of Court is a remedy granted only under exceptional circumstances
where a party, without fault on his part, has failed to avail of the ordinary remedies of new trial, appeal, petition for relief or other
appropriate remedies. Said rule explicitly provides that it is not available as a substitute for a remedy which was lost due to the party’s
own neglect in promptly availing of the same. The underlying reason is traceable to the notion that annulling final judgments goes
against the grain of finality of judgment. Litigation must end and terminate sometime and somewhere, and it is essential to an effective
administration of justice that once a judgment has become final, the issue or cause involved therein should be laid to rest.

While under Section 2, Rule 47 of the Rules of Court a Petition for Annulment of Judgment may be based only on the grounds of
extrinsic fraud and lack of jurisdiction, jurisprudence recognizes lack of due process as additional ground to annul a judgment. In
Arcelona v. Court of Appeals, this Court declared that a final and executory judgment may still be set aside if, upon mere inspection
thereof, its patent nullity can be shown for having been issued without jurisdiction or for lack of due process of law.

In this case, the grant of 5% monthly interest is way beyond the 12% per annum interest sought in the Complaint and smacks of
violation of due process.
Remedial Law I| Page 184

Yuk Ling Ong vs. Co


G.R. No. 206653. February 25, 2015
Facts:

Petitioner Yuk Ling Ong, a British-Hong Kong national, and respondent Benjamin Co a Filipino citizen, were married in the
Philippines. Respondent filed a petition for declaration of nullity on the ground of psychological incapacity before the RTC.
Respondent indicated that petitioner’s address was 23 Sta. Rosa Street, Unit B-2 Manresa Garden Homes, Quezon City. The RTC
issued summons. In his Server’s Return, process server Rodolfo Torres, Jr. stated that, on August 1, 2002, substituted service of
summons with the copy of the petition was effected after several futile attempts to serve the same personally on petitioner. The said
documents were received by Mr. Roly Espinosa, a security officer. The RTC found respondent’s marriage with petitioner as void  ab
initio on the ground of psychological incapacity under Article 36 of the Family Code. It stated that summons was served on petitioner
on August 1, 2002, but she failed to file her responsive pleading within the reglementary period. The public prosecutor also stated that
there were no indicative facts to manifest collusion. Thus, the RTC concluded that petitioner was psychologically incapacitated to
perform her essential marital obligations.

Consequently, petitioner filed a petition for annulment of judgment under Rule 47 of the Rules of Court before the CA claiming that
she was never notified of the cases filed against her. She prayed that the RTC decision be nullified on the grounds of extrinsic fraud
and lack of jurisdiction. CA found the petition for annulment of judgment to be devoid of merit. It held that there was no sufficient
proof to establish that respondent employed fraud to insure petitioner’s non-participation in the trial of the aforementioned case.
Petitioner moved for reconsideration, but her motion was denied by the CA. Hence, this petition.

Issue:

Whether or not the judgment by the RTC rendered without acquiring jurisdiction over the person of the defendant can be annulled
under Rule 47?

Held:

Yes. Annulment of judgment is a recourse equitable in character, allowed only in exceptional cases as where there is no available or
other adequate remedy. Rule 47 of the 1997 Rules of Civil Procedure, as amended, governs actions for annulment of judgments or
final orders and resolutions, and Section 2 thereof explicitly provides only two grounds for annulment of judgment, that is, extrinsic
fraud and lack of jurisdiction. Annulment of judgment is an equitable principle not because it allows a party-litigant another
opportunity to reopen a judgment that has long lapsed into finality but because it enables him to be discharged from the burden of
being bound to a judgment that is an absolute nullity to begin with.

Lack of jurisdiction on the part of the trial court in rendering the judgment or final order is either lack of jurisdiction over the subject
matter or nature of the action, or lack of jurisdiction over the person of the petitioner. The former is a matter of substantive law
because statutory law defines the jurisdiction of the courts over the subject matter or nature of the action. The latter is a matter of
procedural law, for it involves the service of summons or other processes on the petitioner.

Since there was no valid service of summons upon the petitioner, the RTC’s decision must be voided for lack of jurisdiction over the
person of petitioner. The favorable judgment enjoyed by respondent cannot be categorized as a genuine victory because it was fought
against an adversary, who was ignorant of the existing dispute. Whatever prize bestowed upon the victor in such a void decision must
also be undone.
Remedial Law I| Page 185

Santos vs. Santos


G.R. No. 187061, October 08, 2014

Facts:

Ricardo Santos (husband), filed a petition for declaration of presumptive death of his wife for purposes of remarriage. Ricardo claimed
that it was almost 12 years from the date of his Regional Trial Court petition since Celerina left to work as domestic helper in
Honkong. He believed that she had passed away. His petition was granted by the RTC. Celerina Santos (wife) claimed that she learned
about Ricardo’s petition only sometime in October 2008 when she could no longer avail the remedies of new trial, appeal, petition for
relief, or other appropriate remedies. Thus she filed a petition for annulment of judgment on the ground of extrinsic fraud and lack of
jurisdiction. She argued that she was deprived her day in court when Ricardo, despite his knowledge of her true residence,
misrepresented to the court that she was a resident of Tarlac City. The Court of Appeals dismissed Celerina’s petition for being a
wrong mode of remedy. According to the CA, the proper remedy was to file a sworn statement before the civil registry, declaring her
reappearance in accordance with Article 42 of the Family Code.

Issue:

Whether or not Annulment of Judgment under Rule 47 is the proper remedy in assailing the decision of the RTC obtained by extrinsic
fraud declaring the spouse presumptively dead?

Held:

Yes. The grounds for annulment of judgment are extrinsic fraud and lack of jurisdiction. For fraud to become a basis for annulment of
judgment, it has to be extrinsic or actual. It is intrinsic when the fraudulent acts pertain to an issue involved in the original action or
where the acts constituting the fraud were or could have been litigated, it is extrinsic or collateral when a litigant commits acts outside
of the trial which prevents a party from having a real contest, or from presenting all of his case, such that there is no fair submission
of the controversy.

The present petition was filed within the four-year period allowed by law in case of extrinsic fraud, and before the action is barred by
laches, which is the period allowed in case of lack of jurisdiction. There was also no other sufficient remedy available to Celerina at
the time of her discovery of the fraud perpetrated on her. The choice of remedy is important because remedies carry with them certain
admissions, presumptions, and conditions.

The Family Code provides that it is the proof of absence of a spouse for four consecutive years, coupled with a well-founded belief by
the present spouse that the absent spouse is already dead, that constitutes a justification for a second marriage during the subsistence of
another marriage. The Family Code also provides that the second marriage is in danger of being terminated by the presumptively dead
spouse when he or she reappears.

The provision on reappearance in the Family Code as a remedy to effect the termination of the subsequent marriage does not preclude
the spouse who was declared presumptively dead from availing other remedies existing in law. This court had, in fact, recognized that
a subsequent marriage may also be terminated by filing “an action in court to prove the reappearance of the absentee and obtain a
declaration of dissolution or termination of the subsequent marriage.

Celerina does not admit to have been absent. She also seeks not merely the termination of the subsequent marriage but also the
nullification of its effects. She contends that reappearance is not a sufficient remedy because it will only terminate the subsequent
marriage but not nullify the effects of the declaration of her presumptive death and the subsequent marriage.

For the purpose of not only terminating the subsequent marriage but also of nullifying the effects of the declaration of presumptive
death and the subsequent marriage, mere filing of an affidavit of reappearance would not suffice. Celerina’s choice to file an action for
annulment of judgment will, therefore, lie.
Remedial Law I| Page 186

Lasala vs. NFA


G.R. No. 171582, August 19, 2015.

Facts:

Alberto Lasala, through his company PSF Security Agency, used to provide security guard services to the National Food Authority.
Lasala's employees who were deployed to the NFA filed with the NLRC a complaint for underpayment of wages and nonpayment of
other monetary benefits. The NLRC ruled for the employees and held Lasala and the NFA solidarily liable for the employees'
adjudged monetary award. Consequently, the sheriff garnished the NFA's P383,572.90 worth of bank deposits with the Development
Bank of the Philippines.

Believing that it had no liability to Lasala's employees, the NFA filed with the RTC, Branch 220, Quezon City, a complaint for sum of
money with damages and an application for the issuance of a writ of preliminary attachment against Lasala. In response, Lasala filed
an answer with counterclaim and opposition to the prayer for preliminary attachment. Lasala also filed a counterclaim against NFA.

Initially, the trial court granted the NFA's prayer for the issuance of a writ of preliminary attachment. However, this writ was
eventually nullified when Lasala questioned it with the Court of Appeals. The trial court  dismissed the NFA's complaint for failure of
the lawyer deputized by the OGCC, Atty. Mendoza, to present the NFA's evidence-in-chief, due to his repeated hearing absences. The
NFA replaced Atty. Mendoza and administratively charged him with dishonesty, grave misconduct, conduct grossly prejudicial to the
best interests of the service, and gross neglect of duty. It subsequently employed Atty. Atty. Cahucom as its new counsel.

Although the NFA's complaint was dismissed, Lasala's counterclaim remained, and he presented evidence to support it.
Interestingly, Atty. Cahucom, the NFA's new counsel, did not submit any evidence to controvert Lasala's counterclaim
evidence. When asked during trial, Atty. Cahucom simply waived his right to cross-examine Lasala and did not exert any effort to
counter his testimony.

Despite the huge award to Lasala, the NFA failed to appeal its case to the CA. Atty. Cahucom did not inform the NFA's management
about the trial court's adverse ruling. When asked to explain, he reasoned out that he only discovered the decision after the lapse of the
period for appeal.

Having lost its chance to appeal, the NFA filed with the trial court a petition for relief from judgment ( petition for relief) grounded on
excusable negligence. In its petition, the NFA through Atty. Cahucom, attributed its failure to appeal to one of the NFA's employees.
Allegedly, this employee received the copy of the trial court's September 2, 2002 decision but did not inform Atty. Cahucom about it.
It was only after the lapse of the period for the filing of a motion for reconsideration and an appeal that the NFA learned about the
adverse ruling. The trial court denied the petition.

In the meantime, then NFA Administrator Arthur C. Yap had assumed his position. One of his first instructions was the legal audit of
all NFA cases. In doing this, the NFA management found out that the two lawyers (Attys. Mendoza and Cahucom) assigned to the
case against Lasala, grossly mishandled it; hence, causing a huge and unjust liability to the NFA in the amount of P52,788,970.50.

Thus, on the grounds of lack of jurisdiction and extrinsic fraud, the NFA, now through the OGCC, filed with the CA a petition and an
amended petition for annulment of judgment of the trial court's September 2, 2002 decision which had granted a substantially higher
award than what Lasala originally prayed for in his counterclaim. The CA granted the petition and annulled the trial court's September
2, 2002 decision holding that the RTC committed grave abuse of discretion. Hence, this petition.

Issue:

Whether or not the negligence of NFA’s counsels constitutes valid ground to annul the decision of the trial court?

Held:

Yes. The actions of Attys. Mendoza and Cahucom, under the unique circumstances of this case, amount to extrinsic fraud that
warrants the grant of NFA's petition for relief from judgment. The party in the present case, the NFA, is a government agency that
could rightly rely solely on its legal officers to vigilantly protect its interests. The NFA's lawyers were not only its counsel, they were
its employees tasked to advance the agency's legal interests. NFA's lawyers acted negligently several times in handling the case that it
appears deliberate on their part.
Remedial Law I| Page 187

First, Atty. Mendoza caused the dismissal of the NFA's complaint against Lasala by negligently and repeatedly failing to attend the
hearing for the presentation of the NFA's evidence-in-chief. Consequently, the NFA lost its chance to recover from Lasala the
employee benefits that it allegedly shouldered as indirect employer. Atty. Mendoza never bothered to provide any valid excuse for this
crucial omission on his part.

For these failures, Atty. Mendoza merely explained that the NFA's copy of the adverse decision was lost and was only found after the
lapse of the period for appeal. Under these circumstances, the NFA was forced to file an administrative complaint against Atty.
Mendoza for his string of negligent acts.

Atty. Cahucom, Atty. Mendoza's successor in handling the case, notably did not cross-examine Lasala's witnesses, and did not present
controverting evidence to disprove and counter Lasala's counterclaim. Atty. Cahucom further prejudiced the NFA when he likewise
failed to file a motion for reconsideration or an appeal from the trial court's September 2, 2002 decision, where Lasala was awarded
the huge amount of P52,788,970.50, without any convincing evidence to support it. When asked to justify his failure, Atty. Cahucom,
like Atty. Mendoza, merely mentioned that the NFA's copy of the decision was lost and that he only discovered it when the period for
appeal had already lapsed. The trial court's adverse decision, of course, could have been avoided or the award minimized, if Atty.
Cahucom did not waive the NFA's right to present its controverting evidence against Lasala's counterclaim evidence. Strangely, when
asked during hearing, Atty. Cahucom refused to refute Lasala's testimony and instead simply moved for the filing of a memorandum.

The actions of these lawyers, that at the very least could be equated with unreasonable disregard for the case they were handling and
with obvious indifference towards the NFA's plight, lead to the conclusion that Attys. Mendoza's and Cahucom's actions amounted to
a concerted action with Lasala when the latter secured the trial court's huge and baseless counterclaim award.  By this fraudulent
scheme, the NFA was prevented from making a fair submission in the controversy.
Another issue:

Did the CA err in granting the petition for annulment of judgment based on RTC’s grave abuse of discretion?

Yes. The court ruled that the CA violated the restrictive application of a petition for annulment; only extrinsic fraud and/or lack of
jurisdiction may annul a final judgment. By seeking to include acts committed with grave abuse of discretion, the CA's ruling enlarged
the concept of lack of jurisdiction as a ground for annulment. In a petition for annulment based on lack of jurisdiction, the petitioner
cannot rely on jurisdictional defect due to grave abuse of discretion, but on absolute lack of jurisdiction. As we have already held, the
concept of lack of jurisdiction as a ground to annul a judgment does not embrace grave abuse of discretion amounting to lack or
excess of jurisdiction.
Remedial Law I| Page 188

Mangubat vs. Morga-Seva,


G.R. No. 202611. November 23, 2015

Facts:

Gaudencio Mangubat and his wife Aurelia Rellora-Mangubat filed with the Regional Trial Court of Pili, Camarines Sur a Complaint
for Specific Performance with Damages against respondent Belen Morga-Seva Belen and two other defendants. The RTC ruled in
favor of the Mangubats. The case went up to the Supreme Court which affirmed the RTC decision. That decision became final and
executory.

Gaudencio and his children as heirs of the deceased Aurelia filed with the same court a Complaint for Revival of the Decision. They
averred that the writ of execution could not be implemented because Belen and her co-defendants evaded service thereof. And since
five years had already lapsed from the date of its entry, Gaudencio and the heirs prayed for the revival of the RTC Decision.
Gaudencio, assisted by Atty. Reynaldo L. Herrera (Atty. Herrera) and Belen by Atty. Junnel M. Relativo, entered into a Compromise
Agreement. The RTC approved  the agreement and on February 23, 2001 rendered a Decision in accordance therewith. Upon its
finality, the Writ of Execution was ordered issued by the said court.

Belen handed to Atty. Herrera her payment of P91,280.00 in accordance with the Compromise Agreement. Abner, however,
manifested that as far as he is concerned, Belen has not yet made any payment to the heirs as he was not notified by Atty. Herrera of
the same. Thus, Atty. Herrera reported to the court that out of the P91,280.00 handed to him by Belen, he had turned-over the amount
of P84,480.00 to the Clerk of Court and retained 6,800.00 as his attorney's fee. This was duly noted by the RTC.

Subsequently and purportedly in behalf of all the heirs, Abner, through Atty. Vista-Gumba, filed a Motion to Declare the Amicable
Settlement Null and Void. It was alleged therein that Gaudencio acted only on his own behalf when he entered into the compromise
agreement with Belen, hence, the same is null and void for want of consent and participation of the heirs who were indispensable
parties.

RTC ruled on the Motion to Declare the Amicable Settlement Null and Void. Again purportedly on behalf of all the heirs, Abner
moved for the reconsideration but was denied by the RTC. When the same became final, Belen filed a Motion for Execution of
Specific Acts wherein she once more prayed that Abner be ordered to surrender to the RTC the owner's copy of TCT No. 6337 and the
Clerk of Court to execute in her favor and on behalf of the heirs a deed of sale involving the lot covered by the said title. This was
granted by the RTC in an Order dated July 14, 2006. Still, Abner refused to comply. The afore-mentioned order became final on
November 19, 2006. Pursuant thereto, the RTC directed the Registrar of Deeds of Camarines Sur to transfer title to the property under
TCT No. 6337 to Belen.

Abner filed a Petition for Annulment of Final Order with the CA. He contended that under the Compromise Agreement, Belen was
supposed to make her payment on or before June 30, 2001. However, the same was made only on December 18, 2003 or way beyond
the period agreed upon. Moreover, Abner argued that since the February 23, 2001 RTC Decision approving the Compromise
Agreement had long become final and executory, the RTC had already lost its jurisdiction over the case when it issued the
September 25, 2006 Order.

Issue:

Whether or not the petition for annulment of judgment may be granted on the ground that when the RTC issued the assailed Order, it
already lost its jurisdiction since the Order had long become final and executory?

Held:

No. It must be stressed that the remedy of annulment of judgment is only available under certain exceptional circumstances as this is
adverse to the concept of immutability of final judgments. Hence, it is allowed only on two grounds, i.e., extrinsic fraud and lack of
jurisdiction.

Abner anchors his Petition for Annulment of Final Order on lack of jurisdiction. He posits that the RTC had lost jurisdiction over the
case when its February 23, 2001 Decision became final, hence, any issuance subsequent thereto is made without any jurisdiction. The
argument is, however, specious. "Lack of jurisdiction on the part of the trial court in rendering the judgment or final order is either
lack of jurisdiction over the subject matter or nature of the action, or lack of jurisdiction over the person of the petitioner."  Here, it is
Remedial Law I| Page 189

undisputed that the RTC acquired jurisdiction over the person of Abner, he having asked for affirmative relief therefrom several
times. As mentioned, what Abner questions is the RTC's jurisdiction over the case. 

"In a petition for annulment of judgment based on lack of jurisdiction, petitioner must show not merely an abuse of
jurisdictional discretion but an absolute lack of jurisdiction. Lack of jurisdiction means absence of or no jurisdiction, that is,
the court should not have taken cognizance of the petition because the law does not vest it with jurisdiction over the subject
matter. Jurisdiction over the nature of the action or subject matter is conferred by law."

The RTC's jurisdiction over petitions for revival of judgment had already been upheld by the Court. It was held that "[a]n action for
revival of judgment may be filed either 'in the same court where said judgment was rendered or in the place where the plaintiff or
defendant resides, or in any other place designated by the statutes which treat of the venue of actions in general.'" Here, the Complaint
for revival of judgment was filed in the same court which rendered the August 27, 1985 Decision in Civil Case No. P-279.
Undoubtedly, the RTC has jurisdiction over the action. There is therefore no valid ground for the Petition for Annulment of Final
Order that Abner filed with the CA.
Remedial Law I| Page 190

Sibal vs. Buquel


G.R. No. 197825. January 11, 2016

Facts:

Respondents Pedro Buquel, Santiago Buquel, Jr., Rosalinda Buquel and Francisco Buquel inherited from their parents a parcel of land
consisting of 81, 022 sq.m.  Sometime in January 1999, petitioner Camilo Sibal and Tobi Mangoba took possession of a portion of the
property which belonged to Santiago, Sr.  Thereafter, the Buquels made several demands against Sibal and Mangoba for them to
vacate and turn over the property, but the latter refused to do so. Hence, they filed a complaint before the Tuguegarao RTC for
recovery of possession and damages. The RTC ruled in favor of the Buquels and ordered among others, the restoration to them of their
peaceful possession of the land in question, specifically on the share of Santiago Buquel; Jr. This judgment has become final and
executory. Consequently, Sibal filed a Petition for Annulment of the RTC Decision before the CA, where he raised lack of jurisdiction
and that the Buquels were guilty of extrinsic fraud. Sibal asserts that the negligence of his former counsel in handling his defense
during the proceedings in Civil Case No. 6429 resulted in violation of his right to due process. The CA dismissed petition for lack of
merit as well as the Motion for Reconsideration, hence, this petition.

Issue:

Whether or not the decision of the RTC should be annulled because of the negligence of the petitioner’s counsel?

Held:

No. It must be emphasized that not every kind of fraud justifies the action of annulment of judgment. Only extrinsic fraud does. Fraud
is extrinsic when the unsuccessful party has been prevented from fully exhibiting his case, by fraud or deception practiced on him by
his opponent, as by keeping him away from court, a false promise of a compromise; or where the defendant never had knowledge of
the suit, being kept in ignorance by the acts of the plaintiff; or where an attorney fraudulently or without authority connives at his
defeat; these and similar cases which show that there has never been a real contest in the trial or hearing of the case are reasons for
which a new suit may be sustained to set aside and annul the former judgment and open the case for a new and fair hearing. As a
ground for annulment of judgment, extrinsic fraud must arise from an act of the adverse party, and the fraud must be of such nature as
to have deprived the petitioner of its clay in court. The fraud is not extrinsic if the act was committed by the petitioner's own counsel.

Sibal asserts that the negligence of his former counsel in handling his defense during the proceedings in Civil Case No. 6429 resulted
in violation of his right to due process. He claims that his counsel's inexcusable negligence denied him of his clay in court. However,
he admitted that he attended only one stage of the proceedings below, which was the preliminary conference. He was not aware of the
subsequent proceedings as he was totally dependent on his former counsel and would merely wait for the latter to notify him if his
attendance would be required. There was likewise no indication that his counsel was in fact in cahoots with the Buquels to obtain the
assailed judgment. Sibal must therefore bear the unfortunate consequences of his actions. As a litigant, he should not have entirely left
the case in his counsel's hands, for he had the continuing duty to keep himself abreast of the developments, if only to protect his own
interest in the litigation. He could have discharged said duty by keeping in regular touch with his counsel, but he failed to do so.
Remedial Law I| Page 191

Rule 39: Execution and Satisfaction of Judgments

A. Execution Pending Appeal

Bañez vs. Bañez


G.R. No. 132592. January 23, 2002

Facts:

The RTC of Cebu decreed the legal separation between petitioner Aida Bañez and respondent Gabriel Bañez on the ground of the
latter’s sexual infidelity; the dissolution of their conjugal property relations and the division of the net conjugal assets; the forfeiture of
respondent’s one-half share in the net conjugal assets in favor of the common children; the payment to petitioner’s counsel of the sum
of ₱100,000 as attorney’s fees to be taken from petitioner’s share in the net assets; and the surrender by respondent of the use and
possession of a Mazda motor vehicle and the smaller residential house located at Maria Luisa Estate Park Subdivision to petitioner and
the common children within 15 days from receipt of the decision. While Gabriel filed a Notice of Appeal, Aida filed a motion for
execution pending appeal. The trial court only gave due course to petitioner’s motion for execution pending appeal where it ordered
the respondent (1) to vacate the premises of the small residential house situated in Maria Luisa Estate Park Subdivision, Lahug, Cebu
City and for (2) respondent to surrender the use and possession of the Mazda motor vehicle together with its keys and accessories
thereof to petitioner.

In turn, in a petition for certiorari, Gabriel elevated the case to the Court of Appeals. Petitioner Aida focused her claimed on the
residential house by asserting that justice requires that she and her children be allowed to occupy and enjoy the house considering that
during the entire proceedings before the trial court, she did not have the chance to occupy it. However, respondent Gabriel in his
comment, denied petitioner’s allegation that she did not have the chance to occupy the residential house. He averred that she could
have, had she chosen to. According to him, as the inventory of the couple’s properties showed, petitioner owned two houses and lots
and two motor vehicles in the United States, where she is a permanent resident. Respondent contended that there was no compelling
reason for petitioner to have the judgment executed pending appeal. The CA set aside the RTC’s decision granting the motion for
execution pending appeal, hence, this petition.

Issue:

Whether or not the execution of judgment pending appeal was justified?

Held:

No. Execution pending appeal is allowed when superior circumstances demanding urgency outweigh the damages that may result
from the issuance of the writ. Otherwise, instead of being an instrument of solicitude and justice, the writ may well become a tool of
oppression and inequity. In this case, considering the reasons cited by petitioner, there is no superior or urgent circumstance that
outweighs the damage which respondent would suffer if he were ordered to vacate the house. Petitioner did not refute respondent’s
allegations that she did not intend to use said house, and that she has two (2) other houses in the United States where she is a
permanent resident, while he had none at all. Merely putting up a bond is not sufficient reason to justify her plea for execution pending
appeal. To do so would make execution routinary, the rule rather than the exception.
Remedial Law I| Page 192

Santos vs. COMELEC


G.R. No. 155618. March 26, 2003

Facts:

Petitioner Edgar Santos and respondent Pedro Panulaya were both candidates for Mayor of the Municipality of Balingoan, Misamis
Oriental in the May 2001 elections. After the votes were counted and canvassed, the Municipal Board of Canvassers proclaimed
Panulaya as the duly elected Mayor. Santos filed an election protest before the Regional Trial Court. After trial and revision of the
ballots, the trial court found that Santos garnered 2,181 votes while Panulaya received only 2,105. Thus, the trial court proclaimed and
declared Santos as the duly elected Municipal Mayor. Santos thereafter filed a motion for execution pending appeal. Meanwhile,
before the trial court could act on Santos’ motion, Panulaya filed with the COMELEC a petition for certiorari, docketed as SPR No.
20-2002, assailing the decision of the trial court. Likewise Panulaya appealed the trial court’s decision to the COMELEC, where it
was docketed as EAC No. A-12-2002.

The COMELEC, in SPR No. 20-2002, issued a Writ of Preliminary Injunction, which effectively enjoined the trial court from acting
on Santos’ motion for execution pending appeal. Subsequently, the COMELEC dismissed SPR No. 20-2002. Hence, it directed the
trial court to dispose of all pending incidents in SPL Election Protest No. 1-M(2001) with dispatch. Thereafter, the trial court issued an
Order which upholds and approves the Motion for Execution Pending Appeal. After Santos posted the required bond, the trial court
issued the Writ of Execution, thereby installing Santos as Municipal Mayor of Balingoan, Misamis Oriental. Accordingly, Santos took
his oath of office and thereafter assumed the duties and functions of his office.

Later, Panulaya filed with the COMELEC a motion for reconsideration of the dismissal of his petition in SPR No. 20-2002, and later
on filed a supplemental petition. Panulaya also filed another petition with the COMELEC docketed SPR No. 37-2002. On September
2002, the COMELEC issued the assailed Order directing the parties to maintain the status quo ante and enjoining Santos from
assuming the functions of Mayor. Santos filed a motion for reconsideration of the above Order. However, the COMELEC First
Division did not refer the said motion to the COMELEC En Banc. Hence, Santos brought the instant special civil action for certiorari
with the Supreme Court. On October 2002, the COMELEC issued a Resolution in SPR No. 37-2002 granting the petition, setting aside
the trial court’s Order granting Santos’ Motion for Execution pending Appeal as well the Writ of Execution and ordering that Santos is
enjoined from assuming the function of mayor of Balingoan, Misamis Oriental until the final determination of the election appeal
case.

Issue:

Whether or not Santos’ motion for execution pending appeal should be allowed?

Held:

Yes. The grant of execution pending appeal was well within the discretionary powers of the trial court. A valid exercise of the
discretion to allow execution pending appeal requires that it should be based upon good reasons to be stated in a special order.  The
following constitute good reasons and a combination of two or more of them will suffice to grant execution pending appeal: (1.) public
interest involved or will of the electorate; (2.) the shortness of the remaining portion of the term of the contested office; and (3.) the
length of time that the election contest has been pending.

The decision of the trial court in Election Protest No. 1-M(2001) was rendered on April 2, 2002, or after almost one year of trial and
revision of the questioned ballots. It found petitioner as the candidate with the plurality of votes. Respondent appealed the said
decision to the COMELEC. In the meantime, the three-year term of the Office of the Mayor continued to run. The will of the
electorate, as determined by the trial court in the election protest, had to be respected and given meaning. The Municipality of
Balingoan, Misamis Oriental, needed the services of a mayor even while the election protest was pending, and it had to be the
candidate judicially determined to have been chosen by the people.
Remedial Law I| Page 193

City of Iligan vs. Principal Management Group, Inc.


G.R. No. 145260. July 31, 2003
Facts:

The City of Iligan represented by Mayor Quijano entered into a Memorandum of Agreement with Land Bank Realty Development
Corporation (LBRDC) as General Contractor and Principal Management Group, Inc. (PMGI) as Developer - Financing Manager for
the construction of a Sports Complex which upon completion and payment shall be turned over to the City of Iligan. However, the
construction was stopped because of the refusal of the occupants of the property to vacate stating that they have not received their
disturbance compensation yet. By then, PMGI has already accomplished 78.27% of the contracted project equivalent to
₱10,957,800.00 of the total project cost of ₱14,000,000.00. Thereafter, PMGI requested from the City of Iligan the payment
equivalent for such percentage of completion but the City refused stating that what was constructed was only 52.89% or equivalent
only to ₱6,958,861.59 based on the Accomplishment Report as of February 9, 1999 and that the agreement was to pay once the project
has been completed. Since it was not yet completed, they do not have to pay yet.

PMGI filed a complaint against the City of Iligan for rescission of the MOA and damages. After the filing of the City of Iligan’s
Answer, PMGI filed a Motion for Partial Summary Judgment claiming that there was no genuine issue as to the fact of the obligation
of the City of Iligan since it admitted the accomplishment of 52.89% or equivalent to ₱6,958,861.59 of PMGI and that the City of
Iligan had not specifically denied under oath the genuineness of the Letter of Credit and Memorandum of Agreement.

This Motion was granted by the trial court. The City’s MR being denied, it filed a Notice of Appeal. On the other hand, PMGI filed a
Motion for Execution Pending Appeal alleging that the appeal was clearly dilatory and therefore an order for execution may be given
by the Court in its discretion. The motion for execution pending appeal was granted by the trial court stating that its adjudication is
based on the City’s own admission and hence, it would only serve to delay execution of the final order subject of the instant motion.
This was upheld by the CA and thus, this Petition for Review on Certiorari under Rule 45 was filed by the City of Iligan.

Issue:

Whether or not the issuance of the writ of execution pending appeal was proper?

Held:

Yes. There are three requisites for the execution of a judgment pending appeal: a) a motion must be filed by the prevailing party with
notice to the adverse party; b) there must be good reasons for execution pending appeal; and c) the good reasons must be stated in a
special order. Normally, execution cannot be obtained until and unless (a) the judgment has become final and executory; (b) the right
of appeal has been renounced or waived; (c) the period for appeal has lapsed without an appeal having been filed; or (d) having been
filed, the appeal has been resolved and the records of the case have been returned to the court of origin -- in which case, execution
shall issue as a matter of right. On the other hand, when the period of appeal has not yet expired, the execution of a judgment should
not be allowed except if, in the court’s discretion, there are good reasons therefor.

Since the execution of a judgment pending appeal is an exception to the general rule, the existence of "good reasons" is essential.
These reasons must be stated in a special order, because unless these are divulged, it will be difficult to determine on appeal whether
judicial discretion has been properly exercised by the lower court.

Normally, the trial court is not allowed to assess its own judgment and to hold that an appeal may not prosper, or that it would merely
be dilatory. In the present case, however, there are circumstances that undisputedly serve as cogent bases for arriving at such a
conclusion.

First, it is not seriously disputed that the judgment is anchored upon material facts as follows: (1) there is a Memorandum of
Agreement (MOA) for the site development of Sports Complex Project No. 1 signed by the parties; (2) petitioner failed to pay the
occupants of the project site on time, thereby preventing respondent from fully complying with its obligation under the MOA; (3)
respondent admitted that the work accomplished was 52.89 percent, which was equivalent to ₱6,958,861.59. Obviously, there is no
genuine issue as to any material fact on this point.

Second, Article 1191 of the Civil Code states that:

"The power to rescind obligations is implied in reciprocal ones, in case one of the obligors should not comply with what is incumbent
upon him.
Remedial Law I| Page 194

By failing to pay the occupants of the project site within the time required for the completion of the project, petitioner did not comply
with what was incumbent upon it. Applying the law to the undisputed facts, the trial court had prima facie bases for rendering its
partial summary judgment holding that respondent was entitled to rescission and to the payment of ₱6,958,861.59.
Remedial Law I| Page 195

Far East Bank and Trust Co. vs. Toh, Sr.


G.R. No. 144018, June 23, 2003
Facts:

Tomas Toh, Sr. filed a Complaint against Far East Bank and Trust Co. (FEBTC) seeking recovery of his bank deposits in the amount
of P2,560,644.68 plus damages. FEBTC allegedly debited the said amount from Toh’s account without his consent and knowledge
and applied the same as payment for the Letters of Credit availed of by Catmon Sales International Corporation (CASICO). Toh Sr.
filed a Motion for Summary Judgment which the Regional Trial Court (RTC) granted. Toh Sr. then filed a Motion for Discretionary
Execution by invoking Section 2 of Rule 39 on the ground of old age (79 years old) and the probability that he may not be able to
enjoy his money deposited in FEBTC. While this Motion was pending, FEBTC filed a Notice of Appeal. The RTC granted the Motion
for Discretionary Execution. FEBTC then filed a special civil action for certiorari with the Court of Appeals (CA). The CA affirmed
the RTC.

Issue:

Whether or not old age constitutes a good reason to allow execution pending appeal?

Held:

Yes. Discretionary execution is permissible only when “good reasons” exist for immediately executing the judgment before finality or
pending appeal or even before the expiration of the time to appeal. “Good reasons” are compelling circumstances justifying the
immediate execution lest judgment becomes illusory, or the prevailing party may, after the lapse of time, become unable to enjoy it,
considering the tactics of the adverse party who may apparently have no case except to delay. The Rules of Court does not state,
enumerate, or give examples of “good reasons” to justify execution. The determination of what is a good reason must, necessarily, be
addressed to the sound discretion of the trial court. In other words, the issuance of the writ of execution must necessarily be controlled
by the judgment of the judge in accordance with his own conscience and by a sense of justice and equity, free from the control of
another’s judgment or conscience. It must be so for discretion implies the absence of a hard and fast rule. In this case, the trial court
granted Toh’s motion for discretionary execution due to his advanced age.

Toh is already 79 years old. It cannot, by any stretch of imagination, be denied that he is already of advanced age. Not a few might be
fortunate to live beyond 79 years. But no one could claim with certainty that his tribe would be always blessed with long life. It
concluded that old age is a “good reason” to allow execution pending appeal as any delay in the final disposition of the present case
may deny private respondent of his right to enjoy fully the money he has with FEBTC.

Here, Toh obtained a favorable judgment in the trial court. But that judgment in Civil Case No. MC-99-643 is still on appeal before
the Court of Appeals. It might even reach this Court before the controversy is finally resolved with finality. As well said in Borja,
“while we may not agree that a man of his years is practically moribund, the Court can appreciate his apprehension that he will not be
long for this world and may not enjoy the fruit of the judgment before he finally passes away.
Remedial Law I| Page 196

Stronghold Insurance Company, Inc. vs. Felix


G.R. No. 148090, November 28, 2006
Facts:

Emerita Garon filed an action for sum of money against Project Movers Realty & Development Corporation and Stronghold Insurance
Company. RTC rendered a summary judgment in favor of Garon holding Project Movers and Stronghold solidarily liable for the
obligation. Garon then filed a Motion for execution pending appeal on the ground of illness of her husband. RTC granted Garon’s
Motion and it ordered Garon to post a bond of P20 million to answer for any damage that Project Movers and Stronghold Insurance
may sustain by reason of the execution pending appeal. Stronghold Insurance filed a petition for certiorari before the Court of Appeals
to assail the RTC’s Order granting the Motion for Execution Pending Appeal. The CA affirmed the RTC’s Order and held that while it
was not Garon who was ill, Garon needed the money to support her husband’s medical expenses and to support her family.

Issue:

Whether or not the illness of Garon’s husband constitute as a good reason to allow execution pending appeal?

Held:

No. Execution pending appeal is an exception to the general rule. Execution pending appeal is an extraordinary remedy, being more of
the exception rather than the rule. This rule is strictly construed against the movant because courts look with disfavor upon any
attempt to execute a judgment which has not acquired finality. Such execution affects the rights of the parties which are yet to be
ascertained on appeal.

The requisites for the grant of an execution of a judgment pending appeal are the following:
(a) there must be a motion by the prevailing party with notice to the adverse party;
(b) there must be good reasons for execution pending appeal;
(c) the good reasons must be stated in the special order.

As a discretionary execution, execution pending appeal is permissible only when good reasons exist for immediately executing the
judgment before finality or pending appeal or even before the expiration of the period to appeal. Good reasons, special, important,
pressing reasons must exist to justify execution pending appeal; otherwise, instead of an instrument of solicitude and justice, it may
well become a tool of oppression and inequality. Good reasons consist of exceptional circumstances of such urgency as to outweigh
the injury or damage that the losing party may suffer should the appealed judgment be reversed later.

In this case, Garon anchors the motion for execution pending appeal on the the ill health of her spouse and the spouses’ urgent need
for the funds owed to them by Project Movers and Stronghold Insurance constitute good reasons for execution pending appeal; and
Garon is ready and willing to post a bond to answer for any damage Project Movers and Stronghold Insurance may suffer should the
trial court’s decision be reversed on appeal. The posting of a bond, standing alone and absent the good reasons required under Section
2, Rule 39 of the Rules, is not enough to allow execution pending appeal. The mere filing of a bond by a successful party is not a good
reason to justify execution pending appeal as a combination of circumstances is the dominant consideration which impels the grant of
immediate execution. The bond is only an additional factor for the protection of the defendant’s creditor.

The exercise of the power to grant or deny a motion for execution pending appeal is addressed to the sound discretion of the trial
court. However, the existence of good reasons is indispensable to the grant of execution pending appeal. Here, Garon failed to advance
good reasons that would justify the execution pending appeal. It was not Garon, but her husband, who was ill.
Remedial Law I| Page 197

B. Supervening Events

Villaruel, Jr. vs. Fernando


G.R. No. 136726. September 24, 2003

Facts:

Panfilo Villaruel was a former Assistant Secretary of the Air Transportation Office (ATO), Department of Transportation and
Communication (DOTC). Reynaldo Fernando, Modesto Abarca and Marilou Cleofas, were Chief, Chief Administrative Assistant, and
Administrative Assistant, respectively, of the Civil Aviation Training Center (CATC) which is an adjunct office of the ATO. The
controversy started when Villaruel issued a memorandum addressed to Fernando et.al., detailing them under DOTC USec. Primitivo
Cal. They then requested for the reconsideration of such order but complied with the order. Without acting on such request, Villaruel
placed Abarca under preventive suspension 90 days without pay pending investigation for alleged grave misconduct. After the lapse of
90 days, respondents requested from the DOTC Secretary Garcia to lift the order. They also sought the intervention of the
Ombudsman in their case. The Ombudsman then inquired with Sec. Garcia to which he replied that he had issued a memorandum
directing Villaruel to recall respondents to their mother unit. This went unheeded by Villaruel. Thereafter, a Petition for Mandamus
and Damages with Prayer for Preliminary Mandatory Injunction against Villaruel was filed by respondents with the RTC of Pasay
City. The trial court granted the prayer for preliminary mandatory injunction by respondents. For the continuous defiance by Villaruel
of the order, he was cited in indirect contempt by the RTC. The trial court declared Villaruel in default for his failure to file an answer
and accordingly, respondents was allowed to adduce evidence ex-parte. The trial court rendered a judgment declaring the mandamus
permanent and ordering Villaruel to pay damages to respondents. Villaruel, through the OSG filed an appeal before the CA. They
were granted a period of until Dec. 13, 1996, non-extendible, within which to file his memorandum. However, the OSG failed to file
the memorandum. Subsequently, Solicitor Restituto Tuando, Jr. who was handling the case was appointed Regional Trial Court judge
of Dumaguete City. The case was re-assigned to Assistant Solicitor Luciano Joson, Jr. The Court of Appeals issued a Resolution
dismissing petitioners appeal for failure to file the required memorandum. The Resolution became final and executory on 14 June
1997. Consequently, the respondents filed a Motion for Execution with the trial court. Although served a copy of the motion for
execution, the OSG did not file any opposition. Acting on the motion for execution, the trial court issued a Writ of Execution. The
Sheriff issued a Notice of Sheriffs Sale setting on 23 February 1998 the sale of petitioner’s real property.

Villaruel, through his new counsel, filed a Motion to Quash the Writ of Execution and to Suspend Sheriff’s Sale.  In his motion,
petitioner alleged that the trial courts decision never became final and executory as the trial court deprived him of his right to due
process. He claims that the OSG failed to file his memorandum and that he was not informed of the dismissal of the appeal and the
granting of the motion for execution. He further asserts that the Ombudsman’s resolution finding Abarca guilty of violation RA 6713
superseded the decision of the trial court. The CA dismissed his special civil action for certiorari. Hence, this appeal.

Issue:

Whether or not the decision of the Ombudsman finding Abarca guilty of violation RA 6713 superseded the decision of the trial court?

Held:

No. Settled is the rule that a judgment that has acquired finality becomes immutable and unalterable and may no longer be modified in
any respect except only to correct clerical errors or mistakes. True, this rule admits of certain exceptions. One of these exceptions is
whenever circumstances transpire after the finality of the decision rendering its execution unjust and inequitable. This, however, is not
the case here. In the present case, the Ombudsman issued his Resolution prior to the finality of the trial courts decision. The
Ombudsman issued his Resolution on 22 January 1997 while the trial courts decision became final and executory on 14 June
1997. Therefore, the resolution of the Ombudsman is not a supervening event to warrant the stay of the execution of the decision of
the trial court.

The action filed by the petitioner before the Ombudsman is completely different from the action instituted by respondents before the
trial court. The two actions, which are clearly separate and distinct from each other, presented two different causes of
action. Petitioner’s cause of action arose from respondents alleged violation of certain provisions of RA 6713 whereas respondents’
cause of action resulted from petitioner’s refusal to recall respondents to their mother unit at CATC.  In the administrative case before
the Ombudsman, the issue was whether respondents were guilty of violating RA 6713. The findings of the Ombudsman did not render
the execution of the trial courts decision unjust and inequitable. The resolution of the Ombudsman finding Abarca guilty of violating
Section 7(d) of RA 6713 did not state that petitioner had a valid reason to detail respondents to the Office of Undersecretary Cal.
Remedial Law I| Page 198

Morta vs. Bagagñan


A.M. No. MTJ-03-1513. November 12, 2003

Facts:

Spouses Morta filed an administrative complaint against Judge Bagagan and Sheriff Matias charging them of gross ignorance of the
law, incompetence, bias and delay and gross ignorance of the law, negligence and connivance with the defendants respectively. The
case stemmed from an action for Damages with Prayer for a Writ of Preliminary Injunction, filed by the Spouses against a certain
Jaime Occidental. They allege that the MTC of Guinobatan, Albay rendered a decision in their favor and this was appealed before the
RTC which dismissed the action stating that it was under the original and exclusive jurisdiction of DARAB as it was tenancy related.
This was affirmed by the CA. However, the First Division of the Supreme Court affirmed the MTC’s ruling and set aside the RTC and
CA’s decision. They now complain that despite the fact that the decision of the Supreme Court in the aforesaid case had already
become final and executory, the respondent Judge still refused to issue a writ of possession in their favor.

As against the respondent Sheriff, the complainants aver[red] that through his ignorance, negligence and connivance with the
defendants, he failed to execute in full the writ of execution that had been previously issued by the court in Civil Case Nos. 481 and
482. Moreover, it took respondent Sheriff a long time before he finally submitted his Sheriff's Return of Service on the Writ of
Execution.

In his answer, Judge Bagagan stated that he had denied complainants Motion for the issuance of a writ of possession because, by the
time Civil Case Nos. 481 and 482 were finally decided by this Court on June 10, 1999, they had already been ousted from the lots in
question pursuant to the Decisions in DARAB Case No. 2413 and Civil Case No. 1920. In Civil Case No. 1920, respondent judge
ordered complainants to vacate the disputed lots. On the other hand, the DARAB Decision, which became final
and executory on October 27, 1998, directed them to cease and desist from disturbing the peaceful possession of therein Petitioner
Jaime Occidental.

Respondent Sheriff Matias admitted in his Comment that there was delay in the full implementation of the Writ of Execution in Civil
Case Nos. 481 and 482. Explaining that the delay was due to his heavy workload and thus unintentional, he begged for compassion
from this Court.
Issue:
(1) Whether or not Judge Bagagan acted correctly in not issuing the writ of possession?
(2) Whether Sheriff Matias is administratively liable?
Held:
(1) Yes. The respondent judge acted correctly in not issuing a writ of execution/possession. Besides, the latter’s Order directing
defendants not to molest complainants in their peaceful possession was rendered moot when they were ousted from the disputed lots
by virtue of the final and executory judgments in Civil Case No. 1920 and DARAB Case No. 2413.
Indeed, the execution of a final judgment may be refused, as in this case, when there has been a change in the situation of the parties
that would make its execution inequitable.
(2) Yes. He was remiss in his duty to implement the Writ fully in Civil Case Nos. 481 and 482. Those tasked to implement court
orders and processes must see to it that the final stage in the litigation process -- the execution of judgment -- be carried out promptly.
There is no mistaking the mandatory character of the period prescribed under Section 14 of Rule 39 of the Revised Rules of Court on
the Return of a Writ of Execution. A similar rule is stated in Administrative Circular No. 12 dated October 1, 1985, and incorporated
in the Manual for Clerks of Court. According to this Circular, all sheriffs and deputy sheriffs shall submit to the judge concerned a
report on actions taken on all writs and processes assigned to them within 10 days from receipt. Per the records of this case, a Writ of
Execution was issued on November 22, 1999 in Civil Case Nos. 481 and 482. Respondent Sheriffs Return of Service of that Writ was
filed only on May 25, 2000, however, or six months thereafter. There is nothing in the records showing that he submitted before then a
periodic report on the actions he had taken on the Writ every 30 days from the date of receipt as required. On the contrary, the Report
indicates that the Writ was partially executed on December 15-28, 1999 and January 11, 2000; and that the damages adjudged were
partly paid in the amount of P3,500 plus one unit of Karaoke machine. But it was only on May 25, 2000, that this matter was reported
to the trial court.
Remedial Law I| Page 199

C. Properties Exempt from Execution

D’ Armoured Security and Investigation Agency, Inc. vs. Orpia


G.R. No. 151325. June 27, 2005

Facts:

The security guards employed by DArmoured Security and Investigation Agency, Inc., assigned to Fortune Tobacco, Inc. filed with
the Labor Arbiter a complaint for illegal dismissal and various monetary claims against DArmoured Security and Fortune Tobacco.
The Labor Arbiter decided in favor of the security guards. Fortune Tobacco appealed to the NLRC while DArmoured Security did not
appeal. NLRC dismissed the complaint against Fortune Tabacco. This Decision became final and executory. Thus, the award specified
in the Decision of the Arbiter became the sole liability of DArmoured Security. The records were then remanded to the Arbiter for
execution. Upon the security guard’s motion, the Arbiter issued a writ of execution. Eventually, the sheriff served a writ of
garnishment upon the Chief Accountant of Foremost Farms, Inc., a corporation with whom DArmoured Security has an existing
services agreement. Thus DArmoured Security’s receivables with Foremost were garnished.
DArmoured Security filed with the NLRC a Motion to Quash/Recall Writ of Execution and Garnishment claiming that its monthly
receivables are exempt from execution. The Arbiter denied the motion. DArmoured Security motion for reconsideration was denied,
hence it appealed to the NLRC. NLRC dismissed the appeal. Its motion for reconsideration was also denied. Forthwith, DArmoured
Security filed with CA a petition for certiorari and prohibition with prayer for issuance of a writ of preliminary injunction. CA
dismissed the petition. Hence, this petition for review on certiorari under Rule 45.
Issue:
Whether or not the petitioner’s monthly receivables from the Foremost Farms, Inc. (garnishee) are exempt from execution?
Held:
No. Section 13 of Rule 39 of the Rules of Court is plain and clear on what properties are exempt from execution:

(i) So much of the salaries, wages or earnings of the judgment obligor for his personal services within the four months
preceding the levy as are necessary for the support of his family.

The exemption under this procedural rule should be read in conjunction with the Civil Code, the substantive law which proscribes the
execution of employees wages, thus:

ART. 1708. The laborers wage shall not be subject to execution or attachment, except for debts incurred for food, shelter, clothing
and medical attendance.

Obviously, the exemption under Rule 39 of the Rules of Court and Article 1708 of the New Civil Code is meant to favor only laboring
men or women whose works are manual. In this context, exemptions under this rule are confined only to  natural persons and not to
juridical entities such as petitioner.

It stands to reason that only natural persons whose salaries, wages and earnings are indispensable for his own and that of his familys
support are exempted under Section 13 (i) of Rule 39 of the Rules of Court. Undeniably, a corporate entity such as petitioner security
agency is not covered by the exemption.
Remedial Law I| Page 200

D. Revival of Judgment

Panotes vs. City Townhouse Development Corporation


G.R. No. 154739, January 23, 2007

Facts:

Rogelio Panotes filed a Complaint with the National Housing Authority (NHA) against Provident Village Homeowners Association,
Inc., against Provident Securities Corporation (PROSECOR), owner-developer of the Provident Village in Marikina City because
PROSECOR’s failure to provide an open space in the said subdivision. During the proceedings before the NHA, an ocular inspection
showed that the subdivision has no open space. The NHA found, however, that Block 40, with an area of 22,916 square meters, could
be utilized as open space. Thus, NHA, in a Resolution, directed PROSECOR to provide an open space which is Block 40. Considering
that PROSECOR did not appeal from the NHA Resolution, it became final and executory. When Panotes filed a motion for execution
of the NHA Resolution, it was found that the records of the case were “mysteriously missing.” Hence, his motion “was provisionally
dismissed” without prejudice.

Meanwhile, PROSECOR sold to City Townhouse Development Corporation (CTDC), respondent, several lots in the subdivision.
Among the lots sold were those comprising Block 40. CTDC was unaware of the NHA Resolution ordering PROSECOR to have
Block 40 utilized as open space of Provident Village.

Eventually, Panotes was succeeded by Araceli Bumatay as president of the Provident Village Homeowners Association, Inc. She then
filed with the Housing and Land Use Regulatory Board (HLURB) a complaint for revival of the NHA Resolution. HLURB ruled in
favor of Bumatay. The Office of the President affirmed the HLURB decision. However, the CA reversed the HLURB decision and
dismissed Bumatay’s Complaint for revival of judgment.

Issue:

Whether or not the NHA Resolution can be enforced against CTDC?

Held:

No. An action for revival of judgment is no more than a procedural means of securing the execution of a previous judgment which has
become dormant after the passage of five years without it being executed upon motion of the prevailing party. It is not intended to re-
open any issue affecting the merits of the judgment debtor’s case nor the propriety or correctness of the first judgment.

Here, the original judgment or the NHA Resolution sought to be revived was between Rogelio Panotes and PROSECOR, not between
Araceli Bumatay and respondent CTDC.

CTDC purchased from PROSECOR Block 40 in the said village, not as an owner-developer like PROSECOR, but as an ordinary
buyer of lots. Even after the sale, CTDC did not become an owner- developer. The Deed of Sale executed by CTDC, as buyer, and
PROSECOR, as seller, shows that the subject matter of the sale is the unsold lots comprising Block 40 within the subdivision to
CTDC. The contract does not include the transfer of rights of PROSECOR as owner-developer of the said subdivision. Clearly, there
is no basis to conclude that CTDC is the successor-in-interest of PROSECOR.

It bears stressing that when CTDC bought Block 40, there was no annotation on PROSECOR’s title showing that the property is
encumbered. In fact, the NHA Resolution was not annotated thereon. CTDC is thus a buyer in good faith and for value, and as such,
may not be deprived of the ownership of Block 40.

Furthermore, strangers to a case, like CTDC, are not bound by the judgment rendered by a court. It will not divest the rights of a party
who has not and never been a party to a litigation. Execution of a judgment can be issued only against a party to the action and not
against one who did not have his day in Court.
Remedial Law I| Page 201

E. Execution of Judgment by Motion

Rizal Commercial Banking Corporation vs. Serra


G.R. No. 203241. July 10, 2013

Facts:

Federico Serra is the owner of a 374 square meter parcel of land located in Masbate. Serra and RCBC entered into a Contract of Lease
with Option to Buy, wherein Serra agreed to lease his land to RCBC for 25 years. Serra further granted RCBC the option to buy the
land and improvement within 10 years. RCBC informed Serra of its decision to exercise its option to buy the property. However, Serra
replied that he was no longer interested in selling the property. RCBC filed a Complaint for Specific Performance and Damages
against Serra in the RTC Makati. The RTC Makati initially dismissed the complaint. However, the RTC Makati reversed itself and
ordered Serra to execute and deliver the proper deed of sale in favor of RCBC. Serra appealed to the CA. Meanwhile, Serra donated
the property to his mother, Leonida Ablao. Ablao, then, sold the property to Liok. A new land title was issued in favor of Liok. Thus,
RCBC filed a Complaint for Nullification of Deed of Donation and Deed of Sale with Reconveyance and Damages against Liok,
Ablao and Serra before the RTC Masbate. Meanwhile, the CA, and later the Supreme Court, affirmed the order of the RTC Makati in
the Specific Performance case. The decision in the Specific Performance case became final and executory upon entry of judgment.

The RTC Masbate ruled in favor of RCBC, declaring the donation in favor of Ablao and the subsequent sale to Liok null and
void. The CA affirmed the RTC Masbate decision. Thus, Liok filed a Petition for Review on Certiorari, while Serra and Ablao filed a
Petition for Certiorari, before this Court. This Court found neither reversible error nor grave abuse of discretion on the CA’s part.
RCBC moved for the execution of the decision in the Specific Performance case. RCBC alleged that it was legally impossible to ask
for the execution of the decision prior to the annulment of the fraudulent transfers made by Serra. Thus, the period to execute by
motion was suspended during the pendency of the Annulment case. Serra filed his comment and opposition to the motion. Serra
insisted that the motion for execution was already barred by prescription and laches, and that RCBC was at fault for failing to register
as lien in the original title the Contract of Lease with Option to Buy. The RTC Makati denied RCBC’s motion for execution as well as
the motion for reconsideration. Thus, RCBC filed this petition.

Issue:

Whether or not RCBC is barred from having its 05 January 1989 decision executed through motion after the lapse of five years?

Held:

No. The Rules of Court provide that a final and executory judgment may be executed by motion within five years from the date of its
entry or by an action after the lapse of five years and before prescription sets in. This Court, however, allows exceptions when
execution may be made by motion even after the lapse of five years. These exceptions have one common denominator: the delay is
caused or occasioned by actions of the judgment obligor and/or is incurred for his benefit or advantage. Where the delays were
occasioned by the judgment debtor’s own initiatives and for her advantage as well as beyond the judgment creditor’s control, the five-
year period allowed for enforcement of the judgment by motion is deemed to have been effectively interrupted or suspended.

In the present case, there is no dispute that RCBC seeks to enforce the decision which became final and executory on 15 April 1994.
This decision orders Serra to execute and deliver the proper deed of sale in favor of RCBC. However, to evade his obligation to
RCBC, Serra transferred the property to his mother Ablao, who then transferred it to Liok. Serra’s action prompted RCBC to file the
Annulment case. Clearly, the delay in the execution of the decision was caused by Serra for his own advantage. Thus, the pendency of
the Annulment case effectively suspended the five-year period to enforce through a motion the decision in the Specific Performance
case. Since the decision in the Annulment case attained finality on 3 March 2009 and RCBC’s motion for execution was filed on 25
August 2011, RCBC’s motion is deemed filed within the five-year period for enforcement of a decision through a motion.

This Court has reiterated that the purpose of prescribing time limitations for enforcing judgments is to prevent parties from sleeping on
their rights. Far from sleeping on its rights, RCBC has pursued persistently its action against Serra in accordance with law. On the
other hand, Serra has continued to evade his obligation by raising issues of technicality. While strict compliance with the rules of
procedure is desired, liberal interpretation is warranted in cases where a strict enforcement of the rules will not serve the ends of
justice.
Remedial Law I| Page 202

F. Foreign Judgment

Fujiki vs. Marinay


G.R. No. 196049. June 26, 2013

Facts:

Minoru Fujiki (Fujiki) is a Japanese national who married Maria Paz Galela Marinay (Marinay) in the Philippines. Marinay met
another Japanese, Shinichi Maekara (Maekara). Without the first marriage being dissolved, Marinay and Maekara married in the
Philippines. Maekara brought Marinay to Japan. However, Marinay allegedly suffered physical abuse from Maekara. She left Maekara
and started to contact Fujiki. Fujiki and Marinay met in Japan and they were able to reestablish their relationship. Fujiki helped
Marinay obtain a judgment from a family court in Japan which declared the marriage between Marinay and Maekara void on the
ground of bigamy. Fujiki filed a petition for Judicial Recognition of Foreign Judgment or Decree of Absolute Nullity of Marriage in
the RTC. Fujiki prayed that (1) the Japanese Family Court judgment be recognized; (2) that the bigamous marriage between Marinay
and Maekara be declared void ab initio under Articles 35(4) and 41 of the Family Code of the Philippines; and (3) for the RTC to
direct the Local Civil Registrar to annotate the Japanese Family Court judgment on the Certificate of Marriage between Marinay and
Maekara and to endorse such annotation to the NSO.

The RTC immediately issued an order dismissing the petition. The RTC took the view that only "the husband or the wife," in this case
either Maekara or Marinay, can file the petition to declare their marriage void, and not Fujiki. Fujiki moved that the order be
reconsidered. He argued that A.M. No. 02-11-10-SC contemplated ordinary civil actions for declaration of nullity and annulment of
marriage. Thus, A.M. No. 02-11-10-SC does not apply. The RTC resolved to deny petitioner’s motion for reconsideration. The Court
required respondents to file their comment on the petition for review. The public respondents, the Local Civil Registrar of Quezon
City and the Administrator and Civil Registrar General of the NSO, participated through the Office of the Solicitor General. Instead of
a comment, the Solicitor General filed a Manifestation and Motion.

The Solicitor General agreed with the petition. He prayed that the RTC’s "pronouncement that the petitioner failed to comply with x x
x A.M. No. 02-11-10-SC x x x be set aside" and that the case be reinstated in the trial court for further proceedings. The Solicitor
General argued that Fujiki, as the spouse of the first marriage, is an injured party who can sue to declare the bigamous marriage
between Marinay and Maekara void. The Solicitor General cited Juliano-Llave v. Republic which held that Section 2(a) of A.M. No.
02-11-10-SC does not apply in cases of bigamy. The Solicitor General contended that the petition to recognize the Japanese Family
Court judgment may be made in a Rule 108 proceeding. Moreover, the Solicitor General argued that there is no jurisdictional infirmity
in assailing a void marriage under Rule 108, citing De Castro v. De Castro and Niñal v. Bayadog which declared that "[t]he validity of
a void marriage may be collaterally attacked."

Issue:

Whether or not a foreign judgment decreeing a marriage void may be recognized in the Philippines?

Held:

Yes. For Philippine courts to recognize a foreign judgment relating to the status of a marriage where one of the parties is a citizen of a
foreign country, the petitioner only needs to prove the foreign judgment as a fact under the Rules of Court. To be more specific, a
copy of the foreign judgment may be admitted in evidence and proven as a fact under Rule 132, Sections 24 and 25, in relation to Rule
39, Section 48(b) of the Rules of Court. Petitioner may prove the Japanese Family Court judgment through (1) an official publication
or (2) a certification or copy attested by the officer who has custody of the judgment. If the office which has custody is in a foreign
country such as Japan, the certification may be made by the proper diplomatic or consular officer of the Philippine foreign service in
Japan and authenticated by the seal of office.

A foreign judgment relating to the status of a marriage affects the civil status, condition and legal capacity of its parties. However, the
effect of a foreign judgment is not automatic. To extend the effect of a foreign judgment in the Philippines, Philippine courts must
determine if the foreign judgment is consistent with domestic public policy and other mandatory laws. Article 15 of the Civil Code
provides that "[l]aws relating to family rights and duties, or to the status, condition and legal capacity of persons are binding upon
citizens of the Philippines, even though living abroad." This is the rule of lex nationalii in private international law. Thus, the
Philippine State may require, for effectivity in the Philippines, recognition by Philippine courts of a foreign judgment affecting its
citizen, over whom it exercises personal jurisdiction relating to the status, condition and legal capacity of such citizen.

Section 48(b), Rule 39 of the Rules of Court provides that a foreign judgment or final order against a person creates a "presumptive
evidence of a right as between the parties and their successors in interest by a subsequent title." Moreover, Section 48 of the Rules of
Court states that "the judgment or final order may be repelled by evidence of a want of jurisdiction, want of notice to the party,
Remedial Law I| Page 203

collusion, fraud, or clear mistake of law or fact." Thus, Philippine courts exercise limited review on foreign judgments. Courts are not
allowed to delve into the merits of a foreign judgment. Once a foreign judgment is admitted and proven in a Philippine court, it can
only be repelled on grounds external to its merits, i.e. , "want of jurisdiction, want of notice to the party, collusion, fraud, or clear
mistake of law or fact." The rule on limited review embodies the policy of efficiency and the protection of party expectations,  as well
as respecting the jurisdiction of other states.

There is therefore no reason to disallow Fujiki to simply prove as a fact the Japanese Family Court judgment nullifying the marriage
between Marinay and Maekara on the ground of bigamy. While the Philippines has no divorce law, the Japanese Family Court
judgment is fully consistent with Philippine public policy, as bigamous marriages are declared void from the beginning under Article
35(4) of the Family Code. Bigamy is a crime under Article 349 of the Revised Penal Code. Thus, Fujiki can prove the existence of the
Japanese Family Court judgment in accordance with Rule 132, Sections 24 and 25, in relation to Rule 39, Section 48(b) of the Rules
of Court.
Remedial Law I| Page 204

G. Res Judicata

City of Cebu vs. Dedamo, Jr.


G.R. No. 172852. January 30, 2013

Facts:

The present controversy is an off-shoot of civil case for eminent domain over two (2) parcels of land owned by spouses Spouses
Dedamo, filed by the petitioner before the RTC of Cebu City. The petitioner immediately took possession of the lots after
depositing P51,156.00 with the PNB. During the pendency of the case, the petitioner and Spouses Dedamo entered into a Compromise
Agreement whereby the latter agreed to part with the ownership of the parcels of land in favor of the former in consideration of
P1,786,400.00 as provisional payment and just compensation in an amount to be determined by a panel of commissioners. They
recommend the sum of P20,826,339.50 as just compensation and it was approved by the RTC. The RTC Order was affirmed by the
CA and then by the Court, when the matter was elevated for review. When the said decision became final and executory, the case was
remanded for execution to the RTC, before which, a motion for the issuance of a writ of execution was filed by Spouses Dedamo. The
RTC granted the motion and ordered the issuance of the writ. In the meantime, Spouses Dedamo passed away and they were
substituted in the case by herein respondent. The petitioner paid the respondent the sum of P19,039,939.50 which is the difference
between the just compensation due and the provisional payment already made. The respondent filed a motion before the RTC to order
the petitioner to pay interest on the just compensation computed from the time of actual taking of the lands.

The RTC denied the motion and ruled that it can no longer amend a final and executory judgment that did not specifically direct the
payment of legal interest. Adamant, the respondent sought recourse before the CA asserting that the petitioner is liable to pay: (a) 12%
legal interest on the unpaid balance of the just compensation computed from the time of actual taking of the property up to the date of
payment of just compensation; and (b) 12% legal interest from the time the decision awarding just compensation became final and
executory on September 20, 2002 until its satisfaction on December 23, 2003.

The CA rejected the respondents first claim since the issue was belatedly raised and found the respondents second contention
meritorious. The CA awarded legal interest accruing from the time the RTC Order dated December 27, 1996 awarding just
compensation was affirmed with finality by the Supreme Court up to the time of full payment thereof in line with the ruling in Eastern
Shipping Lines, Inc. v. Court of Appeals that when a court judgment awarding a sum of money becomes final and executory, it shall
earn legal interest of 12% per annum reckoned from such finality until satisfaction. In G.R. No. 172942, the Supreme Court denied
respondent’s petition on August 22, 2006 for failure to sufficiently show that the CA committed any reversible error in the questioned
judgment. This present petition, initiated by the City of Cebu, the petitioner seeks to set aside the same CA Decision awarding legal
interest to respondet.

Issue:

Whether or not the petition should be denied on the ground of res judicata in the mode of conclusiveness of judgment?

Held:

Yes. A perusal of the allegations in the present case evidently shows that the petitioner broaches the issues similarly raised and already
resolved in G.R. No. 172942.

Under the principle of conclusiveness of judgment, when a right or fact has been judicially tried and determined by a court of
competent jurisdiction, or when an opportunity for such trial has been given, the judgment of the court, as long as it remains
unreversed, should be conclusive upon the parties and those in privity with them. Stated differently, conclusiveness of judgment bars
the re-litigation in a second case of a fact or question already settled in a previous case.

The adjudication in G.R. No. 172942 has become binding and conclusive on the petitioner who can no longer question the
respondent’s entitlement to the 12% legal interest awarded by the CA. The Courts determination in G.R. No. 172942 on the reckoning
point of the 12% legal interest is likewise binding on the petitioner who cannot re-litigate the said matter anew through the present
recourse. Thus, the judgment in G.R. No. 172942 bars the present case as the relief sought in the latter is inextricably related to the
ruling in the former.

H. Terceria

PSALM vs. Maunlad Homes


G.R. No. 215933. February 8, 2017
Remedial Law I| Page 205

Facts:

Respondent Maunlad Homes, Inc. filed with the Municipal Trial Court in Cities (MTCC), Malolos City, Bulacan, an unlawful detainer
case against National Power Corporation (NPC). The MTCC issued its Decision, ordering NPC to vacate the subject premises and
surrender physical possession thereof to respondent. The NPC appealed the decision to the Regional Trial Court (RTC) of Malolos
City, Bulacan. However, it affirmed the decision of the MTCC. Respondent filed a Motion for Execution which was opposed by the
NPC. The NPC also filed a motion for reconsideration of the RTC decision. RTC denied the MR and granted respondent's motion for
execution. A Writ of Execution pending appeal was issued. The sheriff served a Notice of Demandof payment to the NPC.

Respondent then filed an urgent motion for issuance of a Break Open Order since the sheriff who tried to implement the writ of
execution, by serving the notice of levy on the NPC Warehouse at Barangay Lagundi, Mexico, Pampanga, was prevented by the
security guards assigned therein. The NPC argued that the warehouse is being used both by it and the Power Sector Assets and
Liabilities Management Corporation (herein petitioner PSALM), an entity created and existing by virtue of Republic Act No. 9136,
the Electric Power Industry Reform Act of 2001 (EPIRA Law); that the said law provides that the ownership and all generation assets,
IPP contracts and other NPC disposable assets are transferred to PSALM; and that as of the moment, the ownership of the said items
stored in the said warehouse cannot be established with certainty as they are in the process of determining what properties may be
retained by the latter.

The sheriff issued a Notice of Levy on execution pending appeal of personal properties/sale of seven (7) units transformer radiator
fins, one (1) unit power transformer and four (4) pieces angle bars.

Petitioner PSALM filed an Affidavit of third-party claim with the sheriff pursuant to Section 16, Rule 39 of the Rules of Court, and
alleging that it is the owner of the levied properties pursuant to the EPIRA Law. It also filed a Manifestation with Urgent Ex Parte
Motion for Issuance of Status Quo Order with the RTC arguing that it is the owner of the subject properties pulled out by the sheriff
by operation of law; that it is not a party to the instant case and therefore cannot be bound by the judgment therein; that the obligation
to pay respondent had not been transferred to it. It also prayed for the nullification of the levy of its properties and restoring their
immediate possession to it. RTC denied the motion of PSALM.

Petitioner filed with the CA a petition for certiorari assailing Break Open Order, the notice of levy on execution pending appeal, the
Order dated denying the motion for issuance of Status Quo Order and the third-party claim, and the notice of sale on execution of
personal properties. It alleged that it has no adequate remedy available from the writs and processes issued by the RTC, and that it
acted without or in excess of jurisdiction in issuing the assailed orders despite the fact that petitioner is the owner of the subject
properties. The CA dismissed the petition for certiorari for being an incorrect remedy. Petitioner filed a motion for reconsideration,
which was denied by the CA.

Issue:

Whether or not the third-party claim of PSALM was proper?

Held:

No. The power of the court in executing judgments extends only to properties unquestionably belonging to the judgment debtor alone.
An execution can be issued only against a party and not against one who did not have his day in court. The duty of the sheriff is to
levy the property of the judgment debtor not that of a third person. For, as the saying goes, one man’s goods shall not be sold for
another man’s debts. Thus, if the property levied by virtue of a writ of execution is claimed by a third person who is not the judgment
obligor, Section 16 of Rule 39 of the 1997 Rules of Civil Procedure provides for the remedy of such third party claimant. The third-
party claimant may execute an affidavit of his title or right to the possession of the property levied, and serve the same to the officer
making the levy and a copy thereof to the judgment creditor. This remedy is known as terceria. The officer shall not be bound to keep
the property, unless the judgment creditor files a bond approved by the court to indemnify the third-party claimant in a sum not less
than the value of the property levied on. An action for damages may be brought against the officer within one hundred twenty (120)
days from the date of the filing of the bond.

In Spouses Sy v. Hon. Discaya, the court held that the claim of ownership or right of possession to the levied property by the third-
party claimant must first be unmistakably established, thus:

x x x A third person whose property was seized by a sheriff to answer for the obligation of the judgment debtor may invoke the
supervisory power of the court which authorized such execution. Upon due application by the third person and after summary hearing,
the court may command that the property be released from the mistaken levy and restored to the rightful owner or possessor. What
said court can do in these instances, however, is limited to a determination of whether the sheriff has acted rightly or wrongly in the
Remedial Law I| Page 206

performance of his duties in the execution of judgment, more specifically, if he has indeed taken hold of property not belonging to the
judgment debtor. The court does not and cannot pass upon the question of title to the property, with any character of finality. It can
treat of the matter only insofar as may be necessary to decide if the sheriff has acted correctly or not. It can require the sheriff to
restore the property to the claimant's possession if warranted by the evidence. However, if the claimant's proofs do not persuade the
court of the validity of his title or right of possession thereto, the claim will be denied.

Vous aimerez peut-être aussi